LISTA DE DISCIPLINAS

Exercícios

MA211 - Cálculo II

Selecione os exercícios por

Dificuldade

Categoria

Outros

Os botões acima permitem selecionar que tipos de exercício você deseja ver na lista.
Para retirar alguma categoria da lista, clique sobre o botão para toná-lo inativo. Para adicioná-la, clique novamente no botão.


2252   

Demonstre a identidade abaixo, supondo que $S$ e $E$ satisfaçam as condições do Teorema do Divergente e que as funções escalares e as componentes dos campos vetoriais tenham derivadas parciais de segunda ordem contínuas.

  1. $\displaystyle\iint\limits_{S}{\bf a}\cdot {\bf n}\,dS=0$, onde ${\bf a}$ é um vetor constante.



Dica: Note que $\mbox{div} {\bf a} = 0.$


2302   

Identifique e faça um esboço da imagem da superfície parametrizada dada por ${\bf r}(u,v)=(u,\sqrt{1-u^{2}-v^{2}},v)$, $u^{2}+v^{2}\leq 1.$


Semi superfície esférica $x^2 + y^2 + z^2 = 1,$ $y  \geq 0.$


2325   

Calcule a área da superfície dada por: ${\bf r}(u,v)=(u,v,4-u^{2}-v^{2})$, $(u,v)\in K$, onde $K$ é o conjunto no plano $uv$ limitado pelo eixo $u$ e pela curva (em coordenadas polares) $\rho=e^{-\theta}$,$0\leq \theta \leq \pi.$. (Sugerimos ao leitor desenhar a imagem da superfície.)


$\displaystyle \dfrac{1}{72} \left( \ln\left(3\dfrac{\sqrt{e^{2\pi} + 4} + e^{\pi}}{\sqrt{e^{2\pi} + 4} - e^{\pi}} \right) + 3 \ln\left(\dfrac{\sqrt{5} - 1 }{\sqrt{5} + 1 }\right) - 8e^{3\pi} \sqrt{e^{2\pi} + 4}(e^{2\pi} + 1) + 16\sqrt{5} - 6\pi \right).$


2272   

No item abaixo :

  1.  determine o gradiente de $f$; 
  2.  calcule o gradiente no ponto $P$; e 
  3.  determine a taxa de variação de $f$ em $P$ na direção do vetor $\bf{u}$.

$f(x,y) = y \ln{x},  P = (1, -3),  \bf{u} = \left(-\frac{4}{5}, \frac{3}{5} \right)$.


  1. $\nabla f(x,y) = (y/x,\ln(x)).$
  2. $\nabla f(1,-3) = (-3,0).$
  3. $\displaystyle \frac{12}{5}.$


2538   

Determine o sólido $E$ para o qual a integral $$ \iiint\limits_{  E}(1-x^{2}-2y^{2}-3z^{2})\,dV$$ é máxima.


$E = \left\{ (x,y,z);  x^2 + 2y^2 + 3z^2 \leq 1 \right\}.$


2735   

Determine o plano que passa pelos pontos $(1,1,2)$ e $(-1,1,1)$ e que seja tangente ao gráfico de $f(x,y) = xy$.


$x + 6y - 2z = 3$.


2360   

Encontre a área da superfície $z=1+3x+3y^{2}$ que está acima do triângulo com vértices $(0,0)$, $(0,1)$ e $(2,1).$


$\dfrac{1}{54}\left(46\sqrt{46} - 10\sqrt{10} \right).$


2789   

Determine os valores máximos e mínimos locais e pontos de sela da função $f(x,y)=9-2x+4y-x^{2}-4y^{2}$.


Ponto de máximo: $\displaystyle \left( -1, \frac{1}{2} \right).$


2093   

Seja

$${\bf F}(x,y)=\bigg(\frac{-y}{x^{2}+y^{2}},\frac{x}{x^{2}+y^{2}}+3y\bigg)$$

um campo vetorial em $\mathbb{R}^{2}$. Calcule a integral de linha do campo ${\bf F}$ ao longo das curvas

$C_{1}$ e $C_{2}$, orientadas no sentido anti-horário, onde:

  1. $C_{1}$ é a circunferência de equação $x^{2}+y^{2}=4.$

  2. $C_{2}$ é a fronteira do retângulo $R=\{(x,y)\in \mathbb{R}^{2}|\,-\pi \leq x \leq \pi,-3 \leq y \leq 3\}.$


  1. $0.$

  2. $0.$


2659   

Determine as derivadas parciais de $z=xye^{xy}$.


$\displaystyle \frac{\partial z}{\partial x} = ye^{xy} (1 + xy) \;\;\;\;\;\;\text{e}\;\;\;\;\; \frac{\partial z}{\partial y} = xe^{xy} (1 + xy).$


3133   

Encontre a massa da lâmina descrita como sendo a porção do parabolóide \(2z=x^2+y^2\) que fica dentro do cilindro \(x^2+y^2=8\) e tem densidade \(\delta_0\) constante.


2086   

Calcule $\displaystyle\int_{C}(\sin(xy)+xy\,\cos(xy))\,dx+x^{2}\,\cos(xy)\,dy$, onde $C(t)=(t^{2}-1,t^{2}+1)$, $-1\leq t\leq 1.$


$0.$


1936   

Calcule a integral de linha, onde $C$ é a curva dada.

$\displaystyle\int_{C}x\,dx+y\,dy$,   $C:\,x=t^{2},\,y=\sin t$, $0\leq t\leq \pi/2.$


$\displaystyle \frac{\pi^{4}}{32} + \frac{1}{2}.$


2995   

Calcule a integral, efetuando uma mudança de variáveis apropriada. $\displaystyle\iint\limits_{R}\dfrac{1}{\sqrt{xy}}\, dA$, em que $R$ é a região limitada pela curva $x+y = 1$ e pelos eixos coordenados.


$\pi.$


2799   

Determine os valores máximos e mínimos locais e pontos de sela da função $f(x,y)=(x^{2}+y^{2})e^{y^{2}-x^{2}}$.


Ponto de mínimo: $(0,0);$ pontos de sela: $(1,0)$ e $(-1,0).$


2281   

Defina gradiente de uma função de três variáveis. Calcule $\nabla f(x,y,z)$.

$f(x,y,z) = (x^2 + y^2 + 1)^{z^2}$


$\displaystyle \nabla f(x,y,z) = (x^{2} + y^{2} + 1)^{z^{2}-1}\left(2xz^{2},2yz^{2},2z(x^{2} + y^{2} + 1)\ln(x^{2} + y^{2} + 1)\right).$


2955   

Usando coordenadas esféricas, determine o volume da porção da esfera sólida $\rho \leq a$ que está entre os cones $\phi=\pi/3$ e $\phi=2\pi/3.$


$\dfrac{2\pi a^{3}}{3}.$


3102   

A reta \(y=2-x\) intersecta a parábola \(y=x^2\) nos pontos \((-2,4)\) e \((1,1)\). Mostre que, se \(R\) denotar a região englobada por \(y=2-x\) e \(y=x^2\), então \[ \iint_R\left(1+2y\right)\,dA = \int_{-2}^1\int_{x^2}^{2-x}\left(1+2y\right)\,dydx = 18,9 \]


1950   

Calcule a integral de linha $\int_{C}{\bf F}\cdot d{\bf r}$, onde $C$ é dada pela função vetorial ${\bf r}(t).$

${\bf F}(x,y,z)=(x+y)\,{\bf i}+(y-z)\,{\bf j}+z^{2}\,{\bf k}$, ${\bf r}(t)=t^{2}\,{\bf i}+t^{3}\,{\bf j}+t^{2}\,{\bf k}$, $0\leq t\leq 1.$


$\dfrac{17}{15}.$


2331   

Determine a taxa de variação máxima de $f$ no ponto dado e a direção em que isso ocorre.

$f(x,y) = \sin{xy},  (1,0).$


$1.$


2592   

Considere um escoamento com velocidade ${\bf v}(x,y,z)$ e densidade $\rho(x,y,z)$, tal que ${\bf u}=\rho {\bf v}$ seja dado por ${\bf u}=x{\bf i}+y{\bf j}-2z{\bf k}$. Seja $S$ a superfície $x^{2}+y^{2}+z^{2}=4$, $z\geq \sqrt{2}$, e seja ${\bf n}$ a normal com componente $z>0$. Calcule o fluxo de ${\bf u}$ através de $S$. (Observe que, neste caso, o fluxo tem dimensões $MT^{-1}$ (massa por unidade de tempo).)


$-4\pi\sqrt{2}.$


2865   

Estude com relação a máximos e mínimos a função dada com as restrições dadas.

$f(x,y) = x^2 + 2xy + y^2$ e $x + 2y - 1 = 0.$


Ponto de mínimo: $\displaystyle \left(-1,1 \right)$


2045   

Utilize a Equação
$$ \dfrac{dy}{dx}=-\dfrac{\dfrac{\partial F}{\partial x}}{\dfrac{\partial F}{\partial y}}=-\dfrac{F_x}{F_y}$$
para determinar $\mathrm{d}y/\mathrm{d}x$.
$\cos(x-y)=xe^{y}$


$\displaystyle \frac{dy}{dx} = \frac{\sin(x - y) + e^{y} }{\sin(x - y) -x e^{y}} .$


2752   

A função $f(x,y) = \begin{cases}\dfrac{x^2 - y^2}{x^2 + y^2}, & \quad \text{se } (x,y) \neq (0,0),\\0, & \quad \text{se } (x,y) = (0,0)\\\end{cases}$ é diferenciável em $(0,0)$? Justifique.


Não.


2847   

Passe para coordenadas polares e calcule: $\displaystyle\iint\limits_{R}\cos(x^{2}+y^{2})\,dA$, onde $R$ é a região acima do eixo do $x$ e dentro da circunferência $x^{2}+y^{2}=9.$


$\displaystyle \frac{\pi}{2} \sin(9).$


2713   

Determine uma equação do plano tangente à superfície no ponto especificado.

$z = 3(x-1)^2 + 2(y+3)^2 + 7, \quad (2,-2,12)$.


$z = 6x + 4y + 8$.


2060   

Determine se ${\bf F}(x,y,z)=(e^{x}\,\cos y)\,{\bf i}-(e^{x}\,\sin y)\,{\bf j}+z\,{\bf k}$ é ou não um campo vetorial conservativo. Se for, determine uma função $f$ tal que ${\bf F}=\nabla f.$


Sim. $f(x,y,z) = e^{x}\cos(y) + \dfrac{z^{2}}{2}  + K.$


2505   

Faça um esboço do diagrama de contorno da função cujo gráfico é mostrado.

ma211-list2-ex16.png


$y = 2x \pm \sqrt{C},$ $C \geq 0.$

ma211-list2-ex16_sol.png


2615   

Verifique que o Teorema de Stokes é verdadeiro para o campo vetorial ${\bf F}$ dado e a superfície $S$.

  • ${\bf F}(x,y,z) = y^2{\bf i} + x{\bf j} + z^2{\bf k}$, $S$ é a parte do parabolóide $z = x^2 + y^2$ que está acima do plano $z = 1$, orientado para cima.



$\displaystyle\int_{C} {\bf F} \cdot d{\bf R} = \displaystyle\iint_{S} \mbox{rot} {\bf F} \cdot d{\bf S} = \pi.$


2197   

Encontre os valores de $\partial z/ \partial x$ e $\partial z/\partial y$ no ponto indicado.
$\dfrac{1}{x}+\dfrac{1}{y}+\dfrac{1}{z}-1=0$,  $(2,3,6).$


$\displaystyle \frac{\partial z}{\partial x}(2,3,6) = -9$ e $\displaystyle \frac{\partial z}{\partial x}(2,3,6) = -4.$


2680   

Encontre $\partial f/\partial x$ e $\partial f/\partial y$ para $f(x,y)=(x^{2}-1)(y+2)$.


$\displaystyle \frac{\partial f}{\partial x} = 2x(y + 2) \;\;\;\;\text{e}\;\;\;\; \frac{\partial f}{\partial y} = x^{2} - 1$.


2480   

Esboce o gráfico da função $f(x,y)=y$.


$z = y.$

ma211-list2-ex11_sol_b.png


2367   

Em que direção e sentido a função dada cresce mais rapidamente no ponto dado? E em que direção e sentido decresce mais rapidamente?

$f(x,y) = \ln{||(x,y)||}$ em $(1,-1)$.


 Cresce: $(1,-1)$; descresce: $(-1,1).$


2914   

Considere a integral dada em coordenadas polares por $$\int_{0}^{\pi/4}   \int_{0}^{2\cos{\theta}}r\,dr d\theta,$$ a qual representa a área de uma região $R$ do plano $xy.$

  1. Escreva a região $R$ em coordenadas cartesianas.

  2. Faça um esboço da região $R.$

  3. Calcule a área da região $R.$


  1.  $R = \left\lbrace (x,y);   (x - 1)^2 + y^2 \leq 1,\quad x \leq y,\quad x \geq 0,\quad y \geq 0 \right\rbrace.$

  2. (...)

  3.  $\dfrac{\pi + 2}{4}.$


2491   

Dada a função $f(x,y)=y-x$.

  1. Encontre o domínio da função.

  2. Encontre a imagem da função.

  3. Descreva as curvas de nível da função.


  1. $D_{f} = \mathbb{R}^{2}$.

  2. $Im(f) = \mathbb{R}.$

  3. As curvas de nível são as retas $y - x = C.$


3010   

Uma lâmina ocupa parte do disco $x^2 + y^2 \leq 1$ no primeiro quadrante. Determine o centro de massa se a densidade em qualquer ponto for proporcional à distância do ponto ao eixo $x$.


$\displaystyle \left(\frac{3}{8}, \frac{3\pi}{16} \right).$


2698   

Calcule todas as derivadas parciais de $2^{\underline{a}}$ ordem de $g(x,y)=4x^{3}y^{4}+y^{3}$.


$\displaystyle \frac{\partial^{2} g}{\partial x^{2}}= 24xy^{2},\;\;\;\;\; \frac{\partial^{2} g}{\partial y^{2}}= 48x^{3}  y^{2} \;\;\;\;\;\text{e}\;\;\;\;\; \frac{\partial^{2} g}{\partial x\partial y}= \frac{\partial^{2} g}{\partial y\partial x}= 48x^{2}y^{3}.$


2159   

Calcule a integral de linha $\displaystyle\oint_{C} xy \, dx + x^2y^3 \, dy$, $C$ é o triângulo com vértices $(0,0)$, $(1,0)$ e $(1,2)$ por dois métodos:

  1. diretamente; e

  2. utilizando o Teorema de Green.


$\dfrac{2}{3}.$


1930   

Calcule a integral de linha, onde $C$ é a curva dada.

$\displaystyle\int_{C}y^{3}\,ds$,   $C:\,x=t^{3},\, y=t,\, 0\leq t\leq 2.$


$\displaystyle \frac{1}{54}\left(145^{3/2} - 1 \right).$


2063   

Dados ${\bf F}(x,y)=x^{2}\,{\bf i}+y^{2}\,{\bf j}$, $C$ é o arco da parábola $y=2x^{2}$ de $(-1,2)$ a $(2,8).$

  1. Determine uma função $f$ tal que ${\bf F}=\nabla f$.

  2. Use o resultado anterior para calcular $\int_{C}{\bf F}\cdot d{\bf r}$ sobre a curva $C$ dada.


  1. $f(x,y) = \dfrac{x^{3} + y^{3}}{3};$

  2. $171.$


2017   

Determine o volume do sólido limitado pelos planos coordenados e pelo plano $3x+2y+z=6.$



O sólido cujo volume deve ser calculado é $$E = \{(x,y,z) \in \mathbb{R}^3; (x,y) \in R \mbox{ e } 0 \leq z \leq 6 - 3x - 2y\},$$ em que $R$ é a projeção de $E$ no plano $xy$. Assim, o volume é dado por $$V = \displaystyle\int\!\!\!\!\int\limits_{R}(6-3x-2y)\,dA.$$ A região $R$ é tanto do tipo I como do tipo II, então é possível escrevê-la de pelo menos duas formas. Escrevendo como uma região do tipo I, obtemos: $$R = \left\{(x,y) \in \mathbb{R}^2: 0 \leq x \leq 2 \mbox{ e } 0 \leq y \leq \frac{6-3x}{2}\right\}.$$ Portanto, \begin{eqnarray*} V & = & \displaystyle\int_{0}^{2}\!\int_{0}^{\frac{6-3x}{2}}(6-3x-2y)\,dy dx \\   & = & \displaystyle\int_{0}^{2} \left.\left(6y-3xy-y^2 \right|_{y=0}^{y=\frac{6-3x}{2}} \right) \,dx \\     & = & \displaystyle\int_{0}^{2} \left(9-9x+\frac{9x^2}{4}\right) \,dx \\     & = & \left.9x-\frac{9x^2}{2}+\frac{9x^3}{12} \right|_{x=0}^{x=2} = 6. \end{eqnarray*} Observe que podemos escrever $R$ como uma região do tipo II, obtendo: $$R = \left\{(x,y) \in \mathbb{R}^2: 0 \leq x \leq \frac{6-2y}{3} \text{ e } 0 \leq y \leq 3\right\}.$$ Então, uma outra expressão para $V$ é $$V = \displaystyle\int_{0}^{3}\!\int_{0}^{\frac{6-2y}{3}}(6-3x-2y)\,dx dy = 6.$$


2397   

Determine uma reta que seja tangente à elipse $2x^2 + y^2 = 3$ e paralela à reta $2x + y = 5$.


 $\displaystyle y = -2x + 3$ ou$\displaystyle y = -2x - 3.$


3088   

Mostre que se \(f\), \(f_x\) e \(f_y\) são contínuas numa região circular contendo os pontos \(A=(x_0,y_0)\) e \(B=(x_1,y_1)\), então existe um ponto \((x^\ast,y^\ast)\) no segmento que une \(A\) e \(B\) tal que \[ f(x_1,y_1)-f(x_0,y_0) = f_x(x^\ast,y^\ast)(x_1-x_0)+f_y(x^\ast,y^\ast)(y_1-y_0). \] Este resultado é a versão bidimensional do Teorema do Valor Médio. [Sugestão: expresse o segmento de reta  unindo \(A\) e \(B\) na forma paramétrica e use o Teorema do Valor Médio para funções de uma variável.]


2989   

Calcule a integral, efetuando uma mudança de variáveis apropriada. $\displaystyle\iint\limits_{R} e^{x+y} \, dA$, em que $R$ é dada pela inequação $|x| + |y| \leq 1$.


$e - e^{-1}.$


2805   

Determine os valores máximos e mínimos locais e pontos de sela da função $f(x,y)=x^{3}-12xy+8y^{3}$.


Ponto de mínimo : $\displaystyle \left( 2,1\right);$ ponto de sela: $\displaystyle \left(0,0\right).$


2711   

O elipsoide $4x^{2}+2y^{2}+z^{2}=16$ intercepta o plano $y=2$ em uma elipse. Determine as equações paramétricas da reta tangente à elipse no ponto $(1,2,2).$


$x = 1 + t,$ $y = 2,$ $z = 2 - 2t$.


3105   

Mostre que

\[ \int_0^{+\infty}\int_0^{+\infty}\dfrac{1}{(1+x^2+y^2)^2}\,dxdy= \dfrac{\pi}{4}.\]


2261   

Calcule $\displaystyle\iint\limits_{B} y\,dx dy$, onde $B$ é o conjunto dado.

  1. $B$ é o triângulo de vértices $(0,0)$, $(1,0)$ e $(1,1)$.

  2.  $B=\{(x,y)\in \mathbb{R}^{2}|\;-1\leq x\leq 1,\;0\leq y\leq x+2\}.$

  3.  $B$ é o conjunto de todos $(x,y)$ tais que $x^{2}+4y^{2}\leq 1.$

  4.  $B$ é o triângulo de vértices $(0,0)$, $(1,0)$ e $(2,1).$


  1.  $\dfrac{1}{6}$.

  2.  $\dfrac{13}{3}$.

  3.  $0$.

  4.  $\dfrac{1}{6}$.


3021   

Esboce a região de integração e calcule a integral $\displaystyle\int_{0}^{\pi}\!\!\int_{0}^{x}x\sin{y}\,dy dx$.


$\dfrac{\pi^{2}}{2} + 2.$

ma211-list6-ex25_sol_b.png


2970   

Calcule a integral, transformando para coordenadas esféricas. $\displaystyle\int_{-a}^{a}\int_{-\sqrt{a^{2}-y^{2}}}^{\sqrt{a^{2}-y^{2}}}\int_{-\sqrt{a^{2}-x^{2}-y^{2}}}^{\sqrt{a-x^{2}-y^{2}}}(x^{2}z+y^{2}z+z^{3})\,dzdxdy$.


$0.$


3023   

Esboce a região de integração e calcule a integral $\displaystyle\int_{1}^{\ln 8}\!\!\!\int_{0}^{\ln y}e^{x+y}\,dx dy$.


$8 \ln(8) - 16 + e.$

ma211-list6-ex25_sol_d.png


2421   

Calcule a integral tripla $\displaystyle\iiint\limits_{B}xyz^{2}\,dV$, onde $B$ é a caixa retangular dada por $B=\{(x,y,z) \in \mathbb{R}^3|\;0\leq x\leq 1,\;-1\leq y\leq 2,\;0\leq z\leq 3\}$, integrando primeiro em relação a $y$, depois a $z$ e então a $x$.


$\dfrac{27}{4}.$


2501   

Encontre uma equação para a curva de nível da função $f(x,y)=16-x^{2}-y^{2}$ que passa pelo ponto $(2\sqrt{2},\sqrt{2})$.


$x^{2} + y^{2} = 10.$


2730   

Determine as equações do plano tangente e da reta normal ao gráfico da função dada, no ponto dado. $f(x,y) = x^2 + y^2$ em $(0,1,f(0,1))$.


Plano tangente: $z = 2y - 1$

Reta normal: $(x,y,z) = \left(0,1,1 \right) + \lambda \left(0,2,-1 \right)$.


2767   

Encontre o valor de $\partial z/\partial x$ no ponto $(1,1,1)$ sabendo que a equação

$$xy+z^{3}x-2yz=0$$

define $z$ como uma função de duas variáveis independentes $x$ e $y$ e que a derivada parcial existe.


$\displaystyle \frac{\partial z}{\partial x} (1,1,1) = -2$.


2651   

A lei dos gases para uma massa fixa $m$ de um gás ideal à temperatura absoluta $T$, pressão $P$ e o volume $V$ é $PV=mRT$, onde $R$ é a constante do gás. Mostre que

$$\frac{\mathrm{\partial}P}{\mathrm{\partial}V}\frac{\mathrm{\partial}V}{\mathrm{\partial}T}\frac{\mathrm{\partial}T}{\mathrm{\partial}P}=-1.$$


$\displaystyle \frac{\partial P}{\partial V} = -\frac{mRT}{V^{2}},\;\;\;\frac{\partial V}{\partial T} = \frac{mR}{P}\;\;\;\text{e}\;\;\; \frac{\partial T}{\partial P} = \frac{V}{mR}.$


2721   

Determine a diferencial da função $m = p^5q^3$.


$dm = 5p^{4}q^{3} dp + 3p^{5}q^{2} dq$.


2464   

Calcule a integral de superfície $\displaystyle\iint \limits_{ S}{\bf F}\cdot d{\bf S}$ para o campo vetorial ${\bf F}$ e superfície orientada $S$ dados abaixo. Em outras palavras, determine o fluxo de ${\bf F}$ através de $S$. Para superfícies fechadas, use a orientação positiva (para fora).

  • ${\bf F}(x,y,z)=y{\bf j}-z{\bf k}$ e $S$ é formada pelo parabolóide $y=x^{2}+z^{2}$, $0 \leq y \leq 1$ e pelo círculo $x^{2}+z^{2} \leq 1$, $y=1.$


$0.$


2380   

Determine $\int_{0}^{5}f(x,y)\,dx$ e $\int_{0}^{1}f(x,y)\,dy$, sendo $f(x,y)=12x^{2}y^{3}.$


$\int_{0}^{5} 12x^{2}y^{3} \,dx = 500y^{3}$ e $\int_{0}^{1} 12x^{2}y^{3} \,dy = 3x^{2}.$


3045   

Se uma circunferência $C$ de raio $1$ rola ao longo do interior da circunferência $x^2+y^2=16$, um ponto fixo $P$ de $C$ descreve uma curva chamada epicicloide, com equações paramétricas $x = 5\cos{t}-\cos{5t}$, $y = 5\sin{t} - \sin{5t}$. Faça o gráfico da epicicloide e calcule a área da região que ela envolve.


$30\pi.$

ma211-list12-ex15_sol.png


2912   

Use a integral dupla em coordenadas polares para deduzir a fórmula $$A=\int_{\alpha}^{\beta}\frac{1}{2} r^{2}\,d\theta$$ para a área da região em formato de leque entre a origem e a curva polar $r=f(\theta)$, $\alpha\leq \theta \leq \beta.$


Note que $\displaystyle A = \int_{\alpha}^{\beta}\int_{0}^{f(\theta)} r  dr d\theta. $


2614   

Use o Teorema de Stokes para calcular $\displaystyle\int_C {\bf F}\cdot  d{\bf R}$. $C$ é orientada no sentido anti-horário quando vista de cima.

  • ${\bf F}(x,y,z) = (2xyz-2y,x^2+2x,x^2+2y)$, $C$ é a circunferência $y^2+z^2=1$, $x=2$.


$2\pi$.


3062   

Esboce o campo vetorial $\textbf{F}= \dfrac{y\textbf{i} + x\textbf{j}}{\sqrt{x^2+y^2}}$, desenhando um diagrama.


2945   

Calcule utilizando coordenadas esféricas. $\displaystyle\iiint\limits_{B}x\,dxdydz$, onde $B$ é o conjunto $x\geq 0$ e $x^{2}+y^{2}+z^{2}\leq 4.$


$4\pi$.


2818   

Determine os valores máximo e mínimo absolutos de $f$ no conjunto $D.$

$f(x,y)=3x-y$ no conjunto $D$ de todas $(x,y)$ tais que $x\geq 0$, $y\geq 0$, $y-x\leq 3$, $x+y\leq 4$ e $3x+y\leq 6.$


Valor máximo: $6;$ valor mínimo: $-3.$


2794   

Determine os valores máximos e mínimos locais e pontos de sela da função $f(x,y)=x^{3}-3x^{2}+27y$.


Pontos de sela: $\displaystyle \left(3,\frac{3}{2}\right)$ e $\displaystyle \left(-3,-\frac{3}{2}\right).$


2344   

  1. Determine, mas não calcule, a integral dupla da área da superfície com as equações paramétricas $x=au\cos v$, $y=bu\sin v$, $z=u^{2}$, $0\leq u\leq 2$, $0\leq v\leq 2\pi.$

  2. Elimine os parâmetros para mostrar que a superfície é um paraboloide elíptico e escreva outra integral dupla que forneça sua área.


  1. $\displaystyle \int^{2\pi}_{0}\int_{0}^{2} \sqrt{4b^2 u^4 \cos^{2}v + 4a^2 u^4 \sin^{2} v + a^2 b^2 u^2} dudv.$

  2. $\displaystyle \int_{-2a}^{2a} \int^{b \sqrt{4 - \frac{x^2}{a^2}}}_{-b \sqrt{4 - \frac{x^2}{a^2}}} \sqrt{1 + \left(2\frac{x}{a^2}\right)^{2} + \left(2\frac{y}{b^2} \right)^{2}} dydx.$


2888   

Encontre os pontos da curva $x^2 - 6xy - 7y^2 + 80 = 0$ mais próximos da origem. Desenhe a curva.


$(1,3)$ e $(-1,-3).$ Realizando a mudança de coordenadas $x = \frac{1}{\sqrt{10}} u - \frac{3}{\sqrt{10}} v$ e $y = \frac{3}{\sqrt{10}} u + \frac{1}{\sqrt{10}} v,$ a equação da curva inicial é transformada em $\frac{u^{2}}{10} - \frac{v^{2}}{40} = 1,$.


2786   

Suponha que $(0,2)$ seja um ponto crítico de uma função $g$ com derivadas de segunda ordem contínuas. Em cada caso, o que se pode dizer sobre $g$?

  1. $g_{xx}(0,2)=-1, \quad g_{xy}(0,2)=6, \quad g_{yy}(0,2)=1.$

  2. $g_{xx}(0,2)=-1, \quad g_{xy}(0,2)=2, \quad g_{yy}(0,2)=-8.$

  3. $g_{xx}(0,2)=4, \quad g_{xy}(0,2)=6, \quad g_{yy}(0,2)=9.$


  1. $g$ possui um ponto de sela em $(0,2).$

  2. $g$ possui um ponto de máximo local em $(0,2).$

  3. Não se pode afirmar algo sobre $g$ pelo Teste da Segunda Derivada.



Para fazer essa análise sobre $g$ iremos utilizar o Teste da Segunda Derivada.

Temos que
  1. \[ D=g_{xx}(0,2)g_{yy}(0,2)-g_{xy} ^2(0,2)=-1\cdot 1-6^2=-1-36=-37<0. \] Logo, pelo Teste da Segunda Derivada, segue que $(0,2)$ é um ponto de sela de $g$.
  2. Temos que \[ D=g_{xx}(0,2)g_{yy}(0,2)-g_{xy} ^2(0,2)=(-1)\cdot (-8)-2^2=8-4=4>0. \] Como $D>0$ e $g_{xx}(0,2)<0$, pelo Teste da Segunda Derivada, segue que $(0,2)$ é um ponto de máximo de $g$.
  3. Temos que \[ D=g_{xx}(0,2)g_{yy}(0,2)-g_{xy} ^2(0,2)=4\cdot 9-6^2=36-36=0. \] Como $D=0$ o Teste da Segunda Derivada não nos fornece nenhuma informação sobre $g$.

2671   

Seja $\phi:\mathbb{R}\rightarrow \mathbb{R}$ uma função diferenciável de uma variável real e seja $f(x,y)=(x^{2}+y^{2})\phi \bigg(\dfrac{x}{y}\bigg).$

Mostre que

$$x\;\frac{\partial f}{\partial x}+y\;\frac{\partial f}{\partial y}=2f.$$


$\displaystyle \frac{\partial f}{\partial x} = 2x \phi \left( \frac{x}{y} \right) + \frac{(x^{2} + y^{2})}{y} \phi'\left( \frac{x}{y} \right)\ \;\;\;\;\;\text{e}\;\;\;\;\; \frac{\partial f}{\partial y} = 2y \phi \left( \frac{x}{y} \right) - \frac{x(x^{2} + y^{2})}{y^{2}} \phi'\left( \frac{x}{y} \right).$



2545   

Seja $S$ a superfície $z=f(x,y)$, $(x,y)\in K$, de classe $C^{1}$ num aberto contendo $K$. (Observação: trata-se da superfície dada por $x=u$, $y=v$ e $z=f(u,v)$). Seja ${\bf n}$ a normal a $S$ com componente $z>0$ e seja ${\bf F}=P{\bf i}+Q{\bf j}+R{\bf k}$ um campo vetorial contínuo na imagem de $S$. Mostre que $\displaystyle\iint\limits_{S}{\bf F}\cdot {\bf n}dS=\displaystyle\iint\limits_{K}\left[ -P\dfrac{\partial f}{\partial x}(x,y)-Q\dfrac{\partial f}{\partial y}+R\right]dx dy,$ onde $P$, $Q$ e $R$ são calculadas em $(x,y,f(x,y)).$



Veja a subseção "Integrais de superfície de campos vetoriais"' da seção 16.7 do livro do Stewart.


2626   

Utilizando o Teorema de Stokes, transforme a integral $\displaystyle\iint_{ S}\mbox{rot}{\bf F}\cdot{\bf n}dS$ numa integral de linha e calcule.

  • ${\bf F}(x,y,z) = x{\bf j}$, $S$ a superfície $\{(x,y,z) \in \mathbb{R}^3; 0\leq z\leq 1, x^2+y^2=1,$$x\geq 0, y\geq 0\}$, sendo ${\bf n}$ a normal com componente $x$ positiva.


 $0$.


3096   

  1.  Faça um esboço do sólido no primeiro octante compreendido pelos planos \(x=0\), \(z=0\), \(x=5\), \(z-y=0\) e \(z=-2y+6\).

  2.  Calcule o volume do sólido dividindo-o em duas partes.


2307   

Calcule $D_{\bf{u}}f(x_0,y_0)$, sendo dados

$f(x,y) = x^2 - 3y^2$, $(x_0,y_0) = (1,2)$ e $\bf{u}$ o versor de $2\bf{i} + \bf{j}.$



$\displaystyle D_{\bf{(2,1)}}f(1,2) = -\frac{8}{5}.$ 


2166   

Use o Teorema de Green para calcular $\int_{C}\mathbf{F} \cdot d\mathbf{r}$, onde $\mathbf{F}(x,y) = (2x+y)\mathbf{i} + (3x-y)\mathbf{j}$, $C$ é uma curva fechada, simples, $C^1$ por partes, orientada no sentido positivo, cuja imagem é a fronteira de um compacto $B$ com área $\alpha$. (Verifique a orientação da curva antes de aplicar o Teorema.)


$2\times$(Área de $B$).


2523   

Esboce o gráfico da função $f(x,y)=\ln(\sqrt{x^{2}+y^{2}})$. Em geral, se $g$ é uma função de uma variável, como saber o gráfico de $f(x,y)=g(\sqrt{x^{2}+y^{2}})$ a partir do gráfico de $g$?


O gráfico de $f(x,y) = g(\sqrt{x^{2} + y^{2}})$ pode ser obtido rotacionando o gráfico de $g$ no plano $xz$ ao redor do eixo $z.$

ma211-list2-ex22_sol_c.png


2811   

Mostre que $f(x,y)=x^{2}+4y^{2}-4xy+2$ tem um número infinito de pontos críticos e que $f_{xx}f_{yy} - (f_{xy})^2 = 0$ em cada um. A seguir, mostre que $f$ tem um mínimo local (e absoluto) em cada ponto crítico.


Note que todos os pontos críticos são da forma $\displaystyle \left(x,\frac{1}{2}x \right)$ e que $f(x,y) = (x - 2y)^{2} + 2 \geq 2,$ com igualdade justamente se $\displaystyle y =  \frac{1}{2}x.$



Para encontrar os pontos críticos de $f$, devemos encontrar os pontos $(a,b)$ do domínio de $f$ tal que $f_x(a,b)=0$ e $f_y(a,b)=0$. Temos que $f_x(x,y)=2x-4y$ e $f_y(x,y)=8y-4x$ se anulam simultaneamente se $x=2y$. Logo, todos os pontos do conjunto $\{(x,y)\in \mathbb{R}^{2} | x=2y\}$ são pontos críticos de $f$, provando que $f$ tem infinitos pontos críticos.
Agora, queremos ver que $f_{xx}f_{yy}-f_{xy} ^{2}=0$ em todos os pontos críticos. Para isso, calculemos as segundas derivadas de $f$ \[ f_{xx}(x,y)=2, f_{xy}(x,y)=-4 \text{ e } f_{yy}(x,y)=8. \] Daí temos que $f_{xx}f_{yy}-f_{xy} ^{2}=2\cdot 8-(-4)^{2}=16-16=0$, como queríamos.
Por fim, queremos ver que esses pontos críticos são pontos de mínimo de $f$, mas como $f_{xx}f_{yy}-f_{xy} ^{2}=0$ o Teste de Derivada Segunda é inconclusivo. Mas, note que $f$ pode ser reescrita como \[ f(x,y)=x^2+4y^2-4xy+2=(x-2y)^2+2. \]

Como $(x-2y)^2\geq 0$, segue que $f(x,y)\geq 2$ para todo $(x,y)\in \mathbb{R}^2$. Em particular, se tomamos um ponto crítico de $f$, isto é, um ponto da forma $(2y,y)$ então $f(2y,y)=2$. Provando que os pontos críticos são pontos de mínimo de $f$.


2154   

Use o Teorema do Divergente para calcular o fluxo de ${\bf F}$ através de $S,$ onde ${\bf F}(x,y,z)=(x^{2}+z^{2})\,{\bf i}+(y^{2}-2xy)\,{\bf j}+(4z-2yz)\,{\bf k}$ e $S$ é a superfície da região delimitada pelo cone $x=\sqrt{y^{2}+z^{2}}$ e pelo plano $x=9.$


2492   

Dada a função $f(x,y)=\sqrt{y-x}$.

  1. Encontre o domínio da função.

  2. Encontre a imagem da função.

  3. Descreva as curvas de nível da função.


  1. $D_{f} = \left\lbrace (x,y);\; x \leq y \right\rbrace$.

  2. $Im(f) = \left\lbrace z \in \mathbb{R};\; z \geq 0 \right\rbrace.$

  3. As curvas de nível são as retas $y - x = C,$ com $C \geq 0.$


1960   

Calcule a integral de linha $\displaystyle\int_{C}{\bf F}\cdot d{\bf r}$, onde ${\bf F}(x,y,z)=(yz,2xz,xy+2z)$ e $C$ é o segmento de reta que liga o ponto $(1,0,1)$ ao ponto $(-2,2,2).$


$-7.$


2444   

Calcule a integral de superfície $\displaystyle\iint\limits_{S}x dS$, onde $S$ é a superfície com equações paramétricas $x=u$, $y=v$, $z=u^{2}+v$, $0 \leq u \leq 1$, $u^{2} \leq v \leq 1.$


$\dfrac{\sqrt{2}}{10}(3\sqrt{3} - 2).$


3143   

Considere o campo vetorial \(\mathbf{F}(x,y,z)=x^2\mathbf{i} + y^2\mathbf{j}+z^2\mathbf{k}\) e a superfície \(\sigma\) descrita como sendo a porção do cone \(z=\sqrt{x^2+y^2}\) abaixo do plano \(z=1\) e tendo orientação para cima. Verifique o Teorema de Stokes calculando, separadamente, a integral de linha e a integral dupla e, em seguida, comparando os valores.


2628   

Utilizando o Teorema de Stokes, transforme a integral $\displaystyle\iint_{ S}\mbox{rot}{\bf F}\cdot{\bf n}dS$ numa integral de linha e calcule.

  • ${\bf F}(x,y,z) = y{\bf i}$, $S$ a superfície $x^2+y^2+z^2 = 2$, $x^2+y^2\leq 1$ e $z \geq 0$, sendo ${\bf n}$ a normal apontando para cima.


$-\pi$.


2233   

Seja $\mathbf{r} = x\mathbf{i} + y\mathbf{j} + z\mathbf{k}$ e $r=|\mathbf{r}|$. Verifique a identidade $\nabla^2r^3=12r$.


$\nabla^2r^3= \dfrac{\partial}{\partial x} \left[ \dfrac{3}{2} \sqrt{x^{2} + y^{2} + z^{2}} (2x) \right] + \dfrac{\partial}{\partial y} \left[ \dfrac{3}{2} \sqrt{x^{2} + y^{2} + z^{2}} (2y) \right]\\ + \dfrac{\partial}{\partial z} \left[ \dfrac{3}{2} \sqrt{x^{2} + y^{2} + z^{2}} (2z) \right].$ (Note que: $r = \sqrt{x^{2} + y^{2} + z^{2}}.$)


2953   

Usando coordenadas esféricas, determine o volume da parte da bola $\rho\leq a$ que está entre os cones $\phi=\pi/6$ e $\phi=\pi/3.$


$\displaystyle \left( \sqrt{3} - 1 \right) \dfrac{\pi a^3}{3}.$


2479   

Esboce o gráfico da função $f(x,y)=3$.


$z = 3.$

ma211-list2-ex11_sol_a.png


3132   

Encontre a massa da lâmina descrita como sendo a porção do cilindro circular \(x^2+z^2=4\) que fica diretamente acima do retângulo \(\displaystyle R=\{(x,y)\in\mathbb{R}^2;\ 0\leq x\leq 1,\ 0\leq y\leq 4\}\) e tem densidade \(\delta_0\) constante.


\(\dfrac{4}{3}\pi\delta_0\)


3145   

A Lei de Coulomb afirma que a força eletrostática \(\mathbf{F}(\mathbf{r})\) que uma partícula com carga \(Q\) exerce sobre outra partícula com carga \(q\) é dada pela fórmula \[ \mathbf{F}(\mathbf{r}) = \dfrac{q\,Q}{4\pi\epsilon_0\|\mathbf{r}\|^3}\mathbf{r}, \] onde \(\mathbf{r}\) é o vetor posição da carga \(q\) em relação a \(Q\) e \(\epsilon_0\) é uma constante positiva (chamada permissividade do meio).

  1.  Expresse o campo vetorial \(\mathbf{F}(\mathbf{r})\) em forma de coordenadas \(\mathbf{F}(x,y,z)\) com \(Q\) na origem.

  2.  Calcule o trabalho realizado pelo campo vetorial \(\mathbf{F}\) sobre uma carga \(q\) que se move ao longo de um segmento de reta de \((3,0,0)\) para \((3,1,5)\).


2584   

Considere a função

$$f(x,y) = \begin{cases}x + y, & \quad \text{se } xy = 0, \\k, & \quad \text{caso contrário},\end{cases}$$

em que $k$ é um número real. É possível escolher $k$ de modo que $f$ seja contínua em $(0,0)$? Em caso afirmativo, qual deve ser o valor de $k$?


$k = 0.$


3127   

Verifique que para o vetor posição \(\mathbf{r}=x\mathbf{i}+y\mathbf{j}+z\mathbf{k}\) valem as seguintes propriedades

  1.  \(\displaystyle \mathrm{div\,}\mathbf{r} = 3\)

  2.  \(\displaystyle \nabla\dfrac{1}{\|\mathbf{r}\|} = -\dfrac{\mathbf{r}}{\|\mathbf{r}\|^3} \)


2909   

Utilize coordenadas polares para determinar o volume do sólido dado: delimitado pelo paraboloide $z=9-x^2-y^2$ e pelo plano $z=5.$


$8\pi.$


2613   

Use o Teorema de Stokes para calcular $\displaystyle\int_C {\bf F}\cdot  d{\bf R}$. $C$ é orientada no sentido anti-horário quando vista de cima.

  • ${\bf F}(x,y,z) = (y+z,-z,y)$, $C$ é a curva obtida como interseção do cilindro $x^2+y^2=2y$ com o plano $y = z$.


$\dfrac{4\pi}{3}$.


2216   

Determine o rotacional e o divergente do campo vetorial $\mathbf{F}(x,y,z) = \dfrac{1}{\sqrt{x^2+y^2+z^2}}(x\mathbf{i} + y\mathbf{j} + z\mathbf{k})$.


$\text{rot } \mathbf{F} = \bf{0}.$ $\text{div } \mathbf{F} = \dfrac{2}{\sqrt{x^{2} + y^{2} + z^{2}}}.$


1957   

Calcule a integral de linha $\int_{C}{\bf F}\cdot d{\bf r}$, onde $C$ é dada pela função vetorial ${\bf r}(t).$

${\bf F}(x,y)=(e^{-y}-2x,-xe^{-y}-\sin y)$, ${\bf r}(t)=(t,\tan t)$, $0\leq t\leq \pi/4.$


$\displaystyle \cos(1) - \frac{\pi}{4}e^{-1} - \frac{\pi^{2}}{16} - 1.$


2030   

Use a Regra da Cadeia para determinar $\mathrm{d}z/\mathrm{d} t$ ou $\mathrm{d}w/ \mathrm{d}t.$

$w=xe^{y/z}$, $x=t^{2}$, $y=1-t$, $z=1+2t$.



$\displaystyle \frac{dw}{dt} = e^{\frac{y}{z}} \left(2t - \frac{x}{z} - \frac{2xy}{z^{2}} \right).$


2475   

Determine e faça o esboço do domínio da função $f(x,y)=\sqrt{1-x^{2}}-\sqrt{1-y^{2}}$.



$\left\lbrace (x,y);\; -1 \leq x \leq 1,\;-1\leq y \leq 1 \right\rbrace.$

ma211-list2-ex10_sol_c.png


2893   

Passe para coordenadas polares e calcule: $\displaystyle\int_{-1}^{0} \int_{-\sqrt{1-x^{2}}}^{0}\frac{2}{1+\sqrt{x^{2}+y^{2}}}\,dy dx$


$(1 - \ln(2))\pi.$


3097   

Suponha que a temperatura, em graus Celsius, num ponto \((x,y)\)  de uma chapa metálica plana seja \( T(x,y)=10-8x^2-2y^2 \), onde \(x\) e \(y\) são medidos em metros. Calcule a temperatura média da porção retangular da chapa dada por \(0\leq x\leq 1\) e \(0\leq y\leq 2\).



\(\dfrac{14}{3}\) \({}^\circ\)C


2222   

Seja $f$ um campo escalar e $\mathbf{F}$ um campo vetorial. Diga se cada expressão tem significado. Em caso negativo, explique por quê. Em caso afirmativo, diga se é um campo vetorial ou escalar. 

  1. $\text{grad }{(\text{div }{f})}$;

    $\text{div }{(\text{div }{\mathbf{F}})}$;

    $\text{div }{(\text{rot }{(\text{grad }{f})})}$.


  1. $\text{grad }{(\text{div }{f})}$ não tem significado, pois $f$ é um campo escalar.

  2. $\text{div }{(\text{div }{\mathbf{F}})}$ não tem significado pois $\text{div } \bf{F}$ é um campo escalar.

  3. $\text{div }{(\text{rot }{(\text{grad }{f})})}$ é um campo escalar.


2539   

Encontre o centróide e os momentos de inércia $I_{x}$, $I_{y}$ e $I_{z}$ do tetraedro cujos vértices são os pontos $(0,0,0)$, $(1,0,0)$, $(0,1,0)$ e $(0,0,1).$


Centróide: $\displaystyle \left(\frac{1}{4},\frac{1}{4},\frac{1}{4} \right),$ $I_{x} = I_{y} = I_{z} = \dfrac{1}{30}.$


2315   

Determine a derivada direcional de $f(x,y,z) = xy + yz + zx$ em $P = (1,-1,3)$ na direção de $Q = (2,4,5)$.


$\displaystyle \frac{22}{\sqrt{30}}.$


2446   

Calcule a integral de superfície $\displaystyle\iint\limits_{S}y dS$, onde $S$ é a superfície com equações paramétricas $x=u$, $y=v$, $z=1-u^{2}$, $0\leq u\leq 1$, $0\leq v\leq \sqrt{u}.$




2901   

Determine os valores de máximo e mínimo de $f(x,y,z) = x^2 - yz$ em pontos da esfera $x^2 + y^2 + z^2 = 1$.


Valor máximo: $1;$ valor mínimo: $\displaystyle -\frac{1}{2}.$


2985   

  1. Calcule $\iiint\limits_{E} dV$, em que $E$ é o sólido delimitado pelo elipsoide  $x^2/a^2 + y^2/b^2 + z^2/c^2 = 1$. Utilize a transformação $x = au$, $y = bv$ e $z = cw$.

  2. A Terra não é perfeitamente esférica; como resultado da rotação, os polos foram achatados. Assim, seu formato pode ser aproximado por um elipsoide com $a = b = 6.378$ km e $c = 6.356$ km. Use o item anterior para estimar o volume da Terra.


  1. $\dfrac{4\pi a b c}{3}.$

  2. $\dfrac{4\pi (6378) (6378) (6356)}{3} \approx 1.083 \times 10^{12}$ km$^{3}.$


2363   

Seja $f: \mathbb{R} \to \mathbb{R}$ uma função diferenciável de uma variável. Defina

$$g(x,y) = f(r),  r = \sqrt{x^2 + y^2}.$$

Calcule a derivada direcional da função $g$ no ponto $(x,y) \neq (0,0)$ e na direção do vetor $(x,y)$.


$(f'(r))^{2}.$


3007   

Calcule o centro de massa da região: $D = \{(x,y) \in \mathbb{R}^2: x^2 + 4y^2 \leq 1, \ y \geq 0\}$ e a densidade é proporcional à distância do ponto ao eixo $x$.


$\displaystyle \left(0, \frac{3\pi}{32} \right).$


2715   

Determine uma equação do plano tangente à superfície no ponto especificado.

$z = y \ \mbox{cos}(x-y), \quad (2,2,2)$.


$z = y$.


2211   

Ao calcular, por integração dupla, o volume $V$ do sólido situado abaixo do parabolóide $z=x^{2}+y^{2}$ e limitado inferiormente por uma certa região $D$ no plano $xy$, chegou-se à seguinte expressão: $$V=\int_{0}^{1}\!\!\int_{0}^{y}(x^{2}+y^{2})\,dx dy+\int_{1}^{2}\int_{0}^{2-y}(x^{2}+y^{2})\,dx dy.$$

  1.  Esboce a região $D.$

  2.  Expresse $V$ numa única integral dupla iterada.

  3.  Efetue a integração para calcular $V.$



  1. ...

  2.  $\displaystyle \int_{0}^{1} \int_{x}^{2 - x} x^{2} + y^{2}\;dy\; dx$

  3.  $\dfrac{4}{3}.$


3006   

Calcule o centro de massa do quadrado $D$ dado por $0 \leq x \leq 1, \ 0 \leq y \leq 1$  e com densidade $\quad \rho(x,y) = y$.


$\displaystyle \left(\frac{1}{2}, \frac{2}{3} \right).$


3149   

Seja \(\displaystyle \mathbf{F}(x,y,z)=f(x,y,z)\mathbf{i}+ g(x,y,z)\mathbf{j} + h(x,y,z)\mathbf{k}\) e suponha que \(f\), \(g\) e \(h\) sejam contínuas e tenham derivadas parciais de primeira ordem contínuas numa região. Mostre que se \(\mathbf{F}\) é conservativo numa região esférica aberta então \(\mathrm{rot\,}\mathbf{F} = \mathbf{0}\) nessa região. [Sugestão: use que se \(\mathbf{F}\) for conservativo numa região, então \[ \dfrac{\partial f}{\partial y}=\dfrac{\partial g}{\partial x},\quad \dfrac{\partial f}{\partial z}=\dfrac{\partial h}{\partial x},\quad \dfrac{\partial g}{\partial z}=\dfrac{\partial h}{\partial y} \]  nessa mesma região.]


2996   

Calcule a integral, efetuando uma mudança de variáveis apropriada. $\displaystyle\iint\limits_{R} \dfrac{y - 2x}{3y + 2x} \, dA$, em que $R$ é a paralelogramo de vértices $(1,2)$, $(2,4)$, $(5,2)$ e $(4,0)$.


$-4\ln(2).$


2155   

Use o Teorema de Green para calcular a integral de linha ao longo da curva dada com orientação positiva.

$\displaystyle\int_{C}y^3 \, dx - x^3 \, dy$, $C$ é o círculo $x^2 + y^2 = 4$.



Observe que a curva $C$ com orientação positiva está nas hipóteses do Teorema de Green, assim como o campo $\mathbf{F}(x,y) = (y^3, -x^3)$. Logo,

$$\displaystyle\int_{C}y^3 \, dx - x^3 \, dy  =  \iint\limits_{D} \left(\frac{\partial}{\partial x}(-x^3) - \frac{\partial}{\partial y}(y^3)\right) \, dA = -3 \iint\limits_{D} (x^2 + y^2) \, dA,$$

em que $D = \{(x,y) \in \mathbb{R}^2: x^2 + y^2 \leq 4\}$. Usando coordenadas polares

$$\begin{cases}x = r \cos{\theta} \\y = r \sin{\theta}, \\\end{cases}$$

temos que a região de integração $D$ pode ser escrita como $$\{(r,\theta) \in \mathbb{R}^2: 0 \leq r \leq 2, 0 \leq \theta \leq 2 \pi\}$$ e o jacobiano dessa mudança de coordenadas é igual a $r$. Logo,

$$\iint\limits_{       D} (x^2 + y^2) dA = \displaystyle\int_{0}^{2\pi} \int_{0}^{2}r^2 \cdot r\,dr d\theta = 8\pi.$$


Portanto, $\displaystyle\int_{C}y^3 \, dx - x^3 \,dy = -24\pi$.


2906   

Utilize coordenadas polares para determinar o volume do sólido dado: acima do cone $z=\sqrt{x^2+y^2}$ e abaixo da esfera $x^2+y^2+z^2=1.$


$\displaystyle \frac{\pi}{3}(2 - \sqrt{2}).$


2802   

Determine os valores máximos e mínimos locais e pontos de sela da função $f(x,y)=x^{2}+y^{3}+xy-3x-4y+5$.


Ponto de mínimo : $\displaystyle \left( 1,1\right);$ ponto de sela: $\displaystyle \left(\frac{23}{12},-\frac{5}{6}\right).$


3152   

Mostre que o determinante Jacobiano da mudança de coordenadas cartesianas para esféricas é $-\rho^2 \sin \varphi$.


2423   

A temperatura em um ponto $(x,y,z)$ em uma substância com condutividade $K=6,5$ é $u(x,y,z)=2y^{2}+2z^{2}.$ Determine a taxa de transmissão de calor nessa substância para dentro da superfície cilíndrica $y^{2}+z^{2}=6$, $0\leq x\leq 4.$



O fluxo de calor, com $u(x,y,z)=2y^{2}+2z^{2}$, é dado por

$${\bf F}(x,y,z)=-K \nabla u=-6,5(0{\bf i}+4y{\bf j}+4z{\bf k})=0{\bf i}-26y{\bf j}-26z{\bf k}.$$

Temos que $S$ é a superfície cilíndrica $y^{2}+z^{2}=6$ e $0\leq x \leq 4.$ As equações paramétricas de $S$ são:

$$x=x, y=\sqrt{6}\cos \theta \mbox{e} z=\sqrt{6}\sin \theta$$

onde $0\leq x \leq 4$ e $0\leq \theta \leq 2\pi.$

Então,

$${\bf r}(x,\theta)=x{\bf i}+\sqrt{6}\cos \theta{\bf j}+\sqrt{6}\sin \theta{\bf k}.$$

Como queremos o fluxo de calor para dentro de $S$ devemos calcular

$$\int \int\limits_{S}{\bf F}\cdot dS=\int \int\limits_{ D}{\bf F}({\bf r}(x,\theta))\cdot ({\bf r}_{x}\times {\bf r}_{\theta})dA.$$

Então,

$${\bf r}_{x}(x,\theta)={\bf i}+0{\bf j}+0{\bf k}$$

e

$${\bf r}_{\theta}(x,\theta)=0{\bf i}-\sqrt{6}\sin \theta{\bf j}-\sqrt{6}\cos \theta{\bf k}.$$

Logo,

$\begin{array}{rcl} {\bf r}_{x} \times {\bf r}_{\theta} &=& \left| \begin{array}{ccc}{\bf i} & {\bf j} & {\bf k}\\1 & 0 & 0\\0 & -\sqrt{6}\sin \theta & -\sqrt{6}\cos \theta \\ \end{array} \right| \\ &=& 0{\bf i}-\sqrt{6}\cos \theta{\bf j}-\sqrt{6}\sin \theta{\bf k}, \end{array}$

$${\bf F}({\bf r}(x,\theta))=(0{\bf i}-26\sqrt{6}\cos\theta{\bf j}-26\sqrt{6}\sin \theta{\bf k})$$

e

$${\bf F}({\bf r}(x,\theta))\cdot ({\bf r}_{x}\times {\bf r}_{\theta})=(0{\bf i}-26\sqrt{6}\cos\theta{\bf j}-26\sqrt{6}\sin \theta{\bf k}) \cdot (0{\bf i}-\sqrt{6}\cos \theta{\bf j}-\sqrt{6}\sin \theta{\bf k})=156$$

Assim, a taxa de fluxo de calor para dentro de $S$ é:

$$\int \int\limits_{S}{\bf F}\cdot dS=\int \int\limits_{ D}{\bf F}({\bf r}(x,\theta))\cdot ({\bf r}_{x}\times {\bf r}_{\theta})dA=\int \int\limits_{ D}156 dA=156\int \int\limits_{ D} 1 dA$$

$$=156\int_{0}^{2\pi}\int_{0}^{4}1dxd\theta=156\int_{0}^{2\pi}d\theta\cdot \int_{0}^{4}dx=156\cdot (\theta)\bigg|_{0}^{2\pi}\cdot (x)\bigg|_{0}^{4}=156\cdot 2\pi \cdot 4=1248 \pi.$$


2167   

Use o Teorema de Green para calcular $\int_{C}\mathbf{F} \cdot d\mathbf{r}$, onde $\mathbf{F}(x,y) = 4x^3y^3\mathbf{i} + (3x^4y^2+5x)\mathbf{j}$, $C$ é a fronteira do quadrado de vértices $(-1,0)$, $(0,-1)$, $(1,0)$ e $(0,1)$. (Verifique a orientação da curva antes de aplicar o Teorema.)


$10.$


2667   

Seja $\phi:\mathbb{R}\rightarrow \mathbb{R}$ uma função de uma variável real, diferenciável e tal que $\phi '(1)=4.$ Seja $g(x,y)=\phi\bigg(\dfrac{x}{y}\bigg).$ Calcule

  1. $\dfrac{\partial g}{\partial x}(1,1)$.

  2. $\dfrac{\partial g}{\partial y}(1,1)$.


  1. $\displaystyle \frac{\partial g}{\partial x} = \frac{1}{y} \phi'\left( \frac{x}{y} \right)$

  2. $\displaystyle \frac{\partial g}{\partial y} = -\frac{x}{y^{2}} \phi' \left( \frac{x}{y} \right).$


2115   

Mostre que qualquer função da forma 
$$z=f(x+at)+g(x-at)$$
é uma solução da equação de onda
$$\frac{\partial^{2} z}{\partial t^{2}}=a^{2}\frac{\partial^{2}z}{\partial x^{2}}.$$
(Sugestão: Tome $u=x+at$, $v=x-at$.)



Note que se $u = x + at$ e $v = x - at,$ então $\displaystyle \frac{\partial^{2} z}{\partial t^{2}} = a^{2}f''(u) + a^{2} g''(v)$e\\$\displaystyle \frac{\partial^{2} z}{\partial x^{2}} = f''(u) + g''(v).$


2833   

Uma caixa de papelão sem tampa deve ter um volume de $32000\;cm^{3}$. Determine as dimensões que minimizem a quantidade de papelão utilizado. 


$40$cm $\times$ $40$cm $\times$ $20$cm.


2768   

De acordo com o triângulo abaixo:

ma211-list3-ex36.png

  1. Expresse $A$ implicitamente como uma função de $a$, $b$ e $c$ e calcule $\partial A/\partial a$ e $\partial A/ \partial b.$

  2. Expresse $a$ implicitamente como uma função de $A$, $b$ e $B$ e calcule $\partial a/ \partial A$ e $\partial a/ \partial B.$


  1. $\displaystyle a^{2} = b^{2} + c^{2} -2bc\cos(A),\;\;\;\;\frac{\partial A}{\partial a} = \frac{a}{bc \sin (A)}\;\;\;\text{e}\;\;\;\frac{\partial A}{\partial b} = \frac{c \cos(A) - b}{bc \sin(A)}.$

  2. $\displaystyle \frac{a}{\sin(A)} = \frac{b}{\sin(B)},\;\;\;\;\frac{\partial a}{\partial A} = \frac{a\cos(A)}{\sin(A)}\;\;\;\text{e}\;\;\;\frac{\partial a}{\partial B} = - b\csc(B) \cot(B)\sin(A).$


2055   

Determine se ${\bf F}(x,y)=(e^{x}\,\sin y)\,{\bf i}+(e^{x}\,\cos y)\,{\bf j}.$ é ou não um campo vetorial conservativo. Se for, determine uma função $f$ tal que ${\bf F}=\nabla f.$


Sim. $f(x,y) = e^{x}\sin(y) + K.$


2793   

Determine os valores máximos e mínimos locais e pontos de sela da função $f(x,y)=x^{2}+y^{2}+x^{2}y+4$.


Pontos de mínimo: $(1,1)$ e $(-1,-1);$ ponto de sela: $(0,0).$


2921   

Utilize a integral dupla para determinar a área da região: dentro da cardióide $r=1+\cos{\theta}$ e fora do círculo $r=3\cos{\theta}.$


$\displaystyle \frac{\pi}{4}.$


2757   

Verifique que a função $f(x,y) = x^2y$ é diferenciável.


As derivadas parciais $\frac{\partial f}{\partial x}$ e $\frac{\partial f}{\partial y}$ de cada função $f$ existem e são contínuas em todos os pontos do domínio.


2693   

Encontre $f_{x}$, $f_{y}$ e $f_{z}$ para $f(x,y,z)=e^{-xyz}$.


$\displaystyle f_{x} = -yz e^{-xyz},\;\;\;\; f_{y} = -xz e^{-xyz}\;\;\;\;\text{e}\;\;\;\; f_{z} = -xy e^{-xyz}$.


2370   

Seja $f(x,y) = x \arctan{\dfrac{x}{y}}$. Calcule $D_{\bf{u}}f(1,1)$, em que $\bf{u}$ aponta na direção e sentido de máximo crescimento de $f$, no ponto $(1,1)$.


$\displaystyle D_{\bf{u}}f(1,1) = \sqrt{\left( \frac{\pi}{4} + \frac{1}{2}\right)^{2} + \frac{1}{4}}.$


1953   

Calcule a integral de linha $\int_{C}{\bf F}\cdot d{\bf r}$, onde $C$ é dada pela função vetorial ${\bf r}(t).$

${\bf F}(x,y,z)=(x+y+z)\,{\bf k}$, ${\bf r}(t)=(t,t,-t^{2})$, $0\leq t\leq 1.$


$-\dfrac{11}{6}.$


2593   

Marque o ponto cujas coordenadas cilíndricas são $(2, \pi/4,1)$ e $(4, -\pi/3,5)$. Em seguida, encontre as coordenadas retangulares do ponto.


Para $(2, \pi/4,1):$ $(\sqrt{2},\sqrt{2},1)$ e para $(4, -\pi/3,5):$ $(2, -2\sqrt{3},5)$.


3061   

Esboce o campo vetorial $\textbf{F}= y\textbf{i} + \dfrac{1}{2}\textbf{j}$, desenhando um diagrama.


2280   

Defina gradiente de uma função de três variáveis. Calcule $\nabla f(x,y,z)$.

$f(x,y,z) = x^2 + y^2 + z^2$


 $\displaystyle \nabla f(x,y,z) = (2x,2y,2z).$


2172   

Calcule $\int_{C}\mathbf{F}\cdot\, d\mathbf{r}$, em que

$$\mathbf{F}(x,y) = (x^2+y)\mathbf{i} + (3x-y^2)\mathbf{j}$$

e $C$ é a fronteira orientada positivamente de uma região $D$ que tem área 6.


$12.$


2612   

Use o Teorema de Stokes para calcular $\displaystyle\int_C {\bf F}\cdot d{\bf R}$. Em cada caso, $C$ é orientada no sentido anti-horário quando vista de cima.

  • ${\bf F}(x,y,z) = x^2z{\bf i} + xy^2{\bf j} + z^2{\bf k}$, $C$ é a curva de interseção do plano $x+y+z=1$ com o cilindro $x^2+y^2 = 9$.


$\dfrac{81\pi}{2}.$


3136   

Em 1831, o físico Michael Faraday descobriu que uma corrente elétrica pode ser produzida variando-se o fluxo magnético através de um arco condutor. Suas experiências mostraram que a força eletromotriz \(\mathbf{E}\) está relacionada com a indução magnética \(\mathbf{B}\) pela equação \[ \oint_C\mathbf{E}\cdot\,d\mathbf{r} = - \iint\limits_\sigma\dfrac{\partial\mathbf{B}}{\partial t}\cdot\mathbf{n}\,dS.\] Use este resultado para fazer uma conjectura acerca da relação entre \(\mathrm{rot\,}\mathbf{E}\) e \(\mathbf{B}\). Explique seu raciocínio.


2075   

Seja $R$ o retângulo $1\leq x\leq 2$, $0\leq y\leq 1$. Calcule $\iint\limits_{R} f(x,y)\,dxdy$, sendo $f(x,y)$ igual a

  1.  $ye^{xy}$

  2.  $xy^{2}$


  1.  $\dfrac{(e - 1)^{2}}{2}.$

  2.  $\dfrac{1}{2}.$


2586   

Determine o volume do sólido que está acima do plano $xy$, abaixo do paraboloide $z = x^2 + y^2$ e que se encontra dentro do cilindro $x^2 + y^2 = 2x$ e fora do cilindro $x^2 + y^2 = 1.$



Temos que $0\leq z\leq x^{2}+y^{2}$. Como o sólido se encontra dentro do cilindro $x^{2}+y^{2}=2x$ e fora do cilindro $x^{2}+y^{2}=1$, devemos fazer a interseção desses dois cilindros, isto é, $$\left\{\begin{array}{cc} x^{2}+y^{2}=2x\\ x^{2}+y^{2}=1\\ \end{array} \right.\Rightarrow 2x=1\Leftrightarrow x=\frac{1}{2}$$ Em coordenadas cilíndricas temos que \begin{eqnarray*} x&=&r\cos \theta\\ y&=&r\sin \theta\\ z&=&z\\ dz\,dy\,dx&=&r\,dz\,dr\,d\theta \end{eqnarray*} Da equação $x^{2}+y^{2}=1$ temos que $$r^{2}=1\Longrightarrow r=\pm 1,$$ como devemos ter $r\geq 0$, então nesse caso $r=1.$ Da equação $x^{2}+y^{2}=2x$ temos que $$r^{2}=2r\,\cos \theta \Rightarrow r=2\cos \theta.$$  Agora, sendo $x=\frac{1}{2}$ e $r=1$ temos que $$\cos \theta=\frac{1}{2}\Rightarrow \theta=\pm \frac{\pi}{3}.$$ Assim, em coordenadas cilíndricas temos que o sólido $E$ é dado por $$E=\{(\theta,\,r,\,z)|\, -\frac{\pi}{3}\leq \theta \leq \frac{\pi}{3},\, 1\leq r\leq 2 \cos \theta,\,0\leq z\leq r^{2}\}.$$ Então, $$V=\iiint\limits_{  E}1\,dV= \int_{-\frac{\pi}{3}}^{\frac{\pi}{3}}\int_{1}^{2\cos \theta}\int_{0}^{r^{2}}1\,r\,dz\,dr\,d\theta= \int_{-\frac{\pi}{3}}^{\frac{\pi}{3}}\int_{1}^{2\cos \theta}zr\bigg|_{0}^{r}\,dr\,d\theta$$ $$=\int_{-\frac{\pi}{3}}^{\frac{\pi}{3}}\int_{1}^{2\cos \theta}r^{3}\,dr\,d\theta= \int_{-\frac{\pi}{3}}^{\frac{\pi}{3}}\frac{r^{4}}{4}\bigg|_{1}^{2\cos \theta}\,d\theta =\int_{-\frac{\pi}{3}}^{\frac{\pi}{3}}\bigg(4\cos^{4}\theta-\frac{1}{4}\bigg)\,d\theta$$ $$=4\int_{-\frac{\pi}{3}}^{\frac{\pi}{3}}\underbrace{\cos^{4}\theta}_{\mbox{função   par}}\,d\theta-\int_{-\frac{\pi}{3}}^{\frac{\pi}{3}}\underbrace{\frac{1}{4}}_{\mbox{função    par}}\,d\theta =8\int_{0}^{\frac{\pi}{3}}\cos^{4}\theta\,d\theta-2\int_{0}^{\frac{\pi}{3}}\frac{1}{4}\,d\theta$$ $$=8\bigg[\frac{3}{8}\theta+\frac{1}{4}\sin(2\theta)+\frac{1}{32}\sin(4\theta)\bigg]\bigg|_{0}^{\frac{\pi}{3}} -\bigg(\frac{1}{2}\theta\bigg)\bigg|_{0}^{\frac{\pi}{3}}$$ $$=8\bigg[\frac{3}{8}\cdot \frac{\pi}{3}+\frac{1}{4}\sin\bigg(\frac{2\pi}{3}\bigg)+\frac{1}{32}\sin\bigg(\frac{4\pi}{3}\bigg)\bigg]-\frac{1}{2}\cdot \frac{\pi}{3}$$ $$=\pi+\sqrt{3}-\frac{\sqrt{3}}{8}-\frac{\pi}{6}=\frac{5\pi}{6}+\frac{7\sqrt{3}}{8}.$$


2361   

Considere a superfície parametrizada por

$${\bf r}(u,v)=(uv,u+v,u-v).$$

  1. Determine o valor de $c$ de forma que o ponto $(c,1,0)$ pertença à superfície.

  2. Calcule a área da parte da superfície correspondente à variação $u^{2}+v^{2}\leq 1.$


  1. $\dfrac{1}{4}.$

  2. $\left(\sqrt{6} - \dfrac{4}{3} \right)2\pi.$


2662   

Determine as derivadas parciais de $g(x,y)=x^{y}$.


$\displaystyle \frac{\partial g}{\partial x} = yx^{y - 1}\;\;\;\;\;\;\text{e}\;\;\;\;\; \frac{\partial g}{\partial y} = x^{y} \ln x.$


3042   

  1. Determine a representação paramétrica do toro obtido girando em torno do eixo $z$ o círculo do plano $xz$ com centro em $(b,0,0)$ e raio $a < b.$ [Sugestão: tome como parâmetros os ângulos $\theta$ e $\alpha$ mostrados na figura.]

  2. Use a representação paramétrica do item anterior para achar a área do toro.

ma211-list13-ex38.png


  1. $x = b\cos(\theta) + a\cos(\alpha)\cos(\theta),$ $y = b\sin(\theta) + a\cos(\alpha)\sin(\theta),$ $z = a\sin(\alpha),$ onde $0 \leq \alpha \leq 2\pi,$ $0 \leq \theta \leq 2\pi.$

  2. $4\pi^2 ab.$


2304   

Identifique e faça um esboço da imagem da superfície parametrizada dada por ${\bf r}(u,v)=\bigg(v\cos u,v\sin u,\dfrac{1}{v^{2}}\bigg)$, $0\leq u\leq 2\pi$, $v>0.$


Gráfico de $f(x,y) = \dfrac{1}{x^2 + y^2}.$


2426   

Calcule a integral tripla.

  1.  $\displaystyle\iiint\limits_{  E}2x\,dV$, onde $E=\{(x,y,z)|\;0\leq y\leq 2,\,0\leq x\leq \sqrt{4-y^{2}},\;\\ 0\leq z\leq y\}.$

  2.  $\displaystyle\iiint\limits_{  E}6xy\,dV$, onde $E$ está abaixo do plano $z=1+x+y$ e acima da região do plano $xy$ limitada pelas curvas $y=\sqrt{x}$, $y=0$ e $x=1.$


  1.  $4.$

  2.  $\dfrac{65}{28}.$


2053   

Determine se ${\bf F}(x,y)=(2x-3y)\,{\bf i}+(-3x+4y-8)\,{\bf j}.$ é ou não um campo vetorial conservativo. Se for, determine uma função $f$ tal que ${\bf F}=\nabla f.$


Sim. $f(x,y) = x^2 - 3xy + 2y^2 -8y + K.$


1939   

Calcule a integral de linha, onde $C$ é a curva dada.

$\displaystyle\int_{C}x\,dx+y\,dy+z\,dz$, $C$ é o segmento de extremidades $(0,0,0)$ e $(1,2,1)$, percorrido no sentido de $(1,2,1)$ para $(0,0,0).$


$-3.$


2871   

Utilize os multiplicadores de Lagrange para determinar os valores máximo e mínimo da função sujeita à(s) restrição(ões) dada(s).

$f(x,y) = 4x + 6y; \quad x^2 + y^2 = 13.$


Valor máximo: $26;$ valor mínimo: $-26.$


2588   

Seja $C$ o cilindro de base circular e eixo $(Oz)$, com raio $2$ e altura $3$, com base na origem e densidade inversamente proporcional $\grave{a}$ distância ao eixo.

  1. Determine o momento de inércia de $C$ com relação ao eixo $(Oz)$.

  2. Se $C$ gira em torno do eixo $(Oz)$ com energia cinética $K$, qual a velocidade instantânea nos pontos de sua superfície lateral? (Fórmulas: $\bullet$ Momento de inércia: $I=\iiint\limits_{C}\rho\cdot l^{2}\,dV$, onde $\rho$ é a densidade e $l$  é a distância ao eixo; $\bullet$ Energia cinética de rotação: $K=\dfrac{1}{2}I\omega^{2}.$)


  1.  $6\pi.$

  2.  $\displaystyle \sqrt{\frac{K}{3\pi}}.$


2294   

Determine uma representação paramétrica para a superfície descrita a seguir. O paraboloide $z=9-x^{2}-y^{2}$, $z\geq 0.$


$x = r \cos(\theta),$ $y = r \sin(\theta),$ $z = 9 - r^2,$ onde $0 \leq r \leq 3$ e $0\leq \theta \leq 2\pi.$


3017   

Esboce a região de integração para a integral iterada $\displaystyle\int_{-1}^{2}\!\int_{-\sqrt{4-x^{2}}}^{4-x^{2}}f(x,y)\,dy dx$.


ma211-list6-ex24_sol_a.png

2235   

Seja $\mathbf{r} = x\mathbf{i} + y\mathbf{j} + z\mathbf{k}$ e $r=|\mathbf{r}|$. Verifique a identidade $\nabla \times \mathbf{r} = \mathbf{0}$.


$\nabla \times \mathbf{r} = \left[\dfrac{\partial}{\partial y} (z) - \dfrac{\partial}{\partial z}(y) \right]\mathbf{i} + \left[\dfrac{\partial}{\partial z} (x) - \dfrac{\partial}{\partial x}(z) \right]\mathbf{j} + \left[\dfrac{\partial}{\partial x} (y) - \dfrac{\partial}{\partial y}(x) \right]\mathbf{k}.$ (Note que: $r = \sqrt{x^{2} + y^{2} + z^{2}}.$)


2488   

Esboce o gráfico da função $f(x,y)=y^{2}+1$.


$z = y^{2} + 1$

ma211-list2-ex11_sol_e.png


2740   

Considere a função $f(x,y) = x \ \phi\left(\frac{x}{y}\right)$, em que $\phi(u)$ é uma função derivável de uma variável. Mostre que os planos tangentes ao gráfico de $f$ passam pela origem.


Note que $x \frac{\partial f}{\partial x} (x,y) + y \frac{\partial f}{\partial y}(x,y) = f(x,y).$


2960   

Usando coordenadas esféricas, determine o volume e o centroide do sólido que está acima do cone $\phi=\pi/3$ e abaixo da esfera $\rho=4\cos{\phi}.$


Volume: $10\pi;$ centróide: $(0,0,2,1).$


2808   

Determine os valores máximos e mínimos locais e pontos de sela da função $f(x,y)=\frac{1}{x^{2}}+\frac{1}{y}+xy$, $x>0$ e $y>0$.


Ponto de mínimo: $\displaystyle \left( 2^{2/5}, 2^{-1/5} \right).$


2096   

Seja $\Omega=\{(x,y)\in \mathbb{R}^{2}|\,(x,y)\notin A\}$, onde $A$ é a semirreta $\{(x,y)\in \mathbb{R}^{2}|\,y=0\,e\,x\geq 0\}$. Calcule

$$\int_{C}\frac{-y}{x^{2}+y^{2}}\,dx+\frac{x}{x^{2}+y^{2}}\,dy,$$

onde $C:[0,1]\rightarrow \mathbb{R}^{2}$ é uma curva de classe $C^{1}$ por partes, com imagem contida em $\Omega$, tal que $C(0)=(1,1)$ e $C(1)=(1,-1).$


$\dfrac{3\pi}{2}.$


2158   

Calcule a integral de linha $\displaystyle\oint_{C} xy \, dx + x^2 \, dy$, $C$ é o retângulo com vértices $(0,0)$, $(3,0)$, $(3,1)$ e $(0,1)$ por dois métodos:

  1. diretamente; e

  2. utilizando o Teorema de Green.


$\dfrac{9}{2}.$


2476   

Determine e faça o esboço do domínio da função $\bigstar$ $f(x,y)=\dfrac{\sqrt{y-x^{2}}}{1-x^{2}}$.


$\left\lbrace (x,y);\; y \geq x^{2},\; x\neq \pm 1 \right\rbrace.$

ma211-list2-ex10_sol_d.png


2417   

Determine o volume do sólido.

  1.  Abaixo do paraboloide $z=x^{2}+y^{2}$ e acima da região delimitada por $y=x^{2}$ e $x=y^{2}.$

  2.  Abaixo do paraboloide $z=3x^{2}+y^{2}$ e acima da região delimitada por $y=x$ e $x=y^{2}-y.$

  3.  $ $ Abaixo da superfície $z=xy$ e acima do triângulo com vértices $(1,1)$, $(4,1)$ e $(1,2).$

  4.  Limitado pelo cilindro $y^{2}+z^{2}=4$ e pelos planos $x=2y$, $x=0$ e $z=0$, no primeiro octante.


  1.  $\dfrac{6}{35}.$

  2.  $\dfrac{144}{35}.$

  3.  $\dfrac{31}{8}.$

  4.  $\dfrac{16}{3}.$


2112   

 Suponha que a equação $F(x,y,z)=0$ defina implicitamente cada uma das três variáveis $x$,$y$ e $z$ como função das outras duas: 
$z=f(x,y)$, $y=g(x,y)$ e $x=h(y,z).$ Se $F$ for diferenciável e $F_{x}$,$F_{y}$ e $F_{z}$ forem todas não nulas, mostre que
$$\frac{\partial z}{\partial x} \frac{\partial x}{\partial y}\frac{\partial y}{\partial z}=-1.$$



Note que$\displaystyle \frac{\partial z}{\partial x} = -\frac{F_{x}}{F_{z}},$$\displaystyle \frac{\partial x}{\partial y} = -\frac{F_{y}}{F_{x}}$e$\displaystyle \frac{\partial y}{\partial z} = -\frac{F_{z}}{F_{y}}.$


2310   

 Calcule $D_{\bf{u}}f(x_0,y_0)$, sendo dados

$f(x,y) = xy$, $(x_0,y_0) = (1,1)$ e $\bf{u}$ o versor de $\bf{i} + \bf{j}$.


 $\displaystyle D_{\bf{(1,1)}}f(1,1) = \sqrt{2}.$ 


2351   

Calcule o volume do conjunto dado.

  1.  $x^{2}+y^{2}\leq 1$ e $x+y+2\leq z \leq 4.$

  2.  $x\geq 0$, $y \geq 0$, $x+y\leq 1$ e $0\leq z\leq x^{2}+y^{2}.$


  1.  $2\pi.$

  2.  $\dfrac{1}{6}.$


2340   

Determine a área da superfície dada pela parte do plano $x+2y+z=4$ que está dentro do cilindro $x^{2}+y^{2}=4$.


$4\sqrt{6}\pi.$


2248   

Seja

$$\mathbf{F}(x,y) = \dfrac{x}{(x^2+y^2)^5}\mathbf{i} + \dfrac{y}{(x^2+y^2)^5}\mathbf{j}$$

e $\mathbf{n}$ a normal unitária exterior ao círculo $x^2 + y^2 \leq 1$. Calcule $\int_{C} \mathbf{F} \cdot \mathbf{n} \, ds$, em que $C$ é dada por $\mathbf{r}(t) = (\cos{t},\sin{t})$, $0 \leq t \leq \pi$. (Sugestão: Verifique que $\mathbf{F} \cdot \mathbf{n}$ é constante.)


$\pi.$


2392   

Se $g(x,y) = x^2 + y^2 - 4x$, encontre o vetor gradiente $\nabla g(1,2)$ e use-o para encontrar a reta tangente à curva de nível $g(x,y) = 1$ no ponto $(1,2)$. Esboce a curva de nível, a reta tangente e o vetor gradiente.



 $\nabla g(1,2) = (1,2) = (-2,4);$ reta tangente à curva de nível $g(x,y) = 1$ em $(1,2)$: $-x + 2y = 3.$

MA211-lista04-sol79.png


2130   

$f(t)$ e $g(x,y)$ são funções diferenciáveis tais que $g(t,f(t))=0$ para todo $t$. Suponha $f(0)=1$, 
$\dfrac{\partial g}{\partial x}(0,1)=2$ e $\dfrac{\partial g}{\partial y}(0,1)=4$. Determine a equação da reta tangente a $\gamma(t)=(t,f(t))$, 
no ponto $\gamma(0).$


$\displaystyle (x,y) = (0,1) + \lambda \left(1, - \frac{1}{2}\right),$ $\lambda \in \mathbb{R}.$


2938   

Determine a massa e o centro de massa da lâmina que ocupa a região $D$ e tem função densidade $\rho$, sendo:$D$ a região triangular com vértices $(0,0), (2,1), (0,3)$ e  $\rho(x,y) = x + y$.


Massa: $6;$ centro de massa: $\displaystyle \left(\frac{3}{4},\frac{3}{2} \right).$


2899   

Determine o valor máximo de $f(x,y,z) = 6x + z$ sobre a curva de interseção das superfícies $x^2 + y^2 = 4$ e $z = x^2 - 2y^2$.


$16.$


2005   

Se um arame com densidade linear $\rho(x,y)$ está sobre uma curva plana $C$, seus momentos de inércia em relação aos eixos $x$ e $y$ são definidos por

$$I_{x}=\int_{C}y^{2}\rho(x,y)\,ds        I_{y}=\int_{C}x^{2}\rho(x,y)\,ds.$$

Determine os momentos de inércia de um arame com o formato de um semicírculo $x^{2}+y^{2}=1$, $y\geq 0$, que é mais grosso perto da base do que perto do topo, se a função densidade linear em qualquer ponto for proporcional à sua distância à reta $y=1.$


$I_{x} = k\left(\dfrac{\pi}{2} - \dfrac{4}{3} \right)$ e $I_{y} = k\left(\dfrac{\pi}{2} - \dfrac{2}{3} \right).$


2047   


Determine se o conjunto $\{(x,y)|\,x>0,\,y>0\}$ é ou não:

  1. aberto;

  2. conexo; e

  3. simplesmente conexo.



Temos que o conjunto $D=\{(x,y)|\,x>0,\,y>0\}$ representa o primeiro quadrante, excluindo os eixos. Então:

  1. $D$ é aberto, pois em torno de cada ponto em $D$, podemos colocar um disco que se encontra em $D.$

  2. $D$ é conexo, pois o segmento de reta que une dois pontos quaisquer de $D$ encontra-se em $D.$

  3. $D$ é simplesmente conexo, pois ele é conexo e não tem buracos.


3053   

Esboce o sólido cujo volume é dado pela integral iterada.

$\displaystyle\int_{0}^{1}\int_{0}^{1-x}\int_{0}^{2-2z}\;dy dz dx$


ma211lista8q12ares.png


2792   

Determine os valores máximos e mínimos locais e pontos de sela da função $f(x,y)=x^{3}+2xy+y^{2}-5x$.


Ponto de mínimo: $\displaystyle \left( \frac{5}{3}, -\frac{5}{3} \right);$ ponto de sela: $\displaystyle \left(-1,1\right).$


2823   

Determine os valores máximo e mínimo absolutos de $f$ no conjunto $D.$

$f(x,y)=x^{2}-2xy+2y^{2}$ em $D=\{(x,y)\in \mathbb{R}^2: \;|x|+|y|\leq 1\}.$


Valor máximo: $\displaystyle  2;$ valor mínimo: $0.$


2133   

Mostre que cada a equação a seguir define implicitamente pelo  menos uma função diferenciável $y=y(x).$ 
Expresse $\mathrm{d} y/\mathrm{d} x$ em termos de $x$ e $y.$
$x^{2}y+\sin(y)=x$


 $\displaystyle \frac{d y}{d x} = -\frac{2xy - 1}{x^{2} + \cos(y)}.$



2876   

Utilize os multiplicadores de Lagrange para determinar os valores máximo e mínimo da função sujeita à(s) restrição(ões) dada(s).

$f(x,y,z) = yz + xy; \quad xy = 1, \quad y^2 + z^2 = 1.$


Valor máximo: $\dfrac{3}{2};$ valor mínimo: $\dfrac{1}{2}.$


2450   

Calcule $\displaystyle\iint\limits_{S}g(x,y,z)dS,$ onde $g(x,y,z)=(x^{2}+y^{2}+z^{2})^{1/2}$ e $S$ é a porção do parabolóide $2z=x^{2}+y^{2}$ interior ao cilindro $x^{2}+y^{2}=2y.$


$\dfrac{5\pi}{2}.$


2828   

Determine os pontos do cone $z^{2}=x^{2}+y^{2}$ que estão mais próximos do ponto $(4,2,0).$


$(2,1,\sqrt{5})$ e $(2,1,-\sqrt{5}).$


2000   

Calcule o trabalho realizado pela força ${\bf F}(x,y)=xy\,{\bf i}+y^{2}\,{\bf j}$ ao mover uma partícula da origem ao longo da reta $y=x$ até $(1,1)$ e então de volta à origem ao longo da curva $y=x^{2}.$


$\dfrac{1}{12}.$


2579   

Determine o conjunto dos pontos de continuidade da função $f(x,y) = \sqrt{6 - 2x^2 - 3y^2}$. Justifique sua resposta.


$\left\lbrace (x,y);\; 2x^{2} + 3y^{2} \leq 6 \right\rbrace.$


2611   

Use o Teorema de Stokes para calcular $\displaystyle\int_C {\bf F}\cdot  d{\bf R}$. $C$ é orientada no sentido anti-horário quando vista de cima.

  • ${\bf F}(x,y,z) = (x^2-y){\bf i} + 4z{\bf j} + x^2{\bf k}$, $C$ é a curva de interseção do plano $z=2$ com o cone $z=\sqrt{x^2+y^2}$.


$4\pi$.


2878   

Determine os valores extremos de $f(x,y) = 2x^2 + 3y^2 - 4x - 5$ na região descrita por $x^2 + y^2 \leq 16$.


Valor máximo: $f(-2, \pm 2 \sqrt{3}) = 47$ e valor mínimo $f(1,0) = -7.$


2682   

Encontre $\partial f/\partial x$ e $\partial f/\partial y$ para $f(x,y)=1/(x+y)$.


$\displaystyle \frac{\partial f}{\partial x} = \frac{\partial f}{\partial y} = -\frac{1}{(x^{2} + y^{2})^{2}}$.


2687   

Encontre $f_{x}$, $f_{y}$ e $f_{z}$ para $f(x,y,z)=x-\sqrt{y^{2}+z^{2}}$.


$\displaystyle f_{x} = 1,\;\;\;\; f_{y} = -\frac{y}{\sqrt{y^{2} + z^{2}}}\;\;\;\;\text{e}\;\;\;\; f_{z} =  -\frac{z}{\sqrt{y^{2} + z^{2}}}$.


2706   

Se $z=\sin(x+\sin{y})$, mostre que $\dfrac{\partial z}{\partial x} \;\dfrac{\partial^{2} z}{\partial x \partial y}=\dfrac{\partial z}{\partial y}\;\dfrac{\partial^{2}z}{\partial x^{2}}$.


$\begin{aligned}[t]\frac{\partial z}{\partial x} &= \cos(x + \sin y),\;\;\; \frac{\partial z}{\partial y} = \cos(x + \sin y) \cos y,\\\frac{\partial z^{2}}{\partial x\partial y} &= -\sin (x + \sin y) \cos y\;\;\text{e}\;\; \frac{\partial^{2} z}{\partial x^{2}} = -\sin (x + \sin y).\end{aligned}$


2594   

Mude as coordenadas de $(1,-1,4)$ de retangulares para cilíndricas.


 $\displaystyle (\sqrt{2}, \dfrac{7\pi}{4}, 4).$


2688   

Encontre $f_{x}$, $f_{y}$ e $f_{z}$ para $f(x,y,z)=(x^{2}+y^{2}+z^{2})^{-1/2}$.


$\begin{aligned}[t]f_{x} &= -x(x^{2} + y^{2} + z^{2})^{-3/2},\;\; f_{y} = -y(x^{2} + y^{2} + z^{2})^{-3/2}\;\;\text{e}\\f_{z} &= -z(x^{2} + y^{2} + z^{2})^{-3/2}.\end{aligned}$


2904   

Utilize coordenadas polares para determinar o volume do sólido dado:

dentro da esfera $x^2+y^2+z^2=16$ e fora do cilindro $x^{2}+y^{2}=4.$


$\displaystyle 32\sqrt{3}\pi.$


2016   

Encontre o volume do sólido delimitado pelo parabolóide $z=2+x^{2}+(y-2)^{2}$ e pelos planos $z=1$, $x=1$, $x=-1$, $y=0$ e $y=4.$



Observe que o sólido $E$ está abaixo da superfície $z = 2+x^2+(y-2)^2$ e acima do retângulo $[-1,1]\times [0,4]$ em $z=1$ (ver figura abaixo). 


ma211-list6-ex1_sol.png


Algebricamente, $$E = \{(x,y,z) \in\mathbb{R}^3: -1 \leq x \leq 1, 0 \leq y \leq 4 \mbox{ e } 1 \leq z \leq 2 + x^2 + (y-2)^2\}.$$ Logo, o volume é dado por $$V = \iint\limits_{R}(2+x^2+(y-2)^2)\,dA - \iint\limits_{ R}\,dA,$$ em que $R = \{(x,y) \in \mathbb{R}^2; -1 \leq x \leq 1 \mbox{ e } 0 \leq y \leq 4 \}$. Assim, \begin{eqnarray*} V & = & \displaystyle\int_{-1}^{1}\int_{0}^{4}(x^2+y^2-4y+5)\,dy dx \\   & = & \displaystyle\int_{-1}^{1} \left.\left(x^2y+\frac{y^3}{3}-2y^2+5y \right|_{y=0}^{y=4} \right) \,dx \\     & = & \displaystyle\int_{-1}^{1} \left(4x^2+\frac{28}{3}\right) \,dx \\     & = & \left.\frac{4x^3}{3}+\frac{28x}{3} \right|_{x=-1}^{x=1} = \frac{64}{3}. \end{eqnarray*} Observe que, pelo Teorema de Fubini, podemos optar por calcular a integral $$\int_{0}^{4}\!\int_{-1}^{1}(x^2+y^2-4y+5)\,dy dx,$$ obtendo o mesmo resultado.


3150   

Seja \(\mathbf{F}(x,y)= (ye^{xy}-1)\mathbf{i} + xe^{xy}\mathbf{j}.\)

  1.  Mostre que \(\mathbf{F}\) é um campo vetorial conservativo.

  2.  Calcule uma função potencial de \(\mathbf{F}\).

  3.  Calcule o trabalho realizado pelo campo vetorial sobre uma partícula que se move ao longo da curva representada pelas seguintes equações paramétricas \begin{align*} x  & = t+ \arcsin(\sin t) \\ y & = \dfrac{2}{\pi}\arcsin(\sin t), \ \left(0\leq t\leq 8\pi\right).  \end{align*}


2387   

Determine as equações do plano tangente e da reta normal à superfície dada, no ponto dado.

$2xyz = 3$, em $\left(\dfrac{1}{2},1,3\right)$.

 


Plano tangente: $6x + 3y + z = 9$,
Reta normal: $(x,y,z) = \left(\frac{1}{2},1,3\right) + \lambda (6,3,1),$ $\lambda \in \mathbb{R}.$


2597   

Seja $D$ a região limitada abaixo pelo plano $z=0$, acima pela esfera   $x^2+y^2+z^2=4$ e dos lados pelo cilindo $x^2+y^2=1$. Monte as integrais triplas em coordenadas cilíndricas que dão o volume de $D$ usando as ordens de integração a seguir.

  1.  $dzdrd\theta$

  2.  $drdzd\theta$

  3.  $d\theta dzdr$


  1.  $\displaystyle \int_{0}^{2\pi} \int_{0}^{1} \int_{0}^{\sqrt{4 - r^2}} r dz dr d\theta.$

  2.  $\displaystyle \int_{0}^{2\pi} \int_{0}^{\sqrt{3}} \int_{0}^{1} r  drdzd\theta +  \int_{0}^{2\pi} \int_{\sqrt{3}}^{2} \int_{0}^{\sqrt{4 - z^2}} r  drdzd\theta.$

  3.  $\displaystyle \int_{0}^{1} \int_{0}^{\sqrt{4 - r^2}} \int_{0}^{2\pi} r  d\theta dzdr.$


3008   

Calcule o centro de massa da região: $D$ o triângulo de vértices $(0,0), (0,1)$ e $(1,1)$ e a densidade é proporcional à distância do ponto à origem.


$\displaystyle \left(\frac{3}{4}, \frac{2\sqrt{2} - 1}{2\sqrt{2} + 2\ln(1 + \sqrt{2})} \right).$


2969   

Calcule a integral, transformando para coordenadas esféricas. $\displaystyle\int_{0}^{2}\int_{0}^{\sqrt{4-y^{2}}}\int_{0}^{\sqrt{4-x^{2}-y^{2}}}\dfrac{1}{x^{2}+y^{2}+z^{2}}\,dzdxdy$.


$\pi.$


3113   

A tendência de uma lâmina de resistir a uma mudança no seu movimento de rotação em torno de um eixo é medida pelo seu momento de inércia em torno daquele eixo. Se a lâmina ocupar uma região \(R\) do plano \(xy\) e se sua densidade \(\delta(x,y)\) for uma função contínua em \(R\), então os momentos de inércia em torno dos eixos \(x\), \(y\) e \(z\) são denotados por \(I_x\), \(I_y\) e \(I_z\), respectivamente, e são definidos por \begin{align*} I_x & = \iint\limits_R y^2\delta(x,y)\,dA, \\ I_y & = \iint\limits_R x^2\delta(x,y)\,dA, \\ I_z & = \iint\limits_R (x^2+y^2)\delta(x,y)\,dA. \\ \end{align*} Considere a lâmina retangular que ocupa a região descrita pelas desigualdades \(0\leq x\leq a\) e \( 0\leq y\leq b\). Supondo que a lâmina tenha densidade \(\delta\) constante, mostre que \[ \begin{array}{lll} I_x= \dfrac{\delta ab^3}{3}, & I_y= \dfrac{\delta a^3b}{3}, & I_z= \dfrac{\delta ab(a^2+b^2)}{3}. \end{array} \]


2874   

Utilize os multiplicadores de Lagrange para determinar os valores máximo e mínimo da função sujeita à(s) restrição(ões) dada(s).

$f(x,y,z) = x^4 + y^4 + z^4; \quad x^2 + y^2 + z^2 = 1.$


Valor máximo: $1;$ valor mínimo: $\dfrac{1}{3}.$


1997   

Determine o trabalho realizado pelo campo  de força ${\bf F}(x,y)=x^{2}\,{\bf i}+xy\,{\bf j}$ sobre uma partícula que dá uma volta no círculo $x^{2}+y^{2}=4$ no sentido anti-horário.


$0.$


2631   

Utilizando o Teorema de Stokes, transforme a integral $\displaystyle\iint_{ S}\mbox{rot}{\bf F}\cdot{\bf n}dS$ numa integral de linha e calcule.

  • ${\bf F}(x,y,z) = y{\bf i} + x{\bf j} + xz{\bf k}$, $S$ a superfície $z = x+y+2$ e $x^2 + \dfrac{y^2}{4} \leq 1$, sendo ${\bf n}$ a normal que aponta para baixo.


$4\pi$.


3107   

Encontre a área da superfície descrita como sendo a parte do plano \(2x+2y+z=8\) no primeiro octante.


2774   

Determine o volume máximo da maior caixa retangular no primeiro octante com três faces nos planos coordenados e com um vértice no plano $x+2y+3z=6.$



Vamos maximizar a função:

$$f(x,y)=x\cdot y\cdot \bigg(\dfrac{6-x-2y}{3}\bigg)=\dfrac{6xy-x^{2}y-2xy^{2}}{3},$$

então o volume máximo é $V=x \cdot y \cdot z.$

Para encontrar os pontos críticos devemos encontrar as derivadas parciais $f_{x}$ e $f_{y}.$ Assim,

$$f_{x}(x,y)=\frac{6y-2xy-2y^{2}}{3}\;\;\;\;\;\;\;\; \mbox{e}\;\;\;\;\;\;\;\; f_{y}(x,y)=\frac{6x-x^{2}-4xy}{3}.$$

Fazendo $f_{x}=0$ e $f_{y}=0$, obtemos o seguinte sistema de equações

$$\left \{\begin{array}{cc}6y-2xy-2y^{2}=0\\6x-x^{2}-4xy=0\\\end{array}\right.$$

Da primeira equação obtemos

$$y=0 \;\;\;\;\;\;\;\; \mbox{ou}\;\;\;\;\;\;\;\; y=3-x.$$

Como, $y=0$ não satifaz as condicões, vamos analisar o caso onde $y=3-x.$ Substituindo esse valor na segunda equação obtemos

$$x=0\;\;\;\;\;\;\;\; \mbox{ou}\;\;\;\;\;\;\;\; 3x^{2}-6x=0.$$

Novamente, como $x=0$ não satisfaz as condições, vamos analisar o caso onde $3x^{2}-6=0$. Logo, obtemos

$$x=0\;\;\;\;\;\;\;\; \mbox{ou} \;\;\;\;\;\;\;\; x=2.$$

Novamente, $x=0$ não nos interessa. Assim, sendo $x=2$ obtemos que $y=1$ e $z=\dfrac{2}{3}.$ Portanto, o volume máximo da maior caixa, nas condições do exercício, será

$$V=(2)\cdot(1)\cdot \frac{2}{3}=\frac{4}{3}.$$


3012   

Encontre o centro de massa de uma lâmina em forma de triângulo retângulo isósceles, com os lados iguais tendo comprimento $a$, se a densidade em qualquer ponto for proporcional ao quadrado da distância do vértice oposto à hipotenusa.


$\displaystyle \left(\frac{2a}{5}, \frac{2a}{5} \right).$


2685   

Encontre $\partial f/\partial x$ e $\partial f/\partial y$ para $f(x,y)=\cos^{2}(3x-y^2)$.


$\displaystyle \frac{\partial f}{\partial x} = -6\cos (3x - y^{2}) \sin(3x - y^{2}) \;\;\;\;\text{e}\;\;\;\; \frac{\partial f}{\partial y} = 4y \cos (3x - y^{2}) \sin(3x - y^{2})$.


2208   

Calcule a integral dupla.

  1.  $\displaystyle\iint\limits_{ D}x^{3}y^{2}\,dA, \quad D=\{(x,y) \in \mathbb{R}^2|\;0\leq x\leq 2,\;-x\leq y\leq x\}.$

  2.  $\displaystyle\iint\limits_{D}x\,dA, \quad D=\{(x,y) \in \mathbb{R}^2|\;0\leq x\leq \pi,\;0\leq y\leq \sin{x}\}.$

  3. $\displaystyle\iint\limits_{D}x^{3}\,dA, \quad D=\{(x,y) \in \mathbb{R}^2|\;1\leq x\leq e,\;0\leq y\leq \ln(x)\}.$

  4. $\displaystyle\iint\limits_{D}y^{2}e^{xy}\,dA, \quad D=\{(x,y) \in \mathbb{R}^2|\;0\leq y\leq 4,\;0\leq x\leq y\}.$

  5. $\displaystyle\iint\limits_{D}y^{3}\,dA, \quad D$ região com vértices $(0,2)$, $(1,1)$ e $(3,2).$


  1.  $\dfrac{256}{21}.$

  2.  $\pi.$

  3.  $\dfrac{3e^{4} + 1}{16}.$

  4.  $\dfrac{e^{16} - 17}{2}.$

  5.  $\dfrac{147}{20}.$


2156   

Demonstre que se $R$ é uma região no plano limitada por uma curva $C$ simples, fechada e suave por partes, então a área de $R$, denotada por $A(R)$, pode ser dada por

$$\oint_{C}x\, dy,$$

em que a curva está orientada no sentido positivo.



Temos que

$$A(R) = \iint\limits_{R} 1 \, dA.$$

A fim de utilizar o Teorema de Green, devemos encontrar funções $P$ e $Q$    que tenham derivadas de primeira ordem contínuas em um aberto que contenha a curva $C$ e o interior de $C$ e que satisfaçam a relação $\frac{\partial Q}{\partial x} - \frac{\partial P}{\partial y} = 1$. Observe que a curva $C$ já satisfaz as hipóteses desse teorema e $C$ é a fronteira de $R$. Um exemplo de funções $P$ e $Q$ é $P(x,y) = 0$ e $Q(x,y) = x$. Portanto, pelo Teorema de Green,

$$\iint\limits_{R} 1 \, dA = \oint_{C}0 \, dx + x\, dy = \oint_{C}x\, dy.$$


2785   

Calcule a integral dupla usando coordenadas polares: $\displaystyle\iint\limits_{R}\sqrt{x^{2}+y^{2}}\,dA$, onde $R$ é limitado pelo círculo $y=\sqrt{2x-x^{2}}$ e pela reta $y=x.$


$\displaystyle \frac{8}{9}(2 - \frac{5}{4}\sqrt{2}).$


2192   

Calcule $\displaystyle\iint \limits_{S}{\bf u}\cdot {\bf n}\,dS$, sendo $B=\{(x,y,z)\in \mathbb{R}^{3}|\, x^{2}+y^{2}+z^{2}\leq 1\}$ e ${\bf u}=x\,{\bf i}+y\,{\bf j}+z^{2}\,{\bf k}.$


$\pi.$



1969   

Calcule o limite $\displaystyle \lim_{t \rightarrow 0}\left( \arctan(t), e^{-2t}, \dfrac{\ln{t}}{t} \right)$.



Tomando
${\bf r}(t)=\left(arctg(t), e^{-2t},\frac{\ln t}{t}\right)$, temos que
$\lim\limits_{t\to 0}{\bf r}(t)=\lim\limits_{t\to 0}\left(arctg(t),e^{-2t},\frac{\ln t}{t}\right)=\left(\lim\limits_{t\to 0}arctg(t),\lim\limits_{t\to 0}e^{-2t},\lim\limits_{t\to 0}\frac{\ln t}{t}\right)$
Assim,
$\bullet \lim\limits_{t\rightarrow 0}arctg(t)=0$.
$\bullet \lim\limits_{t\to 0}e^{-2t}=1$.
$\bullet \lim\limits_{t\to 0}\dfrac{\ln t}{t}$ não existe.

Portanto, $\lim\limits_{t\to 0}\left(arctg(t),e^{-2t},\frac{\ln t}{t}\right)$ não existe.


2013   

Calcule $\displaystyle \int_{C}{\bf E}\cdot d{\bf l}$, onde ${\bf E}(x,y)=\dfrac{1}{x^{2}+y^{2}}\dfrac{x\,{\bf i}+y\,{\bf j}}{\sqrt{x^{2}+y^{2}}}$ e $C: {\bf r}(t)=(t,1)$, $-1\leq t\leq 1.$ ( O ${\bf l}$ desempenha aqui o mesmo papel que ${\bf r}:{\bf l}(t)={\bf r}(t).$)


$0.$


2520   

Dada a expressão $g(x,y)=2-f(x,y)$, escreva como o gráfico de $g$ é obtido a partir do gráfico de $f.$


Gráfico de $f$ refletido sobre o plano $xy$ e deslocado para cima por duas unidades.


2463   

Calcule a integral de superfície $\displaystyle\iint \limits_{ S}{\bf F}\cdot d{\bf S}$ para o campo vetorial ${\bf F}$ e superfície orientada $S$ dados abaixo. Em outras palavras, determine o fluxo de ${\bf F}$ através de $S$. Para superfícies fechadas, use a orientação positiva (para fora).

  • ${\bf F}(x,y,z)=x{\bf i}+y{\bf j}+z{\bf k}$, $S$ é a esfera $x^{2}+y^{2}+z^{2}=9.$


$108\pi.$


2723   

Se $z = 5x^2 + y^2$ e $(x,y)$ varia de $(1,2)$ a $(1,05; 2,1)$, compare os valores de $\Delta z$ e $dz$.


$\Delta z = 0.9225$ e $dz = 0.9$.


2676   

Calcule as derivadas parciais de $f(x,y,z) = \sin{(x^2 + y^2 + z^2)}$.


$\begin{aligned}[t]\frac{\partial f}{\partial x} &= 2x \cos (x^{2} + y^{2} + z^{2}),\;\;\;\;  \frac{\partial f}{\partial y} = 2y \cos (x^{2} + y^{2} + z^{2}) \;\;\;\;\;\text{e}\\\frac{\partial f}{\partial z} &= 2z \cos (x^{2} + y^{2} + z^{2}).\end{aligned}$


2072   

Seja $R$ o retângulo $1\leq x\leq 2$, $0\leq y\leq 1$. Calcule $\iint\limits_{R} f(x,y)\,dxdy$, sendo $f(x,y)$ igual a

  1. $\sqrt{x+y}$

  2. $\dfrac{1}{x+y}$



  1. $\dfrac{4(9\sqrt{3} - 8\sqrt{2} + 1)}{15}.$

  2. $\ln\left( \dfrac{27}{16}\right).$



2623   

Utilizando o Teorema de Stokes, transforme a integral $\iint_{ S}\mbox{rot}{\bf F}\cdot{\bf n}dS$ numa integral de linha e calcule.

  • ${\bf F}(x,y,z) = y{\bf i}-x^2{\bf j}+5{\bf k}$, $S$ a superfície parametrizada por ${\bf R}(u,v) = (u,v,1-u^2)$, $u \geq 0$, $v \geq 0$, $u+v\leq 1$, sendo ${\bf n}$ a normal apontando para cima.


$-\dfrac{5}{6}.$


2336   

Determine a área da superfície dada pela parte da superfície $z=xy$ que está dentro do cilindro $x^{2}+y^{2}=1$.


$\dfrac{2\pi}{3}(2\sqrt{2} - 1)$.


2983   

Utilize a transformação dada para calcular a integral. $\displaystyle\iint\limits_{R} (x^2 - xy + y^2) \, dA$, em que $R$ é a região delimitada pela elipse  $x^2 - xy + y^2 = 2$; $x = \sqrt{2}u - \sqrt{\dfrac{2}{3}}v$, $y = \sqrt{2}u + \sqrt{\dfrac{2}{3}}v$.


$\dfrac{4\pi}{\sqrt{3}}.$


2348   

Calcule a área da parte da superfície cilíndrica $z^{2}+x^{2}=4$ que se encontra dentro do cilindro $x^{2}+y^{2}\leq 4$ e acima do plano $xy.$


$16.$


2695   

Calcule todas as derivadas parciais de $2^{\underline{a}}$ ordem de $f(x,y)=x^{3}y^{2}$.


$\displaystyle \frac{\partial^{2} f}{\partial x^{2}}= 2xy^{2},\;\;\;\;\; \frac{\partial^{2} f}{\partial y^{2}}= 2x^{3}\;\;\;\;\;\text{e}\;\;\;\;\; \frac{\partial^{2} f}{\partial x\partial y}= \frac{\partial^{2} f}{\partial y\partial x}= 6x^{2}y.$


2849   

Passe para coordenadas polares e calcule: $\displaystyle\iint\limits_{R}xy\,dx dy$, onde $R$ é o círculo $x^{2}+y^{2}-2y\leq 0$, $x\geq 0.$


$\displaystyle \frac{2}{3}.$


2860   

Estude com relação a máximos e mínimos a função dada com as restrições dadas.

$f(x,y) = 3x + y$ e $x^2 + 2y^2 = 1.$


Ponto de máximo: $\displaystyle \left( \frac{6}{\sqrt{38}}, \frac{1}{\sqrt{38}} \right)$; ponto de mínimo: $\displaystyle \left( -\frac{6}{\sqrt{38}}, -\frac{1}{\sqrt{38}} \right)$.


2436   

Calcule a integral tripla.

  1. $\displaystyle\iiint\limits_{  E}\cos{z} \; dx dy dz$, onde $E$ é o conjunto $0\leq x \leq \dfrac{\pi}{2}$, $0\leq y \leq \dfrac{\pi}{2}$ e $x-y\leq z \leq x+y.$

  2. $\displaystyle\iiint\limits_{  E}(y-x)\;dx dy dz$, onde $E$ é o conjunto $4\leq x+y\leq 8$, $\dfrac{1}{x}\leq y\leq \dfrac{2}{x}$,  $y> x$ e $0\leq z \leq \dfrac{\sqrt[3]{xy}}{\sqrt{x+y}}.$


  1.  $2.$

  2.  $3 - 6\sqrt[3]{2} - 2\sqrt{2} + 6 \sqrt[6]{2^5}.$


2285   

Identifique a superfície que tem equação paramétrica ${\bf r}(u,v)=(u+v)\,{\bf i}+(3-v)\,{\bf j}+(1+4u+5v)\,{\bf k}.$.


$4x - y - z = -4.$


2656   

Determine as derivadas parciais de $z=\dfrac{x^{3}+y^{2}}{x^{2}+y^{2}}$.


$\displaystyle \frac{\partial z}{\partial x} =  \frac{x^{4} + 3x^{2}y^{2} - 2xy^{2}}{(x^{2} + y^{2})^{2}}\;\;\;\;\;\;\text{e}\;\;\;\;\; \frac{\partial z}{\partial y} = \frac{2x^{2}y(1 - x)}{(x^{2} + y^{2})^{2}}.$


2734   

Determine as equações do plano tangente e da reta normal ao gráfico da função dada, no ponto dado. $f(x,y) = xy$ em $\left(\dfrac{1}{2}, \dfrac{1}{2}, f\left(\dfrac{1}{2}, \dfrac{1}{2}\right)\right)$.


Plano tangente: $4z = 2x + 2y - 1$\\

Reta normal: $(x,y,z) = \left(\frac{1}{2}, \frac{1}{2}, \frac{1}{4} \right) + \lambda \left(\frac{1}{2},\frac{1}{2},-1 \right)$.



2366   

Em que direção e sentido a função dada cresce mais rapidamente no ponto dado? E em que direção e sentido decresce mais rapidamente?

$f(x,y) = x^2 + xy + y^2$ em $(1,1)$.


Cresce: $(3,3)$; descresce: $(-3,-3).$



2832   

Determine as dimensões de uma caixa retangular de volume máximo tal que a soma dos comprimentos de suas 12 arestas seja uma constante $c$.


A caixa é um cubo com arestas de comprimento $\dfrac{c}{12}.$


2347   

Seja $f:K\rightarrow \mathbb{R}$ de classe $C^{1}$ no compacto $K$ com fronteira de conteúdo nulo e interior não-vazio. Mostre que a área da superfície $z=f(x,y)$ (isto é, da superfície ${\bf r}$ dada por $x=u$, $y=v$ e $z=f(u,v)$) é dada pela fórmula

$$\iint\limits_{ K}\sqrt{1+\bigg(\frac{\partial f}{\partial x}\bigg)^{2}+\bigg(\frac{\partial f}{\partial y}\bigg)^{2}}dxdy.$$


2894   

Passe para coordenadas polares e calcule: $\displaystyle\iint\limits_{R}\arctan\left(\dfrac{y}{x}\right)\,dA$, onde $R=\{(x,y) \in \mathbb{R}^2| 1\leq x^{2}+y^{2}\leq 4, 0\leq y\leq x\}.$


$\displaystyle \frac{3\pi^2}{64}.$


2411   

Inverta a ordem de integração.

  1.  $\displaystyle\int_{0}^{\dfrac{\pi}{4}}\bigg[\int_{\sin{x}}^{\cos{x}}f(x,y)\,dy\bigg]dx$

  2.  $\displaystyle\int_{-1}^{2}\bigg[\int_{\sqrt{\frac{7+5y^{2}}{3}}}^{\frac{y+7}{3}}f(x,y)\,dx\bigg]dy$

  3.  $\displaystyle\int_{0}^{3}\bigg[\int_{x^{2}-2x}^{\sqrt{3x}}f(x,y)\,dy\bigg]dx$


  1.    $\displaystyle \int_{0}^{\frac{\sqrt{2}}{2}}\bigg[\int_{0}^{\arcsin{y}}f(x,y)\,dx\bigg]dy + \int_{\frac{\sqrt{2}}{2}}^{1}\bigg[\int_{0}^{\arccos{y}}f(x,y)\,dx\bigg]dy$

  2.  $\displaystyle\int_{2}^{3}\bigg[\int_{3x - 7}^{\sqrt{\frac{3x^2 - 7}{5}}}f(x,y)\,dy\bigg]dx$

  3.  $\displaystyle\int_{-1}^{0}\bigg[\int_{1 - \sqrt{1 + y}}^{1 + \sqrt{1 + y}}f(x,y)\,dx\bigg]dy + \int_{0}^{3}\bigg[\int_{\dfrac{y^{2}}{3}}^{1 + \sqrt{1 + y}}f(x,y)\,dx\bigg]dy$


2341   

Determine a área da superfície dada pela porção do cone $z=2\sqrt{x^{2}+y^{2}}$ entre os planos $z=2$ e $z=6.$


$8\sqrt{5}\pi.$


2334   

Determine a taxa de variação máxima de $f$ no ponto dado e a direção em que isso ocorre.

$f(x,y,z) = \tan{(x + 2y + 3z)},  (-5,1,1).$


$\sqrt{14}.$


2453   

Integre $g(x,y,z)=x+y+z$ sobre a porção do plano $2x+2y+z=2$ que está no primeiro octante.


$2.$


3135   

  1.  Seja \(\sigma\) a superfície de um sólido \(G\) com vetor normal unitário \(\mathbf{n}\) orientado para fora de \(\sigma\). Suponha que \(\mathbf{F}\) seja um campo vetorial com derivadas parciais de primeira ordem contínuas em \(\sigma\). Prove que \[\iint\limits_\sigma (\mathrm{rot\,}\mathbf{F})\cdot\mathbf{n}\,dS = 0.\] [Sugestão: tome \(C\) uma curva fechada simples em \(\sigma\) que separa a superfície em duas subsuperfícies \(\sigma_1\) e \(\sigma_2\) com fronteira comum \(C\). Aplique o Teorema de Stokes a \(\sigma_1\) e a \(\sigma_2\) e some os resultados.]

  2.  O campo vetorial \(\mathrm{rot\,}\mathbf{F}\) é denominado campo rotacional de \(\mathbf{F}\). Em palavras, interprete a fórmula do item anterior como uma afirmação sobre o fluxo do campo rotacional.


2650   

Verifique que a função $z=\ln(e^{x}+e^{y})$ é uma solução das equações diferenciais

$$\frac{\mathrm{\partial}z}{\mathrm{\partial}x} + \frac{\mathrm{\partial}z}{\mathrm{\partial}y}=1\;\;\;\;\;\;\; e\;\;\;\;\;\;\; \frac{\mathrm{\partial}^{2}z}{\mathrm{\partial}^{2}x}+\frac{\mathrm{\partial}^{2}z}{\mathrm{\partial}^{2}y}-\bigg(\frac{\mathrm{\partial}^{2}z}{\mathrm{\partial}x\mathrm{\partial}y}\bigg)^{2}=0.$$


$\begin{aligned}[t]\frac{\partial z }{\partial x} &= \frac{e^{x}}{e^{x} + e^{y}},\;\;\; \frac{\partial z }{\partial y} = \frac{e^{y}}{e^{x} + e^{y}},\\\frac{\partial^{2} z }{\partial x^{2}} &= \frac{\partial^{2} z }{\partial y^{2}} = \frac{e^{x + y}}{(e^{x} + e^{y})^{2}},\;\;\; \frac{\partial^{2} z }{\partial x \partial y} = -\frac{e^{x + y}}{(e^{x} + e^{y})^{2}}.\end{aligned}$


1992   

Determine ${\bf r}(t)$ sabendo que

  1. ${\bf r}'(t)=t{\bf i}+2{\bf k}$ e ${\bf r}(0)={\bf i}+{\bf j}.$
  2. ${\bf r}'(t)=\sin(t){\bf i}+\cos(2t){\bf j}+\dfrac{1}{t+1}{\bf k}$, $t\geq 0$ e ${\bf r}(0)={\bf i}-{\bf j}+2{\bf k}.$
  3. ${\bf r}'(t)=\dfrac{1}{1+4t^{2}}{\bf i}+e^{-t}{\bf j}+{\bf k}$ e ${\bf r}(0)={\bf k}.$


3020   

Esboce a região de integração e calcule a integral $\displaystyle\int_{0}^{3}\!\!\int_{0}^{2}(4-y^{2})\,dy dx$.


$16.$

ma211-list6-ex25_sol_a.png


2503   

Encontre uma equação para a curva de nível da função $f(x,y)=\displaystyle \int_{x}^{y}\dfrac{dt}{1+t^{2}}$ que passa pelo ponto $(-\sqrt{2},\sqrt{2})$.


$\arctan(y) - \arctan(x) = 2\arctan(\sqrt{2}).$


2217   

Determine o rotacional e o divergente do campo vetorial $\mathbf{F}(x,y,z) = e^{xy}\sin{z}\mathbf{j} + y\tan^{-1}(x/z)\mathbf{k}$.


$\text{rot } \mathbf{F} = (\arctan(x/z) - e^{xy}\cos(z))\mathbf{i} - \dfrac{yz}{x^{2} + z^{2}} \mathbf{j} + ye^{xy}\sin(z) \mathbf{k}.$ $\text{div } \mathbf{F} = xe^{xy}\sin(z) - \dfrac{xy}{x^{2} + z^{2}}.$


2535   

Ache o centro de massa de $E$, em que:

  1.  A densidade de um ponto $P$ de um sólido cúbico $E$ de aresta $a$ é diretamente proporcional ao quadrado da distância de $P$ a um vértice fixo do cubo.

  2.  $E$ é o tetraedro delimitado pelos planos coordenados e o plano  $2x+5y+z=10$ e a densidade em $P(x,y,z)$ é diretamente proporcional $\grave{a}$ distância do plano $xz$ a $P.$


  1.  $\displaystyle \left( \dfrac{7a}{12},\dfrac{7a}{12},\dfrac{7a}{12} \right).$

  2.  $\displaystyle \left( 1,\dfrac{4}{5},2 \right).$


3093   

Use um software de apoio computacional para mostrar que o volume \(V\) sob a superfície \(z=xy^3\sin(xy)\) e acima do retângulo \([0,\pi]\times[0,1]\) no plano \(xy\)  é dado por \(V=3/\pi\).


1975   

Mostre que a curva com equações paramétricas $x = \sin{t}, \ y = \cos{t}, \ z = \sin^2t$ é a curva de intersecção das superfícies $z = x^2$ e $x^2 + y^2 = 1$. Use esse fato para esboçar a curva.


2230   

Mostre que qualquer campo vetorial da forma

$$\mathbf{F}(x,y,z) = f(y,z)\mathbf{i} + g(x,z)\mathbf{j} + h(x,y)\mathbf{k}$$

é incompressível.


Note que $\text{div } \mathbf{F} = 0.$


2558   

Determine o limite, se existir, ou mostre que o limite não existe.

$\displaystyle \lim_{(x,y) \to (0,0)} \dfrac{x^2ye^y}{x^4 + 4y^2}$.

Não existe.


Não existe.


2322   

Calcule a área da superfície dada por: ${\bf r}(u,v)=(u,v,1-u-v)$, $u\geq 0$,  $v\geq 0$ e $u+v\leq 1.$. (Sugerimos ao leitor desenhar a imagem da superfície.)


$\dfrac{\sqrt{3}}{2}.$


2244   

Calcule $\int_{C}\mathbf{F} \cdot \mathbf{n} \, ds$ ($\mathbf{n}$ é unitário), onde $\mathbf{F}(x,y) = x^2\mathbf{i}$, $C$ dada por $\mathbf{r}(t) = (2\cos{t},\sin{t})$, $0 \leq t \leq 2\pi$ e $\mathbf{n}$ a normal que aponta para fora da região $x^2/4+y^2\leq 1$.


$0$.


2508   

Faça o mapa de contorno da função $f(x,y)=ye^{x}$ mostrando várias de suas curvas de nível.


$y = Ce^{-x}.$

ma211-list2-ex17_sol_c.png


2929   

Esboce o sólido cujo volume é dado pela integral abaixo e calcule-a.

$$\int_{0}^{\pi/6}\!\!\int_{0}^{\pi/2}\!\!\int_{0}^{3}\rho^{2}\sin{\phi}\;d\rho d\theta d\phi$$


ma211-list9-ex10_sol.png

2299   

Identifique e faça um esboço da imagem da superfície parametrizada dada por ${\bf r}(u,v)=(u,v,u^{2}+v^{2})$, $(u,v)\in \mathbb{R}^{2}.$.


Paraboloide de rotação $z = x^2 + y^2.$


2922   

Utilize a integral dupla para determinar a área da região: cortada do primeiro quadrante pela curva $r=2(2-\sin(2\theta))^{1/2}.$


$2(\pi - 1).$


2653   

Seja

$$f(x,y)=\begin{cases}\dfrac{x^{3}y-xy^{3}}{x^{2}+y^{2}}, & \quad \text{se } (x,y)\neq (0,0),\\0, & \quad \text{se } (x,y)=(0,0).\\\end{cases}$$

  1. Use um computador para traçar o gráfico de $f$.

  2. Determine $f_{x}(x,y)$ e $f_{y}(x,y)$ quando $(x,y)\neq (0,0).$

  3. Determine $f_{x}(0,0)$ e $f_{y}(0,0)$ use a definição das derivadas parciais como limite.

  4. Mostre que $f_{xy}(0,0)=-1$ e $f_{yx}(0,0)=1$

  5. O resultado da parte (d) contradiz o Teorema de Clairaut? Use o gráfico de $f_{xy}$ e $f_{yx}$ para ilustrar sua resposta.


  1. Gráfico de $f$:
    ma211-list3-ex20_sol_a.png
  2. $\displaystyle f_{x} = \frac{x^{4}y + 4x^{2}y^{3} - y^{5}}{(x^{2} + y^{2})^{2}}\;\;\text{e}\;\;f_{y} = \frac{x^{5} - 4x^{3}y^{2} - xy^{4} }{(x^{2} + y^{2})^{2}}$ quando $(x,y)\neq (0,0).$
  3. $f_{x}(0,0) = f_{y}(0,0) = 0$.
  4. Use $\displaystyle f_{xy}(0,0)= \lim_{h \to 0} \frac{f_{x}(0,h) - f_{x}(0,0)}{h}\;\;\text{e}\;\;f_{yx}(0,0)= \lim_{h \to 0} \frac{f_{y}(h,0) - f_{y}(0,0)}{h}$.
  5. Para $(x,y) \neq (0,0),$ $\displaystyle f_{xy} = {x^{6} + 9x^{4}y^{2} - 9x^{2}y^{4} - y^{6}}{(x^{2} + y^{2})^{3}}.$ Como $f_{xy}$ não é contínua na origem, não há uma contradição com o Teorema de Clairaut. Os gráficos de $f_{xy}$ e $f_{yx}$ são idênticos, exceto na origem:
    ma211-list3-ex20_sol_e.png

2186   

Encontre os valores de $\partial z/ \partial x$ e $\partial z/\partial y$ no ponto indicado.
$z^{3}-xy+yz+y^{3}-2=0$,  $(1,1,1).$



 $\displaystyle \frac{\partial z}{\partial x}(1,1,1) = \frac{1}{4}$ e $\displaystyle \frac{\partial z}{\partial x}(1,1,1) = -\frac{3}{4}.$


2931   

Calcule a integral em coordenadas esféricas. $\displaystyle\int_{0}^{2\pi}\int_{0}^{\pi/4}\int_{0}^{2}(\rho\cos{\phi})\rho^{2}\sin{\phi}\,d\rho d\phi d\theta$.


$2\pi.$


3034   

Uma região $R$ é mostrada na figura abaixo. Decida se você deve usar coordenadas polares ou retangulares e escreva $\iint \limits_{ R}f(x,y)\,dA$ como uma integral iterada, onde $f$ é uma função qualquer contínua em $R.$


ma211-list7-ex5_a.png



$\displaystyle \int_{0}^{\frac{3\pi}{2}} \int_{0}^{4} f(r\cos(\theta),r\sin(\theta)) r  d r d \theta.$


2059   

Determine se ${\bf F}(x,y,z)=x\,{\bf i}+y\,{\bf j}+z\,{\bf k}$ é ou não um campo vetorial conservativo. Se for, determine uma função $f$ tal que ${\bf F}=\nabla f.$


Sim. $f(x,y,z) = \dfrac{x^{2} + y^{2} + z^{2}}{2} + K.$


2515   

Encontre uma equação para a superfície de nível da função $f(x,y,z)=\sqrt{x-y}-\ln z$ que passa pelo ponto $(3,-1,1)$.


$\sqrt{x - y} - \ln(z) = 2.$


2114   

Se $z=f(x-y)$, mostre que
$$\dfrac{\partial z}{\partial x}+\dfrac{\partial z}{\partial y}=0.$$



Note que se $u = x - y,$ então $\displaystyle \frac{\partial z}{\partial  x} = \frac{dz}{du}$e$\displaystyle \frac{\partial  z}{\partial  y} = -\frac{dz}{du}.$


3098   

Como não há antiderivada elementar da função \(e^{x^2}\), a integral \[ \int_0^2\int_{y/2}^1 e^{x^2}\, dxdy \] não pode ser calculada integrando-se primeiro em relação a \(x\). Calcule essa integral expressando-a como uma integral iterada equivalente com ordem de integração invertida.



A região de integração é dada por \(\displaystyle R=\{(x,y)\in\mathbb{R}^2;\ 0\leq y\leq 2,\ y/2\leq x\leq 1\}\). Vamos inverter a ordem de integração sobre a região \(R\):\begin{align*} \int_0^2\int_{y/2}^1 e^{x^2}\, dxdy  & = \iint\limits_R e^{x^2}\,dA = \int_0^1\int_0^{2x} e^{x^2}\,dydx= \int_0^1\left[e^{x^2}y\right]_{y=0}^{2x}\,dx \\     & = \int_0^1 2xe^{x^2}\,dx = \left.e^{x^2}\right]_0^1 = e-1 \end{align*}


2164   

Use o Teorema de Green para calcular $\int_{C}\mathbf{F} \cdot d\mathbf{r}$, onde $\mathbf{F}(x,y) = (\sqrt{x} + y^3,x^2+\sqrt{y})$, $C$ consiste no arco da curva $y = \sin{x}$ de $(0,0)$ a $(\pi,0)$ e no segmento de reta $(\pi,0)$ a $(0,0)$. (Verifique a orientação da curva antes de aplicar o Teorema.)


$\dfrac{4}{3} - 2\pi.$


2620   

Suponha que  $S$ e $C$ satisfaçam as hipóteses do Teorema de Stokes e $f$ e $g$ tenham derivadas parciais de segunda ordem contínuas. Demonstre que $\displaystyle\int_C (f\nabla f)\cdot d{\bf R} = 0$



Note que $\mbox{rot} (f\nabla f) = {\bf 0}.$


2526   

Seja $f(x,y)=3x+2y.$ Calcule:

  1. $f(1,-1)$;

  2. $f(a,x)$;

  3. $\dfrac{f(x+h,y)-f(x,y)}{h}$;

  4. $\dfrac{f(x,y+k)-f(x,y)}{k}$.


  1. $1.$

  2. $3a + 2x.$

  3. $3.$

  4. $2.$


2883   

Determine a curva de nível de $f(x,y) = x^2 + 16y^2$ que seja tangente à curva $xy = 1$, $x>0$ e $y>0$. Qual o ponto de tangência?


$x^{2} + 16 y^{2} = 8;$ o ponto de tangência é $\displaystyle \left( 2, \frac{1}{2} \right).$


3128   

Sejam \(\mathbf{r}=x\mathbf{i}+y\mathbf{j}+z\mathbf{k}\), \(r=\|\mathbf{r}\|\), \(f\) uma função diferenciável de uma variável e \(\mathbf{F}(\mathbf{r})=f(r)\mathbf{r}\).

  1.  Mostre que \[\nabla f(r) = \dfrac{f'(r)}{r}\mathbf{r}.\]

  2.  Use o resultado anterior para mostrar que \(\displaystyle \mathbf{F}=3f(r)+rf'(r). \)


2527   

Seja $f(x,y)=\dfrac{x-y}{x+2y}$.

  1. Determine o domínio.

  2. Calcule $f(2u+v,v-u).$


  1. $\left\lbrace (x,y);\; x \neq -2y \right\rbrace$

  2. $\frac{u}{v}.$


2657   

Determine as derivadas parciais de $f(x,y)=e^{-x^{2}-y^{2}}$.



$\displaystyle \frac{\partial f}{\partial x} = -2xe^{-x^{2} - y^{2}}\;\;\;\;\;\;\text{e}\;\;\;\;\; \frac{\partial f}{\partial y} = -2ye^{-x^{2} - y^{2}}.$


2655   

Determine as derivadas parciais de $z=\cos(xy)$.


$\displaystyle \frac{\partial z}{\partial x} = -y\sin(xy)\;\;\;\;\;\;\text{e}\;\;\;\;\; \frac{\partial z}{\partial y} = -x\sin(xy).$


2724   

Se $z = x^2 - xy + 3y^2$ e $(x,y)$ varia de $(3;-1)$ a $(2,96;-0,95)$, compare os valores de $\Delta z$ e $dz$.


$\Delta z = -0.7189$ e $dz = -0.73$.


2493   

Dada a função $f(x,y)=xy$.

  1. Encontre o domínio da função.

  2. Encontre a imagem da função.

  3. Descreva as curvas de nível da função.


  1. $D_{f} = \mathbb{R}^{2}$.

  2. $Im(f) = \mathbb{R}.$

  3. As curvas de nível são as hipérboles $xy = C$ quando $C \neq 0$ e os eixos $x$ e $y$ quando $C = 0.$


2697   

Calcule todas as derivadas parciais de $2^{\underline{a}}$ ordem de $z=\ln(1+x^{2}+y^{2})$.



$\begin{aligned}[t]\frac{\partial^{2} z}{\partial x^{2}} &= \frac{2 + 2y^{2} - 2x^{2}}{(1 + x^{2} + y^{2})^{2}},\;\;\;\;\; \frac{\partial^{2} z}{\partial y^{2}}= \frac{2 + 2x^{2} - 2y^{2}}{(1 + x^{2} + y^{2})^{2}} \;\;\;\;\;\text{e}\\\frac{\partial^{2} z}{\partial x\partial y} &= \frac{\partial^{2} z}{\partial y\partial x}= \frac{-4xy}{(1 + x^{2} + y^{2})^{2}}.C\end{aligned}$


2044   

Utilize a Equação
$$ \dfrac{dy}{dx}=-\dfrac{\dfrac{\partial F}{\partial x}}{\dfrac{\partial F}{\partial y}}=-\dfrac{F_x}{F_y}$$
para determinar $\mathrm{d}y/\mathrm{d}x$.
$\sqrt{xy}=1+x^{2}y$



$\displaystyle \frac{dy}{dx} = \frac{4(xy)^{3/2} - y}{x - 2x^{2}\sqrt{xy}} .$


2196   

Se ${\bf F}=(xz,yz,2)$ e $E$ é a região dada por $x^{2}+y^{2}\leq 1$ e $0\leq z \leq 1,$ mostre que o Teorema do Divergente é verdadeiro neste caso. Calcule as duas integrais do enunciado do Teorema e mostre que elas têm o mesmo valor.



2084   

Calcule $\displaystyle\int_{C}y\,dx+x^{2}\,dy$, onde $C$ é a curva cuja imagem é o segmento de extremidades $(1,1)$ e $(2,2)$, orientada de $(1,1)$ para $(2,2).$


$\dfrac{23}{6}.$


2206   

Se $z=f(x,y)$ com $x=u+v$ e $y=u-v$, demonstre que 
$$\frac{\partial z}{\partial u}+\frac{\partial z}{\partial v}=2 \frac{\partial f}{\partial x}.$$



Note que $\displaystyle \frac{\partial z}{\partial u} = \frac{\partial f}{\partial x} + \frac{\partial f}{\partial y}$ e $\displaystyle \frac{\partial z}{\partial v} = \frac{\partial f}{\partial x} - \frac{\partial f}{\partial y}.$


1947   

Calcule a integral de linha, onde $C$ é a curva dada.

$\displaystyle\int_{C}\dfrac{-y}{4x^{2}+y^{2}}\,dx+\dfrac{x}{4x^{2}+y^{2}}\,dy$, $C$ tem por imagem a elipse $4x^{2}+y^{2}=9$ e o sentido de percurso é o anti-horário.


$\displaystyle \pi.$


2245   

Calcule $\int_{C}\mathbf{F} \cdot \mathbf{n} \, ds$ ($\mathbf{n}$ é unitário), onde $\mathbf{F}(x,y) = x^2\mathbf{i}$, $C$ dada por $\mathbf{r}(t) = (2\cos{t},\sin{t})$, $0 \leq t \leq \pi$ e $\mathbf{n}$ a normal com componente $y \geq 0$.



$0$.


2670   

Seja $z=e^{y}\phi(x-y)$, onde $\phi$ é uma função diferenciável de uma variável real. Mostre que $$\dfrac{\partial z}{\partial x}+\dfrac{\partial z}{\partial y}=z.$$


$\displaystyle \frac{\partial z}{\partial x} = e^{y}\phi'(x-y) \;\;\;\;\;\text{e}\;\;\;\;\; \frac{\partial z}{\partial y} = e^{y} \phi(x-y) -e^{y}\phi' (x-y).$


2822   

Determine os valores máximo e mínimo absolutos de $f$ no conjunto $D.$

$f(x,y)=y^{2}-x^{2}$ em $D=\{(x,y)\in \mathbb{R}^2: x^{2}+y^{2}\leq 4\}.$


Valor máximo: $\displaystyle  4;$ valor mínimo: $-4.$


2300   

Identifique e faça um esboço da imagem da superfície parametrizada dada por ${\bf r}(u,v)=(1,u,v)$, $0\leq u\leq 1$, $0\leq v \leq 1.$


Região quadrada do plano $x = 1:$ $0 \leq y \leq 1$ e $0 \leq z \leq 1.$


2531   

Represente graficamente o domínio da função $z=f(x,y)$ dada por $z=\ln(2x^{2}+y^{2}-1)$.


$\left\lbrace (x,y); 2x^{2} + y^{2} > 1 \right\rbrace$

ma211-list2-ex25_sol_d.png


3116   

Seja \(G\) a região sólida dentro da esfera de raio \(2\) centrada na origem e acima do plano \(z=1\). Mostre (ou verifique) os seguintes resultados:

  1.  O volume de \(G\) é dado por \[\iiint\limits_G\,dV = \int_0^{2\pi}\int_0^{\sqrt{3}}\int_1^{\sqrt{4-r^2}}r\,dzdrd\theta \]

  2.  \[\iiint\limits_G\dfrac{z}{x^2+y^2+z^2}\,dV = \int_0^{2\pi}\int_0^{\sqrt{3}}\int_1^{\sqrt{4-r^2}}\dfrac{rz}{r^2+z^2}\,dzdrd\theta \]


2355   

Calcule o volume do conjunto dado.

  1.  $x+y+z\leq 1$, $x\geq 0$, $y\geq 0$ e $z\geq 0.$

  2.  $x\leq y\leq 1$, $x\geq 0$, $z\geq 0$ e $z^{2}+x^{4}+x^{2}y^{2}\leq 2x^{2}.$


  1.  $\dfrac{1}{6}.$

  2.  $\dfrac{\pi(1 - \sqrt{2})}{8} + \dfrac{1}{3}.$


2772   

Determine os valores máximos e mínimos locais e pontos de sela da função $f(x,y)=x^{3}-12xy+8y^{3}$.



Sendo $f(x,y)=x^{3}-12xy+8y^{3}$, vamos inicialmente localizar seus pontos críticos:

$$f_{x}(x,y)=3x^{2}-12y \;\;\;\;\;\;\;\; \mbox{e} \;\;\;\;\;\;\;\; f_{y}(x,y)=-12x+24y^{2}.$$

Igualando essas derivadas parciais a zero, obtemos as equações

$$x^{2}-4y=0 \;\;\;\;\;\;\;\; \mbox{e} \;\;\;\;\;\;\;\; 2y^{2}-x=0.$$

Para resolvê-las, substituímos $x=2y^{2}$ da segunda equação na primeira. Isso resulta em

$$0=y^{4}-y=y(y^{3}-1)$$

e existem duas raízes reais $y=0$ e $y=1.$ Os dois pontos críticos de $f$ são $(0,0)$ e $(2,1).$\\

Agora vamos calcular as segundas derivadas parciais e $D(x,y)$:

$$f_{xx}(x,y)=6x\,\,\,\, f_{xy}(x,y)=-12 \;\;\;\; f_{yy}(x,y)=48y$$

$$\begin{split}D(x,y)&=f_{xx}(x,y)\cdot f_{yy}(x,y)-(f_{xy}(x,y))^{2}\\&=(6x)\cdot (48y)-(-12)^{2}=288xy-144.\end{split}$$

Como $D(0,0)=-144<0$, segue do Teste da Derivada Segunda que $(0,0)$ é um ponto de sela, ou seja, $f$ não tem nem máximo local nem mínimo local em $(0,0).$ Como $D(2,1)=432>0$ e $f_{xx}(2,1)=12>0$, vemos do Teste da Derivada Segunda que $f(2,1)=-8$ é um mínimo local.


3069   

Calcule a integral de linha $\displaystyle\int_{C}{\bf F}\cdot d{\bf r}$, onde ${\bf F}(x,y)=(x+y^{2})\,{\bf j}$ e $C$ é a curva da figura abaixo.

ma211-list10-ex16_a.png


$4.$


2887   

Encontre o ponto da curva $x^2 - 2xy + y^2 - 2x - 2y + 1 = 0$ mais próximo da origem.


$\displaystyle \left( \frac{1}{4}, \frac{1}{4} \right).$


2327   

Calcule a área da superfície dada por: ${\bf r}(u,v)=(\cos u,v,\sin u)$ e $u^{2}+4v^{2}\leq 1.$. (Sugerimos ao leitor desenhar a imagem da superfície.)


$\dfrac{\pi}{2}.$


3031   

No cálculo de uma integral dupla sobre uma região $D$, obtivemos uma soma de integrais iteradas como a que segue:

$$\int\!\!\!\!\int\limits_{\!\!\!\!\!\! D} \! f(x,y)\,dA=\int_{0}^{1}\!\!\int_{0}^{2y} \! f(x,y)\,dx dy+\int_{1}^{3}\!\!\int_{0}^{3-y} \! f(x,y)\,dx dy.$$

Esboce a região $D$ e expresse a integral dupla como uma integral iterada com ordem de integração contrária.


$\displaystyle \int_{0}^{2}\!\!\int_{\frac{x}{2}}^{3-x} \! f(x,y)\,dx dy.$

ma211-list6-ex35_sol.png


2074   

Seja $R$ o retângulo $1\leq x\leq 2$, $0\leq y\leq 1$. Calcule $\iint\limits_{R} f(x,y)\,dxdy$, sendo $f(x,y)$ dada por

  1. $y\cos(xy)$

  2. $x\sin(\pi y)$


  1.  $\cos(1) - \dfrac{(1 + \cos(2))}{2}$

  2.  $\ln\left(\dfrac{4}{3}\right).$


2109   

Sejam $f(x)$ e $g(x)$ duas funções contínuas, respectivamente, nos intervalos $[a,b]$ e $[c,d].$ Use o seguinte resultado $$\iint\limits_{R}f(x)g(y)\,dx dy=\bigg(\int_{a}^{b}f(x)\,dx\bigg)\bigg(\int_{c}^{d}g(y)\,dy\bigg),$$ onde $R$ é o retângulo $a\leq x\leq b$ e $c\leq y\leq d$, para calcular as integrais

  1.  $\displaystyle\iint\limits_{R} x\ln(y)\,dx dy$, onde $R$ é o retângulo $0\leq x\leq 2,\;1\leq y\leq 2.$

  2.  $\displaystyle\iint\limits_{R} xye^{x^{2}-y^{2}}\,dx dy$, onde $R$ é o retângulo $-1\leq x\leq 1,\;0\leq y\leq 3.$


  1.  $2(2\ln(2) - 1).$

  2.  $0.$


2726   

Utilize as diferenciais para estimar a quantidade de estanho em uma lata cilíndrica fechada com $8$ cm de diâmetro e $12$ cm de altura se a espessura da folha de estanho for de $0,04$ cm.


Para $V = \pi r^{2}h$ o volume da lata de raio $r$ e altura $h,$ temos $\Delta V \approx 16$ cm$^{3}.$


2640   

Determine as derivadas parciais de primeira ordem da função $u=\sqrt{x_{1}^{2}+x_{2}^{2}+\cdot \cdot \cdot +x_{n}^{2}}$.


$\displaystyle \frac{\partial u}{\partial x_{i}}= \frac{x_{i}}{\sqrt{x_{1}^{2}+x_{2}^{2}+\cdot \cdot \cdot +x_{n}^{2}}}$ para todo $i = 1, \cdots, n$.


2069   

Utilize as Equações 

$\dfrac{\partial z}{\partial x}=-\dfrac{\dfrac{\partial F}{\partial x}}{\dfrac{\partial F}{\partial z}}$ e $\dfrac{\partial z}{\partial y}=-\dfrac{\dfrac{\partial F}{\partial y}}{\dfrac{\partial F}{\partial z}}$

para determinar $\partial z/\partial x$ e $\partial z/\partial y$.

$xyz=\cos(x+y+z)$


$\displaystyle \frac{dz}{dx} = \frac{yz + \sin(x + y + z)}{xy + \sin(x + y + z)}$ e $\displaystyle \frac{dz}{dy} = \frac{xz + \sin(x + y + z)}{xy + \sin(x + y + z)}.$


2185   

No item abaixo: 

  1. expresse $\partial w/\partial u$ e $\partial w/ \partial v$ como funções de $u$ e $v$, usando a Regra da Cadeia e também expressando $w$ diretamente em termos e $u$ e $v$ antes de diferenciar; 
  2. calcule $\partial w/\partial u$ e $\partial w/ \partial v$ no ponto dado $(u,v)$.

$w=\ln(x^{2}+y^{2}+z^{2})$,  $x=ue^{v}\sin{u}$,  $y=ue^{v}\cos{u}$,  $z=ue^{v}$; $(u,v)=(-2,0).$



  1. $\displaystyle w(u,v) = \ln(2) + 2\ln(u) + 2v,$$\displaystyle \frac{\partial w}{\partial u}(u,v) = \frac{2}{u}$ e $\displaystyle \frac{\partial w}{\partial v}(u,v) = 2.$
  2. $\displaystyle \frac{\partial w}{\partial u}(-2,0) = -1$ e $\displaystyle \frac{\partial w}{\partial v}(-2,0) = 2.$


3000   

Calcule a integral trocando a ordem de integração. $\displaystyle\int_{0}^{1}\!\!\int_{x}^{1}e^{x/y}\,dy dx$.


A região de integração é do tipo I, é dada por

$$\{(x,y) \in \mathbb{R}^2: 0 \leq x \leq 1 \mbox{ e } x \leq y \leq 1\}$$

e pode ser vista geometricamente como a região esboçada na figura abaixo.

ma211-list6-ex4_sol.png

Essa região pode ser descrita como uma região do tipo II da seguinte forma:
$$\{(x,y) \in \mathbb{R}^2: 0 \leq x \leq y \mbox{ e } 0 \leq y \leq 1\}.$$
Assim,
\begin{array}{rcl}\displaystyle\int_{0}^{1}\!\!\int_{x}^{1}e^{x/y}\,dy dx & = & \displaystyle\int_{0}^{1}\!\!\int_{0}^{y} \! e^{x/y}\,dx dy \\  & = & \displaystyle\int_{0}^{1} \! \left. ye^{x/y} \right|_{x=0}^{x=y}\,dx \\    & = & \displaystyle\int_{0}^{1} \! \left. y(e-1) \right|_{x=0}^{x=y}\,dx  \\    & = & \left.(e-1) \frac{y^2}{2}\right|_{0}^{1} = \frac{e-1}{2}.\end{array}


2454   

Calcule as integrais mudando a ordem de integração de maneira apropriada.

  1.  $\displaystyle\int_{0}^{4}\int_{0}^{1}\int_{2y}^{2}\dfrac{4\;\cos(x^{2})}{2\sqrt{z}}\;dx dy dz$

  2.    $\displaystyle\int_{0}^{1}\int_{0}^{1}\int_{x^{2}}^{1}12xze^{zy^{2}}\;dy dx dz$

  3.  $\displaystyle\int_{0}^{1}\int_{\sqrt[3]{z}}^{1}\int_{0}^{\ln 3}\dfrac{\pi e^{2x}\;\sin(\pi y^{2})}{y^{2}}\;dx dy dz$


  1.  $2 \sin(4).$

  2.  $3e - 6.$

  3.  $4.$


2913   

Suponha que a área de uma região no plano de coordenadas polares seja  $$A=\int_{\pi/4}^{3\pi/4}   \int_{\mathrm{cosec\,}{\theta}}^{2\sin{\theta}}r\,dr d\theta.$$ Esboce a região e encontre sua área.


$A = \dfrac{\pi}{2};$ região:


ma211-list7-ex15_sol.png


1989   

Calcule a integral $\displaystyle\int_{0}^{\pi/2}(3\sin^{2}(t) \cos(t){\bf i}+3\sin(t) \cos^{2}(t){\bf j}+2\sin(t)\cos(t){\bf k})\mathrm{d}t$.


2968   

Calcule a integral, transformando para coordenadas esféricas. $\displaystyle\int_{0}^{1}\int_{0}^{\sqrt{1-x^{2}}}\int_{\sqrt{x^{2}+y^{2}}}^{\sqrt{2-x^{2}-y^{2}}}xy\,dzdydx$.


$\dfrac{(4\sqrt{2} - 5)}{15}.$


3153   

Mude a ordem de integração para mostrar que:
$$ \int_0^a \left[ \int_0^y e^{m(a-x)} f(x) \, dx \right] dy = \int_0^a (a-x) e^{m(a-x)} f(x) \, dx,$$
onde $a$ e $m$ são constantes e $a>0$.


2249   

Seja ${\bf F}(x,y,z)=(x+y+z^{2})\,{\bf k}$ e seja $S$ a fronteira do cilindro $x^{2}+y^{2}\leq 4$ e $0\leq z \leq 3.$ Calcule $\displaystyle\iint \limits_{S}{\bf F}\cdot {\bf n}\,dS$ onde ${\bf n}$ é a normal exterior, isto é, ${\bf n}$ é a normal que aponta para fora do cilindro.


2908   

Utilize coordenadas polares para determinar o volume do sólido dado: delimitado pelo cone $z^2=x^2+y^2$ e pelo cilindro $x^2+y^2=2x.$


$\dfrac{8}{9}.$


2868   

Estude com relação a máximos e mínimos a função dada com as restrições dadas.

$f(x,y) = x^3 + y^3 - 3x - 3y$ e $x + 2y = 3.$


Ponto de máximo local: $\displaystyle \left(- \frac{13}{7}, \frac{17}{7} \right)$; ponto de mínimo local:  $\displaystyle \left( 1,1 \right)$.


2148   

Use o Teorema do Divergente para calcular o fluxo de ${\bf F}$ através de $S,$ onde ${\bf F}(x,y,z)=3xy^{2}\,{\bf i}+xe^{z}\,{\bf j}+z^{3}\,{\bf k}$, $S$ é a superfície do sólido delimitado pelo cilindro $y^{2}+z^{2}=1$ e pelos planos $x=-1$ e $x=2.$


2510   

Ache $\displaystyle\iint \limits_{ S}{\bf F}\cdot {\bf n} dS$ se ${\bf n}$ é uma normal unitária superior de $S.$

  • ${\bf F}=x{\bf i}-y{\bf j}$; $S$ é a parte no primeiro octante da esfera $x^{2}+y^{2}+z^{2}=a^{2}.$


$0.$


2213   

Demonstre as identidades, admitindo que as derivadas parciais apropriadas existem e são contínuas. Se $f$ for um campo escalar e $\mathbf{F}$, $\mathbf{G}$ foram campos vetoriais, então $f\mathbf{F}$, $\mathbf{F} \cdot \mathbf{G}$ e $\mathbf{F} \times \mathbf{G}$ serão definidos por

$$\begin{array}{rcl}(f\mathbf{F})(x,y,z) & = & f(x,y,z)\mathbf{F}(x,y,z) \\(\mathbf{F} \cdot \mathbf{G})(x,y,z) & = & \mathbf{F}(x,y,z) \cdot \mathbf{G}(x,y,z) \\(\mathbf{F} \times \mathbf{G})(x,y,z) & = & \mathbf{F}(x,y,z) \times \mathbf{G}(x,y,z).\end{array}$$

  1. $\text{div }{(\mathbf{F}+\mathbf{G})} = \text{div }{\mathbf{F}}+\text{div }{\mathbf{G}}$.

  2. $\text{div }{(f\mathbf{F})} = f\text{div }{\mathbf{F}} + \mathbf{F} \cdot \nabla{f}$.

  3. $\text{div }{(\mathbf{F} \times \mathbf{G})} = \mathbf{G} \cdot \text{rot }{\mathbf{F}} - \mathbf{F}\cdot\text{rot }{\mathbf{G}}$.

  4. $\text{div }{(\nabla{f} \times \nabla{g})} = 0$.



Suponhamos que ${\bf F}=P_{1}\,{\bf i}+Q_{1}\,{\bf j}+R_{1}\,{\bf k}$ e ${\bf G}=P_{2}\,{\bf i}+Q_{2}\,{\bf j}+R_{2}\,{\bf k}.$

  1. Temos que $F+G=(P_{1}+P_{2})\,{\bf i}+(Q_{1}+Q_{2})\,{\bf j}+(R_{1}+R_{2})\,{\bf k}$. Então,

    $$\begin{array}{rcl}\text{div } ({\bf F}+{\bf G})&=&\frac{\partial(P_{1}+P_{2})}{\partial x}+\frac{\partial(Q_{1}+Q_{2})}{\partial y}+\frac{\partial(R_{1}+R_{2})}{\partial z}\\&=&\frac{\partial P_{1}}{\partial x}+\frac{\partial P_{2}}{\partial x}+\frac{\partial Q_{1}}{\partial y}+\frac{\partial Q_{2}}{\partial y}+\frac{\partial R_{1}}{\partial z}+\frac{\partial R_{2}}{\partial z}\\&=&\underbrace{\frac{\partial P_{1}}{\partial x}+\frac{\partial Q_{1}}{\partial y}+\frac{\partial R_{1}}{\partial z}}+\underbrace{\frac{\partial P_{2}}{\partial x}+\frac{\partial Q_{2}}{\partial y}+\frac{\partial R_{2}}{\partial z}}\\&=&          \text{div } {\bf F}           +          \text{div }{\bf G}.\end{array}$$

  2. Temos que $f{\bf F}=(fP_{1})\,{\bf i}+(fQ_{1})\,{\bf j}+(fR_{1})\,{\bf k}.$ Então,

    $$\begin{array}{rcl}\text{div } (f{\bf F})&=&\frac{\partial(fP_{1})}{\partial x}+\frac{\partial(fQ_{1})}{\partial y}+\frac{\partial(fR_{1})}{\partial z}\\&=&\frac{\partial f}{\partial x}\cdot P_{1}+f\cdot\frac{\partial P_{1}}{\partial x}+\frac{\partial f}{\partial y}\cdot Q_{1}+f\cdot\frac{\partial Q_{1}}{\partial y}+\frac{\partial f}{\partial z}\cdot R_{1}+f\cdot \frac{\partial R_{1}}{\partial z}\\&=&f\cdot\bigg(\underbrace{\frac{\partial P_{1}}{\partial x}+\frac{\partial Q_{1}}{\partial y}+\frac{\partial R_{1}}{\partial z}}\bigg)+\underbrace{\frac{\partial f}{\partial x}P_{1}+\frac{\partial f}{\partial y}Q_{1}+\frac{\partial f}{\partial z}R_{1}}\\&=&          f\cdot     \text{div } {\bf F}+              \nabla f\cdot {\bf F}\end{array}$$

  3. Temos que ${\bf F}\times {\bf G}=(Q_{1}R_{2}-Q_{2}R_{1})\,{\bf i}+(P_{2}R_{1}-P_{1}R_{2})\,{\bf j}+(P_{1}Q_{2}-Q_{1}R_{2})\,{\bf k}.$ Então,

    $$\begin{array}{rcl}\text{div } ({\bf F}\times {\bf G})&=&\frac{\partial(Q_{1}R_{2}-Q_{2}R_{1})}{\partial x}+\frac{\partial(P_{2}R_{1}-P_{1}R_{2})}{\partial y}+\frac{\partial(P_{1}Q_{2}-P_{2}Q_{1})}{\partial z}\\&=&\frac{\partial (Q_{1}R_{2})}{\partial x}-\frac{\partial (Q_{2}R_{1})}{\partial x}+\frac{\partial (P_{2}R_{1})}{\partial y}-\frac{\partial (P_{1}R_{2})}{\partial y}+\frac{\partial (P_{1}Q_{2})}{\partial z}-\frac{\partial (Q_{1}R_{2})}{\partial z}\\&=&\frac{\partial Q_{1}}{\partial x}\cdot R_{2}+Q_{1}\cdot \frac{\partial R_{2}}{\partial x}-\frac{\partial Q_{2}}{\partial x}\cdot R_{1}-Q_{2}\cdot \frac{\partial R_{1}}{\partial x}+\frac{\partial P_{2}}{\partial y}\cdot R_{1}+P_{2}\cdot \frac{\partial R_{1}}{\partial y}\\&-&\frac{\partial P_{1}}{\partial y}\cdot R_{2}-P_{1}\cdot \frac{\partial R_{2}}{\partial y}+\frac{\partial P_{1}}{\partial z}\cdot Q_{2}+P_{1}\cdot \frac{\partial Q_{2}}{\partial z}-\frac{\partial P_{2}}{\partial z}-P_{2}\cdot \frac{\partial Q_{1}}{\partial z}\\&=&P_{1}\bigg(\frac{\partial Q_{2}}{\partial z}-\frac{\partial R_{2}}{\partial y}\bigg)+Q_{1}\bigg(\frac{\partial R_{2}}{\partial x}-\frac{\partial P_{2}}{\partial z}\bigg)+R_{1}\bigg(\frac{\partial P_{2}}{\partial y}-\frac{\partial Q_{2}}{\partial x}\bigg)\\&+&P_{2}\bigg(\frac{\partial R_{1}}{\partial y}-\frac{\partial Q_{1}}{\partial z}\bigg)+Q_{2}\bigg(\frac{\partial P_{1}}{\partial z}-\frac{\partial R_{1}}{\partial x}\bigg)+R_{2}\bigg(\frac{\partial Q_{1}}{\partial x}-\frac{\partial P_{1}}{\partial y}\bigg)\\&=&\bigg[-P_{1}\bigg(\frac{\partial R_{2}}{\partial y}-\frac{\partial Q_{2}}{\partial z}\bigg)-Q_{1}\bigg(\frac{\partial P_{2}}{\partial z}-\frac{\partial R_{2}}{\partial x}\bigg)-R_{1}\bigg(\frac{\partial Q_{2}}{\partial x}-\frac{\partial P_{2}}{\partial y}\bigg)\bigg]\\&+&\bigg[P_{2}\bigg(\frac{\partial R_{1}}{\partial y}-\frac{\partial Q_{1}}{\partial z}\bigg)+Q_{2}\bigg(\frac{\partial P_{1}}{\partial z}-\frac{\partial R_{1}}{\partial x}\bigg)+R_{2}\bigg(\frac{\partial Q_{1}}{\partial x}-\frac{\partial P_{1}}{\partial y}\bigg)\bigg]\\&=&-{\bf F}\cdot \text{rot } {\bf G}+{\bf G}\cdot \text{rot } {\bf F}\\&=&{\bf G}\cdot \text{rot } {\bf F}-{\bf F}\cdot \text{rot } {\bf G}.\end{array}$$

  4. Do item anterior temos que

    $$\begin{array}{rcl}\text{div }(\nabla f\times \nabla g)&=&\nabla g \cdot \text{rot } (\nabla f)-\nabla f \cdot \text{rot } (\nabla g).\\\end{array} $$

    Sabemos que, se $f$ é uma função de três variáveis que tem derivadas parciais de segunda ordem contínuas, então $\text{rot } (\nabla f)={\bf 0}.$ Deste resultado, obtemos que

    $$\text{div }(\nabla f\times \nabla g) =\nabla g \cdot {\bf 0}-\nabla f \cdot {\bf 0}=0.$$



2305   

Identifique e faça um esboço da imagem da superfície parametrizada dada por ${\bf r}(u,v)=(u,v,1-u^{2})$, $u\geq 0$, $v\geq 0$ e $u+v\leq 1.$


${\bf r}(u,v)=(u,v,1-u^{2})$, $u\geq 0$,\, $v\geq 0$ e $u+v\leq 1.$


2298   

Determine uma representação paramétrica para a superfície descrita a seguir. A porção do cilindro $(x-2)^{2}+z^{2}=4$ entre os planos $y=0$ e $y=3.$


$x = 4\cos^{2}(v),$ $y = u,$ $z = 4\cos(v)\sin(v),$ onde $-\dfrac{\pi}{2}\leq v \leq \dfrac{\pi}{2}$ e $0 \leq u \leq 3.$


3131   

Use o Teorema de Green para determinar o trabalho realizado pelo campo de forças \(\displaystyle\mathbf{F}(x,y)=\sqrt{y}\textbf{i}+\sqrt{x}\textbf{j}\) sobre uma partícula que percorre uma vez, no sentido anti-horário, a curva fechada dada pelas equações \(y=0\), \(x=2\) e \(y=x^3/4\).


2372   

Seja
$$f(x,y) = x - y\sin{(\pi(x^2 + y^2))}.$$

  1. Calcule a derivada direcional de $f$ no ponto $(0,0)$ na direção de $\bf{v} = (1/2,\sqrt{3}/2)$.

  2. Em que direção a taxa de variação de $f$ no ponto $(0,0)$ é máxima? Qual é o valor da taxa máxima nesse ponto?


  1. $ \displaystyle \frac{1}{2}.$
  2.  Na direção do vetor $(1,0).$ O valor da taxa máxima é $1.$


2039   

Utilize um diagrama em árvore para escrever a Regra da Cadeia para o caso dado. Suponha que todas as funções sejam diferenciáveis.

$t=f(u,v,w)$, onde $u=u(p,q,r,s)$, $v=v(p,q,r,s)$, $w=w(p,q,r,s)$.


$\displaystyle \frac{\partial t}{\partial p} = \frac{\partial t}{\partial u}\frac{\partial u}{\partial p} + \frac{\partial t}{\partial v}\frac{\partial v}{\partial p} + \frac{\partial t}{\partial w}\frac{\partial w}{\partial p},$ $\displaystyle \frac{\partial t}{\partial q} = \frac{\partial t}{\partial u}\frac{\partial u}{\partial q} + \frac{\partial t}{\partial v}\frac{\partial v}{\partial q} + \frac{\partial t}{\partial w}\frac{\partial w}{\partial q},$

$\displaystyle \frac{\partial t}{\partial r} = \frac{\partial t}{\partial u}\frac{\partial u}{\partial r} + \frac{\partial t}{\partial v}\frac{\partial v}{\partial r} + \frac{\partial t}{\partial w}\frac{\partial w}{\partial r}$ e $\displaystyle \frac{\partial t}{\partial s} = \frac{\partial t}{\partial u}\frac{\partial u}{\partial s} + \frac{\partial t}{\partial v}\frac{\partial v}{\partial s} + \frac{\partial t}{\partial w}\frac{\partial w}{\partial s}.$


2954   

Usando coordenadas esféricas, determine o volume do elipsoide $\dfrac{x^{2}}{a^{2}}+\dfrac{y^{2}}{b^{2}}+\dfrac{z^{2}}{c^{2}}\leq 1.$


$\dfrac{4 \pi abc}{3}.$


2457   

Considere a função

$$f(x,y)=\sqrt{x+y^{2}-3}$$

  1. Faça um esboço das curvas de nível de $f$ nos níveis $c=0$, $c=1$ e $c=3.$
  2. Quantas curvas de nível de $f$ passam pelo ponto $(3,-1)$?


  1. As curvas de níveis de $f$ são

    $$\sqrt{x+y^{2}-3}=c\,\,\,\,\mbox{ou}\,\,\,\,x+y^{2}-3=c^2\,\,\,\,\mbox{ou}\,\,\,\,x=3+c^2-y^{2},$$

    ou seja, uma família de parábolas com concavidade para a esquerda. As três curvas de níveis pedidas, obtidas considerando respectivamente $c=0$, $c=1$ e $c=3$, são

    $x=3-y^{2}$, $x=4-y^{2}$ e $x=12-y^{2}.$ Elas estão apresentadas na figura abaixo.

    ma211-list2-ex2.png

  2. Pelo ponto $(3,-1)$ passa uma única curva de nível, isto é, $f(x,y)=1.$ Pois caso contrário o ponto $(3,-1)$ teria duas alturas diferentes, o que é impossível.


1935   

Calcule a integral de linha, onde $C$ é a curva dada.

$\displaystyle\int_{C}xy\,dx+(x-y)\,dy$,   $C$ consiste nos segmentos de reta de $(0,0)$ a $(2,0)$ e de $(2,0)$ a $(3,2).$


$\displaystyle \frac{17}{3}.$


2123   

Suponha que, para todo $t$, $f(t^{2},2t)=t^{3}-3t$. Mostre que 
$$\dfrac{\partial f}{\partial x}(1,2)=-\dfrac{\partial f}{\partial y}(1,2).$$


Tome $t = 1$ em $\displaystyle \frac{df}{dt}(t^{2},2t) = 2t \frac{\partial f}{\partial x}(t^{2},2t) + 2\frac{\partial f}{\partial y}(t^{2},2t) = 3t^{2} - 3.$


3028   

Esboce a região de integração e mude a ordem de integração. $\displaystyle\int_{0}^{4}\!\!\int_{0}^{\sqrt{x}} \! f(x,y)\,dy dx$.


ma211-list6-ex32_sol_a.png

2422   

Calcule a integral iterada.

  1.  $\displaystyle\int_{0}^{1}\!\!\int_{0}^{z}\!\!\int_{0}^{x+z}6xz\;dy dx dz$

  2.  $\displaystyle\int_{0}^{3}\!\!\int_{0}^{1}\!\!\int_{0}^{\sqrt{1-z^{2}}}ze^{y}\;dx dz dy$

  3.  $\displaystyle\int_{0}^{\pi/2}\int_{0}^{y}\int_{0}^{x}\cos(x+y+z)\;dz dx dy$


  1.  $1.$

  2.  $\displaystyle \frac{e^3 - 1}{3}.$

  3.  $-\dfrac{1}{3}.$


2681   

Encontre $\partial f/\partial x$ e $\partial f/\partial y$ para $f(x,y)=(xy-1)^{2}$.


$\displaystyle \frac{\partial f}{\partial x} = 2y(xy - 1)\;\;\;\;\text{e}\;\;\;\; \frac{\partial f}{\partial y} = 2x (xy - 1)$.


3044   

Calcule a área sob um arco da cicloide $x = t-\sin{t}$, $y = 1-\cos{t}$.


Queremos determinar a área da região $R$ mostrada na figura abaixo.

ma211-list12-ex4_sol_a.png

Sabemos que, se $y = f(x)$, então a integral $\int_{a}^{b}f(x)dx$ calcula a área que está abaixo do gráfico de $f$ e acima do eixo $x$, com $x$ variando entre $a$ e $b$. A princípio, poderíamos tentar encontrar uma expressão que relacionasse $x(t)$ e $y(t)$ na parametrização da cicloide, mas esse parece ser um trabalho difícil. Usaremos então o que foi provado no exercício anterior. Temos que
$$A(R) = \oint_{C}x\, dy,$$
em que $C = C_1 \cup C_2$ é a curva descrita na figura a seguir.

ma211-list12-ex4_sol_b.png

Uma parametrização de $C_1$ é $r_1(t) = (x_1(t),y_1(t)) = (t,0)$, em que $0 \leq t \leq 2\pi$. Nesse caso, $y_1'(t) = 0$. Logo,
$$\oint_{C_1}x\, dy = \int_{0}^{2\pi}(t)(0)\, dt = 0.$$
Uma parametrização de $C_2$ é $r_2(t) = (x_2(t),y_2(t)) = (t-\sin{t},1-\cos{t})$, em que $t$ varia de $2\pi$ a $0$. Nesse caso, $y_2'(t) = \sin{t}$. Logo,
$$\begin{array}{rcl}\displaystyle \oint_{C_2}x\, dy & = & \displaystyle \int_{2\pi}^{0}(t-\sin{t}) (\sin{t})\, dt \\ & = & \displaystyle \int_{0}^{2\pi}(\sin^2{t} - t\sin{t}) \, dt \\ & = & \displaystyle \int_{0}^{2\pi}\frac{1-\cos(2t)}{2}\, dt - \int_{0}^{2\pi}t\sin{t} \, dt \\ & = & \pi + 2\pi = 3\pi.\end{array}$$
Portanto, a área da região é $3\pi$.
(Observe que, para resolver a integral $\int_{0}^{2\pi}t\sin{t} \, dt$, usamos integração por partes com $u=t$ e $dv=\sin{t}\,dt$.)


2462   

Calcule a integral de superfície $\displaystyle\iint \limits_{ S}{\bf F}\cdot d{\bf S}$ para o campo vetorial ${\bf F}$ e superfície orientada $S$ dados abaixo. Em outras palavras, determine o fluxo de ${\bf F}$ através de $S$. Para superfícies fechadas, use a orientação positiva (para fora).

  • ${\bf F}(x,y,z)=xze^{y}{\bf i}-xze^{y}{\bf j}+z{\bf k}$ e $S$ é a parte do plano $x+y+z=1$ no primeiro octante, com orientação para baixo.


$-\dfrac{1}{6}.$


2110   

Sejam $f(x)$ e $g(x)$ duas funções contínuas, respectivamente, nos intervalos $[a,b]$ e $[c,d].$ Use o seguinte resultado $$\iint\limits_{R}f(x)g(y)\,dx dy=\bigg(\int_{a}^{b}f(x)\,dx\bigg)\bigg(\int_{c}^{d}g(y)\,dy\bigg),$$ onde $R$ é o retângulo $a\leq x\leq b$ e $c\leq y\leq d$, para calcular as integrais

  1. $\displaystyle\iint\limits_{R} \dfrac{\sin^{2}{x}}{1+4y^{2}}\,dx dy$, onde $R$ é o retângulo $0\leq x\leq \dfrac{\pi}{2},\;0\leq y\leq \dfrac{1}{2}.$

  2. $\displaystyle\iint\limits_{R} \dfrac{xy\sin{x}}{1+4y^{2}}\,dx dy$, onde $R$ é o retângulo $0\leq x\leq \dfrac{\pi}{2},\;0\leq y\leq 1.$


  1.  $\dfrac{\pi^{2}}{32}.$

  2.  $\dfrac{\ln(5)}{8}.$


2170   

Use o Teorema de Green para achar o trabalho realizado pela força $\mathbf{F}(x,y) = x(x+y)\mathbf{i} + xy^2\mathbf{j}$ ao mover uma partícula da origem ao longo do eixo $x$ até $(1,0)$, em seguida ao longo de um segmento de reta até $(0,1)$ e então de volta à origem ao longo do eixo $y$.


$-\dfrac{1}{12}.$


3114   

A tendência de uma lâmina de resistir a uma mudança no seu movimento de rotação em torno de um eixo é medida pelo seu momento de inércia em torno daquele eixo. Se a lâmina ocupar uma região \(R\) do plano \(xy\) e se sua densidade \(\delta(x,y)\) for uma função contínua em \(R\), então os momentos de inércia em torno dos eixos \(x\), \(y\) e \(z\) são denotados por \(I_x\), \(I_y\) e \(I_z\), respectivamente, e são definidos por \begin{align*} I_x & = \iint\limits_R y^2\delta(x,y)\,dA, \\ I_y & = \iint\limits_R x^2\delta(x,y)\,dA, \\ I_z & = \iint\limits_R (x^2+y^2)\delta(x,y)\,dA. \\ \end{align*} Considere a lâmina circular que ocupa a região descrita pelas desigualdades \(0\leq x^2+y^2\leq a^2\). Supondo que a lâmina tenha densidade \(\delta\) constante, mostre que \[  I_x= I_y=\dfrac{\delta\pi a^4}{4}, \quad  I_z= \dfrac{\delta\pi a^4}{2}.\]


3035   

Uma região $R$ é mostrada na figura abaixo. Decida se você deve usar coordenadas polares ou retangulares e escreva $\iint\limits_{R}f(x,y)\,dA$ como uma integral iterada, onde $f$ é uma função qualquer contínua em $R.$


ma211-list7-ex5_b.png



$\displaystyle \int_{-1}^{1} \int_{0}^{1 - x^2} f(x,y)  dy dx .$


2286   

Identifique a superfície que tem equação paramétrica ${\bf r}(u,v)=2\,\sin u\,{\bf i}+3\,\cos u\,{\bf j}+v\,{\bf k}$, $0\leq v\leq 2.$.


$\dfrac{x^2}{4} + \dfrac{y^{2}}{9} = 1,$ com $0\leq z \leq 2.$


2413   

Determine o volume do sólido que está abaixo do paraboloide elíptico $x^{2}/4+y^{2}/9+z=1$ e acima do retângulo $R=[-1,1]\times [-2,2].$


$\dfrac{166}{27}.$


2326   

Calcule a área da superfície dada por: ${\bf r}(u,v)=\bigg(u,v,\dfrac{1}{2}u^{2}\bigg)$,$0\leq v\leq u$ e $u\leq 2.$. (Sugerimos ao leitor desenhar a imagem da superfície.)


$\dfrac{1}{3}\left(5\sqrt{5} - 1  \right).$


2591   

Seja ${\bf F}$ um campo inverso do quadrado, ou seja, ${\bf F}(r)=cr/|r|^{3}$ para alguma constante $c$, onde $r=x{\bf i}+y{\bf j}+z{\bf k}.$ Mostre que o fluxo de ${\bf F}$ por uma esfera $S$ com centro na origem é independente do raio de $S.$


$\displaystyle \iint\limits_{S}{\bf F}\cdot d \bf S = 4\pi c.$


2408   

Inverta a ordem de integração.

  1. $\displaystyle\int_{0}^{1}\bigg[\int_{e^{y-1}}^{e^{y}}f(x,y)\,dx\bigg]dy$

  2.  $\displaystyle\int_{0}^{1}\bigg[\int_{2x}^{x+1}f(x,y)\,dy\bigg]dx$

  3. $\displaystyle\int_{0}^{\frac{\pi}{4}}\bigg[\int_{0}^{\tan(x)}f(x,y)\,dy\bigg]dx$


  1.  $\displaystyle\int_{e^{-1}}^{1}\bigg[\int_{0}^{1 + \ln(x)}f(x,y) \ , dy\bigg]dx + \displaystyle\int_{1}^{e}\bigg[\int_{\ln(x)}^{1}f(x,y)\,dy\bigg]dx$

  2.  $\displaystyle\int_{0}^{1}\bigg[\int_{0}^{y/2}f(x,y)\,dx\bigg]dy + \int_{1}^{2}\bigg[\int_{y - 1}^{y/2}f(x,y)\,dx\bigg]dy$

  3.  $\displaystyle \int_{0}^{1}\bigg[\int_{0}^{\arctan(y)}f(x,y)\,dx \bigg]dy $


3156   

Seja $S$ uma superfície plana paralela ao plano $xy$. Mostre que a fórmula para o cálculo de áreas de superfícies nesse caso reduz à fórmula de integrais duplas para o cálculo de área de regiões planas.


2049   


Determine se o conjunto $\{(x,y)|\,1<x^{2}+y^{2}<4\}$ é ou não:

  1. aberto;

  2. conexo; e

  3. simplesmente conexo.



Temos que o conjunto $D=\{(x,y)|\,1<x^{2}+y^{2}<4\}$ representa a região anelar entre os círculos com centro $(0,0)$ e raio $1$ e $2$. Então:

  1. $D$ é aberto pois, em torno de cada ponto em $D$, podemos colocar um disco que se encontra inteiramente em $D$.

  2. $D$ é conexo pois quaisquer dois pontos de $D$ podem ser conectados por um caminho em $D$.

  3. $D$ não é simplesmente conexo pois,  por exemplo, a região delimitada pela curva simples e fechada $x^{2}+y^{2}=(3/2)^2$ possui pontos que não estão em $D$, por exemplo, a origem $(0,0)$.


3071   

Seja ${\bf F}(x,y)=(e^{x}\,\cos y+y, x-e^{x}\,\sin y)$. Calcule $\int_{C}{\bf F}\cdot d{\bf r}$, onde $C$ é o arco de circunferência que une o ponto $(-\sqrt{2}/2, \sqrt{2}/2)$ ao ponto $(1,0)$. Veja a figura abaixo.

ma211-list11-ex2.png



Notemos que ${\bf F}$ é um campo vetorial conservativo, pois: ${\bf F}$ é definido em todo $\mathbb{R}^{2}$; $P(x,y)=e^{x}\,\cos y+y$ e $Q(x,y)=x-e^{x}\,\sin y$ possuem derivadas parciais de primeira ordem contínuas;  $\dfrac{\partial P}{\partial y}(x,y)=1-e^{x}\,\sin y=\dfrac{\partial Q}{\partial x}(x,y).$

Sendo $F$ conservativo, existe $f$ tal que $\nabla f={\bf F}.$ Vamos encontrar $f$. Temos que 

$$f_{x}(x,y)=P(x,y)     \mbox{ e }      f_{y}(x,y)=Q(x,y).$$

Então,

$$\label{(2)}f_{x}(x,y)=e^{x}\,\cos y+y\Rightarrow f(x,y)=e^{x}\,\cos y+y+g(y).$$

Logo,  temos que

$$f_{y}(x,y)=-e^{x}\,\sin y+x+g'(y).$$

Como $f_{y}(x,y)=Q(x,y)$, obtemos que

$$-e^{x}\,\sin y+x+g'(y)=x-e^{x}\,\sin y\Rightarrow g'(y)=0\Rightarrow g(y)=C.$$

Assim, tomando $C=0$ segue que

$$f(x,y)=e^{x}\,\cos y+xy.$$

Do resultado acima e pelo Teorema Fundamental da Integral de Linha, temos que

$$\int_{C}{\bf F}\cdot d{\bf r}=f(1,0)-f\bigg(-\frac{\sqrt{2}}{2},\frac{\sqrt{2}}{2}\bigg)=e-e^{-\frac{\sqrt{2}}{2}}\,\cdot\cos\bigg(\frac{\sqrt{2}}{2}\bigg)+\frac{1}{2}.$$


2352   

Calcule o volume do conjunto dado.

  1.  $0\leq y\leq 1-x^{2}$ e $0\leq z\leq 1-x^{2}.$

  2.  $x^{2}+y^{2}+3\leq z\leq 4.$


  1.  $\dfrac{16}{15}.$

  2.  $\dfrac{\pi}{2}.$


2777   

Esboce a região cuja área é dada pela integral $\displaystyle\int_{\pi}^{2\pi}  \int_{4}^{7}  r\, dr d\theta$ e calcule-a:


$\displaystyle \frac{33\pi}{2};$ região de integração:

ma211-list7-ex6_sol_a.png



2384   

Calcule a integral iterada.

  1.  $\displaystyle\int_{0}^{1}\!\!\int_{0}^{1}(u-v)^{5}\,du dv$

  2. $\displaystyle\int_{0}^{2}\!\!\int_{0}^{\pi}r\sin^{2}{\theta}\,d\theta dr$


  1. $0.$

  2. $\pi.$


2205   

Suponha que $w=f(x,y)$ é diferenciável e que exista uma constante $\alpha$ tal que 
$x=u\cos(\alpha)-v\sin(\alpha)$
$y=u\sin(\alpha)+v\cos(\alpha).$
Mostre que 
$$\bigg(\frac{\partial w}{\partial u}\bigg)^{2}+\bigg(\frac{\partial w}{\partial v}\bigg)^{2}=\bigg(\frac{\partial w}{\partial x}\bigg)^{2}+\bigg(\frac{\partial w}{\partial y}\bigg)^{2}.$$


Note que $\displaystyle \frac{\partial w}{\partial u} = \cos(\alpha) \frac{\partial w}{\partial x} + \sin(\alpha) \frac{\partial w}{\partial y}$ e $\displaystyle \frac{\partial w}{\partial v} = -\sin(\alpha) \frac{\partial w}{\partial x} + \cos(\alpha) \frac{\partial w}{\partial y}.$


2966   

Dentre as coordenadas cilíndricas ou esféricas, utilize a que lhe parecer mais apropriada para determinaretermine o volume da região limitada abaixo pelo plano $z=0$, lateralmente pelo cilindro $x^{2}+y^{2}=1$ e acima pelo paraboloide $z=x^{2}+y^{2}$.


$\dfrac{\pi}{2}.$


2234   

Seja $\mathbf{r} = x\mathbf{i} + y\mathbf{j} + z\mathbf{k}$ e $r=|\mathbf{r}|$. Verifique a identidade $\nabla{r} = \dfrac{\mathbf{r}}{r}$.


$\nabla{r} = \left(\dfrac{x}{\sqrt{x^2 + y^2 + z^2}},\dfrac{y}{\sqrt{x^2 + y^2 + z^2}} ,\dfrac{z}{\sqrt{x^2 + y^2 + z^2}} \right).$ (Note que: $r = \sqrt{x^{2} + y^{2} + z^{2}}.$)


2324   

Calcule a área da superfície dada por: ${\bf r}(u,v)=(u,v,u^{2}+v^{2})$ e $u^{2}+v^{2}\leq 4.$. (Sugerimos ao leitor desenhar a imagem da superfície.)


$\dfrac{\pi}{6}(17 \sqrt{17} - 1).$


2292   

Determine uma representação paramétrica para a superfície descrita a seguir. A parte do plano $z=x+3$ que está dentro do cilindro $x^{2}+y^{2}=1.$


$x = r \cos(\theta),$ $y = r \sin(\theta),$ $z = 3 + r \cos(\theta),$ onde $0 \leq r \leq 1$ e $0\leq \theta \leq 2\pi.$


2809   

Determine os valores máximos e mínimos locais e pontos de sela da função $f(x,y)=4+x^{3}+y^{3}-3xy$.


Ponto de mínimo: $(1,1);$ ponto de sela: $(0,0).$


2289   

Determine uma representação paramétrica para a superfície descrita a seguir. A parte do paraboloide elíptico $x+y^{2}+2z^{2}=4$ que está em frente ao plano $x=0.$


$y = u,$ $z = v,$ $x = 4 - u^2 - 2v^2,$ onde $u^{2} + 2v^2 \leq 4.$


1983   

Nos itens abaixo: 

  • Esboce o gráfico da curva plana com a equação vetorial dada; 

  • Determine ${\bf r}'(t)$; 

  • Esboce o vetor posição ${\bf r}(t)$ e o vetor tangente ${\bf r}'(t)$ para o valor dado de  $t.$

  1. ${\bf r}(t)=(t-2,t^{2}+1)$ e $t=-1.$

  2. ${\bf r}(t)=\sin(t){\bf i}+2\; \cos(t){\bf j}$ e $t=\pi/4.$

  3. ${\bf r}(t)=(1+\cos{t}){\bf i}+(2+\sin{t}){\bf j}$ e $t=\pi/6.$


2375   

A temperatura em um ponto $(x,y,z)$ é dada por

$$T(x,y,z) = 200e^{-x^2 - 3y^2 - 9z^2},$$

em que $T$ é medido em °C  e $x,y$ e $z$ em metros.

  1.  Determine a taxa de variação da temperatura no ponto $P = (2,-1,2)$ em direção ao ponto $(3,-3,3)$.
  2.  Qual é a direção de maior crescimento da temperatura em $P$?
  3.  Encontre a taxa máxima de crescimento em $P$.


  1.  $\displaystyle \frac{5200\sqrt{6}}{3e^{43}}$ ºC/m.
  2.  $400 e^{-43} (-2,3,-18).$
  3.  $400 e^{-43}\sqrt{337}$ ºC/m.


2088   

Calcule $\displaystyle\int_{C}\dfrac{-y}{x^{2}+y^{2}}\,dx+\dfrac{x}{x^{2}+y^{2}}\,dy$ onde $C:[0,1]\rightarrow \mathbb{R}^{2}$ é uma curva de classe $C^{1}$ por partes, com imagem contida no semiplano $y>0$, tal que $C(0)=(1,1)$ e $C(1)=(-2,3).$


$\pi.$


2583   

  1. Defina continuidade de uma função de duas variáveis $f(x,y)$ em um ponto $(x_0, y_0)$ de seu domínio.

  2. Dada a função

    $$f(x,y) = \begin{cases} \dfrac{x^2\sqrt{y}}{x^2 + y^2}, & \quad \text{se } (x,y) \neq (0,0), \\L, & \quad \text{se } (x,y) = (0,0),\end{cases}$$

    é possível encontrar $L$ de maneira que $f$ seja contínua em $(0,0)$?


  1. $f(x,y)$ é contínua em $(x_{0},y_{0}) \in D_{f}$ se

    $$\lim_{(x,y) \to (x_{0},y_{0})} f(x,y) = f(x_{0},y_{0}).$$

  2. $L = 0.$


2405   

Inverta a ordem de integração.

  1.  $\displaystyle\int_{0}^{1}\bigg[\displaystyle\int_{0}^{x}f(x,y)\,dy\bigg]dx$

  2.  $\displaystyle\int_{0}^{1}\bigg[\int_{x^{2}}^{x}f(x,y)\,dy\bigg]dx$

  3.  $\displaystyle\int_{0}^{1}\bigg[\int_{-\sqrt{y}}^{\sqrt{y}}f(x,y)\,dx\bigg]dy$


  1.  $\displaystyle\int_{0}^{1}\bigg[\displaystyle\int_{y}^{1}f(x,y)\,dx\bigg]dy$

  2.  $\displaystyle\int_{0}^{1}\bigg[\int_{y}^{\sqrt{y}}f(x,y)\,dx\bigg]dy$

  3.  $\displaystyle\int_{-1}^{1}\bigg[\int_{x^2}^{1}f(x,y)\,dy\bigg]dx$


2567   

Sabendo que $\left|\sin\frac{1}{x}\right| \leq 1$, podemos dizer algo sobre

$$\displaystyle \lim_{(x,y) \to (0,0)}y\sin\dfrac{1}{x}?$$

Justifique sua resposta.


$\displaystyle \lim_{(x,y) \to (0,0)} y\sin\left(\frac{1}{x} \right) = 0.$


2717   

Explique por que a função é diferenciável no ponto dado. A seguir, encontre a linearização $L(x,y)$ da função naquele ponto. $f(x,y) = \dfrac{x}{x+y}, \quad (2,1)$.


As derivadas $f_{x}$ e $f_{y}$ de cada $f$ existem e são contínuas no ponto dado, logo $f$ é diferenciável.

$L(x,y) = \frac{1}{9}x - \frac{2}{9}y + \frac{2}{3}$.


2253   

Demonstre a identidade abaixo, supondo que $S$ e $E$ satisfaçam as condições do Teorema do Divergente e que as funções escalares e as componentes dos campos vetoriais tenham derivadas parciais de segunda ordem contínuas.


  1. $V(E)=\dfrac{1}{3}\displaystyle\iint\limits_{S}{\bf F}\cdot dS$, onde ${\bf F}(x,y,z)=x\,{\bf i}+y\,{\bf j}+z\,{\bf k}.$



Dica: Note que $\displaystyle\iiint\limits_{E}{\mbox{div} {\bf F}}\, dV = \iiint \limits_{E}{3}\,dV$.


2634   

Considere a função dada por $z=x \sin\bigg(\dfrac{x}{y}\bigg).$ Verifique que

$$x\;\dfrac{\partial z}{\partial x}+y\;\dfrac{\partial z}{\partial y}=z.$$



Primeiramente, vamos calcular $\dfrac{\partial z}{\partial x}$ e $\dfrac{\partial z}{\partial }.$ Assim,\\

$\bullet $ $\dfrac{\partial z}{\partial x}=$ $\dfrac{\partial}{\partial x}\bigg[x\cdot \sin\bigg(\dfrac{x}{y}\bigg)\bigg]=

1\cdot \sin\bigg(\dfrac{x}{y}\bigg)+x\cdot \cos \bigg(\dfrac{x}{y}\bigg)\cdot \dfrac{1}{y}$

$$=\sin\bigg(\frac{x}{y}\bigg)+\frac{x}{y}\cdot \cos\bigg(\frac{x}{y}\bigg)$$

$\bullet $ $\dfrac{\partial z}{\partial y}=$ $\dfrac{\partial}{\partial y}\bigg[x\cdot \sin\bigg(\dfrac{x}{y}\bigg)\bigg]=

0\cdot \sin\bigg(\dfrac{x}{y}\bigg)+x\cdot \cos \bigg(\dfrac{x}{y}\bigg)\cdot \bigg(-\dfrac{x}{y^{2}}\bigg)$

$$=-\frac{x^{2}}{y^{2}}\cdot \cos\bigg(\frac{x}{y}\bigg).$$

Então,

$$x\cdot \frac{\partial z}{\partial x}+y\frac{\partial z}{\partial y}=x\cdot \bigg[\sin\bigg(\frac{x}{y}\bigg)+\frac{x}{y}\cdot \cos\bigg(\frac{x}{y}\bigg)\bigg] +

y\cdot\bigg[ -\frac{x^{2}}{y^{2}}\cdot \cos\bigg(\frac{x}{y}\bigg)\bigg]$$

$$=x\cdot \sin\bigg(\frac{x}{y}\bigg)+\frac{x^{2}}{y}\cos\bigg(\frac{x}{y}\bigg)-\frac{x^{2}}{y}\cdot \cos\bigg(\frac{x}{y}\bigg)$$

$$x\cdot \sin\bigg(\frac{x}{y}\bigg)=z.$$


3066   

  1. Esboce o campo vetorial $\textbf{F}(x,y) = \textbf{i} + x\textbf{j}$ e algumas linhas de escoamento. Qual é o formato que essas linhas de escoamento parecem ter?
  2. Se as equações paramétricas das linhas de escoamento são $x=x(t)$ e $y=y(t)$, que equações diferenciais essas funções satisfazem? Deduza que $dy/dx = x$.
  3. Se uma partícula está na origem no instante inicial e o campo de velocidade é dado por $\textbf{F}$, determine uma equação para a trajetória percorrida por ela.

2195   

Seja ${\bf F}=(z tg^{-1}(y^{2}),z^{3}\ln(x^{2}+1),z).$ Determine o fluxo de ${\bf F}$ através da parte do parabolóide $x^{2}+y^{2}+z=2$ que está acima do plano $z=1$ e está orientada para cima. (Observe que a superfície acima não é fechada.)


2142   

Verifique que o Teorema do Divergente é verdadeiro para o campo vetorial ${\bf F}$ na região $E.$

${\bf F}(x,y,z)=x^{2}\,{\bf i}+xy\,{\bf j}+z\,{\bf k}$, $E$ é o sólido delimitado pelo paraboloide $z=4-x^{2}-y^{2}$ e pelo plano $xy.$


$\displaystyle\iint_{S} {\bf F} \cdot d{\bf S} = \iiint_{E} \mbox{div} {\bf F} dV = 8\pi.$

3115   

Seja \(G\) a caixa retangular definida pelas desigualdades \(a\leq x\leq b\),  \(c\leq y\leq d\) e \(k\leq z\leq l\). Mostre que \[\iiint\limits_G f(x)g(y)h(z)\,dV = \left[\int_a^bf(x)\,dx\right]\left[\int_c^dg(y)\,dy\right]\left[\int_k^lh(z)\,dz\right].\]


3140   

Prove a seguinte identidade \[ \iint\limits_\sigma\nabla f\cdot\mathbf{n}\,dS = \iiint\limits_G\Delta f\,dV, \] supondo que \(\sigma\) e \(G\) satisfaçam as hipóteses do Teorema da Diverência e que \(f(x,y,z)\) cumpra os requisitos de diferenciabilidade necessários. Acima, \(\displaystyle \Delta f= \dfrac{\partial^2f}{\partial x^2}+\dfrac{\partial^2f}{\partial y^2}+\dfrac{\partial^2f}{\partial z^2}\) é denominado Laplaciano de \(f\).


2731   

Determine as equações do plano tangente e da reta normal ao gráfico da função dada, no ponto dado. $f(x,y) = 3x^3y - xy$ em $(1,-1,f(1,-1))$.


Plano tangente: $z = -8x + 2y + 8$

Reta normal: $(x,y,z) = \left(1,-1,-2 \right) + \lambda \left(-8,2,-1 \right)$.


2054   

Determine se ${\bf F}(x,y)=e^{x}\,\cos y\,{\bf i}+e^{x}\,\sin y\,{\bf j}.$ é ou não um campo vetorial conservativo. Se for, determine uma função $f$ tal que ${\bf F}=\nabla f.$


Não.


2641   

Determine as derivadas parciais de primeira ordem da função $u=te^{w/t}$.


$\displaystyle \frac{\partial u}{\partial t} = e^{w/t} \left( 1 - \frac{w}{t} \right)\;\;\;\text{e}\;\;\; \frac{\partial u}{\partial w} = e^{w/t}$.


2691   

Encontre $f_{x}$, $f_{y}$ e $f_{z}$ para $f(x,y,z)=\ln(x+2y+3z)$.


$\displaystyle f_{x} = \frac{1}{x + 2y + 3z},\;\;\;\; f_{y} = \frac{2}{x + 2y + 3z}\;\;\;\;\text{e}\;\;\;\; f_{z} = \frac{3}{x + 2y + 3z}$.


2709   

Determine a aproximação linear da função $f(x,y,z) = \sqrt{x^2 + y^2 + z^2}$ em $(3,2,6)$ e use-a para aproximar o número $\sqrt{(3,02)^2 + (1,97)^2 + (5,99)^2}$.


Vamos determinar a aproximação  linear da função $f$ em $(3,2,6)$. Primeiramente, calculamos as derivadas parcias $f_{x}$, $f_{y}$ e $f_{z}$, para todo $(x,y,z).$
$\bullet f_{x}(x,y,z)=\dfrac{1}{2}(x^{2}+y^{2}+z^{2})^{-1/2}\cdot 2x=\dfrac{x}{\sqrt{x^{2}+y^{2}+z^{2}}}.$
$\bullet f_{y}(x,y,z)=\dfrac{1}{2}(x^{2}+y^{2}+z^{2})^{-1/2}\cdot 2y=\dfrac{y}{\sqrt{x^{2}+y^{2}+z^{2}}}.$
$\bullet f_{z}(x,y,z)=\dfrac{1}{2}(x^{2}+y^{2}+z^{2})^{-1/2}\cdot 2z=\dfrac{z}{\sqrt{x^{2}+y^{2}+z^{2}}}.$
Agora, calculamos as derivadas parciais de $f$ no ponto $(3,2,6)$, então
$\bullet f_{x}(3,2,6)=\dfrac{3}{\sqrt{3^{2}+2^{2}+6^{2}}}=\dfrac{3}{7}.$
$\bullet f_{x}(3,2,6)=\dfrac{2}{\sqrt{3^{2}+2^{2}+6^{2}}}=\dfrac{2}{7}.$
$\bullet f_{x}(3,2,6)=\dfrac{6}{\sqrt{3^{2}+2^{2}+6^{2}}}=\dfrac{6}{7}.$
Assim, a aproximação linear da função $f$ em $(3,2,6)$ é
\begin{array}{rcl}f(x,y,z)&\approx & f(3,2,6)+f_{x}(3,2,6)(x-3)+f_{y}(3,2,6)(y-2)+f_{z}(3,2,6)(z-6)\\&=&7+\dfrac{3}{7}(x-3)+\frac{2}{7}(y-2)+\frac{6}{7}(z-6)\\&=&\dfrac{3}{7}x+\frac{2}{7}y+\frac{6}{7}z+\bigg(7-\dfrac{9}{7}-\dfrac{4}{7}-\dfrac{36}{7}\bigg)\\&=&\dfrac{3}{7}x+\frac{2}{7}y+\frac{6}{7}z.\end{array}
Agora, vamos aproximar o número $\sqrt{(3,02)^2 + (1,97)^2 + (5,99)^2}.$ Assim,
\begin{array}{rcl}\sqrt{(3,02)^2 + (1,97)^2 + (5,99)^2}&=&f(3,02\,,\,1,97\,,\,5,99)\\&\approx& \frac{3}{7}(3,02)+\frac{2}{7}(1,97)+\frac{6}{7}(5,99)\\&\approx& 6,9914.\end{array}


2432   

Calcule a integral tripla.

  1.  $\displaystyle\iiint\limits_{  E}y\;dx dy dz$, onde $E$ é o conjunto $x^{2}+4y^{2}\leq 1$ e $0\leq z \leq 1.$

  2.  $\displaystyle\iiint\limits_{  E}x\;dx dy dz$, onde $E$ é o conjunto $x^{2}+y^{2}\leq 4$, $x\geq 0$ e \\ $x+y\leq z \leq x+y+1.$


  1.  $0.$

  2.  $\dfrac{16}{3}.$


2050   

Determine se o conjunto $\{(x,y)|\,x^{2}+y^{2}\leq 1\,$ ou\, $4\leq x^{2}+y^{2}\leq 9\}$ é ou não:
  1. aberto;

  2. conexo; e

  3. simplesmente conexo.



Temos que o conjunto $D=\{(x,y)|\,x^{2}+y^{2}\leq 1\,$ ou\, $4\leq x^{2}+y^{2}\leq 9\}$ consiste dos pontos que estão sobre ou dentro do círculo $x^{2}+y^{2}\leq 1$ juntamente com os pontos que estão em ou entre os círculos $x^{2}+y^{2}=4$ e $x^{2}+y^{2}=9$.

  1. $D$ não é aberto pois qualquer disco centrado em $(0,1)$ contém pontos que não estão em $D$.

  2. $D$ não é conexo pois não existe um caminho em $D$ conectando, por exemplo, os pontos $(1,0)$ e $(2,0)$.

  3. $D$ não é simplesmente conexo porque possui um buraco. Com efeito, a região delimitada pela curva simples e fechada $x^2+y^2=(5/2)^2$, contém pontos que não pertencem a $D$, por exemplo, o ponto $(0,3/2)$.


1994   

Sejam ${\bf u}(t)=t{\bf i}+{\bf j}+e^{t}{\bf k}$ e ${\bf v}(t)={\bf i}+{\bf j}+{\bf k}.$ Calcule

  1. $\displaystyle\int_{0}^{1}({\bf u}(t)\times{\bf v}(t))\mathrm{d}t$
  2. $\displaystyle\int_{0}^{1}({\bf u}(t)\cdot {\bf v}(t))\mathrm{d}t$


2264   

Calcule $\displaystyle\iint\limits_{B}f(x,y)\,dx dy$ sendo dados:

  1.  $f(x,y)=\dfrac{1}{\ln(y)}$ e $B=\bigg\{(x,y)\in \mathbb{R}^{2}|\;2\leq y\leq 3,\;0\leq x\leq \dfrac{1}{y}\bigg\}.$

  2.  $f(x,y) = xy\cos{x^{2}}$ e $B=\{(x,y) \in \mathbb{R}^{2}| \; 0 \leq x \leq 1, \; x^{2} \leq y \leq 1\}$.

  3.  $f(x,y) = \cos(2y)\sqrt{4-\sin^{2}{x}}$ e $B$ é o triângulo de vértices $(0,0)$, $\bigg(0,\dfrac{\pi}{2}\bigg)$ e $\bigg(\dfrac{\pi}{2},\dfrac{\pi}{2}\bigg).$

  4.  $f(x,y)=x+y$ e $B$ a região compreendida entre os gráficos das funções $y=x$ e $y=e^{x}$, com $0\leq x\leq 1.$


  1.  $\ln(\ln(3)) - \ln(\ln(2)).$

  2.  $\dfrac{\sin(1) - \cos(1)}{2}$.

  3.  $\dfrac{8}{3} - \sqrt{3}.$

  4.  $\dfrac{1 + e^{2}}{4}.$


2107   

Um campo vetorial inverso do quadrado é da forma:

$${\bf F}({\bf r})=\frac{c{\bf r}}{|{\bf r}|^{3}}$$

para alguma constante $c$, onde ${\bf r}=x\,{\bf i}+y\,{\bf j}+z\,{\bf k}$. Um exemplo de campo inverso do quadrado é o campo elétrico ${\bf F}=\epsilon q Q{\bf r}/|{\bf r}|^{3}$. Suponha que um elétron com carga de $-1,6\times 10^{-19}\, C$ esteja localizado na origem. Uma carga positiva unitária é colocada à distância de $10^{-12}\,m$ do elétron e se move para uma posição que está à metade da distância original do elétron. Determine o trabalho realizado pelo campo elétrico. (Use o valor $\epsilon=8,985\times 10^{9}$.)


$\approx 1,4 \times 10^{4}$ J.


2149   

Use o Teorema do Divergente para calcular o fluxo de ${\bf F}$ através de $S,$ onde ${\bf F}(x,y,z)=3xy^{2}\,{\bf i}+xe^{z}\,{\bf j}+z^{3}\,{\bf k}$ e $S$ é a superfície do sólido delimitado pelo cilindro $y^{2}+z^{2}=1$ e pelos planos $x=-1$ e $x=2.$


2406   

Inverta a ordem de integração.

  1.  $\displaystyle\int_{1}^{e}\bigg[\int_{\ln(x)}^{x}f(x,y)\,dy\bigg]dx.$

  2.  $\displaystyle\int_{0}^{1}\bigg[\int_{y}^{y+3}f(x,y)\,dx\bigg]dy$

  3.  $\displaystyle\int_{-1}^{1}\bigg[\int_{-\sqrt{1-x^{2}}}^{\sqrt{1-x^{2}}}f(x,y)\,dy\bigg]dx$


  1.  $\displaystyle\int_{0}^{1}\bigg[\int_{1}^{e^{y}}f(x,y)\,dx\bigg]dy. + \displaystyle\int_{1}^{e}\bigg[\int_{y}^{1}f(x,y)\,dx\bigg]dy.$

  2.  $\displaystyle\int_{0}^{1}\bigg[\int_{0}^{x}f(x,y)\,dy\bigg]dx + \displaystyle\int_{1}^{3}\bigg[\int_{0}^{1}f(x,y)\,dy\bigg]dx + \displaystyle\int_{3}^{4}\bigg[\int_{x-3}^{1}f(x,y)\,dy\bigg]dx$  

  3.  $\displaystyle\int_{-1}^{1}\bigg[\int_{-\sqrt{1-y^{2}}}^{\sqrt{1-y^{2}}}f(x,y)\,dx\bigg]dy$


3024   

Esboce a região de integração e calcule a integral $\displaystyle\int_{\pi}^{2\pi}\!\!\int_{0}^{\pi}(\sin{x}+\cos{y})\,dx dy$.


$2\pi.$

ma211-list6-ex25_sol_e.png


2903   

Determine os pontos da superfície $xyz = 1$ que estão mais próximos da origem.


$(1,1,1),$ $(1,-1,-1),$ $(-1,1,-1)$ e $(-1,-1,1).$


2732   

Determine as equações do plano tangente e da reta normal ao gráfico da função dada, no ponto dado. $f(x,y) = xe^{x^2 - y^2}$ em $(2,2,f(2,2))$.


Plano tangente: $z = 9x - 8y$

Reta normal: $(x,y,z) = \left(2,2,2 \right) + \lambda \left(9,-8,-1 \right)$.


2884   

Determine o ponto da reta $x + 2y = 1$ cujo produto das coordenadas seja máximo.


$\displaystyle \left( \frac{1}{2}, \frac{1}{4} \right).$


2900   

Use o método dos multiplicadores de Lagrange para determinar o ponto sobre a parábola $y = x^2$ que se encontra mais próximo do ponto $(0,1) \in \mathbb{R}^2.$


$\displaystyle \left(\frac{1}{\sqrt{2}}, \frac{1}{2} \right)$ e $\displaystyle \left(-\frac{1}{\sqrt{2}}, \frac{1}{2} \right).$


2358   

Calcule a área da parte da superfície esférica $x^{2}+y^{2}+z^{2}=1$ que se encontra dentro do cone $z\geq \sqrt{x^{2}+y^{2}}.$


$\pi(2 - \sqrt{2}).$


2175   

Calcule a área da região limitada pela elipse $x = a\cos{t}$, $y=b\sin{t}$,  $0\leq t \leq \pi/2$, em que $a > 0$ e $b > 0$.


$\pi ab.$


2548   

Calcule

$$\displaystyle \lim_{(x,y) \to (0,0)} \dfrac{\mbox{sen}(x^2 + y^2)}{x^2 + y^2}.$$



Considere $t=x^{2}+y^{2}$.
Assim , se $(x,y)\rightarrow (0,0)$ temos que $t \to 0.$ Portanto,

$$\lim_{(x,y) \to (0,0)} \frac{\sin(x^{2}+y^{2})}{x^{2}+y^{2}}=\lim_{t \to 0}\frac{\sin t}{t}=1.$$


2119   

Calcule $\mathrm{d} z/\mathrm{d} t$ por dois processos:

  1. substituindo as expressões para $x(t)$ e $y(t)$ em $z$ e depois derivando diretamente com relação a $t$
  2. aplicando a Regra da Cadeia: $\frac{dz}{dt}=\frac{\partial z}{\partial x}\frac{dx}{dt}+\frac{\partial z}{\partial y }\frac{dy}{dt}$.

$z=\ln(1+x^{2}+y^{2})$, $x=\sin{3t}$ e $y=\cos{3t}.$



$\displaystyle \frac{dz}{dt} (t) = 0.$


2273   

No item abaixo :

  1.  determine o gradiente de $f$; 
  2.  calcule o gradiente no ponto $P$; e 
  3.  determine a taxa de variação de $f$ em $P$ na direção do vetor $\bf{u}$.

$f(x,y,z) = xe^{2yz},  P = (1,-3),  \bf{u} = \left( \frac{2}{3}, -\frac{2}{3}, \frac{1}{3} \right)$.


  1. $\nabla f(x,y,z) = (e^{yz}, 2xze^{2yz}, 2xye^{2yz}).$
  2. $\nabla f(3,0,2) = (1,12,0).$
  3. $\displaystyle -\frac{22}{3}.$


2964   

Dentre as coordenadas cilíndricas ou esféricas, utilize a que lhe parecer mais apropriada para determinar o volume e o centroide do sólido $E$ que está acima do cone $z=\sqrt{x^{2}+y^{2}}$ e abaixo da esfera $x^{2}+y^{2}+z^{2}=1.$


Volume: $\dfrac{\pi(2 - \sqrt{2})}{3};$ centróide: $\left(0,0, \dfrac{3}{8(2 - \sqrt{2})} \right).$


2147   

Use o Teorema do Divergente para calcular o fluxo de ${\bf F}$ através de $S,$ onde ${\bf F}(x,y,z)=e^{x}\,\sin y\,{\bf i}+e^{x}\,\cos y\,{\bf j}+yz^{2}\,{\bf k}$ e $S$ é a superfície da caixa delimitada pelos planos $x=0$, $x=1$, $y=0$, $y=1$, $z=0$ e $z=2.$


2138   

Seja $S$ o gráfico de $f(x,y)=x^{2}+y^{2}$, $x^{2}+y^{2}\leq 1$ e seja ${\bf n}$ a normal a $S$ com componete $z\leq 0$. Seja ${\bf F}(x,y,z)=x^{2}y\,{\bf i}-xy^{2}\,{\bf j}+{\bf k}$. Calcule $\iint \limits_{S}{\bf F}\cdot {\bf n}\, dS.$



Observe que $S$ não é uma superfície fechada (isto é, $S$ não é a fronteira de um sólido $E$). Para que possamos utilizar o Teorema do Divergente, vamos considerar a superfície $S_2$ constituída pelo parabolóide $S$ e pelo círculo $S_1$ dado por $x^2+y^2 \leq 1$ em $z=1$. Como $S_2$ é uma superfície fechada, usamos a escolha da normal ${\bf n_2}$ em $S_2$ que está apontando ``para fora". Sejam ${\bf n_1}$ a normal a $S_1$ (apontando para cima) e ${\bf n}$ a normal a $S$ (apontando para fora).

ma211-list15-ex2_sol.png

Temos
$\displaystyle\iint\limits_{S_2}{\bf F}\cdot {\bf n_2}\,dS = \iint\limits_{S}{\bf F}\cdot {\bf n}\,dS + \iint \limits_{S_1}{\bf F}\cdot {\bf n_1}\,dS,$
isto é,
$\displaystyle\iint\limits_{S}{\bf F}\cdot {\bf n}\,dS = \iint\limits_{S_2}{\bf F}\cdot {\bf n_2}\,dS - \iint \limits_{ S_1}{\bf F}\cdot {\bf n_1}\,dS.$
Pelo Teorema do Divergente,
$$\iint\limits_{S_2}{\bf F}\cdot {\bf n_2}\,dS = \iiint\limits_{E}(2xy-2xy+0)\,dV = 0,$$
em que $E$ é o sólido que possui $S_2$ como fronteira.
Para determinar $\displaystyle\iint\limits_{S_1}{\bf F}\cdot {\bf n_1}\,dS$, devemos encontrar uma parametrização para $S_1$ e determinar o vetor normal ${\bf n_1}$. Considere a seguinte parametrização de $S_1$: $r(u,v) = (u,v,1)$, com $u^2+v^2 \leq 1$. Daí, $r_u(u,v) = (1,0,0)$ e $r_v(u,v) = (0,1,0)$. Logo, $r_u \times r_v = (0,0,1)$ é um vetor normal a $S_1$. Devemos tomar ${\bf n_1} = (0,0,1)$ para que aponte para cima. Então,
$\displaystyle\iint \limits_{S_1}{\bf F}\cdot {\bf n_1}\,dS = \iint\limits_{D}(u^2v,-uv^2,1)\cdot(0,0,1)\,dA,$
em que $D = \{(u,v) \in \mathbb{R}^2; u^2+v^2 \leq 1\}$. Portanto,
$\displaystyle\iint \limits_{S_1}{\bf F}\cdot {\bf n_1}\,dS =  \iint\limits_{D}1\,dA = A(D) = \pi,$
donde concluímos que
$\displaystyle\iint \limits_{S}{\bf F}\cdot {\bf n}\,dS = 0 - \pi = -\pi.$


2238   

Mostre que $f(x,y) = \ln{(x^2+y^2)}$ satisfaz a equação de Laplace $\nabla^2f = 0$, exceto no ponto $(0,0).$


Note que se $(x,y) \neq (0,0),$  $\nabla^2f = \dfrac{\partial}{\partial x} \left[ \dfrac{2x}{x^{2} + y^{2}} \right] + \dfrac{\partial}{\partial y} \left[ \dfrac{2y}{x^{2} + y^{2}} \right].$


3001   

A figura mostra o mapa de contorno de $f$ no quadrado $R = [0,4] \times [0,4]$.

  1. Use a Regra do Ponto Médio com $m = n = 2$ para estimar o valor de $\int\!\!\!\int \limits_{\!\!\!\!\!R} \! f(x,y) \, dA$.

  2. Estime o valor médio de $f$.

ma211-list6-ex7_b.png


2890   

Passe para coordenadas polares e calcule: $\displaystyle\int_{0}^{a} \int_{0}^{\sqrt{a^{2}-x^{2}}}\,dy dx$


$\dfrac{\pi a^2}{4}.$


2661   

Determine as derivadas parciais de $z=\arctan \dfrac{x}{y}$.


$\displaystyle \frac{\partial z}{\partial x} = \frac{y}{x^{2} + y^{2}}\;\;\;\;\;\;\text{e}\;\;\;\;\; \frac{\partial z}{\partial y} = \frac{-x}{x^{2} + y^{2}}.$


2622   

Utilizando o Teorema de Stokes, transforme a integral $\iint_{ S}\mbox{rot}{\bf F}\cdot{\bf n}dS$ numa integral de linha e calcule.

  • ${\bf F}(x,y,z) = y{\bf k}$, $S$ a superfície parametrizada por ${\bf R} (u,v) = (u,v,u^2+v^2)$, $u^2+v^2 \leq 1$, sendo ${\bf n}$ a normal apontando para cima.


 $0.$


2041   

Utilize a Regra da Cadeia para determinar as derivadas parciais indicadas.
$u=\sqrt{r^{2}+s^{2}}$, $r=y+x\;\cos{t}$,  $s=x+y\;\sin{t}$;
$\dfrac{\partial u}{\partial x}$, $\dfrac{\partial u}{\partial y}$, $\dfrac{\partial u}{\partial t}$ quando $x=1$, $y=2$, $t=0$.


$\displaystyle \frac{\partial u}{\partial x} = \frac{4}{\sqrt{10}}$, $\displaystyle \frac{\partial u}{\partial y} = \frac{3}{\sqrt{10}}$, $\displaystyle \frac{\partial u}{\partial t}= \frac{2}{\sqrt{10}}.$


3081   

Verifique que a função \(\displaystyle u(x,t)=\sin(x-ct)\) é uma solução da equação da onda unidimensional \[ \dfrac{\partial^2u}{\partial t^2} = c^2\dfrac{\partial^2u}{\partial x^2}, \] onde \(c\) é uma constante que depende das características da onda.



Calculando diretamente as derivadas parciais da função dada, temos

\[\begin{array}{ll} \dfrac{\partial u}{\partial x} = \cos(x-ct),   & \dfrac{\partial^2u}{\partial x^2}= -\sin(x-ct) \\ \dfrac{\partial u}{\partial t} = -c\cos(x-ct), & \dfrac{\partial^2 u}{\partial t^2}= -c^2\sin(x-ct). \end{array}\] Assim, podemos ver que \(u(x,t)\) satisfaz a equação dada.


2498   

Esboce a região limitada pelos gráficos das equações e use uma integral tripla para calcular seu volume.

  1.  $z+x^{2}=4$, $y+z=4$, $y=0$ e $z=0.$

  2.  $y=2-z^{2}$, $y=z^{2}$, $x+z=4$ e $x=0.$

  3.  $y^{2}+z^{2}=1$, $x+y+z=2$ e $x=0.$


  1.  $\dfrac{128}{5}.$

  2.  $\dfrac{32}{3}.$

  3.  $2\pi.$


2870   

Utilize os multiplicadores de Lagrange para determinar os valores máximo e mínimo da função sujeita à(s) restrição(ões) dada(s).

$f(x,y) = x^2 + y^2; \quad xy = 1.$


Não há valor máximo; valor mínimo: $2.$


2817   

Determine os valores máximo e mínimo absolutos de $f$ no conjunto $D.$

$f(x,y)=(2x-x^{2})(2y-y^{2})$, $D$ é a região do plano $xy$ dada por $0\leq y\leq 2(2x-x^{2})$.


Valor máximo: $1;$  valor mínimo: $0.$


2536   

Calcule a massa do sólido $x+y+z\leq 1$, $x\geq 0$, $y\geq 0$ e $z\geq 0$, sendo a densidade dada por $\rho(x,y,z)=x+y.$


$\dfrac{1}{12}.$


2916   

Calcule utilizando coordenadas esféricas. $\displaystyle\iiint\limits_{B} z \,dxdydz$, onde $B$ é o conjunto $1\leq x^{2}+y^{2}+z^{2}\leq 4$ e $z\geq 0.$



Usando coordenadas esféricas, o sólido pode ser descrito por

$$B = \left\{(\rho, \theta, \phi): 1 \leq \rho \leq 2, 0 \leq \theta \leq 2\pi \mbox{ e } 0 \leq \phi \leq \frac{\pi}{2}\right\}.$$

Lembre que o Jacobiano dessa transformação é $\rho^2 \sin{\phi}$. Assim, obtemos

\begin{array}{rcl}\displaystyle\iiint\limits_{B} z \,dxdydz & = & \displaystyle\int_{0}^{2\pi}\int_{0}^{\frac{\pi}{2}}\int_{1}^{2}(\rho \cos{\phi})(\rho^2 \sin{\phi})\,d\rho d\phi d\theta \\  & = & \displaystyle\int_{0}^{2\pi}\int_{0}^{\frac{\pi}{2}}\left.\left(\frac{\rho^4}{4} \frac{\sin{2\phi}}{2}\right|_{\rho=1}^{\rho=2}\right)\, d\phi d\theta \\  & = & \displaystyle\int_{0}^{2\pi}\left.\left(\frac{(16-1)}{8} \frac{(-\cos{2\phi)}}{2}\right|_{\phi=0}^{\rho=\frac{\pi}{2}}\right)\, d\theta \\  & = & \left.-\frac{15}{16}(-1-1) \theta \right|_{\theta=0}^{\theta=2\pi} = \frac{15\pi}{4}.    \end{array}


2420   

Use a integral tripla para determinar o volume do sólido dado por $x^{2}+y^{2}\leq z\leq \sqrt{4-3x^{2}-3y^{2}}.$



Primeiramente, vamos determinar a projeção no plano $xy$ da interseção de \begin{eqnarray*} z&=&\sqrt{4-3x^{2}-3y^{2}}\\ z&=&x^{2}+y^{2}. \end{eqnarray*} Da primeira equação temos que \begin{eqnarray*} \label{1}z=\sqrt{4-3x^{2}-3y^{2}}\Leftrightarrow z^{2}=4-3x^{2}-3y^{2}\Leftrightarrow z^{2}=4-3(x^{2}+y^{2}). \end{eqnarray*} Substituindo a segunda equação  na primeira, obtemos que $$z^{2}=4-z\Leftrightarrow z^{2}+3z-4=0\Leftrightarrow (z-1)(z-4)=0.$$ Logo, $z=-4$ e $z=1.$ Notemos que $z=-4$ não satisfaz as duas primeiras equações acima, então a projeção $D$ no plano $xy$ é o círculo de raio 1, isto é, $D=\{(x,y)\in \mathbb{R};\;\, x^{2}+y^{2}\leq 1\}.$ Assim, o volume, $V$, do sólido é: $$V=\iint\limits_{D}\bigg[\int_{x^{2}+y^{2}}^{\sqrt{4-3x^{2}-3y^{2}}}1\, dz\bigg]\,dA = \iint\limits_{ D}\sqrt{4-3x^{2}-3y^{2}}-(x^{2}+y^{2})\,dA.$$ Passando para coordenadas polares temos que \begin{eqnarray*}  x=r\cos \theta\\ y=r\sin \theta\\ dA=r\,dr\,d\theta\\ 0\leq r\leq 1\\ 0\leq \theta \leq 2\pi.\\ \end{eqnarray*} Então, $$V=\int_{0}^{2\pi}\int_{0}^{1}(\sqrt{4-3r^{2}}-r^{2})r\,dr\,d \theta=\int_{0}^{2\pi}\int_{0}^{1}(r\sqrt{4-3r^{2}}-r^{3})\,dr\,d\theta$$ $$=\int_{0}^{2\pi}\,d\theta\cdot \bigg[\bigg(\underbrace{\int_{0}^{1}r\sqrt{4-3r^{2}}\,dr}_{\substack{ u=4-3r^{2}\\ du=-6r\,dr}}\bigg)-\bigg(\int_{0}^{1}r^{3}\,dr\bigg)\bigg]$$ $$=\theta\bigg|_{0}^{2\pi}\cdot \bigg[\bigg(\int_{4}^{1}r\cdot u^{1/2}\frac{du}{-6r}\bigg)-\bigg(\frac{r^{4}}{4}\bigg|_{0}^{1}\bigg)\bigg]$$ $$=2\pi\cdot \bigg[\bigg(-\frac{1}{6}\int_{4}^{1}u^{1/2}\,du\bigg)-\frac{1}{4}\bigg]=2\pi \cdot \bigg[\bigg(-\frac{1}{6}\cdot \frac{2}{3}u^{3/2}\bigg|_{4}^{1}\bigg)-\frac{1}{4}\bigg]$$ $$=2\pi \cdot \bigg[-\frac{1}{9}+\frac{1}{9}\cdot 8-\frac{1}{4}\bigg]=2\pi \cdot \frac{19}{36}=\frac{19\pi}{18}.$$


2842   

Passe para coordenadas polares e calcule: $\displaystyle\int_{0}^{1}  \int_{0}^{\sqrt{x-x^{2}}}x\,dy dx$


$\displaystyle \frac{\pi}{16}.$


2168   

A função diferenciável $z=z(x,y)$ é dada implicitamente pela equação $f\bigg(\dfrac{x}{y},\dfrac{z}{x^{\lambda}}\bigg)=0$ ($\lambda\neq 0$ um número real fixo), onde 

$f(u,v)$ é suposta diferenciável e $\dfrac{\partial f}{\partial v}(u,v)\neq 0$. Verifique que 

$$x\frac{\partial z}{\partial x}+y\dfrac{\partial z}{\partial y}=\lambda z.$$


Note que $\displaystyle \frac{\partial z}{\partial x} =   \frac{\lambda z}{x} -\frac{x^{\lambda}}{y} \frac{\partial f}{\partial u} \left(\frac{x}{y},\frac{z}{x^{\lambda}} \right)\left(\frac{\partial f}{\partial v}\left(\frac{x}{y},\frac{z}{x^{\lambda}} \right)\right)^{-1} $  e $\displaystyle \frac{\partial z}{\partial y} =  \frac{x^{\lambda + 1}}{y^{2}} \frac{\partial f}{\partial u} \left(\frac{x}{y},\frac{z}{x^{\lambda}} \right)\left(\frac{\partial f}{\partial v}\left(\frac{x}{y},\frac{z}{x^{\lambda}} \right)\right)^{-1}$.


2256   

Demonstre a identidade abaixo, supondo que $S$ e $E$ satisfaçam as condições do Teorema do Divergente e que as funções escalares e as componentes dos campos vetoriais tenham derivadas parciais de segunda ordem contínuas.

  1. $\displaystyle\iint\limits_{S}(f\nabla g-g\nabla f)\cdot {\bf n}\,dS=\displaystyle\iiint\limits_{E}(f\nabla^{2} g-g\nabla^{2} f)\,dV.$



Use o Teorema da Divergência e que $\nabla f \cdot \nabla g = \nabla g \cdot \nabla f.$



2932   

Calcule a integral em coordenadas esféricas. $\displaystyle\int_{0}^{2\pi}\int_{0}^{\pi}\int_{0}^{(1-\cos{\phi})/2}\rho^{2}\sin{\phi}\,d\rho d\phi d\theta$.


$\dfrac{\pi}{3}.$


2153   

Use o Teorema do Divergente para calcular o fluxo de ${\bf F}$ através de $S,$ onde ${\bf F}(x,y,z)=yz\,{\bf i}+xz\,{\bf j}+xy\,{\bf k}$ e $S$ é o gráfico de $x^{2/3}+y^{2/3}+z^{2/3}=1.$


2140   

Demonstre a identidade $\displaystyle\iint\limits_{S}\mbox{rot}\, {\bf F}\cdot dS=0$, supondo que $S$ e $E$ satisfaçam as condições do Teorema do Divergente e que as funções escalares e as componentes dos campos vetoriais tenham derivadas parciais de segunda ordem contínuas.




Pelo Teorema do Divergente, temos
$\displaystyle\iint\limits_{ S}\mbox{rot} {\bf F}\cdot dS = \iiint\limits_{ E}\mbox{div} (\mbox{rot} {\bf F})\,dV,$
em que $E$ é o sólido que tem $S$ como fronteira. Observe que
\begin{align*}
&\mbox{div} (\mbox{rot} {\bf F})  =\\ & \frac{\partial}{\partial x}(R_y - Q_z) + \frac{\partial}{\partial y}(P_z - R_x) + \frac{\partial}{\partial z}(Q_x - P_y) \\ & R_{xy} - Q_{xz} + P_{yz} - R_{yx} + Q_{zx} - P_{zy} = 0,
\end{align*}
pois, como as derivadas de segunda ordem são contínuas, temos, pelo Teorema de Clairaut, que $P_{yz} = P_{zy}$, $Q_{zx} = Q_{xz}$ e $R_{xy} = R_{yx}$. Portanto,

$\displaystyle\iint\limits_{S}\mbox{rot}{\bf F}\cdot dS=0.$


2875   

Utilize os multiplicadores de Lagrange para determinar os valores máximo e mínimo da função sujeita à(s) restrição(ões) dada(s).

$f(x_1,x_2, \ldots, x_n) = x_1 + x_2 + \cdots + x_n; \quad x_1^2 + x_2^2 + \cdots + x_n^2 = 1.$


Valor máximo: $\sqrt{n};$ valor mínimo: $-\sqrt{n}.$


2125   

Admita que, para todo $(x,y)$, 

$$4y\frac{\partial f}{\partial x}(x,y)-x\frac{\partial f}{\partial y}(x,y)=2.$$

Calcule $g^{'}(t)$, sendo $g(t)=f(2\cos{t},\sin{t})$.


$g^{'}(t) = -1.$


2959   

Usando coordenadas esféricas, determine o volume do sólido que está acima do cone $\phi=\pi/3$ e abaixo da esfera $\rho=4\cos{\phi}.$


$10\pi.$


2821   

Determine os valores máximo e mínimo absolutos de $f$ no conjunto $D.$

$f(x,y)=xy$ em $D=\{(x,y) \in \mathbb{R}^2: x\geq 0,\;y\geq 0\;\text{e}\;2x+y\leq 5\}.$


Valor máximo: $\displaystyle \frac{25}{8};$ valor mínimo: $\displaystyle 0.$


2703   

Considere a função

$$f(x,y)=\log(9-x^{2}-9y^{2}).$$

  1. Esboce no plano $xy$ o domínio de $f.$

  2. Calcule as derivadas parciais $f_{x}$ e $f_{y}.$


  1. $D_{f} = \left\lbrace (x,y) \in \mathbb{R}^{2};\; x^{2} -9y^{2} < 9 \right\rbrace$.
    ma211-list3-ex43_sol_a.png
  2. $\displaystyle f_{x} = \frac{-2x}{9 - x^{2} - 9y^{2}}  \;\;\;\text{e}\;\;\;f_{y} = \frac{-18y}{9 - x^{2} - 9y^{2}}$.

2819   

Determine os valores máximo e mínimo absolutos de $f$ no conjunto $D.$

$f(x,y)=3x-y$ em $D=\{(x,y)\in \mathbb{R}^2: x^{2}+y^{2}\leq 1\}.$


Valor máximo: $\displaystyle \frac{8\sqrt{10}}{10};$ valor mínimo: $-\sqrt{10}.$


2946   

Calcule utilizando coordenadas esféricas. $\displaystyle\iiint\limits_{B}x\,dxdydz$, onde $B$ é o conjunto $\dfrac{x^{2}}{4}+\dfrac{y^{2}}{9}+z^{2}\leq 1$ e $x\geq 0.$


$3\pi.$


2769   

Considere a superfície dada implicitamente por

$$x^{2}+2y^{2}+2z^{2}=-4xyz.$$

  1. Calcule as derivadas $\dfrac{\partial z}{\partial x}$ e $\dfrac{\partial z}{\partial y}$ em um ponto genérico.

  2. Quais os pontos nos quais as derivadas parciais calculadas no item anterior não estão definidas?


  1. $\displaystyle \frac{\partial z}{\partial x} = -\frac{x + 2yz}{2(z + xy)} \;\;\;\text{e}\;\;\; \frac{\partial z}{\partial y} = -\frac{y + xz}{z + xy}.$

  2. $\left\lbrace (x,y,z) \in \mathbb{R}^{3};\; z = -xy \right\rbrace$.


1977   

Mostre que a curva com equações paramétricas $x = t^2$, $y = 1 - 3t$, $z = 1 + t^3$ passa pelos pontos $(1,4,0)$ e $(9,-8,28)$, mas não passa pelo ponto $(4,7,-6).$


2440   

Calcule a integral de superfície $\displaystyle\iint\limits_{S}\sqrt{1+x^{2}+y^{2}}dS$, onde $S$ é o helicóide com equação vetorial ${\bf r}(u,v)=u\cos v{\bf i}+u\sin v{\bf j}+v{\bf k}$, $0 \leq u \leq 1$, $0 \leq v \leq \pi.$


$\dfrac{4\pi}{3}.$


2981   

Utilize a transformação dada para calcular a integral. $\displaystyle \iint\limits_{R}(x - 3y) \, dA$, em que $R$ é a região triangular de vértices $(0,0)$, $(2,1)$ e $(1,2)$; $x = 2u + v$, $y = u + 2v$.


$-3.$


2519   

Dada a expressão $g(x,y)=-f(x,y)$, escreva como o gráfico de $g$ é obtido a partir do gráfico de $f.$


Gráfico de $f$ refletido sobre o plano $xy.$


2048   

Determine se o conjunto $\{(x,y)|\,x\neq 0\}$ é ou não: 
  1. aberto;

  2. conexo; e

  3. simplesmente conexo.



Temos que o conjunto $D=\{(x,y)|\,x\neq 0\}$ consiste de todos os pontos, exceto para aqueles que encontram-se sobre o eixo y. Então:

  1. $D$ é aberto.

  2. Os pontos em lados opostos do eixo $y$ não podem ser conectados por um caminho que se encontra totalmente em $D$, então $D$ não é conexo.

  3. $D$ não é simplesmente conexo, pois não é conexo.


3025   

Esboce a região de integração e calcule a integral $\displaystyle\int_{1}^{2}\!\!\int_{y}^{y^{2}} \,dx dy$.


$\frac{5}{6}.$

ma211-list6-ex25_sol_f.png


3073   

A figura mostra o campo vetorial ${\bf F}(x,y)=(2xy, x^{2})$ e três curvas que começam em $(1,2)$ e terminam em $(3,2).$

ma211-list11-ex6.png

  1. Explique por que $\int_{c}{\bf F}\cdot d{\bf r}$ tem o mesmo valor para as três curvas.
  2. Qual é esse valor comum?


  1. ${\bf F}$ é conservativo, logo $\int_{C} \bf F \cdot d\bf r$ depende somente dos pontos inicial e final de $C.$
  2. $16.$


2177   

  1. Se $C$ é o segmento de reta ligando o ponto $(x_1,y_1)$ ao ponto $(x_2,y_2)$, mostre que

    $$\int_{C}x \, dy - y \, dx = x_1y_2-x_2y_1.$$

  2. Se os vértices de um polígono, na ordem anti-horária, são

    $$ (x_1,y_1), (x_2,y_2), \ldots, (x_n,y_n), $$

    mostre que a área do polígono é

    $$A=\dfrac{1}{2}[(x_1y_2-x_2y_1) + (x_2y_3-x_3y_2) + \cdots + (x_{n-1}y_n - x_ny_{n-1}) + (x_ny_1-x_1y_n)].$$

  3. Determine a área do pentágono com vértices $(0,0)$, $(2,1)$, $(1,3)$, $(0,2)$ e $(-1,1)$.


  1. Use as equações paramétricas do segmento de reta: $x = (1-t)x_{1} + tx_{2}$ e $y = (1-t)y_{1} + ty_{2},$ $0 \leq t \leq 1.$

  2. Aplique o Teorema de Green ao caminho $C = C_{1} \cup C_{2} \cup \cdots \cup C_{n},$ onde $C_{i}$ é o segmento ligando o ponto $(x_{i},y_{i})$ ao ponto $(x_{i + 1},y_{i + 1}),$ para cada $i = 1,\cdots, n-1.$

  3. $\dfrac{9}{2}.$


1937   

Calcule a integral de linha, onde $C$ é a curva dada.
$\displaystyle\int_{C}xy^{3}\,ds$,   $C:\,x=4\,\sin t,\, y=4\,\cos t,\, z=3t,\, 0\leq t\leq \pi/2.$


$320.$


2689   

Passe para coordenadas polares e calcule.     

  1.   $\displaystyle\int_{0}^{1} \int_{1-\sqrt{1-x^{2}}}^{1+\sqrt{1-x^{2}}}xy\,dy dx$

  2.  $\displaystyle\int_{-a}^{a}\!\int_{-\sqrt{a^{2}-x^{2}}}^{\sqrt{a^{2}-x^{2}}}\,dy dx$          



  1. Temos que a região de integração é $$R=\{(x,y)\in\mathbb{R}^2|\, 0\leq x \leq 1\,\mbox{e}\, 1-\sqrt{1-x^{2}}\leq y \leq 1+\sqrt{1-x^{2}}\}.$$
    ma211-list7-ex1_sol_a.png

    Passando para coordenadas polares temos que: $$\left\{ \begin{array}{cc} x=r\,\cos\theta \\ y=r\,\sin\theta \\ dy\,dx=r\,dr\,d\theta \\ \end{array} \right.$$ Agora, \begin{eqnarray*}  x^{2}+y^{2}=2y&\Rightarrow & r^{2}\,\cos^2 \theta+r^{2}\,\sin^{2}\theta=2r\,\sin\theta\\  &\Rightarrow & r^{2}=2r \,\sin\theta\\  &\Rightarrow & r(r-2\sin\theta )=0 \\  &\Rightarrow& r=0 \mbox{ou}  r=2\sin\theta.\end{eqnarray*} Logo, $\displaystyle 0\leq r \leq 2\,\sin\theta$ e $\displaystyle 0\leq\theta \leq\dfrac{\pi}{2}.$  Então, $$\int_{0}^{1}\int_{1-\sqrt{1-x^2}}^{1+\sqrt{1-x^2}}xy\,dy\,dx  =\int_{0}^{\frac{\pi}{2}}\int_{0}^{2\,\sin\theta}(r\,\cos\theta)(r\,\sin\theta)r\,dr d\theta $$
    $$ =\int_{0}^{\frac{\pi}{2}}\int_{0}^{2\,\sin\theta}r^3\,\sin\theta\, \cos\theta\,dr d\theta =\int_{0}^{\dfrac{\pi}{2}}\bigg[\frac{r^{4}} {4}\sin\theta\,\cos\theta\bigg]\bigg|_{0}^{2\,\sin\theta}\,d\theta $$  $$ =\int_0^{\frac{\pi}{2}}\frac{(2\,\sin\theta)^4}{4}\,\sin\theta\,\cos\theta\,d\theta  =4\int_{0}^\frac{\pi}{2}\sin^5\theta\,\cos\theta\, d\theta.$$ Tomando, $u=\sin\theta \Rightarrow du=\cos\theta\, d\theta$ e sendo $\theta =0 \Rightarrow u=0$ e $\theta=\frac{\pi}{2}\Rightarrow u=1.$ Assim,  $$\int_{0}^{1}\int_{1-\sqrt{1-x^{2}}}^{1+\sqrt{1-x^{2}}}xy\,dy dx=4\int_{0}^{1}u^{5}\,du$$  $$=4\cdot \frac{u^{6}}{6}\bigg|_{0}^{1}=\frac{2}{3}.$$

  2. Temos que a região de integração é $$R=\{(x,y)\in \mathbb{R}|\, -a\leq x \leq a,\, -\sqrt{a^{2}-x^{2}}\leq y \leq \sqrt{a^{2}-x^{2}}\}.$$        ma211-list7-ex1_sol_b.png

    Passando para coordenadas polares temos que  $$\left\{ \begin{array}{cc}  x=r\,\cos\theta \\  y=r\,\sin\theta\\  dy\,dx=r\,dr\,d\theta\\ \end{array} \right. $$ Como $x^{2}+y^{2}=a^{2}\Rightarrow r^{2}\,\cos^{2}\theta+r^{2}\,\sin{2}\theta=a^{2}\Rightarrow  r^{2}=a^{2}\Rightarrow r=\pm a.$ Como o raio  deve ser sempre maior ou igual a zero, logo  $$0\leq r\leq a  \mbox{e}  0\leq \theta \leq 2\pi.$$  Então,  $$\int_{-a}^{a}\int_{-\sqrt{a^{2}-x^{2}}}^{\sqrt{a^{2}-x^{2}}}dy\,dx=\int_{0}^{2\pi}\int_{0}^{a}r\,dr\,d \theta=\int_{0}^{2\pi}d\theta \cdot \int_{0}^{a}r\,dr$$   $$=\theta\bigg |_{0}^{2\pi}\cdot \frac{r^{2}}{2}\bigg |_{0}^{a}=(2\pi)\cdot \bigg(\frac{a^{2}}{2}\bigg)=a^{2}\pi.$$  


2254   

Demonstre a identidade abaixo, supondo que $S$ e $E$ satisfaçam as condições do Teorema do Divergente e que as funções escalares e as componentes dos campos vetoriais tenham derivadas parciais de segunda ordem contínuas.

  1. $\displaystyle\iint\limits_{S} D_{n}f\,dS=\displaystyle\iiint\limits_{E}\nabla^{2}f\,dV.$



Lembre que $D_{n} f = \nabla f \cdot {\bf b}$ e $\mbox{div} (\nabla f) = \nabla^{2} f.$


2925   

Mude o ponto $(1,\sqrt{3},2\sqrt{3})$ dado em coordenadas retangulares para esféricas.


$\displaystyle \left( 4, \dfrac{\pi}{3}, \dfrac{\pi}{6} \right).$


2546   

Determine uma fórmula para $\displaystyle\iint \limits_{ S}{\bf F}\cdot d{\bf S}$ semelhante à fórmula

$\displaystyle\iint\limits_{S}{\bf F}\cdot d{\bf S}=\displaystyle\iint\limits_{D}\left(-P\dfrac{\partial f}{\partial x}-Q\dfrac{\partial f}{\partial y}+R\right)dA$ para o caso onde $S$ é dada por $y=h(x,z)$ e ${\bf n}$ é o vetor normal unitário que aponta para a esquerda.


$\displaystyle \iint\limits_{S}{\bf F}\cdot d{\bf S}=\iint\limits_{D}\left(P -Q\dfrac{\partial k}{\partial y}-R\frac{\partial k}{\partial z} \right)dA.$


2029   

Use a Regra da Cadeia para determinar $\mathrm{d}z/\mathrm{d} t$ ou $\mathrm{d}w/ \mathrm{d}t.$

$z=\tan^{-1}(x/y)$, $x=e^{t}$, $y=1-e^{-t}$.


$\displaystyle \frac{dz}{dt} = \frac{xe^{-t} - ye^{t}}{x^{2} + y^{2}}.$


3033   

Dados um hemisfério $H$ e uma parte $P$ de um paraboloide, suponha que ${\bf F}$ seja um campo vetorial sobre $\mathbb{R}^3$ cujas componentes tenham derivadas parciais contínuas. Explique por que

$$\displaystyle\iint\limits_{H}\mbox{rot}{\bf F}\cdot{\bf S} = \iint\limits_{P}\mbox{rot}{\bf F}\cdot{\bf S}.$$

ma211-list14-ex24_a.png

ma211-list14-ex24_b.png


Note que $H$ e $P$ satisfazem as hipóteses do Teorema de Stokes. Logo,
$$\displaystyle \iint \limits_{H} \mbox{rot } {\bf F} \cdot {\bf S} = \int \limits_{C} {\bf F} \cdot d{\bf r} = \iint \limits_{P} \mbox{rot }{\bf F}\cdot{\bf S},$$

onde $C$ é a curva de fronteira.


2937   

Determine a massa e o centro de massa da lâmina que ocupa a região $\displaystyle D = \{(x,y) \in \mathbb{R}^2: 0 \leq x \leq 2, \ -1 \leq y \leq 1\}$ e tem função densidade $\rho(x,y) = xy^2.$


Massa: $\dfrac{4}{3};$ centro de massa: $\displaystyle \left(\frac{4}{3},0 \right).$


2332   

Determine a taxa de variação máxima de $f$ no ponto dado e a direção em que isso ocorre.

$f(x,y,z) = \dfrac{x + y}{z},  (1,1,-1).$


$\sqrt{6}.$


2696   

Calcule todas as derivadas parciais de $2^{\underline{a}}$ ordem de $z=e^{x^{2}-y^{2}}$.


$\begin{aligned}[t]\frac{\partial^{2} z}{\partial x^{2}} &= 2e^{x^{2} - y^{2}}(1 + 2x^{2}),\;\;\;\;\; \frac{\partial^{2} z}{\partial y^{2}}= 2e^{x^{2} - y^{2}}(2y^{2} - 1) \;\;\;\;\;\text{e}\\\frac{\partial^{2} z}{\partial x\partial y} &= \frac{\partial^{2} z}{\partial y\partial x}= -4xye^{x^{2} - y^{2}}.\end{aligned}$


2553   

Determine o limite, se existir, ou mostre que o limite não existe.

$\displaystyle \lim_{(x,y) \to  (5,-2)}(x^5 + 4x^3y - 5xy^2)$.


$2025.$


2073   

Seja $R$ o retângulo $1\leq x\leq 2$, $0\leq y\leq 1$. Calcule $\iint\limits_{ R} f(x,y)\,dxdy$, sendo $f(x,y)$ igual a

  1.  $1$

  2.  $x\cos(xy)$


  1. $1.$

  2. $\cos(1) - \cos(2).$


2460   

Seja $f(x,y)=x^{2}e^{3xy}$.

  1. Calcule $f(2,0).$

  2. Determine o domínio de $f$.

  3. Determine a imagem de $f$.


  1. $4.$

  2. $\mathbb{R}^{2}.$

  3. $[0,\infty).$



  1. Queremos calcular $f(2,0)$ sabendo que $f(x,y)=x^{2}e^{3xy}$. Basta substituir os valores na expressão, assim temos
    \[
    f(2,0)=2^{2}e^{3 \cdot 2 \cdot 0}=4e^{0}=4 \cdot 1=4.
    \]
  2. Por definição, o domínio da função $f$ é o conjunto dos pontos de $\mathbb{R}^{2}$ em que a função está bem definida. Em nosso caso, o domínio é o conjunto de pontos em que a função $f(x,y)=x^{2}e^{3xy}$ está bem definida. Portanto, o domínio de $f$ é $\mathbb{R}^2$.
  3. A imagem da função $f$ é o conjunto dos pontos $\{ z\in \mathbb{R} | z = f(x,y) \text{ e } (x,y)\in D\}$, onde $D$ é o domínio de $f$. Observe que $x^{2} \geq 0$ e $e^{3xy}> 0$ para todo $(x,y)\in \mathbb{R}^{2}$, logo $x^{2}e^{3xy}\geq 0$ para todo $(x,y)\in \mathbb{R}^{2}$. Por exemplo, se fixarmos $x=1$ temos que a imagem da função são os pontos da forma $e^{3y}$ com $y\in \mathbb{R}$, ou seja, é todo o intervalo $(0,\infty)$. Agora, quando colocamos $x=0$ a imagem é $0$. Portanto, a imagem de $f$ é o conjunto $[0,\infty]$.

2031   

Utilize a Regra da Cadeia para determinar $\mathrm{\partial}z/\mathrm{\partial} s$ e $\mathrm{\partial}z/ \mathrm{\partial}t.$

$z=x^{2}y^{3}$, $x=s\cos{t}$, $y=s\sin{t}$. 


$\displaystyle \frac{\partial z}{\partial s} = 2xy^{3} \cos(t) + 3x^{2}y^{2} \sin(t) $ e $\displaystyle \frac{\partial z}{\partial t} = -2sxy^{3} \sin(t) + 3 sx^{2}y^{2} \cos(t)$. 


3155   

A fórmula de Taylor de primeira ordem para $f(\vec{a} + \vec{v})$ pode ser escrita como $ f(\vec{a}) + \nabla f(\vec{a}) \cdot \vec{v}$, já desconsiderando o termo de erro. Calcule-a para $f(x,y) = x^2/2 + y$, $\vec{a} = (0,0)$ e $\vec{v} = (1/2,1/2)$. Calcule também o erro cometido, dizendo se é um erro pequeno ou grande e por quê.


3074   

Mostre (verifique) que o domínio da função $f(x,y)=\ln(x^2-y)$ consiste em todos os pontos abaixo da curva $y<x^2$.


A função $(x,y)\longmapsto\ln(x^2-y)$ só está definida para \( 0<x^2-y \), ou seja, $y<x^2$. Assim, primeiro esboçamos a parábola $y=x^2$ (como uma curva tracejada, por exemplo). A região $y<x^2$ consiste em todos os pontos abaixo dessa curva.


2187   

Use o Teorema do Divergente para calcular o fluxo de ${\bf F}$ através de $S,$ onde ${\bf F}(x,y,z)=3x\,{\bf i}+xz\,{\bf j}+z^{2}\,{\bf k}$ e $S$ é a superfície da região delimitada pelo parabolóide $z=4-x^{2}-y^{2}$ e o plano-$xy.$


2376   

Seja $f$ uma função de duas variáveis que tenha derivadas parciais contínuas e considere os pontos $A = (1,3)$, $B = (3,3)$, $C = (1,7)$ e $D = (6,15)$. A derivada direcional em $A$ na direção do vetor $\overrightarrow{AB}$ é 3, e a derivada direcional em $A$ na direção $\overrightarrow{AC}$ é 26. Determine a derivada direcional de $f$ em $A$ na direção do vetor $\overrightarrow{AD}$.


 $\displaystyle \frac{327}{13}.$


2023   

Calcule a integral dupla, identificando-a antes com o volume de um sólido.

  1.   $\displaystyle\iint\limits_{R} 3 \, dA, \quad R = \{(x,y) \in \mathbb{R}^2: -2 \leq x \leq 2, \ 1 \leq y \leq 6\}.$

  2.   $\displaystyle\iint\limits_{R} (4 - 2y) \, dA, \quad R = [0,1] \times [0,1].$


  1.  $60.$

  2.  $3.$


2424   

Use o Teorema de Stokes para calcular $\displaystyle\int_C {\bf F} \cdot d{\bf r}$, com ${\bf F} (x,y,z) = yz{\bf i} + 2xz{ \bf j} + e^{xy} {\bf k} $ e $C$ é a circunferência $x^2+y^2 = 16$, $z=5$, orientada no sentido anti-horário quando vista de cima.


1952   

Calcule a integral de linha $\int_{C}{\bf F}\cdot d{\bf r}$, onde $C$ é dada pela função vetorial ${\bf r}(t).$

${\bf F}(x,y,z)=x\,{\bf i}+y\,{\bf j}+z\,{\bf k}$, ${\bf r}(t)=(\cos t,\sin t,t)$, $0\leq t\leq 2\pi.$



$2\pi^{2}.$


2540   

Um cubo sólido de $2$ unidades de lado é limitado pelos planos $x=\pm 1$, $z=\pm 1$, $y=3$ e $y=5.$ Encontre o centro de massa e os momentos de inércia desse cubo.


Centro de massa: $\displaystyle \left(0,4,0 \right),$ $I_{x} = \dfrac{400}{3},$ $I_{y} = \dfrac{16}{3},$ $I_{z} = \dfrac{400}{3}.$


2741   

Determine a equação do plano que é tangente ao paraboloide $z = 2x^2 + 3y^2$ e paralelo ao plano $4x - 3y - z = 10$.


$4x - 3y - z = -\frac{11}{4}$.


2404   

Considere a integral iterada dada por $$\int_{0}^{1} \int_{x}^{\sqrt{x}}\frac{e^{y}}{y}\,dy dx.$$

  1.  Desenhe a região de integração no plano $xy.$

  2.  Calcule a integral acima.


  1.  (...)

  2.  $e - 2.$


2126   

Admita que, para todo $(x,y)$, 

$$4y\frac{\partial f}{\partial x}(x,y)-x\frac{\partial f}{\partial y}(x,y)=0.$$

Prove que $f$ é constante sobre a elipse $\dfrac{x^{2}}{4}+y^{2}=1.$



Note que $\displaystyle \frac{dz}{dt} \left(t \right) = 0,$ para $z = f(x,y),$ $x = t$ e $\displaystyle y = \pm \sqrt{1 - \frac{t^{2}}{4}}.$


2956   

Usando coordenadas esféricas, determine o volume da menor região cortada da esfera sólida $\rho \leq 2$ pelo plano $z=1.$


$\dfrac{5\pi}{3}.$


2312   

Determine a derivada direcional da função no ponto dado e na direção do vetor $\bf{v}$.

$g(p,q) = p^4 - p^2q^3,  (2,1),  \bf{v}= \left(-1,2\right).$


 $\displaystyle -\frac{4\sqrt{10}}{5}.$


2886   

Determine o ponto do elipsóide $x^2 + 4y^2 + z^2 = 1$ que maximiza a soma $x + 2y + z$.


$\displaystyle \left( \frac{1}{\sqrt{3}}, \frac{1}{2\sqrt{3}}, \frac{1}{\sqrt{3}} \right).$


2835   

Suponha que um cientista tenha razões para acreditar que duas quantidades $x$ e $y$ estejam relacionadas linearmente, ou seja, $y=mx+b$, pelo menos aproximadamente, para algum valor de $m$ e de $b$. O cientista realiza uma experiência e coleta os dados na forma de pontos $(x_{1},y_{1}), (x_{2},y_{2}), \ldots, (x_{n},y_{n})$, e então coloca-os em um gráfico. Os pontos não estão todos alinhados, de modo que o cientista quer determinar as constantes $m$ e $b$ para que a reta $y=mx+b$ ``ajuste" os pontos tanto quanto possível (veja a figura). Seja $d_{i}=y_{i}-(mx_{i}+b)$ o desvio vertical do ponto $(x_{i},y_{i})$ da reta. O {\bf método dos mínimos quadrados} determina $m$ e $b$ de modo a minimizar $\sum_{i=1}^{n}d_{i}^{2}$, a soma dos quadrados dos desvios. Mostre que, de acordo com esse método, a reta de melhor ajuste é obtida quando

$$m\sum_{i=1}^{n}x_{i}+bn=\sum_{i=1}^{n}y_{i}$$

$$m\sum_{i=1}^{n}x_{i}^{2}+b\sum_{i=1}^{n}x_{i}=\sum_{i=1}^{n}x_{i}y_{i}$$

Assim, a reta é determinada resolvendo esse sistema linear de duas equações nas incógnitas $m$ e $b.$

ma211-list5-ex21.png


As duas equações são obtidas como pontos críticos da função $\displaystyle \sum^{n}_{i = 1} d_{i}^{2} = \sum^{n}_{i = 1} \left(y_{i} - (mx_{i} + b) \right)^{2} = f(m,b).$ Note que de fato pontos satisfazendo as equações são pontos de mínimo de $f.$


2778   

Esboce a região cuja área é dada pela integral e calcule-a:  $\displaystyle\int_{0}^{\pi/2}  \int_{0}^{4\cos{\theta}}   r \, dr d\theta$


$2\pi;$ região de integração:


ma211-list7-ex6_sol_b.png


2461   

Calcule a integral de superfície $\displaystyle\iint \limits_{ S}{\bf F}\cdot d{\bf S}$ para o campo vetorial ${\bf F}$ e superfície orientada $S$ dados abaixo. Em outras palavras, determine o fluxo de ${\bf F}$ através de $S$. Para superfícies fechadas, use a orientação positiva (para fora).

  • ${\bf F}(x,y,z)=xy{\bf i}+yz{\bf j}+zx{\bf k}$ e $S$ é a parte do parabolóide $z=4-x^{2}-y^{2}$ que está acima do quadrado $0\leq x\leq 1$, $0\leq y\leq 1$, com orientação para cima.



$\dfrac{713}{180}.$


2021   

Seja $g(x,y)=f(x^{2}+y^{2})$, onde $f:\mathbb{R}\rightarrow \mathbb{R}$ é uma função diferenciável. Mostre que 
$$y\frac{\partial g}{\partial x}-x\frac{\partial g}{\partial y}=0.$$



Observe que $f$ é uma função de uma variável. Logo, utilizando a Regra da Cadeia para funções de uma variável, obtemos
$$\frac{\partial g}{\partial x}(x,y) = f'(x^2+y^2) (2x)$$
e
$$\frac{\partial g}{\partial y}(x,y) = f'(x^2+y^2) (2y).$$
Portanto
$$y\frac{\partial g}{\partial x}-x\frac{\partial g}{\partial y}=0.$$


2477   

Determine e faça o esboço do domínio da função $f(x,y,z)=\sqrt{1-x^{2}-y^{2}-z^{2}}$.


$\left\lbrace (x,y);\; x^{2} + y^{2} + z^{2} \leq 1 \right\rbrace.$

ma211-list2-ex10_sol_e.png


2635   

A temperatura $T$ de uma localidade do Hemisfério Norte depende da longitude $x$, da latitude $y$ e do tempo $t$, de modo que podemos escrever $T=f(x,y,t)$. Vamos medir o tempo em horas a partir do início de Janeiro.

  1. Qual é o significado das derivadas parciais $\partial T/\partial x$, $\partial T/\partial y$ e $\partial T/\partial t$?

  2. Honolulu (você sabe onde fica?) tem longitude de $158^{\circ}W$ e latitude de $21^{\circ}N$. Suponha que às 9 horas em $1^{\circ}$ de Janeiro esteja ventando para nordeste uma brisa quente, de forma que a oeste e a sul o ar esteja quente e a norte e leste o ar esteja mais frio. Você esperaria que $f_{x}(158,21,9)$, $f_{y}(158,21,9)$ e $f_{t}(128,21,9)$ fossem positivas ou negativas? Explique.


  1. $\partial T/\partial x$ é a taxa de variação da temperatura quando a longitude muda, mas a latitude e o tempo são constantes;
    $\partial T/\partial y$ é a taxa de variação da temperatura quando a latitude muda, mas a longitude e o tempo são constantes;
    $\partial T/\partial t$ é a taxa de variação da temperatura quando o tempo muda, mas a longitude e a latitude são constantes.
  2. $f_{x}(158,21,9) > 0,$ $f_{y}(158,21,9) < 0$ e $f_{t}(158,21,9) > 0.$

3086   

Dado que \(\displaystyle x^3+y^2x-3=0\), determine \(\dfrac{dy}{dx}\) usando derivação implícita.



Derivando implicitamente a equação dada, temos que \(3x^2+y^2+x(2yy')-0=0\). Ou seja,

\[ \frac{dy}{dx}= -\frac{3x^2+y^2}{2xy}.\]


2006   

Determine o trabalho realizado pelo campo de força ${\bf F}(x,y)=x\,{\bf i}+(y+2)\,{\bf j}$ sobre um objeto que se move sobre um arco de cicloide ${\bf r}(t)=(t-\sin t)\,{\bf i}+(1-\cos t)\,{\bf j}$, $0\leq t\leq 2\pi.$


$2\pi^{2}.$


3100   

Mostre (verifique) que as integrais abaixo podem ser calculadas como:

  1.  \[ \int_1^5\int_2^{y/2}6x^2y\,dxdy = \int_1^5\left(\dfrac{1}{4}y^4-16y\right)\,dy \]

  2.  \[ \int_1^5\int_2^{x/2}6x^2y\,dydx = \int_1^5\left(\dfrac{3}{4}x^4-12x^2\right)\,dx \]


2204   

Suponha que $u=f(x,y)$ e $v=g(x,y)$ verifiquem as equações de Cauchy- Riemann $u_{x}=v_{y}$ e $u_{y}=-v_{x}$. Se $x=r\cos{\theta}$ e 
$y=r\sin{\theta}$, mostre que 
$$\frac{\partial u}{\partial r}=\frac{1}{r}\frac{\partial v}{\partial \theta}  \text{ e }  \frac{\partial v}{\partial r}=-\frac{1}{r}\frac{\partial u}{\partial \theta}.$$



Note que $\displaystyle \frac{\partial u}{\partial r} = \cos(\theta) u_{x} + \sin (\theta) u_{y},$ $\displaystyle \frac{\partial v}{\partial r} = \cos(\theta) v_{x} + \sin (\theta) v_{y},$ 
$\displaystyle \frac{\partial u}{\partial \theta} = -r\sin(\theta) u_{x} + r \cos(\theta) u_{y}$ e $\displaystyle \frac{\partial v}{\partial \theta} = - r\sin(\theta) v_{x} + r \cos(\theta) v_{y}$.


2645   

Determine $\partial z/\partial x$ e $\partial z/\partial y$, sendo $z=f(x)+g(y)$.


$\displaystyle \frac{\partial z}{\partial x} =  f'(x)$

$\frac{\partial z}{\partial y} = g'(y)$.


2469   

Calcule a integral de superfície $\displaystyle\iint \limits_{ S}{\bf F}\cdot d{\bf S}$ para o campo vetorial ${\bf F}$ e superfície orientada $S$ dados abaixo. Em outras palavras, determine o fluxo de ${\bf F}$ através de $S$. Para superfícies fechadas, use a orientação positiva (para fora).

  • ${\bf F}(x,y,z)=(x+y){\bf i}+z{\bf j}+xz{\bf k}$ e $S$ é a superfície do cubo de vértices $(\pm 1,\pm 1, \pm 1).$


$8.$


2601   

Seja $E$ a região limitada pelos paraboloides $z = x^2 + y^2$ e $z = 36 - 3x^2 - 3y^2$.

  1.  Ache o volume da região $E$.

  2.  Encontre o centroide de $E$ (centro de massa no caso em que a densidade é constante).


  1.  $162\pi.$

  2.  $(0,0,15)$.


2814   

Determine os valores máximo e mínimo absolutos de $f$ no conjunto $D.$

$f(x,y)=xy^{2}$, $D=\{(x,y) \in \mathbb{R}^2: x\geq 0,\;y\geq 0,\;x^{2}+y^{2}\leq3\}.$


Valor máximo: $2;$ valor mínimo: $0.$


2766   

Use a derivação implicíta para determinar $\partial z/\partial x$ e $\partial z/\partial y$ na expressão $\sin(xyz)=x+2y+3z$.


$\displaystyle \frac{\partial z}{\partial x} = \frac{1 - yz \cos(xyz)}{xy\cos(xyz) - 3}$

$\displaystyle \frac{\partial z}{\partial y} = \frac{2 - xz \cos(xyz)}{xy\cos(xyz) - 3} $.


2328   

Determine a área da superfície dada pela parte do plano $3x+2y+z=6$ que está no primeiro octante.


$3\sqrt{14}.$


2997   

Calcule a integral, efetuando uma mudança de variáveis apropriada. $\displaystyle\iint\limits_{R} \dfrac{\cos{(x-y)}}{\sin{(x+y)}} \, dA$, em que $R$ é a região trapezoidal com vértices $(1,0)$, $(2,0)$, $(0,2)$ e $(0,1)$.


 $1.$


2207   

Calcule as integrais iteradas.

  1. $\displaystyle\int_{0}^{\pi/2}\int_{0}^{\cos{\theta}}e^{\sin{\theta}}\,dr d\theta$

  2. $\displaystyle\int_{0}^{1}\int_{0}^{v}\sqrt{1-v^{2}}\,du dv$


  1. $e - 1.$

  2.  $\dfrac{1}{3}.$


3004   

Determine a massa e o centro de massa da lâmina que ocupa a região $D$ e tem função densidade $\rho$, sendo: $D$ delimitada pelas parábolas $y = x^2$ e $x = y^2; \quad \rho(x,y) = \sqrt{x}$.


Massa: $\dfrac{3}{14};$ centro de massa: $\displaystyle \left(\frac{14}{27},\frac{28}{55} \right).$


2972   

Calcule a integral $\displaystyle\iint\limits_{R} \dfrac{e^{y - x^2}}{y - x^2} dA$, em que $R$ é o conjunto de todos $(x,y)$ tais que $1 + x^2 \leq y \leq 2 + x^2$, $y \geq x + x^2$ e $x \geq 0$, efetuando uma mudança de variáveis apropriada.


Olhando o integrando, é natural pensar que uma das novas variáveis introduzidas deva ser $y-x^2$, mas a outra, a princípio, não está pré-definida. Seja $u = y - x^2$ (escolheremos $v$ apropriadamente depois). Vamos analisar a região de integração dada.

\qquad Como $1 + x^2 \leq y \leq 2 + x^2, \text{ temos } 1 \leq y-x^2 \leq 2$, isto é, $1 \leq u \leq 2$;

\qquad Como $y \geq x + x^2$ e $x \geq 0$, temos $y-x^2 \geq x \geq 0$, isto é, $u \geq x \geq 0$.

Da análise acima, é natural pensar na outra variável como sendo $v = x$. Considere então a mudança de variáveis dada por

$$\begin{cases}x = v, \\y = u+v^2.\end{cases}$$

O Jacobiano dessa transformação é

$$\dfrac{\partial (x,y)}{\partial (u,v)} = \left| \begin{array}{cc} \dfrac{\partial x}{\partial u} & \dfrac{\partial x}{\partial v}  \\ \dfrac{\partial y}{\partial u} & \dfrac{\partial y}{\partial v} \end{array} \right| = \left| \begin{array}{cc} 0 & 1  \\ 1 & 2v \end{array} \right| = -1.$$

Como analisamos anteriormente, a nova região de integração é

$$S = \{(u,v) \in \mathbb{R}^2: 1 \leq u \leq 2 \mbox{ e } 0 \leq v \leq u\}.$$

Assim,

\begin{array}{rcl}\displaystyle\iint\limits_{R} \dfrac{e^{y - x^2}}{y - x^2} \, dA & = & \displaystyle \iint\limits_{S} \frac{e^u}{u} \left|\frac{\partial (x,y)}{\partial (u,v)}\right|\, dv du \\& = & \displaystyle\int_{1}^{2}\int_{0}^{u} \frac{e^u}{u} (1) \, dv du \\  & = & \displaystyle\int_{1}^{2} \left.\left(\frac{v e^u}{u}\right|_{v=0}^{v=u}\right) \, du \\    & = & \displaystyle\int_{1}^{2} e^u \, du \\    & = & e^u |_{1}^{2} = e^2 - e.\end{array}



2728   

Quatro números positivos, cada um menor que $50$, são arredondados até a primeira casa decimal e depois multiplicados. Utilize os diferenciais para estimar o máximo erro possível no cálculo do produto que pode resultar do arredondamento.


Se $x,y,z,w$ são os quatro números e $p(x,y,z,w) = xyzw,$ temos $\Delta p \leq 25000.$


2812   

Determine os valores máximo e mínimo absolutos de $f$ no conjunto $D.$

$f(x,y)=3+xy-x-2y$, $D$ é a região triangular fechada com vértices $(1,0)$, $(5,0)$ e $(1,4).$


Valor máximo: $2;$  valor mínimo: $-2.$


2517   

Dada a expressão $g(x,y)=f(x,y)+2$, escreva como o gráfico de $g$ é obtido a partir do gráfico de $f.$


Gráfico de $f$ deslocado para cima por duas unidades.


2275   

Calcule $\nabla f(x,y)$.
$f(x,y) = x^2y$


$\displaystyle \nabla f(x,y) = (2xy,x^{2}).$


2790   

Determine os valores máximos e mínimos locais e pontos de sela da função $f(x,y)=x^{2}+3xy+4y^{2}-6x+2y$.


Ponto de mínimo: $\displaystyle \left( \frac{54}{7}, -\frac{22}{7} \right).$


3126   

Verifique que para o vetor posição \(\mathbf{r}=x\mathbf{i}+y\mathbf{j}+z\mathbf{k}\) valem as seguintes propriedades

  1.  \(\displaystyle \mathrm{rot\,}\mathbf{r} = \mathbf{0}\)

  2.  \(\displaystyle \nabla\|\mathbf{r}\| = \dfrac{\mathbf{r}}{\|\mathbf{r}\|} \)


3138   

Use o Teorema da Divergência para encontrar todos os valores positivos \(k\) tais que \[ \mathbf{F}(\mathbf{r}) = \dfrac{\mathbf{r}}{\|\mathbf{r}\|^k} \] satisfaça a condição \(\mathrm{div\,}\mathbf{F}=0\) quando \(\mathbf{r}\neq \mathbf{0}\).


2550   

Suponha que $\displaystyle \lim_{(x,y) \to (3,1)}f(x,y) = 6$. O que podemos dizer do valor de $f(3,1)$? E se a função $f$ for contínua?


Nada se pode afirmar. Se $f$ for contínua em $(x_{0},y_{0}),$ $f(3,1) = 6.$


2321   

  1. Determine uma representação paramétrica ${\bf r}:D\subset \mathbb{R}^{2}\rightarrow \mathbb{R}^{3}$ do paraboloide elíptico $z=\dfrac{x^{2}}{a^{2}}+\dfrac{y^{2}}{b^{2}}.$

  2. Calcule a equação do plano tangente à superfície paramétrica dada no item (a) no ponto $(-a\pi,0,\pi^{2}).$


  1. $x = u,$ $y = v,$ $z = \dfrac{u^{2}}{a^{2}}+\dfrac{v^{2}}{b^{2}},$ onde $u,v \in \mathbb{R}.$

  2. $2\pi(x + a\pi) + a(z - \pi^{2}) = 0.$


1926   

Determine o campo vetorial gradiente de  $f(x,y) = \ln(x + 2y)$.


$\nabla f(x,y) = \dfrac{\textbf{i} + 2\textbf{j}}{x + 2y}.$


2410   

Inverta a ordem de integração.

  1.   $\displaystyle\int_{0}^{1}\bigg[\int_{\sqrt{x-x^{2}}}^{\sqrt{2x}}f(x,y)\,dy\bigg]dx$

  2.   $\displaystyle\int_{0}^{3a}\bigg[\int_{\frac{\sqrt{3}}{3}x}^{\sqrt{4ax-x^{2}}}f(x,y)\,dy\bigg]dx, \; a> 0.$

  3.   $\displaystyle\int_{0}^{\pi}\bigg[\int_{0}^{\sin{x}}f(x,y)\,dy\bigg]dx$



  1.  $\  \\ \begin{array}{ll} \displaystyle\int_{0}^{\frac{1}{2}}\bigg[\int_{\frac{y^{2}}{2}}^{\frac{1}{2} - \sqrt{\frac{1}{4} - y^{2}}}f(x,y)\,dx\bigg]dy &+ \displaystyle\int_{0}^{\frac{1}{2}}\bigg[\int_{\frac{1}{2}+ \sqrt{\frac{1}{4} - y^{2}}}^{1}f(x,y)\,dx\bigg]dy\\ &+ \displaystyle\int_{\frac{1}{2}}^{\sqrt{2}}\bigg[\int_{\frac{y^{2}}{2}}^{1}f(x,y)\,dx\bigg]dy \end{array} $

  2.   $\displaystyle\int_{0}^{\sqrt{3}a} \bigg[\int_{2a + \sqrt{4a^2 - y^{2}}}^{\sqrt{3} y}f(x,y)\,dx\bigg]dy.$

  3.   $\displaystyle\int_{0}^{1}\bigg[\int_{\arcsin(y)}^{\pi-\arcsin(y)}f(x,y)\,dx\bigg]dy$


2668   

Seja $\phi:\mathbb{R}\rightarrow \mathbb{R}$ uma função de uma variável real, diferenciável e tal que $\phi '(1)=4.$ Seja $g(x,y)=\phi\bigg(\dfrac{x}{y}\bigg).$ Verifique que, para todo $(x,y)\in \mathbb{R}^{2}$, com $y\neq 0$, temos que

$$x\;\dfrac{\partial g}{\partial x}(x,y)+y\;\dfrac{\partial g}{\partial y}(x,y)=0.$$


2215   

Determine o rotacional e o divergente do campo vetorial $\mathbf{F}(x,y,z) = e^x\sin{y}\mathbf{i} + e^x\cos{y}\mathbf{j} + z\mathbf{k}$.


$\text{rot } \mathbf{F} = \bf{0}.$ $\text{div } \mathbf{F} = 1.$


2415   

Calcule a integral trocando a ordem de integração.

  1.  $\displaystyle\int_{0}^{4}\!\!\int_{\sqrt{x}}^{2}\dfrac{1}{y^{3}+1}\,dy dx$

  2.  $\displaystyle\int_{0}^{\pi}\!\!\int_{x}^{\pi}\dfrac{\sin{y}}{y}\,dy dx$

  3.  $\displaystyle\int_{0}^{2}\!\!\int_{x}^{2}2y^{2}\sin(xy)\,dy dx.$


  1.  $\dfrac{\ln(9)}{3}.$

  2.  $2.$

  3.  $4 - \sin(4).$


2319   

Determine uma equação do plano tangente à superfície parametrizada dada no ponto especificado. ${\bf r}(u,v)=(\arctan (uv),e^{u^{2}-v^{2}},u-v)$, no ponto ${\bf r}(1,-1).$


$(x,y,z) = \left(-\dfrac{\pi}{4},1,2\right) + s\left(-\dfrac{1}{2},2,1\right) + t\left(\dfrac{1}{2},2,-1\right),$ $s,t \in \mathbb{R}.$


3054   

Esboce o sólido cujo volume é dado pela integral iterada.

$\displaystyle\int_{0}^{2}\int_{0}^{2-y}\int_{0}^{4-y^{2}}\;dx dz dy$



ma211lista8q12bres.png

2182   

Nos item abaixo: 

  1. expresse $\mathrm{d} w/\mathrm{d} t$ como uma função de $t$, usando a Regra da Cadeia, expressando $w$ em termos de $t$ e diferenciando em relação a $t$;
  2. calcule $\mathrm{d} w/\mathrm{d} t$ no valor dado de $t$.


$w=x^{2}+y^{2}$,  $x=\cos{t}$,  $y=\sin{t}$;  $t=\pi.$




  1. $\displaystyle \frac{dw}{dt}(t) = 0.$
  2. $\displaystyle \frac{dw}{dt}(\pi) = 0.$


2719   

Determine a aproximação linear da função $f(x,y) = \sqrt{20 - x^2 - 7y^2}$ em $(2,1)$ e use-a para aproximar $f(1,95; 1,08)$.


$L(x,y) = -\frac{2}{3}x - \frac{7}{3}y + \frac{20}{3}$  e $f(1,95; 1,08) \approx 2.847.$


2514   

Faça uma correspondência entre a função: (i) e seu gráfico; (ii) e seus mapas de contorno. Justifique sua escolha.

  1. $z=\sin(xy)$

  2. $z=\sin(x-y)$

  3. $z=(1-x^{2})(1-y^{2})$

  4. $z=e^{x} \; \cos{y}$

  5. $z=\sin{x}-\sin{y}$

  6. $z=\dfrac{x-y}{1+x^{2}+y^{2}}$

ma211-list2-ex19_1.png

ma211-list2-ex19_2.png


1934   

Calcule a integral de linha, onde $C$ é a curva dada.

$\displaystyle\int_{C}(x^{2}y^{3}-\sqrt{x})\,dy$,   $C$ é o arco da curva $y=\sqrt{x}$ de $(1,1)$ a $(4,2).$


$\dfrac{243}{8}.$


2441   

Calcule a integral de superfície $\displaystyle\iint\limits_{S}z dS$, onde $S$ é a superfície $x=y+2z^{2}$, $0 \leq y\leq 1$, $0 \leq z \leq 1.$


$\dfrac{13\sqrt{2}}{12}.$


2364   

Determine as direções em que a derivada direcional da função \linebreak $f(x,y) = x^2 + \sin{xy}$ no ponto $(1,0)$ tem valor 1.


As direções são dadas pelos vetores $(1,0)$ e $\displaystyle \left( \frac{4}{5}, -\frac{3}{5}\right).$


2202   

Os lados iguais e o ângulo correspondente de um triângulo isósceles estão aumentando à razão de $3cm/h$ e $2^{\circ}/h$, respectivamente. Ache a taxa à qual a área do triângulo está aumentando no instante em que o comprimento de cada um dos 

lados iguais é de $6$ metros e o ângulo correspondente é $60^{\circ}.$


$\approx 181559$ cm$^{2}/$h.


2393   

Determine a equação da reta tangente à curva de nível dada, no ponto dado.

$x^2 + xy + y^2 - 3y = 1$, em $(1,2)$.


$y - 2 = -2(x - 1).$


2742   

Determine os planos que são tangentes ao gráfico de $f(x,y) = x^2 + y^2$ e que contenham a interseção dos planos $x + y + z = 3$ e $z = 0$.


$z = 0$ e $z = 6x + 6y - 18.$


2988   

Calcule a integral, efetuando uma mudança de variáveis apropriada. $\displaystyle\iint\limits_{R} \sin{(9x^2 + 4y^2)} \, dA$, em que $R$ é a região do primeiro quadrante limitada pela elipse $9x^2 + 4y^2 = 1$.


$\dfrac{\pi}{24}(1 - \cos(1)).$


2262   

Calcule $\displaystyle\iint\limits_{B} y\,dx dy$, onde $B$ é o conjunto dado.

  1.  $B$ é a região compreendida entre os gráficos de $y=x$ e $y=x^{2}$, com $0\leq x\leq 2.$

  2.  $B$ é o paralelogramo de vértices $(-1,0)$, $(0,0)$, $(1,1)$ e $(0,1).$

  3.  $B$ é o semicírculo $x^{2}+y^{2}\leq 4$, $y\geq 0.$

  4.  $B=\{(x,y)\in \mathbb{R}^{2}|\;x\geq 0,\;x^{5}-x\leq y \leq 0\}.$


  1.  $2$.

  2.  $\dfrac{1}{2}$.

  3.  $\dfrac{16}{3}$.

  4.  $-\dfrac{16}{231}$.


3095   

Cada integral iterada abaixo representa o volume de um sólido. Faça um esboço do sólido. (Não é necessário calcular o volume.)

  1.  \(\displaystyle \int_0^1\int_0^1 (2-x-y)\, dydx\)

  2.  \(\displaystyle \int_{-2}^2\int_{-2}^2(x^2+y^2)\,dxdy\)


3123   

Se \(x=x(u,v,w)\), \(y=y(u,v,w)\) e \(z=z(u,v,w)\) for uma transformação injetora, então \(u=u(x,y,z)\), \(v=v(x,y,z)\) e \(w=w(x,y,z)\). Supondo a diferenciabilidade das funções, mostre que \[\dfrac{\partial(x,y,z)}{\partial(u,v,w)}\cdot\dfrac{\partial(u,v,w)}{\partial(x,y,z)} = 1.\] Use este resultado para mostrar que o volume \(V\) do paralelepípedo oblíquo limitado pelos planos \(x+y+2z=\pm 3\), \(x-2y+z=\pm 2\), \(4x+y+z=\pm 6\) é dado por \(V=16\).


2532   

Seja $f(x,y)=e^{xy}$ uma função de duas variáveis.

  1. Determine o domínio e a imagem de $f.$

  2. Esboce as curvas de nível de $f.$


  1. $D_{f} = \mathbb{R}^{2}$ e $Im(f) = \left\lbrace z \in \mathbb{R};\; z > 0 \right\rbrace.$

  2. $xy = C.$

ma211-list2-ex26_sol.png


2007   

A força em um ponto $(x,y)$ de um plano coordenado é ${\bf F}(x,y)=(x^{2}+y^{2})\,{\bf i}+xy\,{\bf j}$. Ache o trabalho realizado por ${\bf F}(x,y)$ ao longo do gráfico de $y=x^{3}$ de $(0,0)$ a $(2,8).$


$\dfrac{1592}{21}.$


2129   

Considere a função $F(x,y)=f\bigg(\dfrac{x}{y},\dfrac{y}{x}\bigg)$. Mostre que

$$x\dfrac{\partial F}{\partial x}+y\dfrac{\partial F}{\partial y}=0.$$



Note que$\displaystyle \frac{\partial F}{\partial x} = \frac{1}{y}\frac{\partial f}{\partial x}\left(\frac{x}{y}, \frac{y}{x} \right) - \frac{y}{x^{2}} \frac{\partial f}{\partial y}\left(\frac{x}{y}, \frac{y}{x} \right)$ e $\displaystyle \frac{\partial F}{\partial y} = -\frac{x}{y^{2}} \frac{\partial f}{\partial x}\left(\frac{x}{y}, \frac{y}{x} \right) + \frac{1}{x} \frac{\partial f}{\partial y}\left(\frac{x}{y}, \frac{y}{x} \right).$ 


2389   

Determine as equações do plano tangente e da reta normal à superfície dada, no ponto dado.

$x^2 - 2y^2 + z^2 + yz = 2$ em $(2,1,-1).$

 


Plano tangente: $4x - 5y - z = 4$,
Reta normal: $(x,y,z) = (2,1,-1) + \lambda (4,-5,-1),$ $\lambda \in \mathbb{R}.$


2619   

Suponha que $S$ e $C$ satisfaçam as hipóteses do Teorema de Stokes e $f$ e $g$ tenham derivadas parciais de segunda ordem contínuas. Demonstre que $\displaystyle\int_C (f\nabla g)\cdot d{\bf R} = \displaystyle\iint_{S} (\nabla f \times \nabla g)\cdot d{\bf S}$


Note que $\mbox{rot} (f\nabla g) = \nabla f \times \nabla g.$


2528   

Represente graficamente o domínio da função $z=f(x,y)$ dada por $x+y-1+z^{2}=0$, $z\geq 0$.


$\left\lbrace (x,y); x + y \leq 1 \right\rbrace$

ma211-list2-ex25_sol_a.png


2590   

Use a Lei de Gauss para achar a carga contida no hemisfério sólido $x^{2}+y^{2}+z^{2} \leq a^{2}$, $z\geq 0$, se o campo elétrico for ${\bf E}(x,y,z)=x{\bf i}+y{\bf j}+2z{\bf k}$.


$\dfrac{8\pi a^3 \epsilon_{0}}{3}$.


2033   

Utilize a Regra da Cadeia para determinar $\mathrm{\partial}z/\mathrm{\partial} s$ e $\mathrm{\partial}z/ \mathrm{\partial}t.$

$z=e^{x+2y}$, $x=s/t$, $y=t/s$.


$\displaystyle \frac{\partial z}{\partial s} = e^{x + st}\left(\frac{1}{t} - \frac{2t}{s^{2}} \right) $ e $\displaystyle \frac{\partial z}{\partial t} = e^{x + st}\left(\frac{2}{s} - \frac{s}{t^{2}} \right) $.


2124   

Suponha que, para todo $x$,$f(3x,x^{3})=\arctan(x)$.

  1.  Calcule $\dfrac{\partial f}{\partial x}(3,1)$ admitindo $\dfrac{\partial f}{\partial y}(3,1)=2$.
  2.  Determine a equação do plano tangente ao gráfico de $f$ no ponto $(3,1,f(3,1))$.


  1.  $\dfrac{\partial f}{\partial x}(3,1) = -\frac{11}{6}.$
  2.  $\displaystyle z - \frac{\pi}{4} = -\frac{11}{6}(x - 3) + 2(y - 1).$


3108   

Encontre a área da superfície descrita como sendo a parte do cone \(z^2=4x^2+4y^2\) que está acima da região do primeiro quadrante limitada pela reta \(y=x\) e a parábola \(y=x^2\).


\( \dfrac{\sqrt{5}}{6}\)


2437   

Calcule a integral de superfície $\displaystyle\iint\limits_{S}x^{2}yz dS$, onde $S$ é a parte do plano $z=1+2x+3y$ que está acima do retângulo $[0,3]\times [0,2].$


$171\sqrt{14}.$



2727   

Se $R$ é a resistência equivalente de três resistores conectados em paralelo, com resistências $R_1, R_2, R_3$, então

$$\dfrac{1}{R} = \dfrac{1}{R_1} + \dfrac{1}{R_2} + \dfrac{1}{R_3}.$$

Se as resistências medem, em ohms, $R_1 = 25  \Omega$, $R_2 = 40 \Omega$, $R_3 = 50 \Omega$, com margem de erro de $0,5\%$ em cada uma, estime o erro máximo no valor calculado de $R$.


$\Delta R \approx 0.059 \Omega$.


2131   

$f(x,y,z)$ e $g(x,y)$ são funções diferenciáveis tais que, para todo $(x,y)$ no domínio de $g,f(x,y,g(x,y))=0$. 
Suponha $g(1,1)=3$, $\dfrac{\partial f}{\partial x}(1,1,3)=2$, $\dfrac{\partial f}{\partial y}(1,1,3)=5$ e $\dfrac{\partial f}{\partial z}(1,1,3)=10.$ 
Determine a equação do plano tangente ao gráfico de $g$ no ponto $(1,1,3).$


$\displaystyle z - 3 = -\frac{1}{5}(x - 1) - \frac{1}{2} (y-1).$


2684   

Encontre $\partial f/\partial x$ e $\partial f/\partial y$ para $f(x,y)=e^{xy}\ln{y}$.


$\displaystyle \frac{\partial f}{\partial x} = ye^{xy}\ln y\;\;\;\;\text{e}\;\;\;\; \frac{\partial f}{\partial y} = x e^{xy} \ln y + \frac{e^{xy}}{y}$.


2948   

Calcule utilizando coordenadas esféricas. $\displaystyle\iiint\limits_{B}z\,dxdydz$, onde $B$ é o conjunto $z\geq \sqrt{x^{2}+y^{2}}$ e $x^{2}+y^{2}+z^{2}\leq 1.$


$\dfrac{\pi}{8}.$


3157   

Considerando um campo vetorial $\mathbf{F}$ que representa a velocidade em um fluido, a interpretação do rotacional é que partículas em um ponto $(x,y,z)$ de um fluido tendem a rodar em torno do eixo que aponta na direção de $\text{rot }\mathbf{F}(x,y,z)$. Se $\mathbf{F}$ representar a velocidade da corrente de um rio calmo, que corre somente da direção montante à jusante, podemos dizer que $\text{rot } \mathbf{F}$ é igual ou diferente de zero? Por quê? Para auxiliar na interpretação, faça um esboço do gráfico de $\mathbf{F}$ assumindo que ele não varia na direção $z$.


Igual a zero.


2052   

Determine se ${\bf F}(x,y)=-y\,{\bf i}+x\,{\bf j}$ é ou não um campo vetorial conservativo. Se for, determine uma função $f$ tal que ${\bf F}=\nabla f.$


Não.


2700   

Verifique que $\dfrac{\partial ^{2}f}{\partial x^{2}}+\dfrac{\partial ^{2}f}{\partial y^{2}}=0$, onde $f(x,y)=\ln(x^{2}+y^{2}).$


$\displaystyle \frac{\partial^{2} f}{\partial x^{2}}= \frac{2 y^{2} - 2 x^{2}}{(x^{2} + y^{2})^{2}}\;\;\;\;\;\text{e}\;\;\;\;\; \frac{\partial^{2} f}{\partial y^{2}}= \frac{2 x^{2} - 2 y^{2}}{(x^{2} + y^{2})^{2}}.$


2227   

Determine se o campo vetorial $\mathbf{F}(x,y,z) = ye^{-x}\mathbf{i} + e^{-x}\mathbf{j} + 2z\mathbf{k}$ é conservativo ou não. Se for conservativo, determine uma função $f$ tal que $\mathbf{F} = \nabla{f}$.


$\mathbf{F}$ não é conservativo.


2484   

Use a integral tripla para determinar o volume do sólido dado.

  1.  $x^{2}\leq z \leq 1-y$ e $y\geq 0.$

  2.  $x^{2}+2y^{2}\leq z\leq 2a^{2}-x^{2}$ $(a>0).$

  3.  $x^{2}+y^{2}+(z-1)^{2}\leq 1$ e $z\geq x^{2}+y^{2}.$

  4.  $4x^{2}+9y^{2}+z^{2}\leq 4$ e $4x^{2}+9y^{2}\leq 1.$


  1.  $\dfrac{4}{15}.$

  2.  $\pi a^4.$

  3.  $\dfrac{71\pi}{54}.$

  4.  $\dfrac{7\pi}{12}.$


3148   

Seja \(G\) um sólido com a superfície \(\sigma\) orientada por vetores normais unitários para fora, suponha que \(\phi\) tenha derivadas parciais de primeira e segunda ordens contínuas em algum conjunto aberto contendo \(G\) e seja \(D_{\mathbf{n}}\phi\) a derivada direcional de \(\phi\), onde \(\mathbf{n}\) é um vetor normal unitário para fora de \(\sigma\). Mostre que \[ \iint\limits_\sigma D_{\mathbf{n}}\phi\,dS = \iiint\limits_G\left[\dfrac{\partial^2\phi}{\partial x^2}+ \dfrac{\partial^2\phi}{\partial y^2} + \dfrac{\partial^2\phi}{\partial z^2} \right]\,dV. \]


2391   

Calcule a integral de superfície $\displaystyle\iint\limits_{S}x^{2}z^{2}dS$, onde $S$ é a parte do cone $z^{2}=x^{2}+y^{2}$ que está entre os planos $z=1$ e $z=3.$



Temos que $S$ é a porção do cone $z^{2}=x^{2}+y^{2}$ para $1 \leq z \leq 3$, ou equivalentemente, $S$ é a parte da superfície $z=\sqrt{x^{2}+y^{2}}$ sobre a região $D=\{(x,y)| 1 \leq x^{2}+y^{2} \leq 9\}.$ Assim,
$\displaystyle\iint\limits_{S}x^{2}z^{2}dS=\displaystyle\iint\limits_{D}x^{2}(x^{2}+y^{2})\sqrt{\left(\dfrac{\partial z}{\partial x}\right)^{2}
+\left(\frac{\partial z}{\partial y}\right)^{2}+1}dA$
$=\displaystyle\iint\limits_{D}x^{2}(x^{2}+y^{2})\sqrt{\left(\frac{x}{\sqrt{x^{2}+y^{2}}}\right)^{2}+\left(\frac{y}{\sqrt{x^{2}+y^{2}}}\right)^{2}+1}dA$
$=\displaystyle\iint\limits_{D}x^{2}(x^{2}+y^{2})\sqrt{\frac{x^{2}+y^{2}}{x^{2}+y^{2}}+1}dA=\iint\limits_{D}\sqrt{2}x^{2}(x^{2}+y^{2})dA$
$=\sqrt{2}\displaystyle\iint\limits_{D}x^{2}(x^{2}+y^{2})dA.$
Por coordenadas polares, temos que $x=r\cos \theta, y=r\sin \theta, 1\leq r\leq 3 , 0\leq \theta \leq 2\pi \,\mbox{e} \, dA=r dr d\theta.$
Logo,
$\displaystyle\iint\limits_{S}x^{2}z^{2}dS=\sqrt{2}\int_{0}^{2\pi}\int_{1}^{3}(r^{2}\cos^{2}\theta)(r^{2})r dr d\theta =\sqrt{2}\int_{0}^{2\pi}\cos^{2}\theta d\theta \cdot \int_{1}^{3}r^{5}dr$

$=\sqrt{2}\cdot (\theta)\bigg|_{0}^{2\pi}\cdot \bigg(\frac{r^{6}}{6}\bigg)\bigg|_{1}^{3}=\sqrt{2}\cdot \pi \cdot \frac{1}{6}\cdot (3^{6}-1)=\frac{364\sqrt{2}}{3}\pi$


3101   

Seja \(R\) a região triangular de vértices \((0,0)\), \((3,3)\) e \((0,4)\) do plano \(xy\). Expressa como uma integral dupla, qual é área de \(R\)?


\(\displaystyle A(R)=\int_0^3\int_x^{-\frac{1}{3}x+4}\,dydx \)


3052   

Esboce o sólido descrito pelas desigualdades $0 \leq r \leq 2$, $-\pi/2 \leq \theta \leq \pi/2$ e $0 \leq z \leq 1$.


ma211lista8q29res.png


1988   

As curvas ${\bf r}_{1}(t)=(t,t^{2},t^{3})$ e ${\bf r}_{2}(t)=(\sin{t},\sin{2t},t)$ se interceptam na origem. Determine o ângulo 
de intersecção destas com precisão de um grau.


1924   

Calcule a integral de linha, onde $C$ é a curva dada. $\displaystyle\int_{C}x\,dx-y\,dy$, $C$ é o segmento de extremidades $(1,1)$ e $(2,3)$, percorrido no sentido de $(1,1)$ para $(2,3).$



Uma representação paramétrica para o segmento de reta $C$ é

$$\begin{array}{lr}x=1+t \\y=1+2t\\\end{array}\;\;\;\; 0\leq t \leq 1.$$

Logo,

$$\begin{array}{lr}dx=dt \\dy=2\,dt\\\end{array}$$


Assim,

$$\int_{C}x\,dx-y\,dy=\int_{0}^{1}(1+t)\cdot (dt)+(1+2t)\cdot(2\,dt)=\int_{0}^{1}(1+t+2+4t)\,dt$$

$$=\int_{0}^{1}(3+5t)\,dt=\bigg(3t+\frac{5}{2}t^{2}\bigg)\bigg|_{0}^{1}=3+\frac{5}{2}=\frac{11}{2}.$$


3072   

A figura mostra uma curva $C$ e um mapa de contorno de uma função $f$ cujo gradiente é contínuo. Determine $\int_{C}\nabla f\cdot d{\bf r}.$

ma211-list11-ex4.png


$40.$


3104   

Use coordenadas polares para calcular a integral dupla

\[ \iint_R e^{-(x^2+y^2)}\,dA, \]

onde \(R\) é a região contida no círculo \(x^2+y^2=1\).



\(\displaystyle (1-e^{-1})\pi \)


2803   

Determine os valores máximos e mínimos locais e pontos de sela da função $f(x,y)=x^{4}+y^{4}+4x+4y$.


Ponto de mínimo : $(-1,-1).$


2595   

Identifique a superfície cuja equação é dada por $z = 4 - r^2$.


$z = 4 - x^2 - y^2,$ o parabolóide circular com vértice $(0,0,4)$.


1954   

Calcule a integral de linha $\int_{C}{\bf F}\cdot d{\bf r}$, onde $C$ é dada pela função vetorial ${\bf r}(t).$

${\bf F}(x,y)=x^{2}\,{\bf j}$, ${\bf r}(t)=(t^{2},3)$, $-1\leq t\leq 1.$


$0.$


2993   

Calcule a integral, efetuando uma mudança de variáveis apropriada. $\displaystyle\iint\limits_{R} x \, dA$, em que $R$ é o conjunto, no plano $xy$, limitado pela cardioide $\rho = 1 - \cos{\theta}$.


$-\dfrac{5\pi}{4}.$


2303   

Identifique e faça um esboço da imagem da superfície parametrizada dada por ${\bf r}(u,v)=(v\,\cos u,v\sin u,v)$, $0\leq u\leq 2\pi$,\, $0\leq v \leq h$, onde $h>0$ é um real dado.


Face lateral do cone $\sqrt{x^{2} + y^{2}} \leq z \leq h$.


2529   

Represente graficamente o domínio da função $z=f(x,y)$ dada por $f(x,y)=\dfrac{x-y}{\sqrt{1-x^{2}-y^{2}}}$.


$\left\lbrace (x,y); x^{2} + y^{2} < 1 \right\rbrace$

ma211-list2-ex25_sol_b.png


2478   

Determine e faça o esboço do domínio da função $f(x,y,z)=\ln(16-4x^{2}-4y^{2}-z^{2})$.


$\left\lbrace (x,y);\; \frac{x^{2}}{4} + \frac{y^{2}}{4} + \frac{z^{2}}{16} < 1\right\rbrace.$

ma211-list2-ex10_sol_f.png


3059   

Esboce o campo vetorial $\textbf{F}=\dfrac{1}{2}(\textbf{i} + \textbf{j})$, desenhando um diagrama.


2982   

Utilize a transformação dada para calcular a integral. $\displaystyle\iint\limits_{R} x^2 \, dA$, em que $R$ é a região limitada pela elipse $9x^2 + 4y^2 = 36$; $x = 2u$, $y = 3v$.


$6\pi.$


2965   

Dentre as coordenadas cilíndricas ou esféricas, utilize a que lhe parecer mais apropriada para determinaretermine o volume da menor cunha esférica cortada de uma esfera de raio $a$ por dois planos que se interceptam ao longo de um diâmetro com um ângulo de $\pi/6.$


$\dfrac{\pi a^3}{9}.$


2976   

Determine o jacobiano da transformação dada por: $x = \alpha \sin{\beta}, \quad y = \alpha \cos{\beta}$.


$-\alpha.$


2880   

  1. Mostre que o valor máximo de $a^2b^2c^2$ sobre uma esfera de raio $r$ centrada na origem de um sistema de coordenadas cartesianas $(a,b,c)$ é $(r^2/3)^3$.

  2. Usando o item anterior, mostre que, para números não negativos $a$, $b$ e $c$,

    $$(abc)^{\frac{1}{3}} \leq \frac{a + b + c}{3},$$

    isto é, a média geométrica de três números não negativos é menor que ou igual à média aritmética.


  1. Use multiplicadores de Lagrange para maximizar $f(a,b,c) = a^{2}b^{2}c^{2}$ sujeita a restrição $a^{2} + b^{2} + c^{2} = r^{2}.$

  2. Como $(\sqrt{a},\sqrt{b},\sqrt{c})$ está na esfera $a + b + c = r^{2},$ pelo item 1 segue que $abc = f(\sqrt{a},\sqrt{b},\sqrt{c}) \leq \left(\dfrac{r^{2}}{3}\right)^{3} = \left(\dfrac{a + b + c}{3}\right)^{3}.$


2559   

Determine o limite, se existir, ou mostre que o limite não existe.

$\displaystyle \lim_{(x,y) \to (0,0)} \dfrac{x^2 \ \mbox{sen}^2y}{x^2 + 2y^2}$.


$0.$


2144   

Verifique que o Teorema do Divergente é verdadeiro para o campo vetorial ${\bf F}$ na região $E.$

${\bf F}(x,y,z)=x\,{\bf i}+y\,{\bf j}+z\,{\bf k}$, $E$ é a bola unitária $x^{2}+y^{2}+z^{2}\leq 1.$


$\displaystyle\iint_{S} {\bf F} \cdot d{\bf S} = \iiint_{E} \mbox{div} {\bf F} dV =  4\pi.$


2193   

Calcule $\displaystyle\iint \limits_{S}{\bf u}\cdot {\bf n}\,dS$, sendo $B=\{(x,y,z)\in \mathbb{R}^{3}|\, x^{2}+y^{2}\leq 1,\,x^{2}+y^{2}\leq z \leq 5-x^{2}-y^{2}\}$ e ${\bf u}=3xy\,{\bf i}-\dfrac{3}{2}y^{2}\,{\bf j}+z\,{\bf k}.$


$36\pi.$


2220   

Seja $f$ um campo escalar e $\mathbf{F}$ um campo vetorial. Diga se cada expressão tem significado. Em caso negativo, explique por quê. Em caso afirmativo, diga se é um campo vetorial ou escalar.  

  1. $\text{div }{(\text{grad }{f})}$;

  2. $\text{rot }{(\text{rot }{\mathbf{F}})}$;

  3. $(\text{grad }{f}) \times (\text{div }{\mathbf{F}})$.


  1. $\text{div }{(\text{grad }{f})}$ é um campo escalar.

  2. $\text{rot }{(\text{rot }{\mathbf{F}})}$ é um campo vetorial.

  3. $(\text{grad }{f}) \times (\text{div }{\mathbf{F}})$  não tem significado pois $\text{div } \bf{F}$ é um campo escalar.


2078   

Calcule a integral dupla.

  1. $\displaystyle\iint\limits_{R} x\sin(x+y)\,dA, \quad R=[0,\pi/6]\times [0,\pi/3].$

  2.  $\displaystyle\iint\limits_{R} xye^{x^{2}y}\,dA, \quad R=[0,1]\times [0,2].$


  1. $\dfrac{\pi}{12}.$

  2. $\dfrac{(e^{2} - 3)}{2}.$


3005   

Determine os momentos de inércia da lâmina que ocupa a região $D$ e tem função densidade $\rho$ quando: $D$ é a região triangular delimitada pelas retas $x = 0, \ y = x$ e   $2x + y = 6; \quad \rho(x,y) = x^2$.


$\displaystyle I_{x} = \dfrac{1}{16}(e^4 - 1),$ $I_{y} = \dfrac{1}{16}(e^4 - 1)$ e $I_{0} = \dfrac{1}{16}(e^4 + 2e^2 - 3).$


2776   

Encontre os pontos da elipse $x^2 + xy + y^2 = 3$ mais próximos e mais distantes da origem.



A distância entre um ponto $(x,y)$ e a origem $(0,0)$ é

$$d=\sqrt{(x-0)^{2}+(y-0)^{2}}=\sqrt{x^{2}+y^{2}}.$$

Mas a álgebra fica mais simples se maximizarmos e minimizarmos o quadrado da distância:

$$d^{2}=f(x,y)=x^{2}+y^{2}.$$

A restrição é que os pontos pertencem a elipse, ou seja,

$$g(x,y)=x^{2}+xy+y^{2}=3$$

De acordo com os multiplicadores de Lagrange, resolvemos $\nabla f=\lambda \nabla g$ e $g=3.$ Então

$$\nabla f(x,y)=(2x,2y)$$

e

$$\lambda \nabla g(x,y)=\lambda(2x+y,x+2y)=(2x\lambda+y\lambda,2y\lambda+x \lambda).$$

Logo temos,

\begin{array}{rcl}2x=2x\lambda+y\lambda\\2y=2y\lambda+x\lambda\\x^{2}+xy+y^{2}=3\\end{array}

Se $\lambda=0$ teremos que $x=0$ e $y=0$, mas esses valores não satisfazem equação $(3)$. Logo $\lambda \neq 0$ e multiplicando

ambos os lados  da equação $(1)$ por $\dfrac{y}{\lambda}$ e ambos os lados da equação $(2)$ por $\dfrac{x}{\lambda}$, obtemos que

$$\frac{2xy}{y}=2xy+y^{2}\;\;\;\;\;\;\;\; \mbox{e}\;\;\;\;\;\;\;\; \frac{2xy}{y}=2xy+x^{2}.$$

Logo,

$$y^{2}=x^{2}\Rightarrow y=x\;\;\;\; \mbox{ou}\;\;\;\; y=-x.$$

Se $y=x$ temos que da equação $(3)$ que $x^{2}+x^{2}+x^{2}=3\Rightarrow x^{2}=1\Rightarrow x=\pm 1.$

Logo temos os pontos $(1,1)$ e $(-1, -1).$

Se $y=-x$ temos que da equação $(3)$ que $x^{2}-x^{2}+x^{2}=3\Rightarrow x^{2}=3\Rightarrow  x=\pm \sqrt{3}.$

Logo temos os pontos $(\sqrt{3},-\sqrt{3})$ e $(-\sqrt{3},\sqrt{3}).$

Os valores de $f$ nesses pontos são:

$$f(1,1)=f(-1,-1)=2\;\;\;\;\;\;\;\; \mbox{e}\;\;\;\;\;\;\;\; f(\sqrt{3},-\sqrt{3})=f(-\sqrt{3},\sqrt{3})=6.$$

Portanto, $(1,1)$ e $(-1, -1)$ são os pontos mais próximos e $(\sqrt{3},-\sqrt{3})$ e $(-\sqrt{3},\sqrt{3})$ os pontos mais afastados da origem $(0,0).$


2798   

Determine os valores máximos e mínimos locais e pontos de sela da função $f(x,y)=x^{4}+xy+y^{2}-6x-5y$.


Ponto de mínimo: $\displaystyle \left( 1,2\right).$


2881   

O plano $x + y + 2z = 2$ intercepta o paraboloide $z = x^2 + y^2$ em uma elipse. Determine os pontos dessa elipse que estão mais próximo e mais longe da origem.


Mais próximo: $\displaystyle \left( \frac{1}{2}, \frac{1}{2},\frac{1}{2} \right)$ e mais distante: $\displaystyle \left( -1,-1,2 \right).$


2844   

Passe para coordenadas polares e calcule: $\displaystyle\int_{0}^{1}  \int_{0}^{\sqrt{1-y^{2}}}(x^{2}+y^{2})\,dx dy$


$\displaystyle \frac{\pi}{8}.$


2105   

Suponha que ${\bf F}$ seja um campo vetorial inverso do quadrado, ou seja,

$${\bf F}({\bf r})=\frac{c{\bf r}}{|{\bf r}|^{3}}$$

para alguma constante $c$, onde ${\bf r}=x\,{\bf i}+y\,{\bf j}+z\,{\bf k}.$ Determine o trabalho realizado por ${\bf F}$ ao mover um objeto de um ponto $P_{1}$ por um caminho para um ponto $P_{2}$ em termos da distância $d_{1}$ e $d_{2}$ desses pontos à origem.


$c\left(\dfrac{1}{d_{1}} - \dfrac{1}{d_{2}}\right).$


2291   

Determine uma representação paramétrica para a superfície descrita a seguir. A parte do cilindro $y^{2}+z^{2}=16$ que está entre os planos $x=0$ e $x=5.$


$x = u,$ $y = 4\cos (\theta),$ $z = 4\sin(\theta),$ onde $0 \leq u \leq 5,$ $0 \leq \theta \leq 2\pi.$


2083   

Calcule $\displaystyle\int_{(1,1)}^{(2,2)} y\,dx+x\,dy$.


$3.$


1974   

Mostre que a curva com equações paramétricas $x = t \cos{t}, \ y = t \sin{t}, \ z = t$ está no cone $z^2 = x^2 + y^2$ e use esse fato para esboçar a curva.


2191   

Calcule $\displaystyle\iint \limits_{S}{\bf u}\cdot {\bf n}\,dS$, sendo $B=\{(x,y,z)\in \mathbb{R}^{3}|\,x^{2}+y^{2}+z^{2}\leq 1$ e $z\geq x+y\}$ e ${\bf u}=-2xy\,{\bf i}+y^{2}\,{\bf j}+3z\,{\bf k}.$


$\dfrac{3\pi}{2}.$



3015   

  1.  Uma luminária tem duas lâmpadas de um tipo com tempo de vida médio de 1.000 horas. Supondo que possamos modelar a probabilidade de falha dessas lâmpadas por uma função densidade exponencial com média $\mu = 1.000$, determine a probabilidade de que ambas as lâmpadas venham a falhar dentro de um período de 1.000 horas.

  2.  Outra luminária tem somente uma lâmpada do mesmo tipo das do item anterior. Se a lâmpada queima e é trocada por outra to mesmo tipo, determine a probabilidade de que as duas venham a falhar dentro de 1.000 horas.


  1. $(e^{-1} - 1)^2.$

  2. $1 - 2e^{-1}.$


2226   

Determine se o campo vetorial $\mathbf{F}(x,y,z) = 2xy\mathbf{i} + (x^2+2yz)\mathbf{j} + y^2\mathbf{k}$ é conservativo ou não. Se for conservativo, determine uma função $f$ tal que $\mathbf{F} = \nabla{f}$.


$\mathbf{F}$ é conservativo. $f(x,y,z) = x^2 y + y^2 z.$


2848   

Passe para coordenadas polares e calcule: $\displaystyle\iint\limits_{ D}xy\,dA$, onde $D$ é o disco com centro na origem e raio 3.


$0.$


2456   

Dada $f(x,y)=\dfrac{1}{\sqrt{16-x^{2}-y^{2}}}$.

  1. Encontre o domínio da função;

  2. Encontre a imagem da função;

  3. Descreva as curvas de nível da função.



  1. O domínio de $f$ é

    $$D=\{(x,y)|\, 16-x^{2}-y^{2}>0\}=\{(x,y)|\,x^{2}+y^{2}<16\}.$$ma211-list2-ex1.png

  2. A imagem de $f$ é

    $$\bigg\{z|\, z=\frac{1}{\sqrt{16-x^{2}-y^{2}}},\,(x,y)\in D\bigg\}.$$

    Mas,

    $$z=\frac{1}{\sqrt{16-x^{2}-y^{2}}}\geq \frac{1}{\sqrt{16}}=\frac{1}{4}.$$

    Assim, a imagem de $f$ é $\bigg\{z|\, z \geq \dfrac{1}{4}\bigg\}.$

  3. s curvas de níveis de $f$ são da forma $f(x,y)=c$, isto é,

    $$\frac{1}{\sqrt{16-x^{2}-y^{2}}}=c\Leftrightarrow \sqrt{16-x^{2}-y^{2}}=\frac{1}{c}\Leftrightarrow 16-x^{2}-y^{2}=\frac{1}{c^{2}}$$

    $$\Leftrightarrow x^{2}+y^{2}=16-\frac{1}{c^{2}}.$$

    Assim, as curvas de níveis de $f$ são circunferências com centro na origem e raio menor do que $4.$


2720   

Determine a diferencial da função $z = x^3 \ln{y^2}$.


$dz = 3x^{2} \ln (y^{2})dx + \frac{2x^{3}}{y} dy$.


2231   

Seja $\mathbf{r} = x\mathbf{i} + y\mathbf{j} + z\mathbf{k}$ e $r=|\mathbf{r}|$. Verifique a identidade $\nabla \cdot \mathbf{r} = 3$.


$\nabla \cdot \mathbf{r} = \left(\dfrac{\partial}{\partial x}, \dfrac{\partial}{\partial y}, \dfrac{\partial}{\partial z} \right) \cdot \left(x,y,z \right)$ (Note que: $r = \sqrt{x^{2} + y^{2} + z^{2}}.$)


2627   

Utilizando o Teorema de Stokes, transforme a integral $\displaystyle\iint_{ S}\mbox{rot}{\bf F}\cdot{\bf n}dS$ numa integral de linha e calcule.

  • ${\bf F}(x,y,z) = y{\bf i}$, $S$ a superfície $z = x^2+y^2$ com $z \leq 1$, sendo ${\bf n}$ a normal com componente $z$ positiva.


$-\pi$.


2763   

Determine o maior conjunto de pontos em que a função $f(x,y) = \begin{cases}\dfrac{xy^3}{x^2 + y^2}, & \quad \text{se } (x,y) \neq (0,0),\\0, & \quad \text{se } (x,y) = 0\end{cases}$ é diferenciável. Justifique.


$\mathbb{R}^{2}$.


2603   

Determine o volume do sólido limitado pelo cilindro $x^2 + y^2 = 4$ e pelos planos $z = 0$ e $y + z = 3$.


$12\pi.$


3029   

Esboce a região de integração e mude a ordem de integração. $\displaystyle\int_{0}^{3}\!\!\int_{-\sqrt{9-y^{2}}}^{\sqrt{9-y^{2}}}f(x,y)\,dx dy$.


ma211-list6-ex32_sol_b.png

3144   

Encontre o trabalho realizado pelo campo de forças \[ \mathbf{F}(x,y)= y^2\mathbf{i} + xy\mathbf{j} \] para mover uma partícula de \((0,0)\) até \((1,1)\) ao longo da parábola \(y=x^2\).


2350   

Determine o valor médio de $f(x,y)=e^{y}\sqrt{x+e^{y}}$ sobre o retângulo  $R=[0,4]\times [0,1].$


$\dfrac{(4 + e)^{5/2} - e^{5/2} - 5^{5/2} + 1}{15}.$


2027   

Use a Regra da Cadeia para determinar $\mathrm{d}z/\mathrm{d} t$ ou $\mathrm{d}w/ \mathrm{d}t.$

$z=\sqrt{x^{2}+y^{2}}$, $x=e^{2t}$, $y=e^{-2t}$.



$\displaystyle \frac{dz}{dt} = \frac{2xe^{2t} - 2ye^{2t}}{\sqrt{x^{2} + y^{2}}}.$


2137   

A função diferenciável $z=z(x,y)$ é dada implicitamente pela equação  $f\bigg(\dfrac{x}{y},z\bigg)=0$, onde 
$f(u,v)$ é suposta diferenciável e $\dfrac{\partial f}{\partial v}(u,v)\neq 0$. Verifique que 
$$x\frac{\partial z}{\partial x}+y\dfrac{\partial z}{\partial y}=0.$$



Note que $\displaystyle \frac{\partial z}{\partial x} =  - \frac{1}{y} \frac{\partial f}{\partial u} \left(\frac{x}{y},z \right)\left(\frac{\partial f}{\partial v}\left(\frac{x}{y},z \right)\right)^{-1}$  e $\displaystyle \frac{\partial z}{\partial y} =  \frac{x}{y^{2}} \frac{\partial f}{\partial u} \left(\frac{x}{y},z \right)\left(\frac{\partial f}{\partial v}\left(\frac{x}{y},z \right)\right)^{-1}$.


3077   

Determine \(\displaystyle \lim_{(x,y)\to (0,0)}(x^2+y^2)\ln(x^2+y^2). \)



Usando coordenadas polares, teremos que: \[\begin{array}{lll} x=r\cos\theta, & y=r\sin\theta, & r^2=x^2+y^2. \end{array} \] Além disso, como \(r=\sqrt{x^2+y^2}\geq 0\), temos que \( r\rightarrow 0^+\) se, e somente se, \( (x,y)\rightarrow (0,0) \). Assim, segue para o limite dado que \begin{align*} \lim_{(x,y)\to (0,0)}(x^2+y^2)\ln(x^2+y^2) & = \lim_{r\to 0^+} r^2\ln r^2 \\     & = \lim_{r\to 0^+}\underbrace{\dfrac{2\ln r}{1/r^2}}_{\text{do tipo}\ \infty/\infty} \\     & \stackrel{\text{L'Hospital}}{=} \lim_{r\to 0^+} \dfrac{2/r}{-2/r^3} \\    & = \lim_{r\to 0^+} (-r^2) = 0.\end{align*}


2274   

No item abaixo :

  1.  determine o gradiente de $f$; 
  2.  calcule o gradiente no ponto $P$; e 
  3.  determine a taxa de variação de $f$ em $P$ na direção do vetor $\bf{u}$.

$f(x,y,z) = \sqrt{x+yz},  P = (1,3,1),  \bf{u} = \left( \frac{2}{7}, \frac{3}{7}, \frac{6}{7} \right)$.


  1. $\displaystyle \nabla f(x,y,z) = \left(\frac{1}{2\sqrt{x + yz}}, \frac{z}{2\sqrt{x + yz}}, \frac{y}{2\sqrt{x + yz}} \right).$
  2. $\displaystyle \nabla f(1,3,1) = \left(\frac{1}{4}, \frac{1}{4}, \frac{3}{4}\right).$
  3. $\displaystyle \frac{23}{28}.$


2549   

$f(x,y) = \begin{cases}\dfrac{xy^2}{x^2 + y^2}, & \quad \text{se } (x,y) \neq (0,0), \\0, & \quad \text{se } (x,y) = (0,0), \end{cases}$ é contínua em (0,0)? Justifique.



Notemos que para $(x,y)\neq (0,0)$ a função $f$ é contínua, pois $xy^{2}$ e $x^{2}+y^{2}$ são funções contínuas e $x^{2}+y^{2}\neq 0.$ Agora, estudemos a continuidade da função $f$ no ponto $(0,0).$ Temos que

$$\lim_{(x,y)\to (0,0)}f(x,y)=\lim_{(x,y)\to (0,0)}\frac{xy^{2}}{x^{2}+y^{2}}=\lim_{(x,y)\to (0,0)}x\cdot \frac{y^{2}}{x^{2}+y^{2}}.$$

Como

$$\lim_{(x,y)\to (0,0)}x=0\,\,\,\,\,\, \mbox{e}\,\,\,\,\,\, \bigg| \dfrac{y^{2}}{x^{2}+y^{2}}\bigg|\leq 1,\, \forall (x,y)\neq (0,0),$$

obtemos que

$$\lim_{(x,y)\to (0,0)}f(x,y)=0.$$

Assim,

$$\lim_{(x,y)\to (0,0)}f(x,y)=0=f(0,0).$$

Portanto, $f$ é contínua em $(0,0).$


2781   

Calcule a integral dupla usando coordenadas polares: $\displaystyle\iint\limits_{R}e^{x^{2}+y^{2}}\,dx dy$, onde $R$ é o conjunto de todos os $(x,y)$ tais que  $1\leq x^{2}+y^{2}\leq 4$,  $-x\leq y\leq x$ e $x\geq 0.$


$\displaystyle \frac{\pi}{4}(e^4 - e).$


2143   

Verifique que o Teorema do Divergente é verdadeiro para o campo vetorial ${\bf F}$ na região $E.$
${\bf F}(x,y,z)=xy\,{\bf i}+yz\,{\bf j}+zx\,{\bf k}$, $E$ é o cilindro sólido $x^{2}+y^{2}\leq 1$, $0\leq z\leq 1.$


$\displaystyle\iint_{S} {\bf F} \cdot d{\bf S} = \iiint_{E} \mbox{div} {\bf F} dV =  \dfrac{\pi}{2}.$


3037   

Uma região $R$ é mostrada na figura abaixo. Decida se você deve usar coordenadas polares ou retangulares e escreva $\iint \limits_{R}f(x,y)\,dA$ como uma integral iterada, onde $f$ é uma função qualquer contínua em $R.$


ma211-list7-ex5_d.png



$\displaystyle \int_{-\frac{\pi}{2}}^{\frac{\pi}{2}} \int_{3}^{6} f(r\cos(\theta),r\sin(\theta)) r  d r d \theta.$


2951   

Seja $D$ a região limitada abaixo pelo plano $z=0$, acima pela esfera  $x^{2}+y^{2}+z^{2}=4$ e dos lados pelo cilindro $x^{2}+y^{2}=1$. Monte as integrais triplas em coordenadas esféricas que dão o volume de $D$ usando as ordens de integração a seguir.

  1. $d\rho\,d\phi\,d\theta$;

  2. $d\phi\,d\rho\,d\theta$.


  1. $\displaystyle \int_{0}^{2\pi}\int_{0}^{\pi/6}\int_{0}^{2} \rho^{2}\sin(\phi)\; d\rho d\phi d\theta + \int_{0}^{2\pi}\int_{\pi/6}^{\pi/2}\int_{0}^{\csc(\phi)} \rho^{2}\sin(\phi)\; d\rho d\phi d\theta.$

  2. $\displaystyle \int_{0}^{2\pi}\int_{1}^{2}\int_{\pi/6}^{\arcsin(1/\rho)} \rho^{2}\sin(\phi)\; d\phi d\rho d\theta + \int_{0}^{2\pi}\int_{0}^{2}\int_{0}^{\pi/6} \rho^{2}\sin(\phi)\; d\phi d\rho d\theta.$


2068   

Utilize as Equações 

$\dfrac{\partial z}{\partial x}=-\dfrac{\dfrac{\partial F}{\partial x}}{\dfrac{\partial F}{\partial z}}$ e $\dfrac{\partial z}{\partial y}=-\dfrac{\dfrac{\partial F}{\partial y}}{\dfrac{\partial F}{\partial z}}$

para determinar $\partial z/\partial x$ e $\partial z/\partial y$.

$x^{2}+y^{2}+z^{2}=3xyz$


$\displaystyle \frac{dz}{dx} = \frac{3yz - 2x}{2z - 3xy}$ e $\displaystyle \frac{dz}{dy} = \frac{3xz - 2y}{2z - 3xy} .$


2820   

Determine os valores máximo e mínimo absolutos de $f$ no conjunto $D.$

$f(x,y)=x^{2}+3xy-3x$ em $D=\{(x,y)\in \mathbb{R}^2: x\geq 0,\;y\geq 0\; \text{e} \;x+y\leq 1\}.$


Valor máximo: $\displaystyle 0;$ valor mínimo: $-2.$


2502   

Encontre uma equação para a curva de nível da função $f(x,y)=\sqrt{x^{2}-1}$ que passa pelo ponto $(1,0)$.


$x = 1$ ou $x = -1.$


2181   

Utilize o Teorema de Green para demonstrar a fórmula de mudança de variáveis para as integrais duplas para o caso em que $f(x,y) = 1$:

$$\iint\limits_{R} dxdy = \iint\limits_{R}\left|\dfrac{\partial(x,y)}{\partial(u,v)}\right|\, dudv.$$

Aqui, $R$ é a região do plano $xy$ que corresponde à região $S$ do plano $uv$ sob a transformação dada por $x=g(u,v)$, $y=h(u,v)$. (Sugestão: observe que o lado esquerdo é $A(R)$. Converta a integral de linha sobre $\partial R$ para uma integral de linha sobre $\partial S$ e aplique o Teorema de Green no plano $uv$.)


Dica: pelo Teorema de Green, $A(R) = \displaystyle \iint_{R} dxdy = \int_{\partial R} x dy.$ Escolhendo a orientação positiva em $\partial S$ correspondente a orientação positiva em $\partial R,$ segue que

$$\displaystyle \int_{\partial R} x dy = \int_{\partial S} g(u,v) \dfrac{\partial h}{\partial u} du + g(u,v) \frac{\partial h}{\partial v} dv.$$

Conclua utilizando o Teorema de Green no plano $uv$ e a Regra da Cadeia.


2565   

Determine o maior conjunto no qual a função $f(x,y) = \begin{cases}\dfrac{xy}{x^2 + xy + y^2}, & \quad \text{se } (x,y) \neq (0,0), \\0, & \quad \text{se } (x,y) = (0,0)\end{cases}$ é contínua.


$\left\lbrace (x,y);\; (x,y) \neq (0,0) \right\rbrace.$


2091   

Calcule a integral de linha

$$\int_{C}e^{2y}\,dx+(1+2xe^{2y})\,dy,$$

onde $C$ é a curva dada por $r(t)=(te^{t},1+\sin(\pi t/2))$, $0\leq t\leq 1.$ (Sugestão: verifique se o campo é conservativo.)


$e^{5} + 1.$


2660   

Determine as derivadas parciais de $f(x,y)=(4xy-3y^{3})^{3}+5x^{2}y$.


$\displaystyle \frac{\partial f}{\partial x} = 12 y (4xy - 3y^{3})^{2} + 10xy\;\;\;\;\;\;\text{e}\;\;\;\;\; \frac{\partial f}{\partial y} = 3(4xy - 3y^{2})^{2}(4x - 9y^{2}) + 5x^{2}.$


2277   

Calcule $\nabla f(x,y)$.
$f(x,y) = e^{x^2 - y^2}$


$\displaystyle \nabla f(x,y) = e^{x^{2} - y^{2}}(2x,-2y).$


2555   

Determine o limite, se existir, ou mostre que o limite não existe.

$\displaystyle \lim_{(x,y) \to (0,0)} \dfrac{y^4}{x^4 + 3y^4}$.


Não existe.


2121   

Expresse $\partial z/\partial t$ em termos das derivadas parciais de $f$, sendo $z=f(x,y)$ e $x=t^{2}$ e $y=3t.$


$\displaystyle \frac{dz}{dt} (t) = 2t \frac{\partial f}{\partial x}(t^{2},3t) + 3 \frac{\partial f}{\partial y}(t^{2},3t).$


3003   

Determine a massa e o centro de massa da lâmina que ocupa a região $D$ e tem função densidade $\rho$, sendo: $\displaystyle D = \{(x,y) \in\mathbb{R}^2: 0 \leq y \leq \sin{(\pi x/L)}, \ 0 \leq x \leq L\}; \quad \rho(x,y) = y$.



Massa: $\dfrac{L}{4};$ centro de massa: $\displaystyle \left(\frac{L}{2},\frac{16}{9\pi} \right).$


2120   

Seja $g(t)=f(3t,2t^{2}-1).$

  1. Expresse $g^{'}(t)$ em termos das derivadas parciais de $f$.
  2. Calcule $g^{'}(0)$ admitindo $\dfrac{\partial f}{\partial x}(0,-1)=\dfrac{1}{3}.$


  1. $\displaystyle g'(t) =  3\frac{\partial f}{\partial x}(3t,2t^{2} - 1) + 4t \frac{\partial f}{\partial y}(3t,2t^{2} - 1).$
  2. $g'(0) = 1.$


2467   

Calcule a integral de superfície $\displaystyle\iint \limits_{ S}{\bf F}\cdot d{\bf S}$ para o campo vetorial ${\bf F}$ e superfície orientada $S$ dados abaixo. Em outras palavras, determine o fluxo de ${\bf F}$ através de $S$. Para superfícies fechadas, use a orientação positiva (para fora).

  • ${\bf F}(x,y,z)=x{\bf i}+y{\bf j}+z{\bf k}$ e $S$ é a parte no primeiro octante do plano $2x+3y+z=6.$


$18.$


3130   

Use o Teorema de Green para determinar o trabalho realizado pelo campo de forças \(\displaystyle\textbf{F}(x,y)=xy\textbf{i}+(\dfrac{1}{2}x^2+xy)\textbf{j}\) sobre uma partícula que se move ao longo do caminho que começa em \((5,0)\), percorre o semicírculo superior \(x^2+y^2=25\) e retorna ao seu ponto de partida ao longo do eixo \(x\).


\(\dfrac{250}{3}\)


2850   

Passe para coordenadas polares e calcule: $\displaystyle\iint\limits_{R}\,dx dy$, onde $R$ é a região, no plano $xy$, limitada pela curva (dada em coordenadas polares) $\rho=\cos(2\theta)$, $\dfrac{\pi}{8}\leq \theta \leq \dfrac{\pi}{4}.$


 $\displaystyle \frac{3\pi + 2}{32}.$


3060   

Esboce o campo vetorial $\textbf{F}= \dfrac{y\textbf{i} - x\textbf{j}}{\sqrt{x^2+y^2}}$, desenhando um diagrama.


3151   

Enuncie o Teorema da Divergência e o Teorema de Stokes, incluindo todas as hipóteses envolvidas.


2947   

Calcule utilizando coordenadas esféricas. $\displaystyle\iiint\limits_{B}\sqrt{x+y}\sqrt[3]{x+2y-z}\,dxdydz$, onde $B$ é a região $1\leq x+y\leq 2$, $0\leq x+2y-z\leq 1$ e $0\leq z\leq 1.$


$\sqrt{2} - \dfrac{1}{2}.$


2511   

Ache $\displaystyle\iint \limits_{ S}{\bf F}\cdot {\bf n} dS$ se ${\bf n}$ é uma normal unitária superior de $S.$


${\bf F}=2{\bf i}+5{\bf j}+3{\bf k}$; $S$ é a parte do cone $z=(x^{2}+y^{2})^{1/2}$ interior ao cilindro $x^{2}+y^{2}=1.$


$3\pi.$


2221   

Seja $f$ um campo escalar e $\mathbf{F}$ um campo vetorial. Diga se cada expressão tem significado. Em caso negativo, explique por quê. Em caso afirmativo, diga se é um campo vetorial ou escalar.  

  1. $\text{grad }{f}$;

  2. $\text{rot }{(\text{grad }{f})}$;

  3. $\text{grad }{(\text{div }{\mathbf{F}})}$.


  1. $\text{grad }{f}$ é um campo gradiente.

  2. $\text{rot }{(\text{grad }{f})}$ é um campo vetorial.

  3. $\text{grad }{(\text{div }{\mathbf{F}})}$ é um campo vetorial.


2092   

Calcule a integral de linha

$$\int_{C}{\bf F}\cdot d{\bf r}=\int_{C}{\bf F}\cdot r'(t)\,dt$$

onde ${\bf F}=(2xyz^{3},x^{2}z^{3},3x^{2}yz^{2})$ e $C$ é a curva dada por $r(t)=(\sin^{6}t,1-\cos t, e^{t(t-\pi/2)}$, $0\leq t\leq \pi/2.$ (Dica: verifique se ${\bf F}$ é conservativo.)


$1.$


2001   

Uma partícula move-se no plano de modo que no instante $t$ sua posição é dada por ${\bf r}(t)=(t,t^{2})$. Calcule o trabalho realizado pelo campo de forças ${\bf F}(x,y)=(x+y)\,{\bf i}+(x-y)\,{\bf j}$ no deslocamento da partícula de ${\bf r}(0)$ até ${\bf r}(1).$


$1.$


2587   

Calcule a massa do cilindro $x^{2}+y^{2}\leq 4$ e $0\leq z \leq 2$, sabendo que a densidade no ponto $(x,y,z)$ é o dobro da distância do ponto ao plano $z=0.$


$16\pi.$


3041   

Faça uma correspondência entre as equações e os gráficos identificados a seguir,  enumerador respectivamente por $I-VI$, e justifique sua resposta. Determine quais famílias de curvas da grade têm $u$ constante e quais têm $v$ constante.

  1. ${\bf r}(u,v)=u\cos v{\bf i}+u\sin v{\bf j}+v{\bf k}.$

  2. ${\bf r}(u,v)=u\cos v{\bf i}+u\sin v{\bf j}+\sin u{\bf k}$, $-\pi\leq u\leq \pi.$

  3. ${\bf r}(u,v)=\sin v{\bf i}+\cos u\sin 2v{\bf j}+\sin u\sin 2v{\bf k}.$

  4. $x=(1-u)(3+\cos v)\cos 4\pi u$, $y=(1-u)(3+\cos v)\sin 4\pi u$,$z=3u+(1-u)\sin v.$

  5. $x=\cos^{3}u\cos^{3}v$,  $y=\sin^{3}u\cos^{3}v$, $z=\sin^{3}v.$

  6. $x=(1-|u|)\cos v$, $y=(1-|u|)\sin v$, $z=u.$

ma211-list13-ex23_i.png

ma211-list13-ex23_ii.png

ma211-list13-ex23_iii.png

ma211-list13-ex23_iv.png

ma211-list13-ex23_v.png

ma211-list13-ex23_vi.png


  1. IV.
  2. I.
  3. II.
  4. V.
  5. III.
  6. VI

2342   

Determine a área da superfície dada pela porção do cilindro $x^{2}+y^{2}=1$ entre os planos $z=1$ e $z=4.$


$6\pi.$


3118   

Use coordenadas esféricas para encontrar o volume do sólido: limitado acima pela esfera \(\rho=4\) e abaixo pelo cone \(\phi=\pi/3\).


\(\dfrac{64\pi}{3}\)


2009   

Determine o trabalho realizado pelo campo de força ${\bf F}(x,y,z)=(y+z)\,{\bf i}+(x+z)\,{\bf j}+(x+y)\,{\bf k}$ sobre uma partícula que se move ao longo do segmento de reta $(1,0,0)$ a $(3,4,2).$


$26.$


2229   

Mostre que qualquer campo vetorial da forma

$$\mathbf{F}(x,y,z) = f(x)\mathbf{i} + g(y)\mathbf{j} + h(z)\mathbf{k},$$

em que $f,g$ e $h$ são diferenciáveis, é irrotacional.


Note que $\text{rot } \mathbf{F} = \bf{0}.$


2522   

Esboce o gráfico da função $f(x,y)=e^{\sqrt{x^{2}+y^{2}}}$. Em geral, se $g$ é uma função de uma variável, como saber o gráfico de $f(x,y)=g(\sqrt{x^{2}+y^{2}})$ a partir do gráfico de $g$?


O gráfico de $f(x,y) = g(\sqrt{x^{2} + y^{2}})$ pode ser obtido rotacionando o gráfico de $g$ no plano $xz$ ao redor do eixo $z.$

ma211-list2-ex22_sol_b.png


2032   

Utilize a Regra da Cadeia para determinar $\mathrm{\partial}z/\mathrm{\partial} s$ e $\mathrm{\partial}z/ \mathrm{\partial}t.$

$z=\arcsin(x-y)$, $x=s^{2}+t^{2}$,  $y=1-2st$.


$\displaystyle \frac{\partial z}{\partial s} = \displaystyle \frac{\partial z}{\partial t} = \frac{2s + 2t}{\sqrt{1 - (x - y)^{2}}}$.



2279   

Defina gradiente de uma função de três variáveis. Calcule $\nabla f(x,y,z)$.

$f(x,y,z) = \sqrt{x^2 + y^2 + z^2}$


 $\displaystyle \nabla f(x,y,z) = \frac{1}{\sqrt{x^{2} + y^{2} + z^{2}}}.$


2992   

Calcule a integral, efetuando uma mudança de variáveis apropriada. $\displaystyle\iint\limits_{R} \dfrac{\sqrt[3]{y - x}}{1 + y + x} \, dA$, em que $R$ é o triângulo de vértices $(0,0), (1,0)$ e $(0,1)$.


$0$.


2694   

Seja  $w=f(x,y,z)$ uma função de três variáveis independentes. Escreva a definição formal de derivada parcial $\partial f/\partial z$ em $(x_{0},y_{0},z_{0})$. Use essa definição para encontrar $\partial f/\partial z$ em $(1,2,3)$ para $f(x,y,z)=x^{2}yz^{2}.$


$\displaystyle \frac{\partial f}{\partial z}(1,2,3) = 12$.


2533   

Escreva seis integrais triplas iteradas diferentes para o volume do sólido retangular no  primeiro octante limitado pelos planos coordenados e pelos planos $x=1$, $y=2$ e $z=3$. Calcule uma das integrais.


$$\begin{split} 6 &= \int_{0}^{1}\int_{0}^{2}\int_{0}^{3} dz dy dx = \int_{0}^{2}\int_{0}^{1}\int_{0}^{3} dz dx dy = \int_{0}^{3}\int_{0}^{2}\int_{0}^{1} dx dy dz\\ &= \int_{0}^{2}\int_{0}^{3}\int_{0}^{1} dx dz dy = \int_{0}^{3}\int_{0}^{1}\int_{0}^{2} dy dx dz = \int_{0}^{1}\int_{0}^{3}\int_{0}^{2} dy dx dx. \end{split} $$


2223   


Determine a derivada direcional de $f$ no ponto dado e na direção indicada pelo ângulo $\theta$.

$f(x,y) = x^2y^3 - y^4,  (2,1),$  $\theta = \pi/4.$


 $6\sqrt{2}.$


2958   

Usando coordenadas esféricas, determine o volume do sólido que está acima do plano $z=2\sqrt{3}$ e abaixo da esfera $x^{2}+y^{2}+z^{2}=16.$


$\dfrac{88\pi}{3}.$


2563   

Determine o maior conjunto no qual a função $G(x,y) = \ln{(x^2 + y^2 - 4)}$ é contínua.


$\left\lbrace (x,y);\;x^{2} + y^{2} > 4 \right\rbrace.$


2518   

Dada a expressão $g(x,y)=2f(x,y)$, escreva como o gráfico de $g$ é obtido a partir do gráfico de $f.$


Gráfico de $f$ esticado verticalmente ao dobro.


2816   

Determine os valores máximo e mínimo absolutos de $f$ no conjunto $D.$

$f(x,y)=x^{3}-3x-y^{3}+12y$, $D$ é o quadrilátero cujos vértices são $(-2,3)$, $(2,3)$, $(2,2)$ e $(-2,-2).$


Valor máximo: $18;$  valor mínimo: $-18.$


2201   

O raio $r$ e a altura $h$ de um cilindro circular reto aumentam à razão de $0,01cm/min$ e $0,02cm/min$, respectivamente.

  1.  Ache a taxa de variação do volume quando $r=4cm$ e $h=7cm.$
  2.  A que taxa a área da superfície curva está variando nesse instante?


  1.  $0,88\pi$ cm$^{3}/$min.
  2.  $0,3\pi$ cm$^{2}/$min.


3009   

Calcule o centro de massa da região $D$ dada.

  1.  $D$ é o conjunto de todos $(x,y)$ tais que $x^3 \leq y \leq x$ e a densidade é constante e igual a 1.

  2.  $D$ é o conjunto de todos $(x,y)$ tais que $x \leq y \leq x + 1$, $0 \leq x \leq 1$, e a densidade é o produto das coordenadas do ponto.

  3.  $D$ é o conjunto de todos $(x,y)$ tais que $1 \leq x^2 + y^2 \leq 4$, $y \geq 0$, e a densidade é proporcional à distância do ponto à origem.


  1.  $\displaystyle \left(0,0\right).$

  2.  $\displaystyle \left(\frac{5}{7},\frac{9}{7}\right).$

  3.  $\displaystyle \left(0, \frac{45}{14\pi} \right).$


3016   

Esboce o sólido cujo volume é dado pela integral iterada

$$\int_{0}^{1} \!\! \int_{0}^{1}(4-x-2y)\, dx dy.$$


ma211-list6-ex18_sol.png

2699   

Seja $f(x,y)=\dfrac{1}{x^{2}+y^{2}}$. Verifique que

  1. $x\;\dfrac{\partial ^{2}f}{\partial x^{2}}(x,y)+y\;\dfrac{\partial^{2} f}{\partial y \partial x}(x,y)=-3\dfrac{\partial f}{\partial x}(x,y)$

  2. $\dfrac{\partial ^{2}f}{\partial x^{2}}(x,y)+\dfrac{\partial^{2} f}{\partial y^{2}}(x,y)=\dfrac{4}{(x^{2}+y^{2})^{2}}$


$\begin{aligned}[t]\frac{\partial f}{\partial x} &= -\frac{2x}{(x^{2} + y^{2})^{2}},\;\;\;\;\; \frac{\partial^{2} f}{\partial x^{2}}= \frac{6 x^{2} - 2y^{2}}{(x^{2} + y^{2})^{3}},\;\;\;\;\; \frac{\partial^{2} f}{\partial y^{2}}= \frac{6 y^{2} - 2x^{2}}{(x^{2} + y^{2})^{3}} \;\;\;\;\;\text{e}\\ \frac{\partial^{2} f}{\partial y\partial x} &= \frac{8xy}{(x^{2} + y^{2})^{3}}.\end{aligned}$


2861   

Estude com relação a máximos e mínimos a função dada com as restrições dadas.

$f(x,y) = 3x + y$ e $x^2 + 2y^2 \leq 1.$


Ponto de máximo: $\displaystyle \left( \frac{6}{\sqrt{38}}, \frac{1}{\sqrt{38}} \right)$; ponto de mínimo: $\displaystyle \left( -\frac{6}{\sqrt{38}}, -\frac{1}{\sqrt{38}} \right)$.


1928   

Encontre um campo de vetores $\textbf{G} = P(x,y)\textbf{i} + Q(x,y)\textbf{j}$ no plano $xy$ com a propriedade de que, em qualquer ponto $(a,b) \neq (0,0)$, $\textbf{G}$ é um vetor de magnitude $\sqrt{x^2+y^2}$ tangente à circunferência $x^2+y^2=a^2+b^2$ e aponta no sentido horário. (O campo é indefinido em (0,0).)


$\displaystyle \textbf{G} = \frac{y \textbf{i} - x \textbf{j}}{\sqrt{x^{2} + y^{2}}}.$


3154   

A fórmula de Taylor de primeira ordem para $f(\vec{a} + \vec{v})$ pode ser escrita como $ f(\vec{a}) + \nabla f(\vec{a}) \cdot \vec{v}$, já desconsiderando o termo de erro. Calcule-a para $f(x,y) = x^2 + y^2$, $\vec{a} = (1,0)$ e $\vec{v} = (2,1)$. Calcule também o erro cometido, dizendo se é um erro pequeno ou grande e por quê.


2716   

Explique por que a função é diferenciável no ponto dado. A seguir, encontre a linearização $L(x,y)$ da função naquele ponto. $f(x,y) = x\sqrt{y}, \quad (1,4)$.


As derivadas $f_{x}$ e $f_{y}$ de cada $f$ existem e são contínuas no ponto dado, logo $f$ é diferenciável.

$L(x,y) = 2x + \frac{1}{4}y - 1$.


2581   

Determine o conjunto dos pontos de continuidade da função $f(x,y) = \dfrac{x - y}{\sqrt{1 - x^2 - y^2}}$. Justifique sua resposta.


$\left\lbrace (x,y);\; x^{2} + y^{2} < 1 \right\rbrace.$


2759   

Verifique que a função $f(x,y) = x \cos{(x^2 + y^2)}$ é diferenciável.


As derivadas parciais $\frac{\partial f}{\partial x}$ e $\frac{\partial f}{\partial y}$ de cada função $f$ existem e são contínuas em todos os pontos do domínio.


2863   

Estude com relação a máximos e mínimos a função dada com as restrições dadas.

$f(x,y) = x^2 + 4y^2$ e $xy = 1$, $x > 0$ e $y>0.$


Ponto de mínimo: $\displaystyle \left( \sqrt{2}, \frac{\sqrt{2}}{2} \right)$.


2609   

Use o Teorema de Stokes para calcular $\displaystyle\int_C {\bf F}\cdot d{\bf R}$. $C$ é orientada no sentido anti-horário quando vista de cima.

  • ${\bf F}(x,y,z) = (x+y^2){\bf i} + (y+z^2){\bf j} + (z+x^2){\bf k}$, $C$ é o triângulo com vértices $(1,0,0)$, $(0,1,0)$, $(0,0,1)$.


$1$.


2152   

Use o Teorema do Divergente para calcular o fluxo de ${\bf F}$ através de $S,$ onde ${\bf F}(x,y,z)=x^{4}\,{\bf i}-x^{3}z^{2}\,{\bf j}+4xy^{2}z\,{\bf k}$ e $S$ é a superfície do sólido limitado pelo cilindro $x^{2}+y^{2}=1$ e pelos planos $z=x+2$ e $z=0.$


2570   

Calcule $\displaystyle \lim_{(x,y) \to (0,0)} \dfrac{x^2}{\sqrt{x^2 + y^2}}$, caso exista.


$0.$


3124   

  1.  Mostre que se \(R\) for a região triangular de vértices \((0,0)\), \((1,0)\) e  \((0,1)\), então   \[\iint\limits_R f(x+y)\,dA = \int_0^1 uf(u)\,du.\]

  2.  Use o resultado anterior para calcular a integral \[ \iint\limits_R e^{x+y}\,dA. \]


3050   

A figura mostra a região de integração da integral

$$\int_{0}^{1}\int_{\sqrt{x}}^{1}\int_{0}^{1-y}f(x,y,z)\;dz dy dx.$$

Reescreva essa integral como uma integral iterada equivalente nas cinco outras ordens.

ma211lista8q15.png


$\int_{0}^{1}\int_{\sqrt{x}}^{1}\int_{0}^{1-y}f(x,y,z)\;dz dy dx = \int_{0}^{1}\int_{0}^{y^2}\int_{0}^{1-y}f(x,y,z)\;dz dx dy $
$= \int_{0}^{1}\int_{0}^{1 - z}\int_{0}^{y^2}f(x,y,z)\;dx dy dz = \int_{0}^{1}\int_{0}^{1 - y}\int_{0}^{y^2}f(x,y,z)\;dx dz dy $
$= \int_{0}^{1}\int_{0}^{1 - \sqrt{x}}\int_{\sqrt{x}}^{1-z}f(x,y,z)\;dy dz dx = \int_{0}^{1}\int_{0}^{(1 - z)^2}\int_{\sqrt{x}}^{1-z}f(x,y,z)\;dy dx dz.$


2452   

Integre $g(x,y,z)=xyz$ sobre a superfície do sólido retangular cortado do primeiro octante pelos planos $x=a$, $y=b$ e $z=c.$


$\dfrac{abc(ab+ac+bc)}{4}.$


2902   

Determine os valores máximo e mínimo absolutos de

$$f(x,y) = x^2 + 2y^2 - x$$

no conjunto $D = \{(x,y) \in \mathbb{R}^2: x^2 + y^2 \leq 1 \}$.


Valor máximo: $\dfrac{9}{4};$ valor mínimo: $-\dfrac{1}{4}.$


2101   

Considere o campo vetorial

$${\bf F}(x,y)=(1+ye^{xy})\,{\bf i}+(2y+xe^{xy})\,{\bf j}.$$

  1. Determine se ${\bf F}$ é ou não um campo conservativo. Em caso afirmativo, encontre uma função potencial para ${\bf F}.$

  2. Determine o trabalho realizado pelo campo vetorial ${\bf F}$ ao mover uma partícula sobre a hipérbole $x^{2} - y^{2} = 1,$ desde o ponto $(3,-\sqrt{8})$ até o ponto $(3,\sqrt{8}).$


  1. Sim. Função potencial: $f(x,y) = x + e^{xy} + y^{2}.$

  2. $e^{3\sqrt{8}} - e^{-3\sqrt{8}}.$


2371   

Determine todos os pontos nos quais a direção de maior variação da função $f(x,y) = x^2 + y^2 - 2x - 4y$ é $\bf{i} + \bf{j}$.


 $\left\lbrace (x,y) \in \mathbb{R}^{2}; y =x + 1 \right\rbrace.$


2864   

Estude com relação a máximos e mínimos a função dada com as restrições dadas.

$f(x,y) = xy$ e $x^2 + 4y^2 = 8.$


Pontos de máximo: $\displaystyle \left(2,1\right)$ e $(-2,-1)$; pontos de mínimo: $\displaystyle \left(-2,1\right)$ e $(2,-1)$.


3075   

  1.  Esboce o gráfico da função $f(x,y)=e^{-(x^2+y^2)}$.

  2.  Descreva em palavras como o gráfico da função \(\displaystyle g(x,y)= e^{-a(x^2+y^2)}\) está relacionado com o gráfico de \(f\), sendo \(a>0\). Mostre (verifique) que o valor de \(a\) influencia na "largura" do pico presente no gráfico da função.


2267   

Determine a área da superfície dada pela parte do paraboloide hiperbólico $z=y^{2}-x^{2}$ que está entre os cilindros $x^{2}+y^{2}=1$ e $x^{2}+y^{2}=4.$



Temos que $z=f(x,y)=y^{2}-x^{2}$ com $1\leq x^{2}+y^{2}\leq 4$. Então,

$$A(S)=\iint\limits_{ D}\sqrt{1+\bigg(\frac{\partial z}{\partial x}\bigg)^{2}+\bigg(\frac{\partial z}{\partial y}\bigg)^{2}}\,dA$$

$$=\iint\limits_{ D}\sqrt{1+(2y)^{2}+(-2x)^{2}}\,dA=\iint\limits_{ D}\sqrt{1+4y^{2}+4x^{2}}\,dA.$$

Usando coordenadas polares temos que

$$x=r\,\cos \theta,\,\,\,\,\, y=r\,\sin \theta \Rightarrow 0\leq \theta\leq \frac{\pi}{2}\,\, \mbox{e}\,\, 1\leq r \leq 2.$$

Assim,

$$A(S)=\int_{0}^{2\pi}\int_{1}^{2}\sqrt{1+4r^{2}}\,r\,dr\,d\theta=\int_{0}^{2\pi}d\theta \cdot \underbrace{\int_{1}^{2}\sqrt{1+4r^{2}}r\,dr}_{\substack{u=1+4r^{2}\\ du=8r\,dr}}$$

$$=\theta\bigg|_{0}^{2\pi}\cdot \int_{5}^{17}u^{1/2}\cdot r\cdot \frac{du}{8r}=2\pi\cdot \frac{1}{8}\int_{5}^{17}u^{1/2}\,du=\frac{\pi}{4}\cdot \frac{2}{3}u^{3/2}\bigg|_{5}^{17}$$

$$=\frac{\pi}{6}\cdot(17^{3/2}-5^{3/2}).$$


2853   

Considere a função $f(x,y)=x^{2}+y^{2}+2xy-x-y+1$ no quadrado $0\leq x\leq 1$ e $0\leq y\leq 1$.

  1. Mostre que $f$ tem um mínimo absoluto ao longo do segmento de reta $2x+2y=1$ nesse quadrado. Qual é o valor mínimo absoluto?

  2. Encontre o valor máximo absoluto de $f$ no quadrado.


  1. $\displaystyle \frac{3}{4}.$

  2. $f(1,1) = 3.$


3049   

 Calcule a integral tripla $\int\int\int\limits_{T}x^{2}dV$, 

onde $T$ é o tetraedro sólido com vértices $(0,0,0)$, $(1,0,0)$, $(0,1,0)$ e $(0,0,1).$



Para resolvermos a integral tripla, vamos desenhar dois diagramas: um da região sólida $T$ (Figura 1) e o outro a sua projeção $D$ no plano $xy$ (Figura 2). 

ma211lista8q1res.png

A fronteira inferior do tetraedro $T$ é o plano $z=0$ e a superior é o plano $x+y+z=1$ (ou $z=1-x-y$). 

Notemos que os planos $x+y+z=1$ e $z=0$ se interceptam na reta $x+y=1$ (ou $y=1-x$) no plano $xy.$ 

Logo a projeção de $T$ é a região triangular da Figura 2 e temos 

$$T=\{(x,y,z)|\,0\leq x \leq 1,\, 0\leq y \leq 1-x,\, 0\leq z \leq 1-x-y\}.$$

Assim, 

$$\int\int\int\limits_{T}x^{2}\,dV=\int_{0}^{1}\int_{0}^{1-x}\int_{0}^{1-x-z}x^{2}\,dz\,dy\,dx=\int_{0}^{1}\int_{0}^{1-x}x^{2}z\bigg|_{0}^{1-x-y}\,dy\,dx$$

$$=\int_{0}^{1}\int_{0}^{1-x}x^{2}(1-x-y)\,dy\,dx=\int_{0}^{1}\int_{0}^{1-x}(x^{2}-x^{3}-x^{2}y)\,dy\,dx$$

$$=\int_{0}^{1}\bigg(x^{2}y-x^{3}y-x^{2}\frac{y^{2}}{2}\bigg)\bigg|_{0}^{1-x}\,dx=\int_{0}^{1}\bigg(x^{2}(1-x)-x^{3}(1-x)-\frac{x^{2}}{2}(1-x)^{2}\bigg)dx$$

$$=\int_{0}^{1}\bigg(\frac{x^{2}}{2}-x^{3}+\frac{x^{4}}{2}\bigg)\,dx =\bigg[\frac{1}{2}\cdot\frac{x^{3}}{3}-\frac{x^{4}}{4}+\frac{1}{2}\cdot\frac{x^{5}}{5}\bigg]\bigg|_{0}^{1}=\frac{1}{60}.$$


2400   

 Determine os pontos da superfície $x^2 + 2y^2 + 3z^2 = 1$ nos quais o plano tangente é paralelo ao plano $3x - y + 3z = 1$.


$\displaystyle \left(\frac{3\sqrt{2}}{5}, - \frac{1}{5\sqrt{2}}, \frac{\sqrt{2}}{5} \right)$ e $\displaystyle \left(-\frac{3\sqrt{2}}{5}, \frac{1}{5\sqrt{2}}, -\frac{\sqrt{2}}{5} \right).$


2504   

Dois mapas de contorno são mostrados na figura. Um é de uma função $f$ cujo gráfico é um cone. O outro é de uma função $g$ cujo gráfico é um paraboloide. Qual é qual? Por quê?

ma211-list2-ex15.png


O da esquerda corresponde ao cone e o da direita ao paraboloide.


2203   

Quando o tamanho das moléculas e suas forças de atração são levadas em conta, a pressão $P$, o volume $V$ e a temperatura $T$ 
de um mol de gás confinado estão relacionados pela {\it equação de van der Waals}
$$\bigg(P+\frac{a}{V^{2}}\bigg)(V-b)=kT,$$
em que $a$, $b$ e $k$ são constantes positivas. Se $t$ é o tempo, estabeleça uma fórmula para $\mathrm{d}T/ \mathrm{d}t$ em termos de $\mathrm{d}P/\mathrm{d} t$, 
$\mathrm{d} V/\mathrm{d}t$, $P$ e $V$.


$\displaystyle \frac{dT}{dt} = \frac{1}{k} \left( \left(\frac{dP}{dt} - \frac{2a}{V^{3}} \frac{dV}{dt}\right)(V - b) + \left( P + \frac{a}{V^{2}} \right) \frac{dV}{dt} \right).$


2984   

Utilize a transformação dada para calcular a integral. $\displaystyle\iint\limits_{R} xy \, dA$, em que $R$ é a região do primeiro quadrante limitada pelas retas $y = x$ e $y = 3x$ e pelas hipérboles $xy = 1$, $xy = 3$; $x = \dfrac{u}{v}$, $y = v$.


$2 \ln 3.$


2604   

Vamos demonstrar a expressão geral para o volume de um cone circular de altura $h$ e raio da base $R$.

  1.  Representando o cone com vértice na origem e base no plano $z = h$, expresse $V$ por meio de uma integral dupla.

  2.  Calculando a integral, verifique que $V = \dfrac{\pi R^2 h}{3}$.


  1.  $V = 2 \displaystyle \int_{0}^{h} \int_{-\frac{R}{h}z}^{\frac{R}{h}z} \sqrt{\dfrac{R^{2}}{h^{2}} z^{2} - x^{2}} dx dz.$

  2.  Note que $\displaystyle \int_{0}^{h} \int_{-\frac{R}{h}z}^{\frac{R}{h}z} \sqrt{\dfrac{R^{2}}{h^{2}} z^{2} - x^{2}} dx dz = \dfrac{\pi R^{2}h}{6}$ é o volume da parte superior (ou inferior) do cone.


2066   

Dados ${\bf F}(x,y,z)=y^{2}\,\cos z\,{\bf i}+2xy\,\cos z\,{\bf j}-xy^{2}\,\sin z\,{\bf k}$, $C: {\bf r}(t)=t^{2}\,{\bf i}+\sin t\,{\bf j}+t\,{\bf k}$, $0\leq t\leq \pi.$

  1. Determine uma função $f$ tal que ${\bf F}=\nabla f$.

  2. Use o resultado anterior para calcular $\int_{C}{\bf F}\cdot d{\bf r}$ sobre a curva $C$ dada.


  1. $f(x,y,z) = xy^{2} \cos(z);$

  2. $0.$


3030   

Esboce a região de integração e mude a ordem de integração. $\displaystyle\int_{0}^{1}\!\!\int_{\arctan{x}}^{\pi/4}\!f(x,y)\,dy dx$.


ma211-list6-ex32_sol_d.png

2080   

A temperatura em um ponto $(x,y)$ é $T(x,y)$, medida em graus Celsius. Um inseto rasteja de modo que sua posição depois de 

$t$ segundos seja dada por $x=\sqrt{1+t}$ e $y=2+\dfrac{1}{3}t$, onde $x$ e $y$ são medidas em centímetros. A função temperatura satisfaz 

$T_{x}(2,3)=4$ e $T_{y}(2,3)=3$. Quão rápido a temperatura aumenta no caminho do inseto depois de três segundos?



A temperatura aumenta a uma taxa de $2º$C/s.


2157   

Calcule a integral de linha $\displaystyle\oint_{C} (x-y) dx + (x+y)dy$, $C$ é o círculo com centro na origem e raio 2, por dois métodos:

  1. diretamente; e

  2. utilizando o Teorema de Green.


$8\pi.$


2062   

Determine se ${\bf F}(x,y,z)=e^{y+2z}({\bf i}+x\,{\bf j}+2x\,{\bf k})$ é ou não um campo vetorial conservativo. Se for, determine uma função $f$ tal que ${\bf F}=\nabla f.$


Sim. $f(x,y,z) = xe^{y + 2z} + K.$


2061   

Determine se ${\bf F}(x,y,z)=(y\,\sin z)\,{\bf i}+(x\,\sin z)\,{\bf j}+(xy\,\cos z)\,{\bf k}$ é ou não um campo vetorial conservativo. Se for, determine uma função $f$ tal que ${\bf F}=\nabla f.$


Sim. $f(x,y,z) = xy\sin(z) + K.$


2246   

Calcule $\int_{C}\mathbf{F} \cdot \mathbf{n} \, ds$ ($\mathbf{n}$ é unitário), onde $\mathbf{F}(x,y) = x\mathbf{i} + y\mathbf{j}$, $C$ dada por $\mathbf{r}(t) = (t,t^2)$, $0 \leq t \leq 1$ e $\mathbf{n}$ a normal com componente $y < 0$.


$\dfrac{1}{3}$.


2751   

Considere a função

$$f(x,y) = \begin{cases}\dfrac{xy}{x^2 + y^2}, & \quad \text{se } (x,y) \neq (0,0),\\0, & \quad \text{se } (x,y) = (0,0).\\\end{cases}$$

Mostre que $f_x(0,0)$ e $f_y(0,0)$ existem, mas $f$ não é diferenciável em $(0,0)$.


$f_{x}(0,0) = f_{y}(0,0) = 0,$ mas $\lim_{(x,y) \to (0,0)} f(x,y)$ não existe, logo $f$ é discontínua em $(0,0)$ e portanto não é diferenciável neste ponto.


1979   

Determine o domínio da curva de equação vetorial
$$\textbf{r}(t) = \left( \sqrt{\dfrac{t - 2}{t + 1}}, \ln{(5 - t^2)}, e^{-t} \right).$$


2858   

  1. Determine os pontos críticos da função

    $$f(x,y)=-(x^{2}-1)^{2}-(x^{2}y-x-1)^{2}.$$

  2. Calcule os valores assumidos por $f$ nos pontos críticos. É possível classificar os pontos críticos sem utilizar o críterio da derivada segunda? Se for possível, classifique-os e justifique a resposta.


  1. $(1,2)$ e $(-1,0).$

  2. $f(1,2) = f(-1,0) = 0.$ Note que $f(x,y) \leq 0,$ o que implica que $(1,2)$ e $(-1,0)$ são pontos de máximo.


2686   

Encontre $f_{x}$, $f_{y}$ e $f_{z}$ para $f(x,y,z)=1+xy^{2}-2z^{2}$.


$\displaystyle f_{x} = 1+ y^{2} ,\;\;\;\; f_{y} = 2xy \;\;\;\;\text{e}\;\;\;\; f_{z} = -4z$.


2839   

Calcule a integral dupla usando coordenadas polares: $\displaystyle\iint\limits{R}\sin(x^{2}+y^{2})\,dA$, onde $R$ é a região acima do eixo $x$ e dentro da circunferência $x^{2}+y^{2}=9.$


$\displaystyle \frac{\pi}{2}(1 - \cos(9).$


2276   

Calcule $\nabla f(x,y)$.

$f(x,y) = \dfrac{x}{y}$


 $\displaystyle \nabla f(x,y) = \left(\frac{1}{y}, -\frac{x}{y^{2}} \right).$


2936   

Uma carga elétrica é distribuída sobre o retângulo $1 \leq x \leq 3$, $0 \leq y \leq 2$, de modo que a densidade de carga em $(x,y)$ é $\sigma(x,y) = 2xy + y^2$ (medida em coulombs por metro quadrado). Determine a carga total no retângulo.


$\displaystyle \frac{64}{3}$ Coulombs.


2738   

$z = 2x + y$ é a equação do plano tangente ao gráfico de $f(x,y)$ no ponto $(1,1,3)$. Calcule $\dfrac{\partial f}{\partial x}(1,1)$ e $\dfrac{\partial f}{\partial y}(1,1).$


$\displaystyle \frac{\partial f}{\partial x} (1,1) = 2$ e  $\displaystyle \frac{\partial f}{\partial y} (1,1) = 1.$


2547   

Um fluido tem densidade $870kg/m^{3}$ e escoa com velocidade $v=z{\bf i}+y^{2}{\bf j}+x^{2}{\bf k},$ onde $x$, $y$ e $z$ são medidos em metros e as componentes de $v$ em metros por segundo. Encontre a vazão para fora do cilindro $x^{2}+y^{2}=4$, $0\leq z\leq 1.$


$0$ kg/s.


2250   

Verifique que $\mbox{div} {\bf E}=0$ para o campo elétrico ${\bf E}({\bf x})=\dfrac{\epsilon Q}{|{\bf x}|^{3}}{\bf x}.$


2237   

Seja $\mathbf{r} = x\mathbf{i} + y\mathbf{j} + z\mathbf{k}$ e $r=|\mathbf{r}|$. Verifique a identidade $\nabla{\ln{r}} = \dfrac{\mathbf{r}}{r^2}$.


$\nabla{\ln{r}} = \dfrac{1}{2} \nabla \ln (x^{2} + y^{2} + z^{2}).$ (Note que: $r = \sqrt{x^{2} + y^{2} + z^{2}}.$)


2658   

Determine as derivadas parciais de $z=x^{2}\ln(1+x^{2}+y^{2})$.


$\displaystyle \frac{\partial z}{\partial x} = 2x\ln(1+ x^{2} + y^{2}) + \frac{2x^{3}}{1 + x^{2} + y^{2}}\;\;\;\;\;\;\text{e}\;\;\;\;\; \frac{\partial z}{\partial y} = \frac{2x^{2}y}{1 + x^{2} + y^{2}}.$


2891   

Passe para coordenadas polares e calcule: $\displaystyle\int_{-1}^{1} \int_{-\sqrt{1-y^{2}}}^{\sqrt{1-y^{2}}}\ln(x^{2}+y^{2}+1)\,dx dy$


$\displaystyle \pi (\ln(4) - 1).$


2764   

Determine o maior conjunto de pontos em que a função $f(x,y) = \begin{cases}e^{\dfrac{1}{x^2 + y^2 - 1}}, & \quad \text{se } x^2 + y^2 < 1,\\0, & \quad \text{se } x^2 + y^2 \geq 1\end{cases}$ é diferenciável. Justifique.


$\mathbb{R}^{2}$.


2736   

Determine a equação dos planos tangentes ao gráfico de $f(x,y) =  - x^2 - y^2$ que passam por ambos os pontos $(1,0,7)$ e $(3,0,3)$.


$2x + 2y + z = 9$ e $2x - 2y + z = 9.$


2566   

Utilize coordenadas polares $x=r\cos \theta$ e $y=r\sin \theta$, com $r \geq 0$ e $0 \leq \theta < 2 \pi$, e o teorema do confronto para calcular o limite

$$\displaystyle \lim_{(x,y) \to (0,0)}\dfrac{x^3 + y^3}{x^2 + y^2}.$$

Dica: Note que, se $(r, \theta)$ são as coordenadas polares do ponto $(x,y)$, com $r \geq 0$, então $r \to 0^+$ quando $(x,y) \to (0,0)$.


$0.$


2610   

Use o Teorema de Stokes para calcular $\displaystyle\int_C {\bf F}\cdot  d{\bf R}$. $C$ é orientada no sentido anti-horário quando vista de cima.

  • ${\bf F}(x,y,z) = xy{\bf i} + 2z{\bf j} + 3y{\bf k}$, $C$ é a curva de interseção do plano $x+z=5$ com o cilindro $x^2+y^2=9$.


$9\pi$.


2534   

Determine a massa e o centro de massa do cubo dado por $0\leq x\leq a$, $0\leq y\leq a$, $0\leq z\leq a$ e com função densidade:

  1.  $\rho(x,y,z)=x^{2}+y^{2}+z^{2}.$

  2.  $\rho(x,y,z)=x+y+z.$


  1.  Massa: $a^5;$ centro de massa: $\displaystyle \left(\frac{7a}{12},\frac{7a}{12},\frac{7a}{12} \right).$

  2.  Massa: $\dfrac{3a^4}{2};$ centro de massa: $\displaystyle \left(\frac{5a}{9},\frac{5a}{9},\frac{5a}{9} \right).$


2470   

Calcule a integral de superfície $\displaystyle\iint \limits_{ S}{\bf F}\cdot d{\bf S}$ para o campo vetorial ${\bf F}$ e superfície orientada $S$ dados abaixo. Em outras palavras, determine o fluxo de ${\bf F}$ através de $S$. Para superfícies fechadas, use a orientação positiva (para fora).

  • ${\bf F}(x,y,z)=x{\bf i}-y{\bf j}+z{\bf k}$ e $S$ é a superfície do sólido delimitado pelos gráficos de $z=x^{2}+y^{2}$ e $z=4.$


$8\pi.$


2036   

Seja $W(s,t)=F(u(s,t),v(s,t))$, onde $F$, $u$ e $v$ são diferenciáveis, e $u(1,0)=2$, $u_{s}(1,0)=-2$, $u_{t}(1,0)=6$, $F_{u}(2,3)=-1$, $v(1,0)=3$, $v_{s}(1,0)=5$, $v_{t}(1,0)=4$, $F_{v}(2,3)=10.$ Determine $W_{s}(1,0)$ e $W_{t}(1,0).$


$W_{s}(1,0) = 52$ e $W_{t}(1,0) = 34.$


2435   

Calcule a integral tripla.

  1. $\displaystyle\iiint\limits_{  E}2z\;dx dy dz$, onde $E$ é o conjunto $x^{2}+y^{2}+z^{2}\leq 4$ e $z\geq 0.$

  2. $\displaystyle\iiint\limits_{  E}2z\;dx dy dz$, onde $E$ é o conjunto $4x^{2}+9y^{2}+z^{2}\leq 4$ e $z\geq 0.$


  1.  $8\pi.$

  2.  $0.$


3076   

Calcule \(\displaystyle \lim_{(x,y)\to (-1,2)} \dfrac{xy}{x^2+y^2}\).



Como a função \(\displaystyle f(x,y)=\dfrac{xy}{x^2+y^2}\) é contínua no ponto \((-1,2)\) (de acumulação), basta avaliá-la neste mesmo ponto. Ou seja, \[ \lim_{(x,y)\to (-1,2)}\dfrac{xy}{x^2+y^2} = \dfrac{(-1)2}{(-1)^2+2^2} = -\dfrac{2}{5}. \]


2396   

Mostre que a equação do plano tangente ao elipsoide $x^2/a^2 + y^2/b^2 + z^2/c^2 = 1$ no ponto $(x_0,y_0,z_0)$ pode ser escrita como
$$\dfrac{xx_0}{a^2} + \dfrac{yy_0}{b^2} + \dfrac{zz_0}{c^2} = 1.$$



 Note que se $F(x,y,z) = x^2/a^2 + y^2/b^2 + z^2/c^2 - 1,$ então 
$$
\nabla F(x_{0},y_{0},z_{0}) = 2 \left(\frac{x_{0}}{a^{2}},\frac{y_{0}}{b^{2}},\frac{z_{0}}{c^{2}} \right)
$$
e a equação do plano tangente em $(x_{0},y_{0},z_{0})$ é 
$$
\nabla F(x_{0},y_{0},z_{0}) \cdot (x,y,z) = \nabla F(x_{0},y_{0},z_{0}) \cdot (x_{0},y_{0},z_{0}) = 2.
$$


2390   

A função diferenciável $z = f(x,y)$ é dada implicitamente pela equação $x^3 + y^3 + z^3 = 10$. Determine a equação do plano tangente ao gráfico de $f$ no ponto $(1,1,f(1,1))$.


 $x + y + 4z = 10.$


2301   

Identifique e faça um esboço da imagem da superfície parametrizada dada por ${\bf r}(u,v)=(u,v,1-u-v)$, $u\geq 0$, $v\geq 0$ e $u+v\leq 1.$


Região triangular do plano $x + y + z = 1:$ $0 \leq x \leq 1, $ $0 \leq y \leq 1,$ $0 \leq z \leq 1.$


2466   

Calcule a integral de superfície $\displaystyle\iint \limits_{ S}{\bf F}\cdot d{\bf S}$ para o campo vetorial ${\bf F}$ e superfície orientada $S$ dados abaixo. Em outras palavras, determine o fluxo de ${\bf F}$ através de $S$. Para superfícies fechadas, use a orientação positiva (para fora).

  • ${\bf F}(x,y,z)=x^{2}{\bf i}+y^{2}{\bf j}+z^{2}{\bf k}$ e $S$ é a fronteira do semicilindro sólido $0 \leq z \leq \sqrt{1-y^{2}}$, $0 \leq x \leq 2.$


$2\pi + \dfrac{8}{3}.$


3117   

Seja \(G\) a região sólida dentro da esfera de raio \(2\) centrada na origem e acima do plano \(z=1\). Mostre (ou verifique) os seguintes resultados:

  1.  O volume de \(G\) é dado por \[\iiint\limits_G\,dV = \int_0^{2\pi}\int_0^{\dfrac{\pi}{3}}\int_{\sec\phi}^{2}\rho^2\sin\phi\,d\rho d\phi d\theta \]

  2.  \[\iiint\limits_G\dfrac{z}{x^2+y^2+z^2}\,dV = \int_0^{2\pi}\int_0^{\pi/3}\int_{\sec\phi}^{2}\rho\cos\phi\sin\phi\,d\rho d\phi d\theta \]


2433   

Calcule a integral tripla.

  1.  $\displaystyle\iiint\limits_{  E}2z\;dx dy dz$, onde $E$ é o conjunto $x^{2}+y^{2}\leq 1$, $x^{2}+y^{2}+z^{2}\leq 4$ e $z\geq 0.$

  2.  $\displaystyle\iiint\limits_{  E}x\;dx dy dz$, onde $E$ é o conjunto $x^{2}-y^{2}\leq z \leq 1-2y^{2}.$


  1.  $\dfrac{7\pi}{2}.$

  2.  $0.$


2028   

Use a Regra da Cadeia para determinar $\mathrm{d}z/\mathrm{d} t$ ou $\mathrm{d}w/ \mathrm{d}t.$

$z=\sin{x}\cos{y}$, $x=\pi t$, $y=\sqrt{t}$.


$\displaystyle \frac{dz}{dt} = \pi \cos(x) \cos(y) - \frac{1}{2\sqrt{t}} \sin(x) \sin(y).$


2188   

Use o Teorema do Divergente para calcular o fluxo de ${\bf F}$ através de $S,$ onde ${\bf F}(x,y,z)=2xz\,{\bf i}+xyz\,{\bf j}+yz\,{\bf k}$ e $S$ é a superfície da região delimitada pelos planos coordenados e os planos $x+2z=4$ e $y=2.$


2753   

A função $f(x,y) = \begin{cases}\dfrac{x^2y}{x^2 + y^2}, & \quad \text{se } (x,y) \neq (0,0),\\0, & \quad \text{se } (x,y) = (0,0)\\\end{cases}$ é diferenciável em $(0,0)$? Justifique.


Não.


2770   

 Utilize coordenadas polares para combinar a soma $$\int_{\frac{1}{\sqrt{2}}}^{1} \int_{\sqrt{1-x^{2}}}^{x}xy\,dy dx+\int_{1}^{\sqrt{2}}   \int_{0}^{x}xy\,dy dx+\int_{\sqrt{2}}^{2} \int_{0}^{\sqrt{4-x^{2}}}xy\,dy dx$$ em uma única integral dupla. Em seguida, calcule essa integral dupla.



Queremos combinar a soma, abaixo, de integrais em uma única: $$\underbrace{\int_\frac{1}{\sqrt{2}}^{1} \int_\sqrt{1-x^{2}}^{x}xy\,dy dx}_{1}+\underbrace{\int_{1}^{\sqrt{2}}\int_{0}^{x}xy\,dy dx}_{2}+ \underbrace{\int_{\sqrt{2}}^{2} \int_{0}^{\sqrt{4-x^{2}}}xy\,dy dx}_{3}$$ Na figura abaixo, temos que a região da esquerda corresponde à região de integração da integral $(1)$, a região do meio corresponde à região de integração  da integral $(2)$ e a região da esquerda corresponde à região de integração da integral $(3)$.


ma211-list7-ex2_sol.png


Notemos que com a junção das três regiões, podemos olhar como uma única região. Assim, em coordenadas  polares teremos que $0\leq \theta \leq \frac{\pi}{4}$ e $1\leq r \leq 2.$ Então: $$\int_{\frac{1}{\sqrt{2}}}^{1} \int_{\sqrt{1-x^{2}}}^{x}xy\,dy dx+\int_{1}^{\sqrt{2}}   \int_{0}^{x}xy\,dy dx+\int_{\sqrt{2}}^{2} \int_{0}^{\sqrt{4-x^{2}}}xy\,dy dx$$ $$=\int_{0}^{\frac{\pi}{4}}\int_{1}^{2}(r\,\cos \theta)\cdot (r\,\sin \theta)\,r\,dr\,d\theta=\int_{0}^{\frac{\pi}{4}}\int_{1}^{2}r^{3}\cos\theta \sin \theta\,dr\,d\theta$$ $$=\underbrace{\int_{0}^{\frac{\pi}{4}}\cos \theta\, \sin \theta\,d \theta}_{\substack{ u=\sin \theta\\ du=\cos\, d\theta}}\cdot \int_{1}^{2}r^{3}\,dr =\int_{0}^{\frac{\sqrt{2}}{2}}u\,du\cdot \frac{r^{4}}{4}\bigg|_{1}^{2}$$ $$=\frac{u^{2}}{2}\bigg|_{0}^{\frac{\sqrt{2}}{2}}\cdot \bigg(\frac{16}{4}-\frac{1}{4}\bigg)=\frac{1}{4}\cdot \frac{15}{4}=\frac{15}{16}.$$


2111   

Utilize simetria para calcular $\iint\limits_{D}(2-3x+4y)\,dA$, onde $D$ é a região limitada pelo quadrado com vértices $(\pm 5,0)$ e $(0,\pm 5).$


$100.$


2345   

Mostre que as equações paramétricas $x=a \cosh u\cos v$, $y=b\cosh u \sin v$, $z=c\sinh u$, representam um hiperboloide de uma folha.


Note que $\dfrac{x^{2}}{a^{2}} + \dfrac{y^{2}}{b^{2}} - \dfrac{z^{2}}{c^{2}} = 1$.


2747   

Explique por que a função é diferenciável no ponto dado. $f(x,y) = x\sqrt{y}, \quad (1,4)$.


As derivadas $f_{x}$ e $f_{y}$ de cada $f$ existem e são contínuas no ponto dado, logo $f$ é diferenciável.


2398   

Determine uma reta que seja tangente à curva $x^2 + xy + y^2 = 7$ e paralela à reta $4x + 5y = 17$.


 $\displaystyle y - 2 = -\frac{4}{5} (x - 1)$ ou $\displaystyle y + 2 = -\frac{4}{5} (x + 1).$


2744   

Considere a função $f(x,y) = x \ g(x^2 - y^2)$, em que $g(u)$ é uma função derivável de uma variável. Mostre que o plano tangente ao gráfico de $f$ no ponto $(a,a,f(a,a))$ passa pela origem.


Note que $a \frac{\partial f}{\partial x} (a,a) + a \frac{\partial f}{\partial y}(a,a) = f(a,a).$



2308   

 Calcule $D_{\bf{u}}f(x_0,y_0)$, sendo dados

$f(x,y) = e^{x^2-y^2}$, $(x_0,y_0) = (1,1)$ e $\bf{u}$ o versor de $(3,4)$.


 $\displaystyle D_{\bf{(3,4)}}f(1,1) = -\frac{2}{5}.$ 


2100   

Considere o campo vetorial

$${\bf F}(x,y)=(\ln(y^{2}+1))\,{\bf i}+\bigg(\frac{2y(x-1)}{y^{2}+1}\bigg)\,{\bf j}.$$

  1. Determine se $F$ é ou não um campo conservativo.

  2. Determine o trabalho realizado pelo campo vetorial ${\bf F}$ ao mover uma partícula desde o ponto $(-1,1)$ até o ponto $(2,3).$


  1. Sim.

  2. $\ln(10) + 2\ln(2).$


2081   

 O comprimento $l$, a largura $w$ e a altura $h$ de uma caixa variam com o tempo. Em certo instante, as dimensões da caixa são $l=1m$ e $w=h=2m$. $l$ e $w$ aumentam a uma taxa de $2m/s$, ao passo que $h$ diminui a uma taxa de $3m/s$. Nesse instante, determine as taxas nas quais as seguintes quantidades estão variando.

  1. O volume.
  2. A área da superfície.
  3. O comprimento da diagonal.


  1. $6$ m$^3$/s.
  2. $10$ m$^2$/s.
  3. $0$ m/s.


2846   

Passe para coordenadas polares e calcule: $\displaystyle\iint\limits_{D}e^{-x^{2}-y^{2}}\,dA$, onde $D$ é a região delimitada pelo semicírculo $x=\sqrt{4-y^{2}}$ e o eixo $y.$


$\displaystyle \frac{\pi}{2} (1 - e^{-4}).$


2551   

Se $f(x_0,y_0) = 3$, o que podemos dizer sobre

$$\displaystyle \lim_{(x,y) \to (x_0,y_0)}f(x,y)$$

se $f$ for contínua em $(x_0,y_0)$? E se $f$ não for contínua em $(x_0,y_0)$? Justifique sua resposta.


Se $f$ for contínua em $(x_{0},y_{0}),$ então o limite é igual a $f(x_{0},y_{0}) = 3.$ Se não for contínua em $(x_{0},y_{0}),$ então o limite pode ter qualquer valor diferente de $3.$


3055   

 Faça o esboço do sólido cujo volume é dado pela integral e calcule essa integral.

$\displaystyle \int_0^4 \int_0^{2\pi}\int_r^4 r \, dz d\theta dr$


ma211lista8q32ares.png


2309   

 Calcule $D_{\bf{u}}f(x_0,y_0)$, sendo dados

$f(x,y) = \arctan{\dfrac{x}{y}}$, $(x_0,y_0) = (3,3)$ e $\bf{u} = \left(\frac{1}{\sqrt{2}},\frac{1}{\sqrt{2}}\right)$.


 $\displaystyle D_{\bf{\left(\frac{1}{\sqrt{2}},\frac{1}{\sqrt{2}} \right)}}f(3,3) = 0.$


2930   

Calcule a integral em coordenadas esféricas. $\displaystyle\int_{0}^{\pi}\int_{0}^{\pi}\int_{0}^{2\,\sin{\phi}}\rho^{2}\sin{\phi}\,d\rho d\phi d\theta$.


$\pi^2.$


2003   

Sejam $A=(3,0)$, $B=(1,1)$ e $C=(0,3)$ pontos de $\mathbb{R}^{2}$ e $C$ a trajetória que vai em linha reta de $A$ até $B$ e em seguida de $B$ até $C$. Determine o trabalho ao longo de $C$ do campo de forças ${\bf F}:\mathbb{R}^{2}\rightarrow \mathbb{R}^{2}$, sendo

$${\bf F}(x,y)=\bigg(-\frac{y}{x^{2}+y^{2}},\frac{x}{x^{2}+y^{2}}\bigg).$$


$\displaystyle 2\arctan(2) + \arctan\left(\frac{1}{2} \right) - \arctan\left(\frac{1}{3} \right).$


2242   

Calcule $\int_{C}\mathbf{F} \cdot \mathbf{n} \, ds$ ($\mathbf{n}$ é unitário, onde $\mathbf{F}(x,y) = x\mathbf{i} + y\mathbf{j}$, $C$ dada por $\mathbf{r}(t) = (\cos{t},\sin{t})$, $0 \leq t \leq 2\pi$ e $\mathbf{n}$ a normal exterior.


$2\pi.$


2025   

Suponha que em uma certa região do espaço o potencial elétrico $V$ seja dado por $V(x,y,z) = 5x^2 - 3xy + xyz.$

  1. Determine a taxa de variação do potencial em $P = (3,4,5)$ na direção do vetor $\bf{v} = \bf{i} + \bf{j} - \bf{k}$.
  2. Em que direção $V$ varia mais rapidamente em $P$?
  3. Qual a taxa máxima de variação em $P$?



  1. Queremos determinar o valor de $D_\bf{u}f(P)$, em que $\bf{u}$ é o vetor unitário que tem mesma direção de $\bf{v}$, isto é, $\bf{u} = \frac{1}{\sqrt{3}}(1,1,-1)$. Como $V$ é diferenciável, segue que  $D_\bf{u}f(P) = \nabla V(P) \cdot \mathbf{u}$. Observe que \linebreak $\nabla V(x,y,z) = (10x-3y+yz,-3x+xz,xy)$. Logo $\nabla V(P) = (38,6,12)$. Portanto, $$D_\bf{u}f(P) = \nabla V(P) \, \cdot \, \mathbf{u} = (38,6,12) \, \cdot \, \dfrac{1}{\sqrt{3}}(1,1,-1) = \dfrac{32\sqrt{3}}{3}.$$
  2. A direção em que $V$ varia mais rapidamente no ponto $P$ é a direção do gradiente de $V$ no ponto $P$, isto é, na direção de $\nabla V(P) = (38,6,12)$. Observe que aqui não é necessário normalizar o vetor, pois o exercício pede apenas a direção.
  3. A taxa de variação máxima é $|\nabla V(P)| = 2\sqrt{406}$.


1963   

Calcule a integral de linha $\displaystyle\int_{C}y^{2}\,dx+x\,dy -\,dz$, onde $C$ é a poligonal de vértices $A_{0}=(0,0,0)$, $A_{1}=(1,1,1)$, $A_{2}=(1,1,0)$, orientada de $A_{0}$ para $A_{2}.$


$\displaystyle \frac{5}{6}.$


2011   

Um homem pesando $160$ lb carrega uma lata de tinta de $25$ lb por uma escada helicoidal em torno de um silo com raio de $20$ pés. Se o silo tem $90$ pés de altura e o homem dá três voltas completas em torno do silo. Além disso, $9$ lb de tinta vazam da lata de modo contínuo e uniforme durante a subida do homem. Quanto trabalho é realizado?


$16245$ ft-lb.


1945   

Calcule a integral de linha, onde $C$ é a curva dada.

$\displaystyle\int_{C}dx+xy\,dy+z\,dz$, $C$ é a interseção de $x^{2}+y^{2}+z^{2}=2$, $x\geq 0$, $y\geq 0$ e $z\geq 0$, com o plano $y=x$; o sentido de percurso é do ponto $(0,0,\sqrt{2})$ para $(1,1,0).$


$\displaystyle \frac{1}{3}.$


2758   

Verifique que a função $f(x,y) = \ln{(1 + x^2 + y^2)}$ é diferenciável.


As derivadas parciais $\frac{\partial f}{\partial x}$ e $\frac{\partial f}{\partial y}$ de cada função $f$ existem e são contínuas em todos os pontos do domínio.


2473   

Determine e faça o esboço do domínio da função $f(x,y)=\sqrt{x+y}$.


$\left\lbrace (x,y);\; y \geq -x \right\rbrace$

ma211-list2-ex10_sol_a.png


2841   

Passe para coordenadas polares e calcule: $\displaystyle\int_{0}^{1}  \int_{x^{2}}^{\sqrt{2-x^{2}}}\sqrt{x^{2}+y^{2}}\,dy dx$


$\displaystyle \frac{2}{45}(1 + \sqrt{2}) + \frac{\pi}{3\sqrt{2}}.$


3141   

Supondo que \(\sigma\) e \(G\) satisfaçam as hipóteses do Teorema da Divergência e que \(f\) e \(g\) sejam funções suficientemente regulares, prove as seguintes identidades (de Green):

  1.  \[\iint\limits_\sigma\left(f\nabla g\right)\cdot\mathbf{n}\,dS = \iiint\limits_G\left( f\Delta g+\nabla f\cdot\nabla g\right)\,dV, \]

  2.  \[\iint\limits_\sigma\left(f\nabla g-g\nabla f\right)\cdot\mathbf{n}\,dS = \iiint\limits_G\left( f\Delta g- g\Delta f\right)\,dV, \] onde \(\displaystyle \Delta f= \dfrac{\partial^2f}{\partial x^2}+\dfrac{\partial^2f}{\partial y^2}+\dfrac{\partial^2f}{\partial z^2}\) é denominado Laplaciano de \(f\).


2346   

Encontre a área da parte da esfera $x^{2}+y^{2}+z^{2}=a^{2}$ que está dentro do cilindro $x^{2}+y^{2}=ax.$


$2a^2 (\pi - 2).$


2919   

Utilize a integral dupla para determinar a área da região: no interior do círculo $x^{2}+(y-1)^{2}=1$ e fora do círculo $x^{2}+y^{2}=1.$


$\displaystyle \frac{\pi}{3} + \frac{\sqrt{3}}{2}.$


1931   

Calcule a integral de linha, onde $C$ é a curva dada.

$\displaystyle\int_{C}xy^{4}\,ds$,   $C$ é a metade direita do círculo $x^{2}+y^{2}=16.$


$\dfrac{2^{13}}{5}.$


3040   

  1. Definimos a integral imprópria (sobre todo o plano $\mathbb{R}^{2}$) $$I=\displaystyle\iint\limits_{ \mathbb{R}^{2}}e^{-(x^{2}+y^{2})}\,dA=\int_{-\infty}^{\infty} \int_{-\infty}^{\infty}e^{-(x{^2}+y^{2})}\,dy dx= \lim_{a\rightarrow\infty}\displaystyle\iint\limits_{D_{a}}e^{-(x^{2}+y^{2})}\,dA,$$ onde $D_{a}$ é o disco com raio $a$ e centro na origem. Mostre que $$\int_{-\infty}^{\infty} \int_{-\infty}^{\infty}e^{-(x^{2}+y^{2})}\,dA=\pi.$$

  2. Uma definição equivalente da integral imprópria da parte (a) é $$\iint\limits_{\mathbb{R}^{2}}e^{-(x^{2}+y^{2})}\,dA=\lim_{a\rightarrow\infty}\displaystyle\iint\limits_{S_{a}}e^{-(x^{2}+y^{2})}\,dA,$$ onde $S_{a}$ é o quadrado com vértices $(\pm a,\pm a)$. Use esse resultado para mostrar que $$\int_{-\infty}^{\infty}e^{-x^{2}}\,dx\,\int_{-\infty}^{\infty}e^{-y^{2}}\,dy=\pi.$$

  3. Deduza que $$\int_{-\infty}^{\infty}e^{-x^{2}}\,dx=\sqrt{\pi}.$$

  4. Fazendo a mudança de variável $t=\sqrt{2} x$, mostre que $$\int_{-\infty}^{\infty}e^{-x^{2}/2} dx=\sqrt{2\pi}.$$

    (Este é um resultado fundamental em probabilidade e estatística.)


  1. Note que $$\displaystyle\iint\limits_{D_{a}}e^{-(x^{2}+y^{2})}\,dA =\int_{0}^{2\pi}\int_{0}^{a} r e^{-r^2} dr d\theta = \pi (1 - e^{-a^2})$$ para cada $a.$

  2. Note que $$\int\limits_{S_{a}}e^{-(x^{2}+y^{2})}\,dA = \int_{-a}^{a} \int_{-a}^{a} e^{-x^2} e^{-y^2} dxdy = \left(\int_{-a}^{a} e^{-x^2} dx\right) \left(\int_{-a}^{a} e^{-y^2} dy\right) $$ para cada $a.$

  3. Troque $y$ por $x$ no item (b).

  4. Note que fazendo a mudança de variável sugerida, $$\int_{-\infty}^{\infty}e^{-x^{2}/2} dx= \frac{1}{\sqrt{2}}\int_{-\infty}^{\infty}e^{-t^{2}/2} dt = \sqrt{\pi}.$$


2270   

Determine uma equação do plano tangente à superfície parametrizada dada no ponto especificado. ${\bf r}(u,v)=(u-v,u^{2}+v^{2},uv)$, no ponto ${\bf r}(1,1).$



Temos que ${\bf r}(u,v)=\underbrace{(u-v)}_{x(u,v)}\,{\bf i}+\underbrace{(u^{2}+v^{2})}_{y(u,v)}\,{\bf j}+\underbrace{uv}_{z(u,v)}\,{\bf k}$

Primeiro, vamos calcular os vetores tangentes:

$$\begin{array}{rcl}{\bf r}_{u}&=&\frac{\partial x(u,v)}{\partial u}\,{\bf i}+\frac{\partial y(u,v)}{\partial u}\,{\bf j}+\frac{\partial z(u,v)}{\partial u}\,{\bf k}\\&=& \,{\bf i}+2u\,{\bf j}+v\,{\bf k}\end{array}$$

e

$$\begin{array}{rcl}{\bf r}_{v}&=&\frac{\partial x(u,v)}{\partial v}\,{\bf i}+\frac{\partial y(u,v)}{\partial v}\,{\bf j}+\frac{\partial z(u,v)}{\partial v}\,{\bf k}\\&=& -\,{\bf i}+2v\,{\bf j}+u\,{\bf k}\end{array}$$


Assim, o vetor normal ao plano tangente é:

$$\begin{array}{rcl}{\bf r}_{u}\times {\bf r}_{v}&=&\left|\begin{array}{ccc}{\bf i}& {\bf j}&{\bf k}\\1 & 2u & v\\-1 & 2v & u\\\end{array}\right|\\&=&(-2u^{2}-2v^{2})\,{\bf i}-(u+v)\,{\bf j}+(2u+2v)\,{\bf k}\end{array}$$


Como $u=1$ e $v=1$ temos que o vetor normal é $-4\,{\bf i}-2\,{\bf j}+4\,{\bf k}.$

Portanto, uma equação do plano tangente no ponto ${\bf r}(1,1)=(0,2,1)$ é

$$-4\cdot(x-0)-2\cdot(y-2)+4\cdot (z-1)=0$$

$$-4x-2y+4+4z-4=0$$

$$-4x-2y+4z=0    \mbox{ou}     2x+y-2z=0$$


2646   

Determine a derivada parcial indicada. $u=e^{r\theta}\sin{\theta}$; $\dfrac{\partial ^{3}u}{\partial r^{2}\partial \theta}$.


$\dfrac{\partial ^{3}u}{\partial r^{2}\partial \theta} = \theta e^{r\theta} (2\sin \theta + \theta \cos \theta + r\theta \sin \theta)$.


2773   

Determine os valores máximos e mínimos locais e pontos de sela da função $f(x,y)=y\cos{x}$.



Sendo $f(x,y)=y\,\cos x$, vamos inicialmente localizar seus pontos críticos:

$$f_{x}(x,y)=-y\,\sin x \;\;\;\;\;\;\;\; \mbox{e} \;\;\;\;\;\;\;\; f_{y}(x,y)=\cos x.$$

Igualando essas derivadas parciais a zero, obtemos as equações

$$y\,\sin x=0 \;\;\;\;\;\;\;\; \mbox{e} \;\;\;\;\;\;\;\; \cos x=0.$$

Da segunda equação obtemos que $x=\bigg(\dfrac{\pi}{2}+n\pi\bigg)$, $n\in \mathbb{Z}.$ Da primeira equação temos que $y=0$ para todos essas $x$-valores.

Assim, os pontos críticos são $\bigg(\dfrac{\pi}{2}+n\pi,0\bigg).$ Agora,

$$f_{xx}(x,y)=-y\,\cos x,\;\;\;\;\;\; f_{xy}(x,y)=-\sin x\;\;\;\;\;\;\; \mbox{e}\;\;\;\;\;\; f_{yy}(x,y)=0.$$

Então

\begin{array}{rcl}D(x,y)&=&(f_{xx}(x,y))\cdot (f_{yy}(x,y))-(f_{xy}(x,y))^{2}\\&\Rightarrow& D\bigg(\dfrac{\pi}{2}+n\pi,0\bigg)=0-\sin^{2}x=-\sin^{2}x<0.\end{array}

Portanto, cada ponto crítico é ponto de sela.


1993   

Calcule.

  1. $\displaystyle\int_{0}^{1}(t{\bf i}+e^{t}{\bf j})\mathrm{d}t$
  2. $\displaystyle\int_{-1}^{1}\!\bigg(\sin(3t){\bf i}+\dfrac{1}{1+t^{2}}{\bf j}+{\bf k}\bigg)\mathrm{d}t$
  3. $\displaystyle\int_{1}^{2}(3{\bf i}+2{\bf j}+{\bf k})\mathrm{d}t$


3064   

Determine o campo vetorial gradiente $\nabla f$ de $f(x,y) = \sqrt{x^2+y^2}$ e o esboce.


2509   

Ache $\displaystyle\iint \limits_{ S}{\bf F}\cdot {\bf n} dS$ se ${\bf n}$ é uma normal unitária superior de $S.$


  • ${\bf F}=x{\bf i}+y{\bf j}+z{\bf k}$; $S$ é o hemisfério superior de $x^{2}+y^{2}+z^{2}=a^{2}.$


$2\pi a^3.$


2866   

Estude com relação a máximos e mínimos a função dada com as restrições dadas.

$f(x,y) = x^2 - 2xy + y^2$ e $x^2 + y^2 = 1.$


Pontos de máximo: $\displaystyle \left( \frac{1}{\sqrt{2}}, -\frac{1}{\sqrt{2}} \right)$ e $\displaystyle \left( -\frac{1}{\sqrt{2}}, \frac{1}{\sqrt{2}} \right)$; ponto de mínimo: $\displaystyle \left( \frac{1}{\sqrt{2}}, \frac{1}{\sqrt{2}}\right)$.


2243   

Calcule $\int_{C}\mathbf{F} \cdot \mathbf{n} \, ds$ ($\mathbf{n}$ é unitário), onde $\mathbf{F}(x,y) = y\mathbf{j}$, $C$ a fronteira do quadrado de vértices $(0,0)$, $(1,0)$, $(1,1)$, $(0,1)$ e $\mathbf{n}$ a normal que aponta para fora do quadrado, sendo $C$ orientada no sentido anti-horário.



$1.$


2506   

Faça o mapa de contorno da função $f(x,y)=(y-2x)^{2}$  mostrando várias de suas curvas de nível.


$y=2x\pm \sqrt{C}, C \geq0.$

ma211-list2-ex17_sol_a.png


2879   

A produção total $P$ de certo produto depende da quantidade $L$ de trabalho empregado e da quantidade $K$ de capital investido. Nas Seções 14.1 e 14.3 do livro do Stewart, foi discutido o modelo Cobb-Douglas $P = bL^\alpha K^{1-\alpha}$ seguido de certas hipóteses econômicas, em que $b$ e $\alpha$ são constantes positivas e $\alpha < 1$. Se o custo por unidade de trabalho for $m$ e o custo por unidade de capital for $n$, e uma companhia puder gastar somente uma quantidade $p$ de dinheiro como despesa total, então a maximização da produção $P$ estará sujeita à restrição $mL + nK = p$. Mostre que a produção máxima ocorre quando

$$L = \dfrac{\alpha p}{m} \quad \text{e} \quad K = \dfrac{(1 - \alpha)p}{n}.$$


2725   

O comprimento e a largura de um retângulo foram medidos como $30$ cm e $24$ cm, respectivamente, com um erro de medida de, no máximo, $0,1$ cm. Utilize as diferenciais para estimar o erro máximo cometido no cálculo da área do retângulo.


$\Delta A \approx 5.4$ cm$^{2}$.


2701   

Verifique que $x\;\dfrac{\partial ^{2}z}{\partial x \partial y}+y\;\dfrac{\partial ^{2}z}{\partial y^{2}}=0$, onde $z=(x+y)e^{x/y}.$



$\displaystyle \frac{\partial^{2} z}{\partial x \partial y}= \frac{-3xy - x^{2}}{y^{3}}e^{\frac{x}{y}}  \;\;\;\;\;\text{e}\;\;\;\;\; \frac{\partial^{2} z}{\partial y^{2}}= \frac{3x^{2}y + x^{3}}{y^{4}}e^{\frac{x}{y}}.$


2314   

Determine a derivada direcional da função no ponto dado e na direção do vetor $\bf{v}$.

$f(x,y,z) = \sqrt{xyz},  (3,2,6),  \bf{v} = \left(-1,-2,2\right).$


$-1.$


3014   

  1.  Verifique que $$f(x,y) = \begin{cases} 4xy, & \quad \text{se } 0 \leq x \leq 1, \ 0 \leq y \leq 1,\\ 0, & \quad \text{caso contrário}, \end{cases}$$ é uma função densidade conjunta.

  2.  Se $X$ e $Y$ são variáveis aleatórias cuja função densidade conjunta é a função $f$ do item anterior, determine: (i) $P(X \geq \frac{1}{2})$,          (ii) $P(X \geq \frac{1}{2}, Y \leq \frac{1}{2})$.

  3.  Determine os valores esperados de $X$ e $Y$.


  1.  Note que $$\iint_\limits{\mathbb{R}^2} f(x,y)\,dA = \int_{0}^{1} \int_{0}^{1} 4xy\, dydx = 1. $$

  2.  (i) $\dfrac{3}{4}.$               (ii) $\dfrac{3}{16}.$

  3.  $\dfrac{3}{16}.$


2500   

Dada a função $f(x,y)=e^{-(x^{2}+y^{2})}$.

  1. Encontre o domínio da função.

  2. Encontre a imagem da função.

  3. Descreva as curvas de nível da função.


  1. $D_{f} = \mathbb{R}^{2}$.

  2. $Im(f) = \left\lbrace z \in \mathbb{R};\; 0 < z \leq 1 \right\rbrace.$

  3. As curvas de nível são as os círculos $x^{2} + y^{2} = C$ com $C > 0$ e a origem.


2806   

Determine os valores máximos e mínimos locais e pontos de sela da função $f(x,y)=x^{2}-4xy+4y^{2}-x+3y+1$.


Não há pontos críticos.


3032   

Considere a integral

$$\int_{0}^{1}\!\!\int_{x^{2}}^{1}x^{3}\sin{y^{3}}\,dy dx.$$

  1. Desenhe a região de integração.

  2. Calcule o valor da integral.


  1. ma211-list6-ex38_sol.png

  2. $\dfrac{1 - \cos(1)}{12}$.


1976   

Trace a curva com equações paramétricas
\begin{eqnarray}
x & = & \sqrt{1 - 0,25 \cos^210t} \cos{t} \nonumber \\
y & = & \sqrt{1 - 0,25 \cos^210t} \sin{t} \nonumber \\
z & = & 0,5 \cos{10t}. \nonumber
\end{eqnarray} 
Explique a aparência da curva, mostrando que ela está em uma esfera.


2845   

Passe para coordenadas polares e calcule: $\displaystyle\int_{0}^{6}  \int_{0}^{y}x\,dx dy$


$36.$


2481   

Use a integral tripla para determinar o volume do sólido dado.

  1.  $0\leq x \leq 1$, $0\leq y \leq 1$ e $0\leq z \leq 5-x^{2}-3y^{2}.$

  2.  $0\leq x \leq 1$, $0\leq y \leq x^{2}$ e $0\leq z \leq x+y^{2}.$

  3. $x^{2}+y^{2}\leq z \leq 4.$

  4.  $x^{2}+4y^{2}\leq z \leq 1.$


  1.  $\dfrac{11}{3}.$

  2. $\dfrac{25}{84}.$

  3. $8\pi.$

  4.  $\dfrac{\pi}{4}.$


2034   

Utilize a Regra da Cadeia para determinar $\mathrm{\partial}z/\mathrm{\partial} s$ e $\mathrm{\partial}z/ \mathrm{\partial}t.$

$z=e^{r}\cos{\theta}$, $r=st$, $\theta=\sqrt{s^{2}+t^{2}}$.



$\displaystyle \frac{\partial z}{\partial s} = e^{r} \left( t\cos(\theta) - \frac{s}{\sqrt{s^{2} + t^{2}}} \sin(\theta) \right) $ e $\displaystyle \frac{\partial z}{\partial t} = e^{r} \left( s\cos(\theta) - \frac{t}{\sqrt{s^{2} + t^{2}}} \sin(\theta) \right).$


2306   

Seja $f$ uma função de três variáveis independentes $x,y$ e $z$. Mostre que $D_{\bf{i}}f = f_x$, $D_{\bf{j}}f = f_y$ e $D_{\bf{k}}f = f_z$.


Lembre que $\bf{i} = (1,0,0),$ $\bf{j} = (0,1,0),$ $\bf{k} = (0,0,1)$ e $D_{\bf{u}}f = \nabla f \cdot \bf{u}.$


2755   

Verifique que a função $f(x,y) = e^{x - y^2}$ é diferenciável.


As derivadas parciais $\frac{\partial f}{\partial x}$ e $\frac{\partial f}{\partial y}$ de cada função $f$ existem e são contínuas em todos os pontos do domínio.


3121   

Usando coordenadas esféricas, calcule a massa do sólido compreendido entre as esferas \(x^2+y^2+z^2=1\) e \(x^2+y^2+z^2=4\), com densidade \(\delta(x,y,z)=(x^2+y^2+z^2)^{-1/2}.\)


2173   

Calcule o trabalho realizado pela força $\mathbf{F}(x,y) = xy\mathbf{i}+y^2\mathbf{j}$ ao mover uma partícula da origem ao longo da reta $y=x$ até $(1,1)$ e então de volta à origem ao longo da curva $y=x^2$.


$\dfrac{1}{12}.$


2487   

Esboce o gráfico da função $f(x,y)=\cos{x}$.


$z = \cos(x)$

ma211-list2-ex11_sol_d.png


2019   

Uma piscina de 8 por 12 metros está cheia de água. A profundidade é medida em intervalos de 2 metros, começando em um canto da piscina, e os valores foram registrados na tabela. Estime o volume de água na piscina.

$$ \begin{array}{|c|c|c|c|c|c|c|c|} \hline  & 0 & 2   & 4   & 6   & 8   & 10  & 12  \\ \hline  0      & 1 & 1,5  & 2   & 2,4 & 2,8 & 3   & 3   \\ 2       & 1 & 1,5 & 2   & 2,8 & 3   & 3,6 & 3   \\ 4        & 1 & 1,8 & 2,7 & 3   & 3,6 & 4   & 3,2 \\ 6        & 1 & 1,5 & 2   & 2,3 & 2,7 & 3   & 2,5 \\   8     & 1 & 1   & 1   & 1   & 1,5 & 2   & 2   \\ \hline\end{array}$$


$\approx 227.$


2580   

Determine o conjunto dos pontos de continuidade da função $f(x,y) = \mbox{ln} \ \dfrac{x - y}{x^2 + y^2}$. Justifique sua resposta.


$\left\lbrace (x,y);\; x > y \right\rbrace.$


2831   

Encontre o volume máximo de uma caixa retangular que está inscrita em uma esfera de raio $r.$


$\displaystyle \frac{8}{3\sqrt{3}} r^{3}.$


2386   

Determine as equações do plano tangente e da reta normal à superfície dada, no ponto dado.

$x^2 + 3y^2 + 4z^2 = 8$, em $(1,-1,1)$.


Plano tangente: $x - 3y + 4z = 8$
Reta normal: $(x,y,z) = (1,-1,1) + \lambda (2,-6,8),$ $\lambda \in \mathbb{R}.$


2459   

Verifique que, para a função de produção de Cobb-Douglas

$$P(L,K)=1,01L^{0,75}K^{0,25}$$

discutida no Exemplo 3 da Seção 14.1 do Stewart, a produção dobrará se as quantidades de trabalho e a de capital investido forem dobradas. Determine se isto também é verdade para uma função de produção genérica

$$P(L,K)=bL^{\alpha}K^{1-\alpha}.$$


Sim.


2260   

Escreva a integral dupla $$\iint\limits_{R}x\cos{y}\;dA,$$ onde $R$ é limitada pelas retas $y=0$, $x=\pi/4$ e $y=x$, das duas formas possíveis (mudando a ordem de integração). Escolha uma dessas formas e calcule o valor dessa integral.


$\displaystyle \int_{0}^{\pi/4} \int_{0}^{x} x \cos(y)\;dy\;dx = \int_{0}^{\pi/4} \int_{y}^{\pi / 4} x \cos(y)\;dx\;dy = -\frac{\pi - 4}{4\sqrt{2}}.$


2564   

Determine o maior conjunto no qual a função $f(x,y) = \begin{cases}\dfrac{x^2y^3}{2x^2 + y^2}, & \quad \text{se } (x,y) \neq (0,0), \\1, & \quad \text{se } (x,y) = (0,0) \end{cases}$ é contínua.


$\left\lbrace (x,y);\; (x,y) \neq (0,0) \right\rbrace.$


2425   

Se $S$ é uma esfera e ${\bf F}$ satisfaz as hipóteses do Teorema de Stokes, mostre que $\displaystyle\iint\limits_{S}\mbox{rot}{\bf F} \cdot d{\bf S} = 0$.


3056   

 Faça o esboço do sólido cujo volume é dado pela integral e calcule essa integral.

$\displaystyle \int_0^{\pi/2}\int_0^2\!\!\int_0^{9 - r^2} r dz dr d\theta$


ma211lista8q32bres.png


2897   

Determine o plano tangente à superfície $\frac{x^2}{4} + \frac{y^2}{9} + \frac{z^2}{16} = 1$, com $x > 0$, $y > 0$ e $z > 0$, que forma com os planos coordenados um tetraedro de volume mínimo. (Dica: O volume do tetraedro formado pelos planos coordenados e o plano $ax + by + cz = d$ no primeiro octante é dado por $V = d^3/(6abc)$.)


$6x + 4y + 3z = 12\sqrt{3}.$


2998   

Seja $f$ uma função contínua em $[0,1]$ e seja $R$ a região triangular com vértices $(0,0), (1,0)$ e $(0,1)$. Mostre que

$$\iint\limits_{R} f(x,y) \, dA = \int_0^1 uf(u) \, du.$$


Utilize a mudança de variáveis $u = x + y$ e $v = y.$


2485   

Use a integral tripla para determinar o volume do sólido dado.

  1.  O tetraedro limitado pelos planos coordenados e o plano $2x+y+z=4.$

  2.  O sólido limitado pelo paraboloide $x=y^{2}+z^{2}$ e pelo plano $x=16.$

  3.  O sólido delimitado pelo cilindro $x=y^{2}$ e pelos planos $z=0$ e $x+z=1$.


  1.  $\dfrac{16}{3}.$

  2.  $128\pi.$

  3.  $\dfrac{8}{15}.$


2064   

Dados ${\bf F}(x,y)=xy^{2}\,{\bf i}+x^{2}y\,{\bf j}$, $C: {\bf r}(t)=(t+\sin\frac{1}{2}\pi t, t+\cos \frac{1}{2}\pi t)$, $0\leq t\leq 1.$

  1. Determine uma função $f$ tal que ${\bf F}=\nabla f$.

  2. Use o resultado anterior para calcular $\int_{C}{\bf F}\cdot d{\bf r}$ sobre a curva $C$ dada.


  1. $f(x,y) = \dfrac{x^{2}y^{2}}{2};$

  2. $2.$


2825   

Determine $(x,y)$, com $x^{2}+4y^{2}\leq 1$, que maximiza a soma $2x+y.$


$\displaystyle \left( \frac{4\sqrt{17}}{17}, \frac{\sqrt{17}}{34} \right).$


2056   

Determine se ${\bf F}(x,y)=(\ln y+2xy^{3})\,{\bf i}+(3x^{2}y^{2}+x/y)\,{\bf j}.$ é ou não um campo vetorial conservativo. Se for, determine uma função $f$ tal que ${\bf F}=\nabla f.$


Sim. $f(x,y) = x^{2}y + xy^{-2} + K.$


2838   

Calcule a integral dupla usando coordenadas polares: $\displaystyle\iint\limits_{R}y\,dA$, onde $R$ é a região no primeiro quadrante limitada pelo semi-círculo $x^{2}+y^{2}=2x.$


$\displaystyle \frac{2}{3}.$


2513   

Uma placa fina de metal, localizada no plano $xy$, tem temperatura $T(x,y)$ no ponto $(x,y)$. As curvas de nível de $T$ são chamadas isotérmicas porque todos os pontos em uma isotérmica têm a mesma temperatura. Faça o esboço de algumas isotérmicas se a função temperatura for dada por

$$T(x,y)=\dfrac{100}{1+x^{2}+2y^{2}}.$$


As isotérmicas são dadas pela família de elipses: $x^{2} + 2y^{2} = \frac{100}{C} - 1,$ com $0 < C \leq 100.$

ma211-list2-ex18_sol.png


2927   

Escreva a equação $z^{2}=x^{2}+y^{2}$ em coordenadas esféricas.


$\cos^2 \phi = \sin^2 \phi.$


2765   

Use a derivação implicíta para determinar $\partial z/\partial x$ e $\partial z/\partial y$ na expressão $x-z=\arctan(yz)$.


$\displaystyle \frac{\partial z}{\partial x} = \frac{1 + y^{2}z^{2}}{1 + y + y^{2}z^{2}}$

$\displaystyle \frac{\partial z}{\partial y} = -\frac{z}{1 + y + y^{2}z^{2}}$.


2183   

Nos item abaixo: 

  1. expresse $\mathrm{d} w/\mathrm{d} t$ como uma função de $t$, usando a Regra da Cadeia, expressando $w$ em termos de $t$ e diferenciando em relação a $t$;
  2. calcule $\mathrm{d} w/\mathrm{d} t$ no valor dado de $t$.

$w=x^{2}+y^{2}$,  $x=\cos{t}+\sin{t}$,  $y=\cos{t}-\sin{t}$;  $t=0.$



  1. $\displaystyle \frac{dw}{dt}(t) = 0.$
  2. $\displaystyle \frac{dw}{dt}(0) = 0.$


2589   

A água do mar tem densidade $1025 kg/m^{3}$ e escoa em um campo de velocidade ${\bf v}=y{\bf i}+x{\bf j}$, onde $x$, $y$ e $z$ são medidos em metros e as componentes de ${\bf v}$ em metros por segundo. Encontre a vazão para fora do hemisfério $x^{2}+y^{2}+z^{2}=9$, $z\geq 0.$


$0$ kg/s.


2943   

Calcule utilizando coordenadas esféricas. $\displaystyle\iiint\limits_{E}z\,dV$, onde $E$ está entre as esferas $x^{2}+y^{2}+z^{2}=1$ e $x^{2}+y^{2}+z^{2}=4$, no primeiro octante.



$\dfrac{15\pi}{16}.$


2416   

Determine o volume do sólido descrito abaixo.

  1.  Limitado pelo cilindro $x^{2}+y^{2}=1$ e pelos planos $y=z$, $x=0$ e $z=0$, no primeiro octante.

  2.  Cuja base é a região no plano $xy$ que é limitada pela parábola $y=4-x^{2}$ e pela reta $y=3x$, enquanto o topo do sólido é limitado pelo plano $z=x+4.$

  3.  No primeiro octante limitado pelos planos coordenados, pelo cilindro   $x^{2}+y^{2}=4$ e pelo plano $z+y=3.$


  1.  $\dfrac{1}{3}.$

  2.  $\dfrac{625}{12}.$

  3.  $\dfrac{9\pi - 8}{3}.$


3109   

Encontre a área da região descrita como sendo a parte do cone \(z=\sqrt{x^2+y^2}\) dentro do cilindro \(x^2+y^2=2x\).


1958   

Calcule a integral de linha $\displaystyle\int_{C}{\bf F}\cdot d{\bf r}$, onde ${\bf F}(x,y,z)=(yz,xz,xy+2y)$ e $C$ é o segmento de reta que liga o ponto $(1,0,1)$ ao ponto $(-2,2,2).$


$-6.$


2552   

Explique por que cada função é contínua ou descontínua.

  1. A temperatura externa como função da latitude, da longitude e do tempo.

  2. A altura acima do nível do mar como função da longitude, da latitude e do tempo.

  3. O custo da tarifa do táxi como função da distância percorrida e do tempo gasto.


  1. Contínua.

  2. Descontínua.

  3. Descontínua.


2010   

Um homem pesando $160$ lb carrega uma lata de tinta de $25$ lb por uma escada helicoidal em torno de um silo com raio de $20$ pés. Se o silo tem $90$ pés de altura e o homem dá três voltas completas em torno do silo, quanto trabalho é realizado pelo homem contra a gravidade para subir ao topo?


$16650$ ft-lb.


2977   

Determine o jacobiano da transformação dada por: $x = uv, \quad y = vw, \quad z = uw$.


$2uvw.$


2896   

Utilize coordenadas polares para determinar o volume do sólido dado: delimitado pelo hiperboloide $-x^2-y^2+z^2=1$ e acima do plano $xy.$


$\displaystyle \frac{4\pi}{3}.$


2179   

Seja $D$ a região limitada por um caminho fechado e simples $C$ no plano $xy$. As coordenadas do centroide $(\bar{x},\bar{y})$ de $D$ são

$$\bar{x} = \dfrac{1}{2A}\oint_{C}x^2 \, dy \quad \quad\quad\quad \bar{y} = -\dfrac{1}{2A}\oint_{C}y^2 \, dx,$$

em que $A$ é a área de $D$. Encontre o centroide de um quarto de uma região circular de raio $a$.


$\displaystyle \left(\frac{4a}{3\pi},\frac{4a}{3\pi} \right),$ se a região for a parte do disco $x^{2} + y^{2} = a^{2}$ no primeiro quadrante.


3120   

Usando coordenadas esféricas, calcule a massa da esfera sólida de raio \(a\) com densidade proporcional à distância ao centro (tomando \(k\) como a constante de proporcionalidade).


 \(k\pi a^4\)


2094   

Considere o campo vetorial

$${\bf F}(x,y)=\bigg(e^{x}\ln(y)-\frac{e^{y}}{x}\bigg)\,{\bf i}+\bigg(\frac{e^{x}}{y}-e^{y}\ln(x)\bigg)\,{\bf j}.$$

  1. O campo ${\bf F}$ é conservativo? Justifique sua resposta.

  2. Calcule $\int_{C}{\bf F}\cdot d{\bf r}$, onde $C$ é qualquer caminho ligando o ponto $(1,1)$ ao ponto $(3,3).$


  1. Sim.

  2. $0.$


2630   

Utilizando o Teorema de Stokes, transforme a integral $\displaystyle\iint_{ S}\mbox{rot}{\bf F}\cdot{\bf n}dS$ numa integral de linha e calcule.

  • ${\bf F}(x,y,z) = -y^2{\bf i} + x^2{\bf j} + z^2{\bf k}$, $S$ a superfície $x^2 + \dfrac{y^2}{4} + z^2 = 2$, $z \geq 1$, sendo ${\bf n}$ a normal que aponta para cima.


 $0$.


2040   

Utilize a Regra da Cadeia para determinar as derivadas parciais indicadas.
$z=x^{2}+xy^{3}$, $x=uv^{2}+w^{3}$, $y=u+ue^{w}$;
$\dfrac{\partial z}{\partial u}$, $\dfrac{\partial z}{\partial v}$, $\dfrac{\partial z}{\partial w}$ quando $u=2$,  $v=1$, $w=0$.


$\dfrac{\partial z}{\partial u} = 85$, $\dfrac{\partial z}{\partial v} = 178$, $\dfrac{\partial z}{\partial w} = 54.$


2663   

Determine as derivadas parciais de $z=(x^{2}+y^{2})\ln(x^{2}+y^{2})$.


$\displaystyle \frac{\partial z}{\partial x} = 2x(1 + \ln(x^{2} + y^ {2}))\;\;\;\;\;\;\text{e}\;\;\;\;\; \frac{\partial z}{\partial y} = 2y(1 + \ln(x^{2} + y^ {2})).$


2295   

Determine uma representação paramétrica para a superfície descrita a seguir. A porção no primeiro octante do cone $z=\sqrt{x^{2}+y^{2}}/2$ entre os planos $z=0$ e $z=3.$


$x = r \cos(\theta),$ $y = r \sin(\theta),$ $z = \dfrac{r}{2},$ onde $0 \leq r \leq 6$ e $0\leq \theta \leq \dfrac{\pi}{2}.$


2702   

Seja $f(x,y)=\dfrac{x^{2}y^{2}}{x^{2}+y^{2}}.$

  1. Calcule as derivadas parciais $\dfrac{\partial f}{\partial x}(x,y)$ e $\dfrac{\partial f}{\partial y}(x,y)$, num ponto  $(x,y)\neq\;(0,0).$

  2. Calcule o limite, se existir.

    $$\lim_{(x,y)\rightarrow (0,0)}\frac{\partial f}{\partial x}(x,y)$$


  1. $\displaystyle \frac{\partial f}{\partial x} = \frac{2xy^{4}}{(x^{2} + y^{2})^{2}} \;\;\;\text{e}\;\;\; \frac{\partial f}{\partial y} = \frac{2x^{4}y}{(x^{2} + y^{2})^{2}}$.

  2. $\displaystyle \lim_{(x,y)\rightarrow (0,0)}\frac{\partial f}{\partial x}(x,y) = 0$.


2174   

Calcule a área da região $R$ delimitada pela cardioide $\mathbf{r}(t) = (x(t),y(t))$, em que $x(t) = 2\cos{t}-\cos{2t}$ e $y(t) = 2\sin{t}-\sin{2t}$, $t \in [0,2\pi]$.


$6\pi.$


2939   

Determine a massa e o centro de massa da lâmina que ocupa a região $D$ e tem função densidade $\rho$ quando: $D$ é a região triangular delimitada pelas retas $x = 0, \ y = x$ e   $2x + y = 6; \quad \rho(x,y) = x^2$.


Massa: $4;$ centro de massa: $\displaystyle \left(\frac{6}{5},\frac{12}{5} \right).$


2733   

Determine as equações do plano tangente e da reta normal ao gráfico da função dada, no ponto dado. $f(x,y) = \arctan{(x - 2y)}$ em $\left(2, \dfrac{1}{2},f\left(2,\dfrac{1}{2}\right)\right)$.


Plano tangente: $4z = 2x - 4y + (\pi - 2)$

Reta normal: $(x,y,z) = \left(2,\frac{1}{2},\frac{\pi}{4} \right) + \lambda \left(\frac{1}{2},-1,-1 \right)$.


2961   

Usando coordenadas esféricas, determine o volume do sólido que está dentro da esfera $x^{2}+y^{2}+z^{2}=4$, acima do plano $xy$ e abaixo do cone $z=\sqrt{x^{2}+y^{2}}.$



$\dfrac{8\sqrt{2}\pi}{3}.$


2543   

Encontre o fluxo exterior do campo ${\bf F}(x,y,z)=z^{2}{\bf i}+x{\bf j}-3z{\bf k}$ através da superfície cortada do cilindro parabólico $z=4-y^{2}$ pelos planos $x=0$, $x=1$ e $z=0.$


$-32.$


2224   

Determine a derivada direcional de $f$ no ponto dado e na direção indicada pelo ângulo $\theta$.

$f(x,y) = ye^{-x},  (0,4),$ $\theta = 2\pi/3$.


$2 + \frac{\sqrt{3}}{2}.$


2973   

Determine o jacobiano da transformação dada por: $x = 5u - v, \quad y = u + 3v$.


$16.$


2560   

Determine o limite, se existir, ou mostre que o limite não existe.

$\displaystyle \lim_{(x,y) \to (0,0)} \dfrac{x^2 + y^2}{\sqrt{x^2 + y^2 + 1} - 1}$.


$2.$


2368   

Em que direção e sentido a função dada cresce mais rapidamente no ponto dado? E em que direção e sentido decresce mais rapidamente?

$f(x,y) = \sqrt{4 - x^2 - 2y^2}$ em $\left(1,\dfrac{1}{2}\right)$.


 Cresce: $(-1,-1)$; descresce: $(1,1).$


2097   

Mostre que a integral de linha $\int_{C}2x\,\sin y\,dx+(x^{2}\,\cos y-3y^{2})\,dy$, onde $C$ é qualquer caminho entre $(-1,0)$ a $(5,1)$, é independente do caminho e calcule a integral.


$\mathbf{F} (x,y) = 2x \sin(y) \mathbf{i} + x^{2} \cos(y) - 3y^{2} \bf j$ é um campo conservativo com uma função potencial $f(x,y) = x^{2} \sin(y) - y^{3};$ o valor da integral é $25 \sin(1) - 1.$


2649   

Verifique que a função $u=1/\sqrt{x^{2}+y^{2}+z^{2}}$ é uma solução da equação de Laplace tridimensional $u_{xx}+u_{yy}+u_{zz}=0.$


$\displaystyle u_{xx} = \frac{2x^{2} - y^{2} - z^{2}}{(x^{2} + y^{2} + z^{2})^{5/2}},\;\;\; u_{yy} = \frac{2y^{2} - x^{2} - z^{2}}{(x^{2} + y^{2} + z^{2})^{5/2}}\;\;\;\text{e}\;\;\;u_{zz} = \frac{2z^{2} - x^{2} - y^{2}}{(x^{2} + y^{2} + z^{2})^{5/2}}$.


3139   

Prove a seguinte identidade \[ \iint\limits_\sigma\mathrm{rot\,}\mathbf{F}\cdot\mathbf{n}\,dS = 0, \] supondo que \(\mathbf{F}\) e \(\sigma\) satisfaçam as hipóteses do Teorema da Divergência.


2782   

Calcule a integral dupla usando coordenadas polares: $\displaystyle\iint\limits_{R}(x^{2}+y^{2})^{3/2}\,dA$, onde $R$ é limitado pelo círculo $x^{2}+y^{2}=4.$


$\displaystyle \frac{64\pi}{5}.$


2419   

Encontre o volume do sólido no primeiro octante limitado pelo cilindro $z=16-x^{2}$ e pelo plano $y=5.$


 $\dfrac{640}{3}.$


2672   

Determine $\dfrac{ \partial f}{\partial x}$ e $\dfrac{\partial f}{\partial y}$, sendo $f(x,y)= \begin{cases}\dfrac{x+y^{4}}{x^{2}+y^{2}}, & \quad \text{se } (x,y)\neq (0,0),\\0, & \quad \text{se } (x,y)=(0,0).\\\end{cases}$


$\begin{aligned}[t]\frac{\partial f}{\partial x} &= \begin{cases}\dfrac{y^{2} - x^{2} - 2xy^{4}}{(x^{2}+y^{2})^{2}}, & \quad \text{se } (x,y)\neq (0,0),\\\text{não existe} & \quad \text{se } (x,y)=(0,0)\\\end{cases} \;\;\;\; \text{e}\\\frac{\partial f}{\partial y} &= \begin{cases}\dfrac{4x^{2}y^{3} + 2y^{5} - 2xy}{x^{2}+y^{2}}, & \quad \text{se } (x,y)\neq (0,0),\\0, & \quad \text{se } (x,y)=(0,0).\\\end{cases}\end{aligned}$


2382   

Calcule a integral iterada.

  1. $\displaystyle\int_{0}^{\pi/2} \int_{0}^{\pi/2}\sin{x}\cos{y} \, dy dx$

  2. $\displaystyle\int_{0}^{2}\!\!\int_{0}^{1}(2x+y)^{8}\,dx dy$


  1. $1.$

  2. $\dfrac{4^{10} - 2^{11}}{180}.$



2394   

Determine a equação da reta tangente à curva de nível dada, no ponto dado.

$e^{2x - y} + 2x + 2y = 4$, em $\left(\dfrac{1}{2},1\right)$.


 $y = -4x + 3.$


2949   

Calcule utilizando coordenadas esféricas. $\displaystyle\iiint\limits_{B}\sqrt{x^{2}+y^{2}+z^{2}}\,dxdydz$, onde $B$ é a interseção da semi-esfera  $x^{2}+y^{2}+z^{2}\leq 4$, $z\geq 0$, com o cilindro $x^{2}+y^{2}\leq 1.$


$\displaystyle \dfrac{\pi}{4}\left( 32- 14\sqrt{3} + \ln(2 + \sqrt{3})\right).$


2214   

Determine o rotacional e o divergente do campo vetorial $\mathbf{F}(x,y,z) = xyz\mathbf{i} - x^2y\mathbf{k}$.


$\text{rot } \mathbf{F} = -x^2 \mathbf{i} + 3xy \mathbf{j} -xz \mathbf{k}.$ $\text{div } \mathbf{F} = yz.$


1938   

Calcule a integral de linha, onde $C$ é a curva dada.

$\displaystyle\int_{C}xe^{yz}\,ds$,   $C$ é o segmento de reta de $(0,0,0)$ a $(1,2,3).$


$\dfrac{\sqrt{14}}{12}\left(e^{6} - 1 \right).$


2428   

Calcule a integral tripla.

  1.  $\displaystyle\iiint\limits_{  E}z\,dV$, onde $E$ é limitado pelo cilindro $y^{2}+z^{2}=9$ e pelos planos $x=0$, $y=3x$ e $z=0$ no primeiro octante.

  2.  $\displaystyle\iiint\limits_{  E}xyz\;dx dy dz$, onde $E$ é o paralelepípedo $0\leq x\leq 2$, $0\leq y\leq 1$, e $1\leq z\leq 2.$


  1.  $\dfrac{27}{8}.$

  2.  $\dfrac{3}{2}.$


1987   

Determine as equações paramétricas para a reta tangente $\grave{a}$ curva dada pelas equações paramétricas 
$x=e^{-t}\; \cos{t}$, $y=e^{-t}\; \sin{t}$, $z=e^{-t}$ no ponto $(1,0,1)$.



2407   

Inverta a ordem de integração.

  1.  $\displaystyle\int_{-1}^{1}\bigg[\int_{x^{2}}^{\sqrt{2-x^{2}}}f(x,y)\,dy\bigg]dx$

  2.  $\displaystyle\int_{0}^{1}\bigg[\int_{y-1}^{2-2y}f(x,y)\,dx\bigg]dy$

  3.  $\displaystyle\int_{0}^{1}\bigg[\int_{x^{2}}^{1}f(x,y)\,dy\bigg]dx$


  1.  $\displaystyle\int_{0}^{1}\bigg[\int_{-\sqrt{y}}^{\sqrt{y}}f(x,y)\,dx\bigg]dy +  \displaystyle\int_{1}^{\sqrt{2}}\bigg[\int_{-\sqrt{2 - y^{2}}}^{\sqrt{2-y^{2}}}f(x,y)\,dx\bigg]dy$

  2.  $\displaystyle\int_{-1}^{0}\bigg[\int_{0}^{x + 1}f(x,y)\,dy \bigg] dx + \int_{0}^{2}\bigg[\int_{0}^{\frac{2-x}{2}}f(x,y)\,dy \bigg] dx$

  3.  $\displaystyle\int_{0}^{1}\bigg[\int_{0}^{\sqrt{y}}f(x,y)\,dx\bigg]dy$


1966   

Calcule a integral de linha $\int_{C}{\bf F}\cdot d{\bf r}$, onde ${\bf F}(x,y)=e^{x-1}\,{\bf i}+xy\,{\bf j}$ e $C$ é dada por ${\bf r}(t)=t^{2}\,{\bf i}+t^{3}\,{\bf j},   0\leq t\leq 1.$


$\displaystyle \frac{11}{8} - \frac{1}{e}.$


2737   

Determine o plano que é paralelo ao plano $z = 2x + y$ e tangente ao gráfico de $f(x,y) = x^2 + y^2$.


$z = 2x + y - \frac{5}{4}$.


2895   

Utilize coordenadas polares para determinar o volume do sólido dado: abaixo do cone $z=\sqrt{x^{2}+y^{2}}$ e acima do disco $x^{2}+y^{2}\leq 4.$


$\displaystyle \frac{16\pi}{3}.$


2872   

Utilize os multiplicadores de Lagrange para determinar os valores máximo e mínimo da função sujeita à(s) restrição(ões) dada(s).

$f(x,y) = x^2y; \quad x^2 + 2y^2 = 6.$


Valor máximo: $4;$ valor mínimo: $-4.$


3022   

Esboce a região de integração e calcule a integral $\displaystyle\int_{0}^{3}\!\!\int_{-2}^{0}(x^{2}y-2xy)\,dy dx$.



$0.$

ma211-list6-ex25_sol_c.png


2399   

Existem pontos no hiperboloide $x^2 - y^2 - z^2 = 1$ nos quais o plano tangente é paralelo ao plano $z = x + y$?


Não.


2714   

Determine uma equação do plano tangente à superfície no ponto especificado.

$z = \sqrt{xy}, \quad (1,1,1)$.


$x + y - 2z = 0$.


2915   

Um sólido está acima do cone $z=\sqrt{x^{2}+y^{2}}$ e abaixo da esfera $x^{2}+y^{2}+z^{2}=z.$ Escreva uma descrição do sólido em termos de desigualdades envolvendo coordenadas esféricas.



A mudança de coordenadas retangulares para coordenadas cartesianas é dada por

$$\begin{cases}x = \rho \cos{\theta} \sin{\phi} \\y = \rho \sin{\theta} \sin{\phi}\\z = \rho \cos{\phi},\end{cases}$$

em que $\rho \geq 0$, $\theta \in [0,2\pi]$ e $\phi \in [0,\pi]$. Observe que $\sin{\phi} \geq 0$ quando $\phi \in [0,\pi]$. Logo, a equação do cone em coordenadas esféricas pode ser escrita como $\rho \cos{\phi} = \sqrt{\rho^2 \sin^2{\phi}} = \rho\sin{\phi}$. A origem $(0,0,0)$ pertence ao cone e é dada por $\rho = 0$. Nos demais pontos, $\rho \neq 0$, donde $\phi = \pi/4$.

A equação da esfera em coordenadas esféricas pode ser escrita como $\rho^2=\rho\cos{\phi}$. A origem $(0,0,0)$ pertence à esfera e é dada por $\rho=0$. Nos demais pontos, $\rho \neq 0$, donde $\rho = \cos{\phi}$.

Portanto, o sólido pode ser descrito em coordenadas esféricas por

$$E = \left\{(\rho, \theta, \phi): 0 \leq \rho \leq \cos{\phi}, 0 \leq \theta \leq 2\pi \mbox{ e } 0 \leq \phi \leq \frac{\pi}{4}\right\}.$$

ma211-list9-ex1_sol.png


2729   

Determine as equações do plano tangente e da reta normal ao gráfico da função dada, no ponto dado. $f(x,y) = 2x^2y$ em $(1,1,f(1,1))$.


Plano tangente: $z = 4x + 2y - 4$

Reta normal: $(x,y,z) = \left(1,1,2 \right) + \lambda \left(4,2,-1 \right)$.


2600   

Calcule as seguintes integrais triplas.

  1.  $\displaystyle\iiint\limits_{  E}  x^2 \, dV$, em que $E$ é o sólido que está dentro do cilindro $x^2 + y^2 = 1$, acima do plano $z = 0$ e abaixo do cone $z^2 = 4x^2 + 4y^2$.

  2.  $\displaystyle\iiint\limits_{  E}   xyz \, dV,$ em que $E$ é o sólido limitado pelos paraboloides $z = x^2 + y^2$, $z = 8 - x^2 - y^2$.

  3.  $\displaystyle\int_{-2}^2\int_{-\sqrt{4 - y^2}}^{\sqrt{4 - y^2}}\int_{\sqrt{x^2 + y^2}}^2 xz \, dz dx dy$


  1.  $\dfrac{2\pi}{5}$.

  2.  $0.$

  3.  $0.$


2675   

Calcule as derivadas parciais de $w = \dfrac{xyz}{x + y + z}$.


$\displaystyle \frac{\partial w}{\partial x} = \frac{yz(y+z)}{(x+y+z)^{2}},\;\;\;\;  \frac{\partial w}{\partial y} = \frac{xz(x+z)}{(x+y+z)^{2}}\;\;\;\;\;\text{e}\;\;\;\;\;\frac{\partial w}{\partial z} = \frac{xy(x+y)}{(x+y+z)^{2}}.$


2113   

Se $u=f(x,y)$, onde $x=e^{s}\cos{t}$ e $y=e^{s}\sin{t}$, mostre que
$$\bigg(\dfrac{\partial u}{\partial x}\bigg)^{2}+ \bigg(\dfrac{\partial u}{\partial y}\bigg)^{2}=
e^{-2s}\bigg[ \bigg(\dfrac{\partial u}{\partial s}\bigg)^{2}+\bigg(\dfrac{\partial u}{\partial t}\bigg)^{2}\bigg].$$



Note que $\displaystyle \frac{\partial u}{\partial s} = e^{s} \cos(t) \frac{\partial u}{\partial x}  + e^{s} \sin(t) \frac{\partial u}{\partial y} $e
$\displaystyle \frac{\partial u}{\partial t} = -e^{s} \sin(t) \frac{\partial u}{\partial x}  + e^{s} \cos(t) \frac{\partial u}{\partial y} .$


1996   

Seja ${\bf F}:\mathbb{R}^{2} \to \mathbb{R}^{2}$ um campo vetorial contínuo tal que, para todo $(x,y)$, ${\bf F}(x,y)$ é paralelo ao vetor $x\,{\bf i}+y\,{\bf j}$. Calcule $\int_{C}{\bf F}\cdot d{\bf r}$, onde ${\bf r}:[a,b]\to \mathbb{R}^{2}$ é uma curva de classe $C^{1}$, cuja imagem está contida na circunferência de centro na origem e raio $r>0$. Interprete geometricamente.


$0.$


2134   

Mostre que cada a equação a seguir define implicitamente pelo  menos uma função diferenciável $y=y(x).$ 
$y^{4}+x^{2}y^{2}+x^{4}=3$
 


$\displaystyle \frac{d y}{d x} = - \frac{2xy^{2} + 4x^{3}}{4y^{3} + 2x^{2}y}.$


1948   

Calcule o limite $\displaystyle \lim_{t \rightarrow 0}\left(\dfrac{e^t - 1}{t}, \dfrac{\sqrt{1+t}-1}{t}, \dfrac{3}{t+1}\right)$.




Consideremos ${\bf r}(t)=\bigg(\frac{e^{t}-1}{t},\frac{\sqrt{1+t}-1}{t},\frac{3}{t+1}\bigg).$

Temos que o limite de ${\bf r}$ é o vetor cujas componentes são os limites das funções componentes de ${\bf r}$, se esses limites existirem.

Então,

$\lim\limits_{t \to 0}{\bf r}(t)=\lim\limits_{t \to 0}\left(\frac{e^{t}-1}{t},\frac{\sqrt{1+t}-1}{t},\frac{3}{t+1}\right)=\left(\lim\limits_{ t\to 0}\frac{e^{t}-1}{t},\lim\limits_{t \to 0}\frac{\sqrt{1+t}-1}{t},\lim\limits_{t \to 0}\frac{3}{t+1}\right)$

Assim,

$\bullet \lim\limits_{t\rightarrow 0}\dfrac{e^{t}-1}{t}=1$.

$\bullet \lim\limits_{t\to 0}\frac{\sqrt{1+t}-1}{t}=\lim\limits_{t\to 0}\frac{(\sqrt{1+t}-1)(\sqrt{1+t}+1)}{t(\sqrt{1+t}+1)}=\lim\limits_{t\to 0}\frac{1+t-1}{t(\sqrt{1+t}+1)}=\lim\limits_{t\to 0}\frac{t}{t(\sqrt{1+t}+1)}=\lim\limits_{t\to 0}\frac{1}{\sqrt{1+t}+1}=\frac{1}{2}$.

$\bullet \lim\limits_{t\to 0}\dfrac{3}{t+1}=3$.

Portanto,

$\lim\limits_{t\to 0}\bigg(\frac{e^{t}-1}{t},\frac{\sqrt{1+t}-1}{t},\frac{3}{t+1}\bigg)=\bigg(1,\frac{1}{2},3\bigg).$


2333   

Determine a taxa de variação máxima de $f$ no ponto dado e a direção em que isso ocorre.

$f(x,y,z) = \sqrt{x^2 + y^2 + z^2},  (3,6,-2).$


$1.$


3036   

Uma região $R$ é mostrada na figura. Decida se você deve usar coordenadas polares ou retangulares e escreva $\iint \limits_{R}f(x,y)\,dA$ como uma integral iterada, onde $f$ é uma função qualquer contínua em $R.$


ma211-list7-ex5_c.png


$\displaystyle \int_{-1}^{1} \int_{0}^{\frac{(x + 1)}{2}} f(x,y)  dy dx .$


2910   

Calcule a integral iterada $\int_{-3}^{3} \int_{0}^{\sqrt{9-x^2}}\sin(x^{2}+y^{2})\,dy dx$, convertendo-a antes para coordenadas polares.


$\displaystyle \frac{\pi}{2}(1 - \cos(9)).$


3112   

Encontre a área da parte da superfície \(z=\sqrt{4-x^2}\) que fica acima do retângulo \(R\) do plano \(xy\) cujas coordenadas satisfazem \(0\leq x\leq 1\) e \(0\leq y\leq 4\).



A superfície é uma parte do cilindro \(x^2+z^2=4\) localizada no primeiro octante. Neste caso, como \(z=f(x,y)\), podemos tomar \(x=u\) e \(y=v\) como parâmetros. Assim, teremos que \(\displaystyle \mathbf{r}=u\mathbf{i}+v\mathbf{j}+f(u,v)\mathbf{k} \) e \[ \|\dfrac{\partial\mathbf{r}}{\partial u}\times \dfrac{\partial \mathbf{r}}{\partial v}\| = \sqrt{\left(\dfrac{\partial z}{\partial x}\right)^2+\left(\dfrac{\partial z}{\partial y}\right)^2+1}.\] Segue para a área que \begin{align*} S & = \iint\limits_R\sqrt{\left(\dfrac{\partial z}{\partial x}\right)^2+\left(\dfrac{\partial z}{\partial y}\right)^2+1}\,dA \\  & = \iint\limits_R\sqrt{\left(-\dfrac{x}{\sqrt{4-x^2}}\right)^2+ 0 + 1}\,dA = \int_0^4\int_0^1\dfrac{2}{\sqrt{4-x^2}}\,dxdy \\   & = 2\int_0^4\left[\arcsin\left(\dfrac{1}{2}x\right)\right]_{x=0}^1\,dy = 2\int_0^4\dfrac{\pi}{6}\,dy = \dfrac{4}{3}\pi. \end{align*}


2754   

A função $f(x,y) = \begin{cases}\dfrac{x^4}{x^2 + y^2}, & \quad \text{se } (x,y) \neq (0,0),\\0, & \quad \text{se } (x,y) = (0,0)\\\end{cases}$ é diferenciável em $(0,0)$? Justifique.


Sim.



2139   

Use o Teorema do Divergente para calcular $\displaystyle\iint \limits_{S}(2x+2y+z^{2})\,dS$ onde $S$ é a esfera $x^{2}+y^{2}+z^{2}=1.$



A superfície $S$ em questão é a esfera unitária, que é a fronteira da bola unitária $B$ dada por $x^2+y^2+z^2 \leq 1$ e tem vetor normal num ponto $(x,y,z)$ igual a $(x,y,z)$ (o qual aponta para ``fora").

ma211-list15-ex3_sol.png

Observe que podemos transformar o integrando $2x+2y+z^{2}$ em $(2,2,z) \cdot (x,y,z)$ e essa escrita é interessante, já que o segundo vetor é exatamente o vetor normal a $S$. Agora estamos em condições de aplicar o Teorema do Divergente quando tomamos o campo ${\bf F}(x,y,z) = (2,2,z)$. Assim,
\begin{array}{rcl}\displaystyle\iint\limits_{S}(2x+2y+z^{2})\,dS & = & \iint\limits_{ S}(2,2,z) \cdot (x,y,z)\,dS \\& = & \int\int\int \limits_{ S}{\bf F} \cdot {\bf n}\,dS \\& = & \iiint\limits_{ B}\text{div } F\,dV \\& = & \iiint\limits_{ B}(0+0+1)\,dV \\& = & V(B) = \frac{4\pi}{3}.\end{array}


2575   

Calcule $\displaystyle \lim_{(x,y) \to (0,0)} \dfrac{xy^2}{x^2 - y^2}$, caso exista.


Não existe.


2176   

Calcule a área da região limitada pela astroide $x=\cos^3{t}$, $y = \sin^3{t}$,  $0 \leq t \leq 2\pi$.


$\dfrac{3\pi}{8}.$


1972   

Determine a equação da reta tangente à trajetória da função \newline${\bf r}(t)=\bigg(\dfrac{1}{t},
\dfrac{1}{t},t^{2}\bigg)$, no ponto ${\bf r}(2)$.



$${\bf r}(2)=\bigg(\frac{1}{2},\frac{1}{2},4\bigg)\,\,\,\,\,\, e \,\,\,\,\,\, \frac{d{\bf r}}{dt}=\bigg(-\frac{1}{t^{2}},-\frac{1}{t^{2}},2t\bigg).$$
Assim, 
$$\frac{d{\bf r}}{dt}(2)=\bigg(-\frac{1}{4},-\frac{1}{4},4\bigg).$$
Portanto, a equação da reta tangente em ${\bf r}(2)$ é:
$${\bf x}={\bf r}(2)+\lambda \frac{d{\bf r}}{dt}(2),\,\, \lambda \in \mathbb{R},$$
ou seja, 
$$(x,y,z)=\bigg(\frac{1}{2},\frac{1}{2},4\bigg)+\lambda \bigg(-\frac{1}{4},-\frac{1}{4},4\bigg),\,\, \lambda \in \mathbb{R}.$$



2268   

Determine a área da superfície dada pela parte de baixo da esfera $x^{2}+y^{2}+z^{2}=2$ cortada pelo cone $z=\sqrt{x^{2}+y^{2}}.$



Sejam

$$\left \{\begin{array}{cc}x=r\,\sin \phi\,\cos \theta\\y=r\,\sin \phi\,\sin \theta\\z=r\,\cos \phi\\\end{array}\right. \Rightarrow r=\sqrt{x^{2}+y^{2}+z^{2}}=\sqrt{2},\, \mbox{na\,esfera}.$$

Temos que

$$x^{2}+y^{2}+z^{2}=2    \mbox{e}\,\,\,\, z=\sqrt{x^{2}+y^{2}}\Rightarrow z^{2}+z^{2}=2\Rightarrow z^{2}=1\Rightarrow z=1\,(\mbox{pois}\, z\geq 0).$$

Logo, $\phi=\frac{\pi}{4}.$ Para a parte inferior da esfera cortado pelo cone, temos que $\phi=\pi.$

Então,

$$r(\phi,\theta)=(\sqrt{2}\,\sin \phi,\,\cos\theta)\,{\bf i}+(\sqrt{2}\,\sin \phi\,\sin \theta)\,{\bf j}+(\sqrt{2}\,\cos \phi)\,{\bf k},$$

$$\frac{\pi}{4}\leq \phi\leq \pi\,\,\,\, \mbox{e}\,\,\,\, 0\leq \theta \leq 2\pi.$$

Isso implica que

$$r_{\phi}(\phi,\theta)=(\sqrt{2}\,\cos \phi,\,\cos\theta)\,{\bf i}+(\sqrt{2}\,\cos \phi\,\sin \theta)\,{\bf j}-(\sqrt{2}\,\sin \phi)\,{\bf k}$$

e

$$r_{\theta}(\phi,\theta)=(-\sqrt{2}\,\sin \phi,\,\sin\theta)\,{\bf i}+(\sqrt{2}\,\sin \phi\,\cos \theta)\,{\bf j}+0\,{\bf k}$$

Logo,

$$\begin{array}{rcl}r_{\phi}\times r_{\theta}&=&\left|\begin{array}{ccc}{\bf i}&{\bf j}&{\bf k}\\\sqrt{2}\,\cos \phi\,\cos \theta & \sqrt{2}\,\cos \phi\,\sin \theta& -\sqrt{2}\,\sin \phi\\-\sqrt{2}\,\sin \phi\,\sin \theta & \sqrt{2}\,\sin \phi\,\cos \theta & 0\end{array}\right|\\&=&(2\,\sin^{2}\phi\,\cos \theta)\,{\bf i}+(2\sin^{2}\phi\,\sin \theta)\,{\bf j}+(2\,\sin \phi \,\cos \phi)\,{\bf k}.\\\end{array}$$

Isso resulta que

$$\begin{array}{rcl}|r_{\phi}\times r_{\theta}|&=&\sqrt{4\sin^{2}\phi\,\cos^{2}\theta+4\,\sin^{4}\,\sin^{2}\theta+4\sin^{2}\phi\,\cos^{2}\phi}\\&=&\sqrt{4\,\sin^{2}\phi}=2|\sin\phi|=2\sin \phi   \bigg(\mbox{pois},\, \frac{\pi}{4}\leq \phi \leq \pi\bigg).\end{array}$$

Assim,

$$A=\iint\limits_{ D}|r_{\phi}\times r_{\theta}|\,dA=\int_{\frac{\pi}{4}}^{\pi}\int_{0}^{2\pi}2\sin \phi\, d\theta d \phi=2\int_{\frac{\pi}{4}}^{\pi}\sin \phi\,d\phi \cdot \int_{0}^{2\pi}d\theta$$

$$=2\cdot (-\cos \phi)\bigg|_{\frac{\pi}{4}}^{\pi}\cdot \theta\bigg|_{0}^{2\pi}=2\cdot \bigg(1-\frac{\sqrt{2}}{2}\bigg)\cdot 2\pi=4\pi\bigg(1-\frac{\sqrt{2}}{2}\bigg)=\pi(4-2\sqrt{2})$$


2141   

Verifique que o Teorema do Divergente é verdadeiro para o campo vetorial ${\bf F}$ na região $E.$

${\bf F}(x,y,z)=3x\,{\bf i}+xy\,{\bf j}+2xz\,{\bf k}$, $E$ é o cubo limitado pelos planos $x=0$, $x=1$, $y=0$, $y=1$,  $z=0$ e $z=1.$


$\displaystyle\iint_{S} {\bf F} \cdot d{\bf S} = \iiint_{E} \mbox{div} {\bf F} dV = \dfrac{9}{2}.$



3011   

A fronteira de uma lâmina consiste nos semicírculos $y = \sqrt{1 - x^2}$ e   $y = \sqrt{4 - x^2}$, juntamente com as partes do eixo $x$ que os une. Encontre o centro de massa da lâmina se a densidade em qualquer ponto é proporcional à sua distância da origem.


$\displaystyle \left(0, \frac{45}{14\pi} \right).$


2290   

Determine uma representação paramétrica para a superfície descrita a seguir. A parte da esfera $x^{2}+y^{2}+z^{2}=4$ que está acima do cone $z=\sqrt{x^{2}+y^{2}}.$



$x = 2\sin(\phi)\cos(\theta),$ $y = 2\sin(\phi)\sin(\theta),$ $z = 2\cos(\phi),$ onde $0\leq \phi \leq \frac{\pi}{4}$ e $0 \leq \theta \leq 2\pi.$


2859   

Considere a função

$$f(x,y)=-\frac{y^{2}}{2}+3x^{2}-2x^{3}.$$

  1. Determine e classifique os pontos críticos de $f.$

  2. Mostre que a curva de nível $f(x,y)=0$ com $x\geq 0$ é uma curva fechada, isto é, é a fronteira de uma região $R$ limitada do plano $xy.$ Calcule o valor máximo de $f$ nessa região $R$.


  1. Pontos críticos: $(0,0)$ e $(1,0).$ Ponto de máximo: $(1,0);$ ponto de sela: $(0,0).$

  2. $\max \{ f(s); s\in R \} =1$.


2556   

Determine o limite, se existir, ou mostre que o limite não existe.

$\displaystyle \lim_{(x,y) \to (0,0)} \dfrac{xy \ \mbox{cos} \ y}{3x^2 + y^2}$.


Não existe.



3094   

Cada integral iterada abaixo representa o volume de um sólido. Faça um esboço do sólido. (Não é necessário calcular o volume.)

  1.  \(\displaystyle \int_0^5\int_1^2 4\, dxdy\)

  2.  \(\displaystyle \int_0^3\int_0^4\sqrt{25-x^2-y^2}\,dydx\)


2784   

Calcule a integral dupla usando coordenadas polares: $\displaystyle\iint\limits_{R}\sqrt{x^{2}+y^{2}}\,dA$, onde $R$ é limitado pelo triângulo de vértices $(0,0)$, $(3,0)$ e $(3,3).$



$\displaystyle \frac{9}{2} (\sqrt{2} + \ln(\sqrt{2} + 1)).$


2058   

Determine se ${\bf F}(x,y,z)=\dfrac{x}{(x^{2}+y^{2}+z^{2})^{2}}\,{\bf i}+\dfrac{y}{(x^{2}+y^{2}+z^{2})^{2}}\,{\bf j}+\dfrac{z}{(x^{2}+y^{2}+z^{2})^{2}}\,{\bf k}$ é ou não um campo vetorial conservativo. Se for, determine uma função $f$ tal que ${\bf F}=\nabla f.$


Sim. $f(x,y,z) = -\dfrac{1}{2(x^2 + y^{2} +z^{2})} + K.$


2035   

Utilize a Regra da Cadeia para determinar $\mathrm{\partial}z/\mathrm{\partial} s$ e $\mathrm{\partial}z/ \mathrm{\partial}t.$

$z=\tan(u/v)$, $u=2s+3t$, $v=3s-2t$.


$\displaystyle \frac{\partial z}{\partial s} =  \frac{2u - 3v}{v^{2}} \sec^{2}\left(\frac{u}{v} \right)$ e $\displaystyle \frac{\partial z}{\partial t} = \frac{2u + 3v}{v^{2}} \sec^{2}\left(\frac{u}{v} \right))$.


2209   

Calcule a integral dupla.

  1. $\displaystyle\iint\limits_{D}(2x-y)\,dA, \quad D$ limitada pelo círculo de centro na origem e raio 2.

  2. $\displaystyle\iint\limits_{D}\dfrac{x}{y}\,dA, \quad D$ região no primeiro quadrante limitada pelas retas $y=x$, $y=2x$, $x=1$ e $x=2.$

  3. $\displaystyle\iint\limits_{D}\dfrac{1}{xy}\,dA, \quad D$ o quadrado $1\leq x\leq 2$, \;$1\leq y\leq 2.$

  4. $\displaystyle\iint\limits_{D}(x-\sqrt{y})\,dA, \quad D$ região triangular cortado do primeiro quadrante do plano $xy$ pela reta $x+y=1.$


  1.  $0.$

  2.  $\dfrac{3\ln(2)}{2}.$

  3.  $(\ln(2))^{2}.$

  4. $-\dfrac{1}{10}.$


2924   

Marque o ponto cujas coordenadas esféricas é $(1,0,0)$ e encontre as coordenadas retangulares do ponto.


$(0,0,1).$

ma211-list9-ex5_sol.png


2571   

Calcule $\displaystyle \lim_{(x,y) \to (0,0)} \dfrac{xy}{x^2 + y^2}$, caso exista.


Não existe.


2051   

Determine se ${\bf F}(x,y)=y\,{\bf i}+x\,{\bf j}$ é ou não um campo vetorial conservativo. Se for, determine uma função $f$ tal que ${\bf F}=\nabla f.$


Sim. $f(x,y) = xy + K.$


2455   

Encontre a constante $a$ tal que $$\int_{0}^{1}\int_{0}^{4-a-x^{2}}\int_{a}^{4-x^{2}-y}\;dz dy dx=\frac{4}{15}.$$


$\dfrac{13}{3}$ ou $3.$


2082   

Se $z=f(x,y)$, onde $x=r\cos{\theta}$ e $y=r\sin{\theta}$,

  1. Determine $\dfrac{\partial z}{\partial r}$ e $\dfrac{\partial z}{\partial \theta}.$
  2. Mostre que $\bigg(\dfrac{\partial z}{\partial x}\bigg)^{2}+ \bigg(\dfrac{\partial z}{\partial y}\bigg)^{2}=\bigg(\dfrac{\partial z}{\partial r}\bigg)^{2}+\dfrac{1}{r^{2}}\bigg(\dfrac{\partial z}{\partial \theta}\bigg)^{2}$.


  1. $\displaystyle \frac{\partial z}{\partial r} = \cos(\theta) \frac{\partial z}{\partial x}  + \sin(\theta) \frac{\partial z}{\partial y} $e$\displaystyle \frac{\partial z}{\partial \theta} = -r \sin(\theta)\frac{\partial z}{\partial x}  + r\cos(\theta) \frac{\partial z}{\partial y}.$
  2. Use $(a)$ para calcular $\bigg(\dfrac{\partial z}{\partial r}\bigg)^{2}+\dfrac{1}{r^{2}}\bigg(\dfrac{\partial z}{\partial \theta}\bigg)^{2}$.


2994   

Calcule a integral, efetuando uma mudança de variáveis apropriada. $\displaystyle\iint\limits_{R} x \, dA$, em que $R$ é o círculo $x^2 + y^2 - x \leq 0$.


$\dfrac{\pi}{8}.$


2374   

A temperatura $T$ em uma bola de metal é inversamente proporcional à distância do centro da bola, que tomamos como a origem. A temperatura no ponto $(1,2,2)$ é de 120°.

  1.  Determine a taxa de variação de $T$ em $(1,2,2)$ em direção ao ponto $(2,1,3)$.
  2.  Mostre que em qualquer ponto da bola a direção de maior crescimento na temperatura é dada por um vetor que aponta para a origem.


  1. $\displaystyle -\frac{40}{3\sqrt{3}}.$
  2.  Note que $\nabla T = -360 (x^{2} + y^{2} + z^{2})^{-3/2} (x,y,z)$ sempre aponta para a origem. 


2940   

Determine a massa e o centro de massa da lâmina que ocupa a região $D$ e tem função densidade $\rho$, quando: $D$ é delimitada por $y = e^x$, $y = 0$, $x = 0$ e $x = 1; \quad \rho(x,y) = y$.


Massa: $\dfrac{1}{4}(e^{2} - 1);$ centro de massa: $\displaystyle \left(\frac{e^2 + 1}{2(e^2 - 1)},\frac{4(e^3 - 1)}{9 (e^2 - 1)} \right).$


2401   

 Determine um plano que seja tangente à superfície $x^2 + 3y^2 + 2z^2 = \dfrac{11}{6}$ e paralelo ao plano $x + y + z = 10$.


 $\displaystyle x + y + z = \frac{11}{6}$ ou $\displaystyle x + y + z = -\frac{11}{6}.$


3078   

Verifique que a função \(f(x,y)=\sqrt{1-x^2-y^2}\) é contínua no disco unitário fechado \(x^2+y^2\leq 1\).



O domínio de \(f\) é o disco unitário fechado \(x^2+y^2\leq 1\). Para todo ponto \((x_0,y_0)\) na fronteira do disco, temos \[ \lim_{(x,y)\to(x_0,y_0)}\sqrt{1-x^2-y^2} = \sqrt{1-x_0^2-y_0^2} = 0.\] Como o mesmo vale também para pontos interiores ao disco, temos que \(f\) é contínua no disco fechado.


3047   

O campo vetorial $\mathbf{F}$ é mostrado no plano $xy$ e é o mesmo em todos os planos horizontais (em outras palavras, $\mathbf{F}$ é independente de $z$ e sua componente $z$ é 0).

  1. O $\text{div }{\mathbf{F}}$ será positivo, negativo ou nulo? Justifique.

  2. Determine se o $\text{rot }{\mathbf{F}} = 0$. Se não, em que direção rot $\mathbf{F}$ aponta?

ma211-list13-ex6_ii.png


  1. Positivo.

  2. $\text{rot } \bf{F} = \bf{0}.$


2317   

Determine uma equação do plano tangente à superfície parametrizada dada no ponto especificado. $x=u^{2}$, $y=v^{2}$, $z=uv$; $u=1$, $v=1.$


$x + y - 2z = 0.$


2189   

Use o Teorema do Divergente para calcular o fluxo de ${\bf F}$ através de $S,$ onde ${\bf F}(x,y,z)=(5x^{3}+12xy^{2})\,{\bf i}+(y^{3}+e^{y}\,\sin z)\,{\bf j}+(5z^{3}+e^{y}\,\cos z)\,{\bf k}$ e $S$ é a superfície do sólido entre as esferas $x^{2}+y^{2}+z^{2}=1$ e $x^{2}+y^{2}+z^{2}=2.$


2855   

Determine os pontos do gráfico de $xy^{3}z^{2}=16$ mais próximos da origem.


$\displaystyle \left( \frac{2}{\sqrt[4]{12}}, \sqrt[4]{12}, \frac{2 \sqrt{2}}{\sqrt[4]{12}}\right),$ $\displaystyle \left( \frac{2}{\sqrt[4]{12}}, \sqrt[4]{12}, - \frac{2 \sqrt{2}}{\sqrt[4]{12}}\right),$ $\displaystyle \left( -\frac{2}{\sqrt[4]{12}}, \sqrt[4]{12}, \frac{2 \sqrt{2}}{\sqrt[4]{12}} \right)$ e $\displaystyle \left( -\frac{2}{\sqrt[4]{12}}, \sqrt[4]{12}, - \frac{2 \sqrt{2}}{\sqrt[4]{12}} \right).$


2801   

Determine os valores máximos e mínimos locais e pontos de sela da função $f(x,y)=x^{4}+y^{4}-2x^{2}-2y^{2}$.



Pontos de mínimo: $(-1,1)$ e $(-1,-1);$ ponto de máximo: $(0,0);$ pontos de sela: $(0,1), (0,-1), (1,0)$ e $(-1,0).$


2362   

Encontre a derivada direcional de $f(x,y) = x^2 + y^2$ na direção do versor tangente da curva

$$\bf{r}(t) = (\cos{t} + t\sin{t})\bf{i} + (\sin{t} - t\cos{t})\bf{j},  t > 0.$$


Versor tangente a $\mathbf{r}(t):$ $\mathbf{u} = \cos(t)\mathbf{i} + (\sin(t))\mathbf{j};$ $D_{\mathbf{u}} f = 2.$


2652   

Disseram-lhe que existe uma função $f$ cujas derivadas parciais são \[f_{x}(x,y)=x+4y  \quad \mbox{e} \quad f_{y}(x,y)=3x-y,\] e cujas derivadas parciais de segunda ordem são contínuas. Você deve acreditar nisso?


Não, pois pelo Teorema de Clairaut deveria ser verdade que $f_{xy} = f_{yx},$ mas temos $f_{xy} = 4 \neq 3 = f_{yx}.$


1940   

Calcule a integral de linha, onde $C$ é a curva dada.

$\displaystyle\int_{C}(2x+9z)\,ds$,   $C:\,x=t,\, y=t^{2},\, z=t^{3},\, 0\leq t\leq 1.$


$\displaystyle \frac{1}{6}\left(14^{3/2} - 1\right).$


2923   

Utilize a integral dupla para determinar a área da região: limitada pelo eixo $x$ positivo e pela espiral $r=4\theta/3$, $0\leq \theta \leq 2\pi.$ A região se parece com uma concha de caracol.


$\dfrac{64\pi^3}{27}.$


2829   

Determine os pontos da superfície $y^{2}=9+xz$ que estão mais próximos da origem.


$(0,3,0)$ e $(0,-3,0).$


1944   

Calcule a integral de linha, onde $C$ é a curva dada.

$\displaystyle\int_{C}x\,dx+dy+2\,dz$, $C$ é a interseção do paraboloide $z=x^{2}+y^{2}$ com o plano $z=2x+2y-1$; caminhe no sentido anti-horário.


$0.$


2297   

Determine uma representação paramétrica para a superfície descrita a seguir. A superfície cortada do cilindro parabólico $z=4-y^{2}$ pelos planos $x=0$, $x=2$ e $z=0.$


$x = u,$ $y = v,$ $z = 4 - v^2,$ onde $0\leq u \leq 2$ e $-2 \leq v \leq 2.$


2409   

Calcule a integral de superfície $\displaystyle\iint\limits_{S}\dfrac{z}{\sqrt{1+4x^{2}+4y^{2}}}dS$, onde $S$ é a parte do parabolóide

$z=1-x^{2}-y^{2}$ que se encontra dentro do cilindro $x^{2}+y^{2}\leq 2y.$



Parametrizando a superfície $S$, temos as equações paramétricas:

$x=u, y=v \, \mbox{e} \, z=1-u^{2}-v^{2}.$
Então,
${\bf r}(u,v)=u{\bf i}+v{\bf j}+(1-u^{2}-v^{2}){\bf k}.$
Logo,
$f({\bf r}(u,v))=\dfrac{1-u^{2}-v^{2}}{\sqrt{1-4u^{2}-4v^{2}}},$ ${\bf r}_{u}={\bf i}+0{\bf j}-2u{\bf k}$ e ${\bf r}_{v}=0{\bf i}+{\bf j}-2v{\bf k}.$
Temos que

${\bf r}_{u}\times {\bf r}_{v}=\left| \begin{array}{ccc} {\bf i} & {\bf j} & {\bf k}\\ 1 & 0 & -2u\\ 0 & 1 & -2v \end{array} \right| = 2u{\bf i}+2v{\bf j}+{\bf k}$,

implicando que $|{\bf r}_{u}\times {\bf r}_{v}|=\sqrt{(2u)^{2}+(2v)^{2}+1^{2}}=\sqrt{1+4u^{2}+4v^{2}}.$ Assim,

$\displaystyle\iint\limits_{S}\dfrac{z}{\sqrt{1+4x^{2}+4y^{2}}}dS=\displaystyle\iint\limits_{D} f({\bf r}(u.v))|{\bf r}_{u}\times {\bf r}_{v}| du dv$ $=\displaystyle\iint\limits_{D} \frac{1-u^{2}-v^{2}}{\sqrt{1-4u^{2}-4v^{2}}} \sqrt{1+4u^{2}+4v^{2}} du dv=\displaystyle\iint\limits_{D}(1-u^{2}-v^{2})du dv$.
Notemos que
$D=\{(u,v)| u^{2}+v^{2}\leq 2v\}=\{(u,v)|u^{2}+(v-1)^{2}\leq 1\}.$
Em coordenadas polares teremos que
$u=r\cos \theta, v-1=r\sin \theta,$
$du dv=\left| \begin{array}{cc}
\dfrac{\partial u}{\partial r} & \dfrac{\partial u}{\partial \theta}\\
\dfrac{\partial v}{\partial r} & \dfrac{\partial v}{\partial \theta}
\end{array} \right|$, $ dr d\theta=\left| \begin{array}{cc} \cos \theta & -r\sin \theta\\ \sin \theta & r\cos \theta \end{array} \right| \, e \, du dv=r dr d\theta.$

Como $u^{2}+u^{2}=2u \Rightarrow r^{2}\cos^{2}\theta+r^{2}\sin^{2}\theta=r\sin \theta \Rightarrow r=2\sin \theta,$ então $0\leq r \leq 2\sin \theta \, \mbox{e} \, 0 \leq \theta \leq \pi.$
Logo
$\displaystyle\iint\limits_{S}\dfrac{z} {\sqrt{1+4x^{2}+4y^{2}}}dS=\displaystyle\int_{0}^{\pi}\displaystyle\int_{0}^{2\sin \theta}(1-r^{2}\cos^{2} \theta-r^{2}\sin^{2}\theta)r dr d\theta$

$\displaystyle\int_{0}^{\pi}\displaystyle\int_{0}^{2\sin \theta}(1-r^{2})r dr d\theta=\displaystyle\int_{0}^{\pi}\int_{0}^{2\sin \theta}(r-r^{3})dr d\theta$ $=\displaystyle\int_{0}^{\pi}(2\sin^{2}\theta-4\sin^{4}\theta)\bigg|_{0}^{2\sin \theta}d\theta=2\int_{0}^{\pi}\sin^{2}\theta d\theta-4\int_{0}^{\pi}\sin^{4}\theta$

$=2\cdot\left(\dfrac{\theta}{2}-\frac{1}{4}\sin 2\theta\right)\bigg|_{0}^{\pi}-4\cdot \left(-\dfrac{1}{4}\sin^{3}
\theta \cos \theta+\dfrac{3}{8}\theta-\dfrac{3}{16}\sin 2\theta\right)\bigg|_{0}^{\pi}$
$=2\cdot \dfrac{\pi}{2}-4\cdot\left(\dfrac{3}{8}\pi\right)=-\dfrac{\pi}{2}.$


3147   

  1.  Use o Teorema de Green para provar que\[ \int_Cf(x)\,dx + g(y)\,dy = 0\] se \(f\) e \(g\) forem funções diferenciáveis e \(C\) for uma curva fechada simples lisa por partes.

  2.  O que isso nos diz sobre o campo vetorial \[ \mathbf{F}(x,y) = f(x)\mathbf{i}+g(y)\mathbf{j}?\]


2674   

Calcule as derivadas parciais de $w = x^2 \arcsin{\dfrac{y}{z}}$.


$\displaystyle \frac{\partial w}{\partial x} = 2x \arcsin \left( \frac{t}{z}\right),\;\;\;\;  \frac{\partial w}{\partial y} = \frac{x^{2}|z|}{z\sqrt{z^{2} - y^{2}}} \;\;\;\;\;\text{e}\;\;\;\;\;\frac{\partial w}{\partial z} = - \frac{x^{2}y}{|z|\sqrt{z^{2} - y^{2}}}.$


2585   

Encontre o volume da região sólida limitada abaixo pelo plano $z = 0$, lateralmente pelo cilindro $x^2 + y^2 = 1$ e acima pelo paraboloide $z = x^2 + y^2$.



Temos que a região sólida $E$ está acima do plano $z=0$, abaixo do paraboloide $z=x^{2}+y^{2}$ e limitado lateralmente pelo cilindro $x^{2}+y^{2}=1$. Notemos que podemos dividir a região sólida em quatro porções simétricas. Assim, levando em consideração a porção da região sólida $E$ que está no primeiro octante, temos em coordenadas cilíndricas $$0\leq \theta \leq \frac{\pi}{2},\, 0\leq r \leq 1\,\, \mbox{e}\,\, 0\leq z\leq x^{2}+y^{2}=r^{2}.$$ Assim, o volume da região sólida $E$ é: $$V=\iiint\limits_{  E}1\,dV=4\int_{0}^{\frac{\pi}{2}}\int_{0}^{1}\int_{0}^{r^{2}}1\,r\,dz\,dr\,d\theta$$ $$=4\int_{0}^{\frac{\pi}{2}}\int_{0}^{1}zr\,\bigg|_{0}^{r^{2}}\,dr\,d\theta=4\int_{0}^{\frac{\pi}{2}}\int_{0}^{1}r^{3}\,dr\,d\theta$$ $$=4\int_{0}^{\frac{\pi}{2}}\,d\theta\cdot \int_{0}^{1}r^{3}\,dr=4\cdot \theta\bigg|_{0}^{\frac{\pi}{2}}\cdot \frac{r^{4}}{4}\bigg|_{0}^{1}$$ $$=4\cdot \frac{\pi}{2}\cdot \frac{1}{4}=\frac{\pi}{2}.$$


2991   

Calcule a integral, efetuando uma mudança de variáveis apropriada. $\displaystyle\iint\limits_{R} \sin{(4x^2 + y^2)} \, dA$, em que $R$ é o cojunto de todos $(x,y)$ tais que $4x^2 + y^2 \leq 1$ e $y \geq 0$.


$\dfrac{\pi}{4}(1 - \cos(1)).$


2942   

Calcule utilizando coordenadas esféricas. $\displaystyle\iiint\limits_{H}(9-x^{2}-y^{2})\,dV$, onde $H$ é o hemisfério sólido $x^{2}+y^{2}+z^{2}\leq 9$ e $z\geq 0.$


$\dfrac{486\pi}{5}.$


3084   

Suponha que \(f(x,y)\) seja uma função diferenciável no ponto \((x_0,y_0)\) e seja \(z_0=f(x_0,y_0)\). Mostre que a função \(\displaystyle g(x,y,z)=z-f(x,y)\) é diferenciável em \((x_0,y_0,z_0)\).


1968   

Calcule $\int_{C}(x+y+z)\,dx+(x-2y+3z)\,dy+(2x+y-z)\,dz$, onde $C$ é a curva de $(0,0,0)$ a $(2,3,4)$ se

  1. $C$ consiste em três segmentos de reta, o primeiro paralelo ao eixo $x$, o segundo paralelo ao eixo $y$ e o terceiro paralelo ao eixo $z$.

  2. $C$ consite em três segmentos de reta, o primeiro paralelo ao eixo $z$, o segundo ao eixo $x$ e o terceiro paralelo ao eixo $y.$

  3. $C$ é um segmento retilíneo.


  1. $19.$

  2. $35.$

  3. $27.$


2132   

Seja $g(t)=f(3t^{2},t^{3},e^{2t})$ e suponha $\dfrac{\partial f}{\partial z}(0,0,1)=4.$ 

  1.  Expresse $g^{'}(t)$ em termos das derivadas parciais de $f.$
  2. Calcule $g^{'}(0).$


  1. $\displaystyle g^{'}(t) = 6t \frac{\partial f}{\partial x}(3t^{2},t^{3},e^{2t}) +  3t^{2} \frac{\partial f}{\partial y}(3t^{2},t^{3},e^{2t}) + 2e^{2t} \frac{\partial f}{\partial z}(3t^{2},t^{3},e^{2t}).$ 
  2.  $g^{'}(0) = 8.$


2598   

Considere a integral tripla iterada $$\int_{-\sqrt{2}}^{\sqrt{2}}\int_{-\sqrt{2-x^2}}^{\sqrt{2-x^2}}\int_{x^2 + y^2}^{4-x^2-y^2} dz dy dx.$$

  1.  Transforme a integral utilizando coordenadas cilíndricas.

  2.  Calcule a integral.

  3.  Descreva o sólido cujo volume é dado por essa integral.


  1.  $\displaystyle \int_{0}^{2\pi}\int_{0}^{\sqrt{2}}\int_{ r^2}^{4-r^{2}} r dz dr d\theta.$

  2.  $4\pi.$

  3.  Região entre os parabolóides $z = x^2 + y^2$ e $z = 4 - x^2 - y^2$.


2318   

Determine uma equação do plano tangente à superfície parametrizada dada no ponto especificado. ${\bf r}(u,v)=(u,v,u^{2}+v^{2})$, no ponto ${\bf r}(1,1).$


$(x,y,z) = (1,1,2) + s(1,0,2) + t(0,1,2),$ $s,t \in \mathbb{R}.$


2497   

Esboce o sólido cujo volume é dado pela integral iterada.

  1.  $\displaystyle\int_{0}^{1}\int_{0}^{1-x}\int_{0}^{2-2z}\;dy dz dx$

  2.  $\displaystyle\int_{0}^{2}\int_{0}^{2-y}\int_{0}^{4-y^{2}}\;dx dz dy$

  3.  $\displaystyle\int_{0}^{1}\int_{\sqrt{1-z}}^{\sqrt{4-z}}\int_{2}^{3}\;dx dy dz$

  4.  $\displaystyle\int_{0}^{2}\int_{x^{2}}^{2x}\int_{0}^{x+y}\;dz dy dx$


  1. (... fig)

  2. (... fig.)

  3.  $\displaystyle \left\lbrace (x,y,z); 2 \leq x \leq 3,  \sqrt{1 - z} \leq y \leq \sqrt{4 - z} , 0 \leq z \leq 1\right\rbrace.$

  4.  $\displaystyle \left\lbrace (x,y,z); 0 \leq x \leq 2,  x^{2} \leq y \leq 2x , 0 \leq z \leq x + y\right\rbrace.$


1973   

 Esboce o gráfico da curva cuja equação vetorial é dada. Indique com setas a direção na qual o parâmetro cresce.

  1. $\textbf{r}(t) = (t, \cos{2t}, \sin{2t})$
  2. $\textbf{r}(t) = (1 + t, 3t, -t)$



2373   

Considere a função

$$f(x,y) = \ln{(x^2 + y^2)}.$$

  1. Determine a taxa de variação máxima de $f$ em $(1,1)$ e a direção em que isso ocorre.
  2.  Determine a derivada direcional de $f$ em $(1,1)$ na direção do vetor $\bf{v} = (3,4)$.


  1. Na direção do vetor $(1,1).$ O valor da taxa máxima é $\sqrt{2}.$ 
  2.  $ \displaystyle \frac{7}{5}.$


2512   

Ache $\displaystyle\iint \limits_{ S}{\bf F}\cdot {\bf n} dS$ se ${\bf n}$ é uma normal unitária superior de $S.$


${\bf F}=x{\bf i}+y{\bf j}+z{\bf k}$; $S$ é a parte do plano $3x+2y+z=12$ intersectada pelos planos $x=0$,$y=0$, $x=1$ e $y=2.$


$24.$


2472   

Seja $g(x,y,z)=\ln(25-x^{2}-y^{2}-z^{2}).$

  1. Calcule $g(2,-2,4).$

  2. Determine o domínio de $g$.

  3. Determine a imagem de $g$.


  1. $0.$

  2. $\left\lbrace (x,y,z): x^{2} + y^{2} + z^{2} < 25 \right\rbrace.$

  3. $(-\infty, \ln(25)].$


2669   

A função $p=p(V,T)$ é dada implicitamente pela equação $pV=nRT$, onde $n$ e $R$ são constantes não-nulas (Lei dos Gases Ideais). Calcule $\dfrac{\partial p}{\partial V}$ e $\dfrac{\partial p}{\partial T}.$


$\displaystyle \frac{\partial p}{\partial V} = -\frac{nRT}{V^{2}}\;\;\;\;\;\text{e}\;\;\;\;\; \frac{\partial p}{\partial T} = \frac{nR}{V}.$


2775   

Use multiplicadores de Lagrange para demonstrar que o triângulo com área máxima, e que tem um perímetro constante $p$, é equilátero.

(Sugestão: Utilize a fórmula de Heron para a área:

$$A = \sqrt{s(s-x)(s-y)(s-z)},$$

em que $s = p/2$ e $x,y$ e $z$ são os comprimentos dos lados.)



Utilizando a fórmula de Heron temos que a área e um triânulo é

$$A=\sqrt{s(s-x)(s-y)(s-z)},$$

com $s=p/2$ e $x,\,y,\,z$ lados do triângulo.

Mas a álgebra fica mais simples se maximizarmos o quadrado da área, isto é,

$$A^{2}=f(x,y,z)=s(s-x)(s-y)(s-z).$$

A restrição é que o triângulo têm perímetro constante $p$, ou seja,

$$g(x,y,z)=x+y+z=p.$$

De acordo com o método dos multiplicadores de Lagrange, resolvemos $\nabla f=\lambda \nabla g$ e $g=p.$ Então

$$\nabla f(x,y,z)=(\,-s(s-y)(s-z),\, -s(s-x)(s-z),\,-s(s-x)(s-y)\,)$$

e

$$\lambda \nabla g(x,y,z)=\lambda (1,1,1)=(\lambda, \lambda, \lambda).$$

Logo temos as seguintes equações

\begin{array}{rcl}-s(s-y)(s-z)&=&\lambda\\-s(s-x)(s-z)&=&\lambda\\-s(s-x)(s-y)&=&\lambda\\x+y+z&=&p\end{array}

Assim, das três primeiras equações, temos que

$$-s(s-y)(s-z)=-s(s-x)(s-z)=-s(s-x)(s-y).$$

Da primeira igualdade obtemos que $s-y=s-x\Rightarrow y=x$ e da segunda igualdade obtemos que $s-z=s-y\Rightarrow z=y$, resultando que $x=y=z.$

Portanto, o triângulo com área máxima e perímetro constante $p$ é um triângulo equilátero.


2666   

Considere a função $z=\dfrac{xy^{2}}{x^{2}+y^{2}}.$ Verifique que $x\dfrac{\partial z}{\partial x}+y\dfrac{\partial z}{\partial y}=z.$


$\displaystyle \frac{\partial z}{\partial x} = \frac{y^{4} - x^{2}y^{2}}{(x^{2} + y^{2})^{2}}\;\;\;\;\;\;\text{e}\;\;\;\;\; \frac{\partial z}{\partial y} = \frac{2x^{3}y}{(x^{2} + y^{2})^{2}}.$


2999   

Esboce a região de integração e mude a ordem de integração. $\displaystyle\int_{1}^{2}\!\!\int_{0}^{\ln(x)} \! f(x,y)\,dy dx$.


Note que a região de integração é do tipo I, é dada por

$$\{(x,y) \in \mathbb{R}^2: 1 \leq x \leq 2 \mbox{ e } 0 \leq y \leq \ln(x)\}$$

e pode ser vista geometricamente como a região esboçada na figura abaixo.

ma211-list6-ex3_sol.png

Além disso, ela pode ser descrita como uma região do tipo II da seguinte forma:
$$\{(x,y) \in \mathbb{R}^2: e^y \leq x \leq 2 \mbox{ e } 0 \leq y \leq \ln{2}\}.$$
Portanto, a integral pode ser reescrita como
$\displaystyle\int_{0}^{\ln{2}}\!\!\int_{e^y}^{2} \! f(x,y)\,dx dy$.


2232   

Seja $\mathbf{r} = x\mathbf{i} + y\mathbf{j} + z\mathbf{k}$ e $r=|\mathbf{r}|$. Verifique a identidade $\nabla \cdot (r\mathbf{r}) = 4r$.


$\nabla \cdot (r\mathbf{r}) = \left( \dfrac{x^{2}}{\sqrt{x^{2} + y^{2} + z^{2}}} + \sqrt{x^{2} + y^{2} + z^{2}} \right) + \left( \dfrac{x^{2}}{\sqrt{y^{2} + y^{2} + z^{2}}} + \sqrt{y^{2} + y^{2} + z^{2}} \right)\\ + \left( \dfrac{z^{2}}{\sqrt{x^{2} + y^{2} + z^{2}}} + \sqrt{x^{2} + y^{2} + z^{2}} \right)$ (Note que: $r = \sqrt{x^{2} + y^{2} + z^{2}}.$)


1964   

Calcule a integral de linha $\displaystyle\int_{C}x^{2}\,dx+y^{2}\,dy+z^{2}\,dz$, onde $C$ é o segmento de reta que liga o ponto $(1,0,1)$ ao ponto $(-2,2,2).$.


$\displaystyle \frac{2}{3}.$


2918   

Considere uma pá quadrada de um ventilador com lados de comprimento 2 e com o canto inferior esquerdo colocado na origem. Se a densidade da pá for $\rho(x,y) = 1 + 0,1\cdot x$, é mais difícil girar a pá em torno do eixo $x$ ou do eixo $y$?



Se calcularmos os momentos de inércia sobre $x$ e $y$, poderemos determinar em qual direção será mais difíciel de girar a pá do ventilador. Notemos que a região de integração é o quadrado com lados de comprimento 2 e com o canto inferior esquerdo colocado na origem em ambas as integrais. Então, o momento de inércia sobre o eixo $x$ é dada por: $$I_{x}=\iint\limits_{D}y^{2}\rho(x,y)\,dA=\int_{0}^{2}\int_{0}^{2}y^{2}(1+0,1x)dydx$$ $$=\int_{0}^{2}(1+0,1x)\,dx\cdot \int_{0}^{2}y^{2}\,dy=\bigg(x+0,1\frac{x^{2}}{2}\bigg)\bigg|_{0}^{2}\cdot \bigg(\frac{y^{3}}{3}\bigg)\bigg|_{0}^{2}$$ $$=\bigg[(2+0,2)-0\bigg]\cdot \bigg[\frac{8}{3}\bigg]=\frac{17,6}{3}.$$ Da mesma forma, o momente de inércia sobre o eixo $y$ é dado por: $$I_{y}=\iint\limits{D}x^{2}\rho(x,y)\,dA=\int_{0}^{2}\int_{0}^{2}x^{2}(1+0,1x)dydx$$ $$=\int_{0}^{2}(x^{2}+0,1x^{3})\,dx\cdot \int_{0}^{2}\,dy=\bigg(\frac{x^{3}}{3}+0,1\frac{x^{4}}{4}\bigg)\bigg|_{0}^{2}\cdot \bigg(y\bigg)\bigg|_{0}^{2}$$ $$=\bigg[\bigg(\frac{8}{3}+0,4\bigg)-0\bigg]\cdot \bigg[2-0\bigg]=\frac{18,4}{3}.$$ Como $I_{y}>I_{x}$ é mais difícil girarmos a pá do ventilador em torno do eixo $y.$


2963   

O centróide de uma região $E$ é dado por

$$\overline{x}=\frac{1}{vol(E)}\int_{E}x\,dV,\;\;\;\; \overline{y}=\frac{1}{vol(E)}\int_{E}y\,dV\;\; \text{e}\;\; \overline{z}=\frac{1}{vol(E)}\int_{E}z\,dV.$$

Calcule o centróide da região dada em coordenadas esféricas por $0\leq \rho \leq 1$, $0\leq\phi \leq \pi/3$ e $0\leq \theta \leq 2\pi$ (observe que, devido à simetria da região, $\overline{x}$ e $\overline{y}$ se anulam, bastando calcular a terceira coordenada).


$\overline{z} = \dfrac{9}{16}.$


2596   

 Uma casca cilíndrica tem $20$ cm de comprimento, com raio interno de 6 cm e raio externo de $7$ cm. Escreva desigualdades que descrevam a casca em um sistema de coordenadas adequado. Explique como você posicionou o sistema de coordenadas em relação à casca.


$6 \leq r \leq 7,$ $0 \leq \theta \leq 2\pi,$ $0 \leq z \leq 20.$


2761   

Determine o maior conjunto de pontos em que a função $f(x,y) = \begin{cases}\dfrac{xy}{x^2 + y^2}, & \quad \text{se } (x,y) \neq (0,0),\\0, & \quad \text{se } (x,y) = (0,0)\end{cases}$ é diferenciável. Justifique.


$\mathbb{R}^{2} \setminus \left\lbrace (0,0) \right\rbrace$.


2057   

Determine se ${\bf F}(x,y,z)=(x-y)\,{\bf i}+(x+y+z)\,{\bf j}+z^{2}\,{\bf k}$ é ou não um campo vetorial conservativo. Se for, determine uma função $f$ tal que ${\bf F}=\nabla f.$


Não.


2190   

Calcule $\displaystyle\iint \limits_{S}{\bf u}\cdot {\bf n}\,dS$, sendo $S$ a fronteira de $B$ com normal exterior ${\bf n}$, sendo $B=\{(x,y,z)\in \mathbb{R}^{3}|\, 0\leq x\leq 1,\,0\leq y\leq x$ e $0\leq z\leq 4\}$ e ${\bf u}=xy\,{\bf i}+yz\,{\bf j}+z^{2}\,{\bf k}.$


$\dfrac{13\pi}{20}.$



2200   

Suponha que substituamos coordenadas polares $x=r\cos{\theta}$ e $y=r\sin{\theta}$ em uma função diferenciável $w=f(x,y).$

  1.  Mostre que $$\frac{\partial w}{\partial r}=f_{x}\cos{\theta}+f_{y}\sin{\theta}$$ e $$\frac{1}{r}\frac{\partial w}{\partial \theta}=-f_{x}\sin{\theta}+f_{y}\cos{\theta}.$$
  2.  Resolva as equações no item 1. para expressar $f_{x}$ e $f_{y}$ em termos de $\partial w/ \partial r$ e $\partial w/\partial \theta$.
  3.  Mostre que  $$(f_{x})^{2}+(f_{y})^{2}=\bigg(\frac{\partial w}{\partial r}\bigg)^{2}+\frac{1}{r^{2}}\bigg(\frac{\partial w}{\partial \theta}\bigg)^{2}.$$



  1.  $\displaystyle f_{x} = \cos(\theta) \frac{\partial w}{\partial r} - \frac{\sin (\theta)}{r} \frac{\partial w}{\partial \theta}$ e $\displaystyle f_{y} = \sin(\theta) \frac{\partial w}{\partial r} + \frac{\cos (\theta)}{r} \frac{\partial w}{\partial \theta}.$


3083   

A resistência total \(R\) de três resistores \(R_1\), \(R_2\) e \(R_3\) ligados em paralelo é dada por \[ \frac{1}{R}=\frac{1}{R_1}+\frac{1}{R_2}+\frac{1}{R_3}. \] Suponha que \(R_1\), \(R_2\) e \(R_3\) tenham sido medidos como \(100\ \Omega\), \(200\  \Omega\) e \(300\  \Omega\), respectivamente, com um erro máximo de \(10\%\) em cada um e sendo \(\Omega\)(Ohm) a unidade de medida no sistema internacional de unidades. Use diferenciais para aproximar o erro percentual máximo no valor calculado de \(R\).


2952   

Seja $E$ o sólido limitado pelos dois planos $z=1$ e $z=2$ e lateralmente pelo cone $z=\sqrt{x^{2}+y^{2}}$. Expresse o volume de $E$ como integral tripla em coordenadas esféricas (não é necessário calcular a integral).


$\displaystyle \int_{0}^{2\pi} \int_{0}^{\pi/4} \int_{\sec(\phi)}^{2\sec(\phi)} \rho^{2}\sin(\phi)\;d\rho d\phi d\theta.$


2490   

Faça uma correspondência entre a função e seu gráfico (indicado por I-VI). Dê razões para sua escolha.

  1. $f(x,y)=|x|+|y|$.

  2. $f(x,y)=\dfrac{1}{1+x^{2}+y^{2}}$.

  3. $f(x,y)=(x-y)^{2}$.

  4. $f(x,y)=|xy|$.

  5. $f(x,y)=(x^{2}-y^{2})^{2}$.

  6. $f(x,y)=\sin(|x|+|y|)$.

ma211-list2-ex12.png


2218   

Determine o rotacional e o divergente do campo vetorial $\mathbf{F}(x,y,z) = (\ln{x},\ln{(xy)},\ln{(xyz)})$.


$\text{rot } \mathbf{F} = \dfrac{1}{y}\mathbf{i} - \dfrac{1}{x} \mathbf{j} +\dfrac{1}{x} \mathbf{k}.$ $\text{div } \mathbf{F} = \dfrac{1}{x} + \dfrac{1}{y} + \dfrac{1}{z}.$


2542   

Encontre o fluxo do campo ${\bf F}$ ao longo da porção da superfície dada no sentido especificado.

  • ${\bf F}(x,y,z)=yx^{2}{\bf i}-2{\bf j}+xz{\bf k}$; $S$ é a superfície retangular $y=0$, $-1\leq x \leq 2$, $2\leq z \leq 7$, sentido $-{\bf j}.$


$30.$


2496   

Para qual valor de $c$ o volume do elipsóide $x^{2}+(y/2)^{2}+(z/c)^{2}=1$ é igual a $8\pi$?


$3.$


2042   

Utilize a Regra da Cadeia para determinar as derivadas parciais indicadas.
$Y=w\tan^{-1}(uv)$, $u=r+s$,  $v=s+t$; $w=t+r$
$\dfrac{\partial Y}{\partial r}$, $\dfrac{\partial Y}{\partial s}$, $\dfrac{\partial Y}{\partial t}$ quando $r=1$, $s=0$, $t=1$.


$\displaystyle \frac{\partial Y}{\partial r} = 1 + \frac{\pi}{4}$ ,$\dfrac{\partial Y}{\partial s} = 2$, $\displaystyle \dfrac{\partial Y}{\partial t} = 1 + \frac{\pi}{4}.$


2337   

Determine a área da superfície dada pela parte da superfície $y=4x+z^{2}$ que está entre os planos $x=0$, $x=1$, $z=0$ e $z=1.$


$\dfrac{\sqrt{21}}{2} + \dfrac{17}{4} \left( \ln(2 + \sqrt{21}) - \ln(\sqrt{17}) \right).$


2037   

Se $z=f(x,y)$, onde $f$ é diferenciável, e $x=g(t)$, $g(3)=2$, $g'(3)=5$, $f_{x}(2,7)=6$, $y=h(t)$, $h(3)=7$, $h'(3)=-4$, $f_{y}(2,7)=-8,$ determine $\mathrm{d}z/ \mathrm{d}t$ quando $t=3.$



$\displaystyle \frac{dz}{dt}(3) = 62.$


3038   

Esboce a região cuja área é dada pela integral e calcule-a, sendo: $\displaystyle\int_{\pi}^{2\pi}  \int_{4}^{7}   r \, dr d\theta.$


$\displaystyle \frac{33\pi}{2};$ região de integração:

ma211-list7-ex6_sol_a.png



2907   

Utilize coordenadas polares para determinar o volume do sólido dado: dentro do cilindro $x^2+y^2=4$ e do elipsoide $4x^2+4y^2+z^2=64.$


$\displaystyle \frac{8\pi}{3} (64 - 24\sqrt{3}).$


1962   

Calcule a integral de linha $\displaystyle\int_{C}\,dx+\,dy$, onde $C$ é a poligonal de vértices $A_{0}=(0,0)$, $A_{1}=(1,2)$, $A_{2}=(-1,3)$, $A_{3}=(-2,1)$ e $A_{4}=(-1,-1)$, sendo $C$ orientada de $A_{0}$ para $A_{4}.$


$\displaystyle -2.$


2296   

Determine uma representação paramétrica para a superfície descrita a seguir. A porção da esfera $x^{2}+y^{2}+z^{2}=3$ entre os planos $z=\sqrt{3}/2$ e $z=-\sqrt{3}/2.$


$x = \sqrt{3}\sin(\phi)\cos(\theta),$ $y = \sqrt{3}\sin(\phi)\sin(\theta),$ $z = \sqrt{3}\cos(\phi),$ onde $\dfrac{\pi}{3} \leq \phi \leq \dfrac{2\pi}{3}$ e $0 \leq \theta \leq 2\pi.$


2813   

Determine os valores máximo e mínimo absolutos de $f$ no conjunto $D.$

$f(x,y)=x^{2}+y^{2}+x^{2}y+4$, $D=\{(x,y) \in \mathbb{R}^2: |x|\leq 1, \; |y|\leq 1\}.$


Valor máximo: $7;$ valor mínimo: $4.$


2240   

A primeira identidade de Green é dada por:

$$\iint\limits_{ D} f\nabla^2g \, dA = \oint_{C}f(\nabla{g}) \cdot \mathbf{n} \, ds - \iint\limits_{ D}\nabla{f} \cdot \nabla{g} \, dA,$$

em que $D$ e $C$ satisfazem as hipóteses do Teorema de Green e as derivadas parciais apropriadas de $f$ e $g$ existem e são contínuas. (A quantidade $ \nabla{g} \cdot \mathbf{n} = D_{\mathbf{n}}g$ aparece na integral de linha. Essa é a derivada direcional na direção do vetor normal $\mathbf{n}$ e é chamada derivada normal de $g$.) Use-a para demonstrar a segunda identidade de Green:

$$\iint\limits_{ D} (f\nabla^2g - g\nabla^2f)\, dA = \oint_{C}(f\nabla{g} - g\nabla{f}) \cdot \mathbf{n} \, ds,$$

em que $D$ e $C$ satisfazem as hipóteses do Teorema de Green e as derivadas parciais apropriadas de $f$ e $g$ existem e são contínuas.


Note que pela primeira identidade de Green,

$$\iint\limits_{ D} (f\nabla^2g - g\nabla^2f)\, dA = \oint_{C}(f\nabla{g} \cdot \mathbf{n} - g\nabla{f} \cdot \mathbf{n}) \, ds, + \iint\limits_{ D} (\nabla f \cdot \nabla g - \nabla g \cdot \nabla f)\, dA.$$


2783   

Calcule a integral dupla usando coordenadas polares: $\displaystyle\iint\limits_{R}\frac{x^{2}}{x^{2}+y^{2}}\,dA$, onde $R$ é a região anular limitada por $x^{2}+y^{2}=a^{2}$ e $x^{2}+y^{2}=b^{2}$, $0< a< b.$


$\displaystyle \frac{\pi}{2}(b^2 - a^2).$


2269   

Determine uma equação do plano tangente à superfície parametrizada dada no ponto especificado. ${\bf r}(u,v)=u^{2}\,{\bf i}+2u\,\sin v\,{\bf j}+u\,\cos v\,{\bf k}$; $u=1$, $v=0.$



Temos que ${\bf r}(u,v)=\underbrace{u^{2}}_{x(u,v)}\,{\bf i}+\underbrace{2u\,\sin v}_{y(u,v)}\,{\bf j}+\underbrace{u\,\cos v}_{z(u,v)}\,{\bf k}$

Primeiro, vamos calcular os vetores tangentes:

$$\begin{array}{rcl}{\bf r}_{u}&=&\frac{\partial x(u,v)}{\partial u}\,{\bf i}+\frac{\partial y(u,v)}{\partial u}\,{\bf j}+\frac{\partial z(u,v)}{\partial u}\,{\bf k}\\&=& 2u\,{\bf i}+2\,\sin v\,{\bf j}+\cos v\,{\bf k}\end{array}$$

e

$$\begin{array}{rcl}{\bf r}_{v}&=&\frac{\partial x(u,v)}{\partial v}\,{\bf i}+\frac{\partial y(u,v)}{\partial v}\,{\bf j}+\frac{\partial z(u,v)}{\partial v}\,{\bf k}\\&=& 0\,{\bf i}+2u\,\cos v\,{\bf j}-u\sin v\,{\bf k}\end{array}$$


Assim, o vetor normal ao plano tangente é:

$$\begin{array}{rcl}{\bf r}_{u}\times {\bf r}_{v}&=&\left|\begin{array}{ccc}{\bf i} & {\bf j} & {\bf k}\\2u & 2\sin v & \cos v\\0 & 2u\cos v & -u\sin v\\\end{array}\right|\\&=&(-2u\,\sin^{2}v-2u\cos^{2}v)\,{\bf i}+(2u^{2}\,\sin v)\,{\bf j}+(4u^{2}\,\cos v)\,{\bf k}\end{array}$$


Como $u=1$ e $v=0$ temos que o vetor normal é $-2\,{\bf i}+0\,{\bf j}+4\,{\bf k}.$

Portanto, uma equação do plano tangente no ponto ${\bf r}(1,0)=(1,0,1)$ é

$$-2\cdot(x-1)+0\cdot(y-0)+4\cdot (z-1)=0$$

$$-2x+2+4z-4=0$$

$$-2x+4z-2=0    \mbox{ou}     x-2z+1=0$$


1929   

Uma partícula se move em um campo de velocidade $\textbf{V}(x,y) = (x^2,x+y^2)$. Se ela está na posição $(2,1)$ no instante $t=3$, estime sua posição no instante $t=3,01$.


$(2,04;1,03).$


2578   

Determine o conjunto dos pontos de continuidade da função $f(x,y) = 3x^2y^2 - 5xy + 6$. Justifique sua resposta.


$\mathbb{R}^{2}.$


2606   

Use o Teorema de Stokes para calcular $\displaystyle\iint\limits_{ S}\mbox{rot}{\bf F}\cdot d{\bf S}.$

  • ${\bf F}(x,y,z) = xyz{\bf i} + xy{\bf j} + x^2yz{\bf k}$ e $S$ é formada pelo topo e pelos quatro lados (mas não pelo fundo) do cubo com vértices $(\pm 1,\pm 1,\pm 1)$, com orientação para fora.


$0.$


2962   

Usando coordenadas esféricas, determine o centroide e o momento de inércia em relação a um diâmetro de sua base do hemisfério sólido homogêneo de raio $a.$


Centróide: $\left(0,0,\dfrac{3a}{8} \right);$ momento de inércia: $\dfrac{4 K a^5 \pi}{15},$ onde $K$ é a densidade constante.


2877   

Embora $\nabla f = \lambda \nabla g$ seja uma condição necessária para a ocorrência de um valor extremo de $f(x,y)$ sujeito à restrição $g(x,y) = 0$, ela não garante por si só que ele exista. Como um exemplo, tente usar o método dos multiplicadores de Lagrange para encontrar um valor máximo de $f(x,y) = x + y$ sujeito à restrição $xy = 16$. O método identificará os dois pontos $(4,4)$ e $(-4,-4)$ como candidatos para a localização dos valores extremos. Ainda assim, a soma $x + y$ não tem valor máximo sobre a hipérbole. Quanto mais distante você está da origem nessa hipérbole no primeiro quadrante, maior se torna a soma $f(x,y) = x + y$.


Note que quando $x \to 0,$ tem-se $y \to \infty$ e $f(x,y) \to \infty;$ e quando $x \to -\infty,$ tem-se $y \to 0$ e $f(x,y) \to -\infty,$ logo não há valores máximo e mínimo de $f$ sujeito a esta restrição.


2349   

Calcule o volume do conjunto dado.

  1.   $\{(x,y,z)\in \mathbb{R}^{3}|  0\leq x\leq 1, 0\leq y\leq 1,0\leq z\leq x+2y\}$

  2.  $\{(x,y,z)\in \mathbb{R}^{3}| 0\leq x\leq 2,  1\leq y\leq 2, 0\leq z\leq \sqrt{xy}\}$

  3.  $\{(x,y,z)\in \mathbb{R}^{3}| 0\leq x\leq 1,  0\leq y\leq 1, 0\leq z\leq xye^{x^{2}-y^{2}}\}$

  4.  $ \{(x,y,z)\in \mathbb{R}^{3}| 0\leq x\leq 1,  0\leq y\leq 1, x^{2}+y^{2}\leq z\leq 2\}$

  5.  $\{(x,y,z)\in \mathbb{R}^{3}| 1\leq x\leq 2, 0\leq y\leq 1,\;x+y\leq z\leq x+y+2\}$

  6.  $\{(x,y,z)\in \mathbb{R}^{3}|\;0\leq x\leq 1, 0\leq y\leq 1,1\leq z\leq e^{x+y}\}$


  1.   $\dfrac{3}{2}.$

  2.   $\dfrac{8\sqrt{2}(2\sqrt{2} - 1)}{9}.$

  3.   $\dfrac{(e - 1)(1 - e^{-1})}{4}.$

  4.   $\dfrac{4}{3}.$

  5.   $2.$

  6.   $e^{2}-2e.$


3085   

Mostre que se \(f\) é diferenciável e \(z=xf(x/y)\), então todos os pontos planos tangentes ao gráfico dessa equação passam pela origem.


2495   

Dada a função $f(x,y)=\dfrac{y}{x^{2}}$.

  1. Encontre o domínio da função.

  2. Encontre a imagem da função.

  3. Descreva as curvas de nível da função.


  1. $D_{f} = \left\lbrace (x,y);\; (x,y) \neq (0,y) \right\rbrace$.

  2. $Im(f) =\mathbb{R}.$

  3. As curvas de nível são as parábolas $y = C x^{2}$ sem a origem se $C \neq 0$ e o eixo $x$ se $C \neq 0.$


2957   

Usando coordenadas esféricas, determine o volume da região cortada do cilindro sólido $x^{2}+y^{2}\leq 1$ pela esfera $x^{2}+y^{2}+z^{2}=4.$


$\dfrac{4\pi(8 - 3\sqrt{3})}{3}.$


2283   

Determine se os pontos $P(7,10,4)$ e $Q(5,22,5)$ estão na superfície ${\bf r}(u,v)=(2u+3v,1+5u-v,2+u+v)$.


$P$ não está na superfície; $Q$ está na superfície.


2098   

Suponha que ${\bf F}=\nabla f$ seja um campo vetorial conservativo e

$$g(x,y,z)=\int_{(0,0,0)}^{(x,y,z)}{\bf F}\cdot d{\bf r}.$$

Mostre que $\nabla g={\bf F}.$


Como $g(x,y,z) = f(x,y,z) - f(0,0,0),$ segue que $\nabla g = \nabla f = \mathbf{F}.$



2815   

Determine os valores máximo e mínimo absolutos de $f$ no conjunto $D.$

$f(x,y)=2x^{3}+y^{4}$, $D=\{(x,y) \in \mathbb{R}^2: x^{2}+y^{2}\leq 1\}.$


Valor máximo: $2;$  valor mínimo: $-2.$


2194   

Use o Teorema do Divergente para calcular $\displaystyle\iint \limits_{S}{\bf F}\cdot dS$, onde ${\bf F}(x,y,z)=z^{2}x\,{\bf i}+(\frac{1}{3}y^{3}+tg z)\,{\bf j}+(x^{2}z+y^{2})\,{\bf k}$ e $S$ é a metade de cima da esfera $x^{2}+y^{2}+z^{2}=1.$
[Sugestão: observe que $S$ não é uma superfície fechada. Calcule primeiro as integrais sobre $S_{1}$ e $S_{2}$, onde $S_{1}$ é o círculo $x^{2}+y^{2}\leq 1$, orientado para baixo, e $S_{2}=S\cup S_{1}.$]



Note que $\dfrac{\partial}{\partial x} \left( \dfrac{x}{|{\bf x}|^3} \right) = \dfrac{|{\bf x}|^2 - 3x^2}{|{\bf x}|^5},$ $\dfrac{\partial}{\partial y} \left( \dfrac{y}{|{\bf x}|^3} \right) = \dfrac{|{\bf x}|^2 - 3y^2}{|{\bf x}|^5}$ e $\dfrac{\partial}{\partial z} \left( \dfrac{x}{|{\bf x}|^3} \right) = \dfrac{|{\bf x}|^2 - 3z^2}{|{\bf x}|^5}.$


2664   

Determine as derivadas parciais de $f(x,y)=\sqrt[3]{x^{3}+y^{2}+3}$.


$\displaystyle \frac{\partial f}{\partial x} = \frac{x^{2}}{\sqrt[3]{(x^{3} + y^{3} + 3)^{2}}}\;\;\;\;\;\;\text{e}\;\;\;\;\; \frac{\partial f}{\partial y} = \frac{2y}{3 \sqrt[3]{(x^{3} + y^{3} + 3)^{2}}} .$


2978   

Determine a imagem do conjunto $S$ sob a transformação dada. $S = \{(x,y) \in \mathbb{R}^2: 0 \leq u \leq 3, \, 0 \leq v \leq 2\}$;$x = 2u + 3v$, $y = u - v$.


O paralelogramo com vértices $(0,0),$ $(6,3),$ $(12,1),$ $(6,-2).$


3089   

Mostre que se \(f_x(x,y)=0\) e \(f_y(x,y)=0\) em toda uma região circular, então \(f(x,y)\) é constante nessa região.


2150   

Use o Teorema do Divergente para calcular o fluxo de ${\bf F}$ através de $S,$ onde ${\bf F}(x,y,z)=x^{3}y\,{\bf i}-x^{2}y^{2}\,{\bf j}-x^{2}yz\,{\bf k}$ e $S$ é a superfície do sólido delimitado pelo hiperbolóide $x^{2}+y^{2}-z^{2}=1$ e pelos planos $z=-2$ e $z=2.$


2178   

Seja $D$ a região limitada por um caminho fechado e simples $C$ no plano $xy$. Utilize o Teorema de Green para demonstrar que as coordenadas do centroide $(\bar{x},\bar{y})$ de $D$ são

$$\bar{x} = \dfrac{1}{2A}\oint_{C}x^2 \, dy \quad \quad\quad\quad \bar{y} = -\dfrac{1}{2A}\oint_{C}y^2 \, dx,$$

em que $A$ é a área de $D$.


$\dfrac{1}{2A}\oint_{C}x^2 \, dy = \dfrac{1}{2A} \iint_{D} 2x \, dA = \bar{x}$ e $-\dfrac{1}{2A}\oint_{C}y^2 \, dx = -\dfrac{1}{2A}\iint_{D} (-2y) \, dA = \bar{y}$


1932   

Calcule a integral de linha, onde $C$ é a curva dada.

$\displaystyle\int_{C}x\,\sin{y}\,ds$,   $C$ é o segmento de reta que liga $(0,3)$ a $(4,6).$


$\displaystyle \frac{20}{6} \left(\sin(6) - 3\cos(6) - \sin(3) \right).$


2926   

Identifique a superfície cuja equação é $\rho=\sin{\theta}\sin{\phi}.$


Esfera de raio $\dfrac{1}{2}$ centrada no ponto $\left(0,\dfrac{1}{2},0\right).$


2106   

Um campo vetorial inverso do quadrado é da forma:

$${\bf F}({\bf r})=\frac{c{\bf r}}{|{\bf r}|^{3}}$$

para alguma constante $c$, onde ${\bf r}=x\,{\bf i}+y\,{\bf j}+z\,{\bf k}$. Um exemplo de um campo inverso do quadrado é o campo gravitacional ${\bf F}=-(mMG){\bf r}/|{\bf r}|^{3}$. Determine o trabalho realizado pelo campo gravitacional quando a Terra se move do afélio (em uma distância máxima de $1,52\times 10^{8}\,km$ do Sol) ao periélio (em uma distância mínima de $1,47\times 10^{8}\,km)$. (Use os valores $m=5,97\times 10^{24}\,kg$, $M=1,99\times 10^{30}\,kg$ e $G=6,67\times 10^{-11}\,N\cdot m^{2}/kg^{2}.$)


$\approx 1,77 \times 10^{35}$ J.


3043   

Calcule

$$\oint_{C} \dfrac{-y}{x^2+y^2} \, dx + \dfrac{x}{x^2 + y^2} \, dy,$$

em que $C$ é a curva

ma211-list12-ex2_sol_a.png



Podemos escrever $C$ como $C_1 \cup C_2$, em que $C_1$ e $C_2$ são as curvas dadas abaixo.

ma211-list12-ex2_sol_b.png

Seja $A$ um aberto simplesmente conexo que contém $C_1$ e não contém a origem. O campo $\mathbf{F}$ restrito a $A$ é conservativo, pois $A$ é aberto e simplesmente conexo, $P(x,y) = \dfrac{-y}{x^2 + y^2}$ e $Q(x,y) = \dfrac{x}{x^2 + y^2}$ possuem derivadas de primeira ordem contínuas em $A$ e $P$ e $Q$ satisfazem a relação $\frac{\partial Q}{\partial x} = \frac{\partial P}{\partial y}$. Então,
$$\oint_{C_1}\mathbf{F} \cdot\, d\mathbf{r} = 0.$$
Não podemos proceder de maneira análoga em $C_2$, já que todo aberto $B$ que contém a curva $C_2$ e não contém a origem não será simplesmente conexo. Com isso, não conseguimos garantir que o campo $\mathbf{F}$ restrito a $B$ é conservativo (observe que, a princípio, não podemos afirmar que o campo é não conservativo).
A ideia para contornar esse problema é ``isolar" a origem com uma curva fechada $C_3$, a princípio arbitrária. Vamos escolher essa curva $C_3$ de maneira conveniente para que consigamos resolver o problema. Seja $\varepsilon > 0$ pequeno o suficiente para que a curva $C_3$ parametrizada por $r(t) = (\varepsilon \cos{t}, \varepsilon \sin{t})$, com $t$ variando de $2\pi$ a $0$, não intercepte a curva $C_2$ e esteja entre a curva $C_2$ e a origem.

ma211-list12-ex2_sol_c.png

Considere $D_1 = \{(x,y) \in \mathbb{R}^2: (x,y) \mbox{ está entre } C_2 \mbox{ e } C_3 \mbox{ e } y \geq 0\}$ e $D_2 = \{(x,y) \in \mathbb{R}^2: (x,y) \mbox{ está entre } C_2 \mbox{ e } C_3 \mbox{ e } y \leq 0\}$. As curvas que delimitam $D_1$ e $D_2$ são $C_{D_1}= C_{2}^+\cup C_{a}\cup C_{3}^+\cup C_{b}$ e $C_{D_2}=C_{2}^-\cup -C_{b}\cup C_{3}^- \cup -C_{a}$, respectivamente, e estão ilustradas a seguir.

ma211-list12-ex2_sol_d.png


Note que
$$\oint_{C_{D_1}} \mathbf{F} \cdot d\mathbf{r} = \int_{C^+_2} \mathbf{F} \cdot d\mathbf{r} + \int_{C_a} \mathbf{F} \cdot d\mathbf{r} + \int_{C^+_3} \mathbf{F} \cdot d\mathbf{r} + \int_{C_b} \mathbf{F} \cdot d\mathbf{r} \qquad (\star)$$
e
$$\oint_{C_{D_2}} \mathbf{F} \cdot d\mathbf{r} = \int_{C^-_2} \mathbf{F} \cdot d\mathbf{r} + \int_{-C_a} \mathbf{F} \cdot d\mathbf{r} + \int_{C^-_3} \mathbf{F} \cdot d\mathbf{r} + \int_{-C_b} \mathbf{F} \cdot d\mathbf{r}  \qquad (\star \star).$$
Como $\frac{\partial Q}{\partial x} = \frac{\partial P}{\partial y}$, temos, pelo Teorema de Green,
$$\displaystyle\oint_{C_{D_1}} \mathbf{F} \cdot d\mathbf{r} = \iint\limits_{D_1} 0 \, dA = 0$$
e
$$\displaystyle\oint_{C_{D_2}} \mathbf{F} \cdot d\mathbf{r} = \iint\limits_{D_2} 0 \, dA = 0.$$
Somando as equações ($\star$) e ($\star \star$), obtemos
$$\int_{C^+_2} \mathbf{F} \cdot d\mathbf{r} + \int_{C^+_3} \mathbf{F} \cdot d\mathbf{r} + \int_{C^-_2} \mathbf{F} \cdot d\mathbf{r} + \int_{C^-_3} \mathbf{F} \cdot d\mathbf{r} = 0,$$
isto é,
$$\int_{C_2} \mathbf{F} \cdot d\mathbf{r} = -\int_{C_3} \mathbf{F} \cdot d\mathbf{r} = \int_{-C_3} \mathbf{F} \cdot d\mathbf{r}.$$
Assim, basta determinar $\int_{-C_3} \mathbf{F} \cdot d\mathbf{r}$. A parametrização de $-C_3$ é $r(t) = (\varepsilon \cos{t}, \varepsilon \sin{t})$, com $t$ variando de $0$ a $2\pi$. Daí,
$$\begin{array}{rcl}\displaystyle \int_{-C_3} \mathbf{F} \cdot d\mathbf{r} & = & \displaystyle \int_{0}^{2\pi} \left(\frac{-\varepsilon \sin{t}}{\varepsilon^2},\frac{\varepsilon \cos{t}}{\varepsilon^2}\right) \cdot (-\varepsilon \sin{t}, \varepsilon \cos{t}) \, dt \\& = & \displaystyle \int_{0}^{2\pi} 1 \, dt =  2\pi.\end{array}$$
Portanto,
$$\oint_{C} \mathbf{F} \cdot d\mathbf{r} = \oint_{C_1} \mathbf{F} \cdot d\mathbf{r} + \oint_{C_2} \mathbf{F} \cdot d\mathbf{r} = 0 + 2\pi = 2\pi.$$


2356   

Calcule o volume do conjunto dado.

  1.  $x^{2}+y^{2}\leq z\leq 2x.$

  2.  $x\leq z\leq1-y^{2}$ e $x\geq 0.$


  1.  $\dfrac{\pi}{2}.$

  2.  $\dfrac{8}{15}.$


2287   

Determine uma representação paramétrica para a superfície descrita a seguir. O plano que passa pelo ponto $(1,2,-3)$ e contém os  vetores ${\bf i}+{\bf j}-{\bf k}$ e ${\bf i}-{\bf j}+{\bf k}.$


$x= 1 + u + v,$ $y = 2 + u - v,$ $z = 3 - u + v.$


2642   

Determine as derivadas parciais de primeira ordem da função $u=x^{y/z}$.


$\displaystyle \frac{\partial u}{\partial x} = \frac{y}{z} x^{(y/z) - 1},\;\;\; \frac{\partial u}{\partial y} = x^{y/z} \ln x \;\;\;\text{e}\;\;\; \frac{\partial u}{\partial z} = - \frac{yx^{y/z}}{z^{2}} \ln x$.


2971   

Mostre que

$$\int_{-\infty}^{\infty}\int_{-\infty}^{\infty}\int_{-\infty}^{\infty}\sqrt{x^{2}+y^{2}+z^{2}}\,e^{-(x^{2}+y^{2}+z^{2})}\,dxdydz=2\pi.$$

(A integral imprópria tripla é definida como o limite da integral tripla sobre uma esfera sólida quando o raio da esfera aumenta indefinidamente.)


Note que $$\begin{split}&\int_{-\infty}^{\infty}\int_{-\infty}^{\infty}\int_{-\infty}^{\infty}\sqrt{x^{2}+y^{2}+z^{2}}\,e^{-(x^{2}+y^{2}+z^{2})}\,dxdydz  \\&= \lim_{R \to \infty} \int_{0}^{2\pi}\int_{0}^{\pi}\int_{0}^{R} \rho e^{-\rho^2}\rho^2 \sin(\phi)\;d\rho d\phi d\theta.\end{split}$$



2797   

Determine os valores máximos e mínimos locais e pontos de sela da função $f(x,y)=e^{x}\cos{y}$.



Não há pontos críticos.


1999   

Calcule o trabalho realizado por uma partícula andando sobre a espiral dada por $C:\,x=t\,\cos t$, $y=t\,\sin t$, com $0\leq t\leq 2\pi$, sob a ação do campo ${\bf F}(x,y)=(x,y)$, ou seja, calcule a integral $\int_{C}x\,dx+y\,dy.$


$2\pi^{2}.$


1995   

Seja ${\bf F}(t)$ uma força dependendo do tempo $t$, que atua sobre uma partícula entre os instantes $t_{1}$

e $t_{2}$. Supondo ${\bf F}$ integrável em $[t_{1},t_{2}]$, o vetor 

$${\bf I}=\int_{t_{1}}^{t_{2}}{\bf F}(t)\mathrm{d}t$$

denomina-se impulso de ${\bf F}$ no intervalo de tempo $[t_{1},t_{2}]$. Calcule o impulso de ${\bf F}$ no intervalo 

de tempo dado.

  1. ${\bf F}(t)=t{\bf i}+{\bf j}+t^{2}{\bf k}$, $t_{1}=0$ e $t_{2}=2.$
  2. ${\bf F}(t)=\dfrac{1}{t+1}{\bf i}+t^{2}{\bf j}+{\bf k}$, $t_{1}=0$ e $t_{2}=1.$


2265   

Calcule $\displaystyle\iint\limits_{B}f(x,y)\,dx dy$ sendo dados:

  1.  $f(x,y)=y^{3}e^{xy^{2}}$ e $B$ o retângulo $0\leq x\leq 1$, $1\leq y\leq 2.$

  2.  $f(x,y)=x^{5}\cos{y^{3}}$ e $B=\{(x,y)\in \mathbb{R}^{2}|\;y\geq x^{2},\;x^{2}+y^{2}\leq 2\}.$

  3.  $f(x,y)= x^{2}$ e $B$ o conjunto de todos $(x,y)$ tais que $x\leq y\leq -x^{2}+2x+2.$

  4.  $f(x,y)=x$ e $B$ a região compreendida entre os gráficos de $y=\cos{x}$ e $y=1-\cos{x}$, com $0\leq x\leq \dfrac{\pi}{2}.$


  1.  $\dfrac{e^{4} - e - 3}{2}.$

  2.  $0.$

  3.  $\dfrac{63}{20}.$

  4.  $\left(\dfrac{5}{72} -\dfrac{ \sqrt{3}}{18}\right)\pi^{2} + \left( \dfrac{4\sqrt{3}}{3} - 1 \right) \pi.$


2085   

Calcule $\displaystyle\int_{(-1,0)}^{(1,0)}\dfrac{x}{x^{2}+y^{2}}\,dx+\dfrac{y}{x^{2}+y^{2}}\,dy$.


$\displaystyle \dfrac{\pi}{4} + \arctan\left( \dfrac{2}{3} \right).$


2241   

As equações de Maxwell relacionam o campo elétrico $\mathbf{E}$ e o campo magnético $\mathbf{H}$, quando eles variam com o tempo em uma região que não contenha carga nem corrente, como segue:

$$\text{div }{\mathbf{E}} = 0,\text{     }\text{div }{\mathbf{H}} = 0$$

$$\text{rot }{\mathbf{E}} = -\dfrac{1}{c}\dfrac{\partial\mathbf{H}}{\partial t},\text{     }\text{rot }{\mathbf{H}}  =  \dfrac{1}{c}\dfrac{\partial\mathbf{E}}{\partial t},$$

em que $c$ é a velocidade da luz. Use essas equações para demonstrar o seguinte:

  1. $\nabla \times (\nabla \times \mathbf{E}) = - \dfrac{1}{c^2}\dfrac{\partial^2\mathbf{E}}{\partial t^2}$,

  2. $\nabla \times (\nabla \times \mathbf{H}) = - \dfrac{1}{c^2}\dfrac{\partial^2\mathbf{H}}{\partial t^2}$,

  3. $\nabla^2{\mathbf{E}} =  \dfrac{1}{c^2}\dfrac{\partial^2\mathbf{E}}{\partial t^2}$,

  4. $\nabla^2{\mathbf{H}} =  \dfrac{1}{c^2}\dfrac{\partial^2\mathbf{H}}{\partial t^2}$.


  1. $\nabla \times (\nabla \times \mathbf{E}) = \nabla \times (\text{rot } \mathbf{E}) = \nabla \times \left( -\dfrac{1}{c} \dfrac{\partial \mathbf{H}}{\partial t} \right) = -\dfrac{1}{c} \dfrac{\partial}{\partial t} \text{rot } \mathbf{H} = -\dfrac{1}{c} \dfrac{\partial}{\partial t} \left(\dfrac{1}{c} \dfrac{\partial \mathbf{E}}{\partial t} \right)$

  2. Análogo ao item 1.

  3. Note que $\nabla^2{\mathbf{E}} =  \nabla \text{div } \mathbf{E} - \text{rot } \text{rot }(\mathbf{E}).$

  4. Análogo ao item 3.


2647   

Determine as derivadas parciais indicadas. $w=\dfrac{x}{y+2z}$; \;\;\;\;$\dfrac{\partial^{3}w}{\partial z\partial y \partial x}$, \;\;\;\;$\dfrac{\partial^{3}w}{\partial x^{2}\partial y}$.


$\displaystyle \frac{\partial^{3}w}{\partial z\partial y \partial x} = \frac{4}{(y + 2z)^{3}}\;\;\;\text{e} \;\;\;\; \frac{\partial^{3}w}{\partial x^{2}\partial y} = 0$.


2465   

Calcule a integral de superfície $\displaystyle\iint \limits_{ S}{\bf F}\cdot d{\bf S}$ para o campo vetorial ${\bf F}$ e superfície orientada $S$ dados abaixo. Em outras palavras, determine o fluxo de ${\bf F}$ através de $S$. Para superfícies fechadas, use a orientação positiva (para fora).

  • ${\bf F}(x,y,z)=x{\bf i}+2y{\bf j}+3z{\bf k}$ e $S$ é o cubo com vértices $(\pm 1, \pm 1,\pm 1).$


$48.$


1949   

Calcule a integral de linha $\int_{C}{\bf F}\cdot d{\bf r}$, onde $C$ é dada pela função vetorial ${\bf r}(t).$

${\bf F}(x,y)=xy\,{\bf i}+3y^{2}\,{\bf j}$, ${\bf r}(t)=11t^{4}\,{\bf i}+t^{3}\,{\bf j}$, $0\leq t\leq 1.$


$45.$


2892   

Passe para coordenadas polares e calcule: $\displaystyle\int_{0}^{\ln 2}  \int_{0}^{\sqrt{(\ln 2)^{2}-y^{2}}}e^{\sqrt{x^{2}+y^{2}}}\,dx dy$


$\displaystyle \frac{\pi(2\ln(2) - 1)}{2}.$


2796   

Determine os valores máximos e mínimos locais e pontos de sela da função $f(x,y)=\sqrt[3]{x^{2}+2xy+4y^{2}-6x-12y}$.


Ponto de mínimo: $\displaystyle \left( 2,1\right).$


2165   

Use o Teorema de Green para calcular $\int_{C}\mathbf{F} \cdot d\mathbf{r}$, onde $\mathbf{F}(x,y) = (e^x+x^2y,e^y-xy^2)$, $C$ é a circunferência $x^2+y^2=25$, orientada no sentido horário. (Verifique a orientação da curva antes de aplicar o Teorema.)


$\dfrac{625\pi}{2}.$


2151   

Use o Teorema do Divergente para calcular o fluxo de ${\bf F}$ através de $S,$ onde ${\bf F}(x,y,z)=(\cos z+xy^{2})\,{\bf i}+xe^{-z}\,{\bf j}+(\sin y+x^{2}z)\,{\bf k}$ e $S$ é a superfície do sólido limitado pelo parabolóide $z=x^{2}+y^{2}$ e pelo plano $z=4.$


2980   

Determine a imagem do conjunto $S$ sob a transformação dada. $S$ é a região triangular com vértices $(0,0), (1,1), (0,1)$;$x = u^2$, $y = v$.


A região limitada pela reta $y = 1,$ pelo eixo $y$ e por $y = \sqrt{x}.$


1965   

Verifique que
$$\int_{C} P\,dx+Q\,dy=\iint\limits_{B}\bigg(\frac{\partial Q}{\partial x}-\frac{\partial P}{\partial y}\bigg)\,dxdy,$$

onde $B$ é o triângulo de vértices $(0,0)$, $(1,0)$ e $(1,1)$, $C$ é a fronteira de $B$ orientada no sentido anti-horário, $P(x,y)=x^{2}-y$ e $Q(x,y)=x^{2}+y.$


$\displaystyle \int_{C} P\,dx+Q\,dy = \dfrac{7}{6} =  \iint\limits_{B}\bigg(\frac{\partial Q}{\partial x}-\frac{\partial P}{\partial y}\bigg)\,dxdy.$


2198   

Encontre $\partial w/ \partial r$ quando $r=1$, $s=-1$ se $w=(x+y+z)^{2}$, $x=r-s$, $y=\cos(r+s)$, $z=\sin(r+s).$



$\displaystyle \frac{\partial w}{\partial r}(x(1,-1),y(1,-1),z(-1,1)) = 12.$


3080   

Mostre que os limites não existem, considerando que \((x,y)\rightarrow (0,0) \) ao longo dos eixos coordenados.

  1.  \[ \lim_{(x,y)\to(0,0)}\dfrac{x-y}{x^2+y^2} \]

  2.  \[ \lim_{(x,y)\to(0,0)}\dfrac{\cos(xy)}{x^2+y^2} \]


2739   

$2x + y + 3z = 6$ é a equação do plano tangente ao gráfico de $f(x,y)$ no ponto $(1,1,1)$.

  1. Calcule $\dfrac{\partial f}{\partial x}(1,1)$ e $\dfrac{\partial f}{\partial y}(1,1)$.

  2. Determine a equação da reta normal no ponto $(1,1,1).$


  1. $\displaystyle \frac{\partial f}{\partial x} (1,1) = -\frac{2}{3}$ e  $\displaystyle \frac{\partial f}{\partial y} (1,1) = -\frac{1}{3}.$

  2. $(x,y,z) = (1,1,1) + \lambda (2,1,3)$.


2099   

Determine o trabalho realizado pelo campo de força ${\bf F}(x,y)=2y^{3/2}\,{\bf i}+3x\sqrt{y}\,{\bf j}$ ao mover um objeto de $P(1,1)$ a $Q(2,4).$


$30.$


2800   

Determine os valores máximos e mínimos locais e pontos de sela da função $f(x,y)=x^{5}+y^{5}-5x-5y$.


Ponto de mínimo: $\displaystyle \left( 1,1\right);$ ponto de máximo: $\displaystyle \left( -1,-1\right);$ pontos de sela: $\displaystyle \left(1,-1\right)$ e $\displaystyle \left(-1,1\right).$


2429   

Calcule a integral tripla.

  1.  $\displaystyle\iiint\limits_{  E}x\;dx dy dz$, onde $E$ é o conjunto $0\leq x \leq 1$, $0\leq y \leq 1$ e \\ $x+y\leq z \leq x+y+1.$

  2.  $\displaystyle\iiint\limits_{  E}\sqrt{1-z^{2}}\;dx dy dz$, onde $E$ é o conjunto $0 \leq x \leq 1$, $0\leq z\leq 1$ e $0\leq y \leq z.$


  1.  $\dfrac{1}{2}.$

  2.  $\dfrac{1}{3}.$


3091   

Encontre todos os extremos relativos de \(x^2y^2\) sujeitos à restrição \(4x^2+y^2=8\). Faça-o de duas maneiras: primeiro, usando restrições para eliminar uma variável e, em seguida, utilizando multiplicadores de Lagrange como variáveis auxiliares.


Ocorre máximo absoluto de \(4\) em \((\pm 1,\pm 2)\); mínimo absoluto de valor \(0\) em \((\pm\sqrt{2},0)\) e \((0,\pm 2\sqrt{2})\).


2576   

Seja $f(x,y) = \dfrac{2xy^2}{x^2 + y^4}$.

  1. Considere a reta $\gamma(t) = (at, bt)$, com $a^2 + b^2 > 0$; mostre que, quaisquer que sejam $a$ e $b$,

    $$\displaystyle \lim_{t \to 0} f(\gamma(t)) = 0.$$

    Tente visualizar este resultado através das curvas de nível de $f$.

  2. Calcule $\displaystyle \lim_{t \to 0} f(\delta(t))$, onde $\delta(t) = (t^2,t).$ (Antes de calcular o limite, tente prever o resultado olhando para as curvas de nível de $f$.)

  3. $\displaystyle \lim_{(x,y) \to (0,0)}\dfrac{2xy^2}{x^2 + y^4}$ existe? Por quê?


  1. Demonstração.

  2. $1.$

  3. Não existe.


2439   

Calcule a integral de superfície $\displaystyle\iint\limits_{S}yz dS$, onde $S$ é a superfície com equações paramétricas $x=u^{2}$, $y=u \sin v$, $z=u\cos v$, $0 \leq u \leq 1$, $0 \leq v \leq \pi/2.$


$\dfrac{5\sqrt{5}}{48} + \dfrac{1}{240}.$


2644   

Use a definição de derivadas parciais como limites para encontrar $f_{x}(x,y)$ e $f_{y}(x,y)$, sendo $f(x,y)=x^{2}y-x^{3}y$.


$\displaystyle f_{x} = y^{2} - 3x^{2}y \;\;\;\text{e}\;\;\; f_{y} = 2xy - x^{3}$.


2637   

As seguintes superfícies, rotuladas $a$, $b$ e $c$ de cima para baixo, são gráficos de uma função $f$ e de suas derivadas parciais $f_{x}$ e $f_{y}$. Identifique cada superfície e dê razões para sua escolha.

ma211-list3-ex7.png


a) $f_{y},$ b) $f_{x},$ c) $f$.


2146   

Aplique o Teorema da Divergência para achar $\displaystyle\iint \limits_{S}{\bf F}\cdot {\bf n}\,dS,$  sendo ${\bf F}(x,y,z)=(x^{2}+\sin yz)\,{\bf i}+(y-xe^{-z})\,{\bf j}+z^{2}\,{\bf k}$ e $S$ a superfície da região delimitada pelo cilindro $x^{2}+y^{2}=4$ e os planos $x+z=2$ e $z=0.$


$20\pi.$


2917   

Determine a imagem do conjunto $S$ sob a transformação dada. $S$ é o disco dado por $u^2 + v^2 \leq 1$;$x = au$, $y = bv$.



Suponha $a$ e $b$ não-nulos. Por essa mudança de coordenadas, temos que $u = x/a$ e $v = y/b$. Substituindo na equação dada, obtemos

$$\frac{x^2}{a^2}+\frac{y^2}{b^2} \leq 1,$$

isto é, o disco $S$ é transformado em uma elipse.



2228   

Determine se o campo vetorial $\mathbf{F}(x,y,z) = y\cos{xy}\mathbf{i} + x\cos{xy}\mathbf{j} - \sin{z}\mathbf{k}$ é conservativo ou não. Se for conservativo, determine uma função $f$ tal que $\mathbf{F} = \nabla{f}$.


$\mathbf{F}$ é conservativo. $f(x,y,z) = \sin(xy) + \cos(z).$


2381   

Calcule a integral iterada.

  1. $\displaystyle\int_{1}^{3} \!\! \int_{0}^{1}(1+4xy)\, dx dy$

  2. $\displaystyle\int_{2}^{4}\!\!\int_{-1}^{1}(x^{2}+y^{2})\,dy dx$


  1.  $10.$

  2.  $\dfrac{116}{3}.$


2474   

Determine e faça o esboço do domínio da função $f(x,y)=\ln(9-x^{2}-9y^{2})$.


$\left\lbrace (x,y);\; \frac{x^{2}}{9} + y^{2} < 1 \right\rbrace.$


ma211-list2-ex10_sol_b.png


2089   

Calcule $\int_{C}2x\,\cos y\,dx-x^{2}\,\sin y\,dy$ ao longo dos caminhos $C$ a seguir no plano $xy.$

  1. A parabóla $y=(x-1)^{2}$ de $(1,0)$ a $(0,1).$

  2. O segmento de reta de $(-1,\pi)$ a $(1,0).$

  3. O eixo $x$ de $(-1,0)$ a $(1,0).$

  4. O astróide ${\bf r}(t)=(\cos^{3} t)\,{\bf i}+(\sin^{3}t)\,{\bf j}$, $0\leq t\leq 2\pi$, no sentido anti-horário de $(1,0)$ de volta a $(1,0).$


  1. $-1.$

  2. $2.$

  3. $0.$

  4. $0.$


2365   

Existe uma direção $\bf{u}$ na qual a taxa de variação de $f(x,y) = x^2 - 3xy + 4y^2$ em $P = (1,2)$ é igual a 14? Justifique sua resposta.


 Não, já que $|\nabla f(1,2)| = \sqrt{185} < 14.$


2934   

Uma piscina circular tem diâmetro de 10 metros. A profundidade é constante ao longo das retas de leste a oeste e cresce linearmente de 1 metro na extremidade sul para dois metros na extremidade norte. Encontre o volume de água da piscina.


$1800 \pi$ m$^3.$


3070   

Experiências mostram que uma corrente contínua $I$ em um fio comprido produz um campo magnético ${\bf B}$ que é tangente a qualquer círculo em um plano perpendicular ao fio cujo centro seja o eixo do fio (como na figura). A Lei de Ampère relaciona a corrente elétrica ao campo magnético criado e afirma que

$$\int_{C}{\bf B}\cdot d{\bf r}=\mu_{0}I,$$
onde $I$ é a corrente total que passa por qualquer superfície limitada por uma curva fechada $C$ e $\mu_{0}$ é uma constante, chamada permeabilidade no vácuo. Tomando $C$ como um círculo de raio $r$, mostre que o módulo $B=|{\bf B}|$ do campo magnético a uma distância $r$ do centro do fio é dado por 
$$B=\frac{\mu_{0}I}{2\pi r}.$$

ma211-list10-ex39.png


Note que $\textbf{B}$ é tangente a qualquer círculo que está no plano perpendicular ao fio. Logo, $\textbf{B} = |\textbf{B}| \textbf{T},$ onde $\textbf{T}$ é a tangente unitária ao círculo $\textbf{C}$ parametrizado por $x = r \cos(\theta),$ $y = r \sin(\theta).$ Daí,
$\textbf{B} = |\textbf{B}| \left(-\sin(\theta),\cos(\theta) \right)$ e
$$\int_{C} \textbf{B} \cdot d\textbf{r} = \int_{0}^{2\pi} |\textbf{B}| \left( -\sin(\theta), \cos(\theta)\right)\cdot (\left(-r \sin(\theta), r\cos(\theta) \right)  d\theta = 2\pi r |\textbf{B}|.$$


2414   

Inverta a ordem de integração, integrando primeiro em $y$ e depois em $x$ para calcular a integral:

  1.  $\displaystyle\int_{0}^{1}\!\!\int_{\sqrt{y}}^{1}\sqrt{x^{3}+1}\,dx dy$

  2.  $\displaystyle\int_{0}^{1}\!\!\int_{\sqrt{y}}\sin{x^{3}}\,dx dy$


  1.  $\dfrac{2(2\sqrt{2} - 1)}{9}.$

  2.  $\dfrac{2}{3} \sin^{2}\left(\dfrac{1}{2} \right).$


2568   

Calcule $\displaystyle \lim_{(x,y) \to (0,0)} x \ \sin{\dfrac{1}{x^2 + y^2}}$, caso exista.


$0.$


2935   

Utilize o resultado $\displaystyle \int_{-\infty}^{\infty}e^{-x^{2}}\,dx=\sqrt{\pi}$  para calcular as integrais:

  1. $\displaystyle\int_{0}^{\infty}   x^{2}e^{-x^{2}}\,dx$

  2. $\displaystyle\int_{0}^{\infty}\sqrt{x}e^{-x}\,dx$


  1. $\displaystyle \frac{\sqrt{\pi}}{4}.$

  2. $\displaystyle \frac{\sqrt{\pi}}{2}.$


2920   

Utilize a integral dupla para determinar a área da região: um laço da rosácea $r=\cos(3\theta).$


$\displaystyle \frac{\pi}{12}.$


2582   

Considere a função

$$f(x,y) = \begin{cases}\dfrac{x^2 - xy}{x^2 + y^2}, & \quad \text{se } (x,y) \neq (0,0), \\0, & \quad \text{se } (x,y) = (0,0).\end{cases}$$

  1. Calcule o limite $\displaystyle \lim_{(x,y) \to (0,0)}f(x,y)$ ou mostre que esse limite não existe.

  2. Calcule o limite $\displaystyle \lim_{(x,y) \to (1,1)}f(x,y)$ ou mostre que esse limite não existe.

  3. $f$ é contínua em $(0,0)$? Justifique.

  4. $f$ é contínua em $(1,1)$? Justifique.



  1. O limite não existe.

  2. $0.$

  3. Não.

  4. Sim.


2163   

Use o Teorema de Green para calcular a integral de linha ao longo da curva dada com orientação positiva. $\displaystyle\int_{C}\dfrac{-y}{x^2+y^2} \, dx + \dfrac{ x}{x^2+y^2} \, dy$, $C$ curva fechada, $C^1$ por partes, simples e fronteira de um conjunto $B$ cujo interior contém o círculo $x^2 + y^2 \leq 1$. (Sugestão: Aplique o Teorema de Green à região $K$ compreendida entre a curva $C$ e a circunferência.)



$2\pi.$


3110   

A parte da superfície \[ z= \dfrac{h}{a}\sqrt{x^2+y^2}\quad\left(a,\ h>0\right) \] entre o plano \(xy\) e o plano \(z=h\) é um cone circular reto de altura \(h\) e raio \(a\). Use uma integral dupla para mostrar que a área da superfície lateral desse cone é dada por \(\displaystyle S=\pi a\sqrt{a^2+h^2}\).


2468   

Calcule a integral de superfície $\displaystyle\iint \limits_{ S}{\bf F}\cdot d{\bf S}$ para o campo vetorial ${\bf F}$ e superfície orientada $S$ dados abaixo. Em outras palavras, determine o fluxo de ${\bf F}$ através de $S$. Para superfícies fechadas, use a orientação positiva (para fora).

  • ${\bf F}(x,y,z)=(x^{2}+z){\bf i}+y^{2}z{\bf j}+(x^{2}+y^{2}+z){\bf k}$ e $S$ é a parte no primeiro octante do parabolóide $z=x^{2}+y^{2}$ intersectada pelo plano $z=4.$


$4\pi - \dfrac{320}{7}.$


2572   

Calcule $\displaystyle \lim_{(x,y) \to (0,0)} \dfrac{xy(x - y)}{x^4 + y^4}$, caso exista.


Não existe.


2004   

Um arame fino é entortado no formato da semicircunferência $x^{2}+y^{2}=4$, $x\geq 0$. Se a densidade linear for uma constante $k$, determine a massa e o centro de massa do arame.


Massa: $k2\pi;$ centro de massa: $\displaystyle \left( \frac{4}{\pi},0 \right).$


2561   

Determine $h(x,y) = g(f(x,y))$ e o conjunto no qual $h$ é contínua, em que

$$g(t) = t^2 + \sqrt{t}, \ \ \ f(x,y) = 2x + 3y - 6.$$


$h(x,y) = (2x+3y-6)^{2} + \sqrt{2x + 3y - 6}$ é contínua em $\left\lbrace (x,y);\; y \geq -\frac{2x}{3} + 2 \right\rbrace.$   


3002   

A integral $\int \!\!\! \int\limits_{\!\!\!\!\!R} \! \sqrt{9 - y^2} \, dA$, em que $R = [0,4] \times [0,2]$, representa o volume de um sólido. Esboce o sólido.


ma211-list6-ex9_sol.png

2070   

Utilize as Equações 

$\dfrac{\partial z}{\partial x}=-\dfrac{\dfrac{\partial F}{\partial x}}{\dfrac{\partial F}{\partial z}}$ e $\dfrac{\partial z}{\partial y}=-\dfrac{\dfrac{\partial F}{\partial y}}{\dfrac{\partial F}{\partial z}}$

para determinar $\partial z/\partial x$ e $\partial z/\partial y$.

$yz=\ln(x+z)$


$\displaystyle \frac{dz}{dx} = \frac{1}{y(x+z)-1}$ e $\displaystyle \frac{dz}{dy} = \frac{z(x+z)}{y(x+z)-1}.$


2323   

Calcule a área da superfície dada por: ${\bf r}(u,v)=(u,v,2-u-v)$ e $u^{2}+v^{2}\leq 1.$. (Sugerimos ao leitor desenhar a imagem da superfície.)


$\pi \sqrt{3}.$


2402   

 Determine as equações paramétricas da reta tangente à curva formada pela intersecção do paraboloide $z = x^2 + y^2$ com o elipsoide $4x^2 + y^2 + z^2 = 9$ no ponto $(-1,1,2)$.


 $(x,y,z) = (-1,1,2) + \lambda (-10, -16, -12),$ $\lambda \in \mathbb{R}.$


3099   

Mostre (verifique) que as integrais abaixo podem ser calculadas como:


1. \[ \int_1^5\int_2^{y/2}6x^2y\,dxdy = \int_1^5\left(\dfrac{1}{4}y^4-16y\right)\,dy \]


2. \[ \int_1^5\int_2^{x/2}6x^2y\,dydx = \int_1^5\left(\dfrac{3}{4}x^4-12x^2\right)\,dx \]



2665   

Determine as derivadas parciais de $z=\dfrac{x\sin{y}}{\cos(x^{2}+y^{2})}$.


$\begin{aligned}[t]\frac{\partial z}{\partial x} &= \frac{\sin y ( \cos(x^{2} + y^{2}) + 2x^{2} \sin(x^{2} + y^{2}))}{(\cos(x^{2} + y^{2}))^{2}}\;\;\;\;\;\;\text{e}\\\frac{\partial z}{\partial y} &= \frac{x \cos y \cos(x^{2} + y^{2}) + 2xy \sin y \sin(x^{2} + y^{2})}{(\cos(x^{2} + y^{2}))^{2}}.\end{aligned}$


2807   

Determine os valores máximos e mínimos locais e pontos de sela da função $f(x,y)=x^{3}+2xy+y^{2}-5$.


Ponto de mínimo : $\displaystyle \left( \frac{5}{3}, -\frac{5}{3}\right);$ ponto de sela: $\displaystyle \left(-1,1\right).$


2046   

Determine se ${\bf F}(x,y)=(ye^{x}+\sin y)\,{\bf i}+(e^{x}+x\,\cos y)\,{\bf j}$ é ou não um campo vetorial conservativo. Se for, determine uma função $f$ tal que ${\bf F}=\nabla f.$



Primeiramente, temos que o domínio de ${\bf F}$ é todo o $\mathbb{R}^{2}$, o qual é uma região aberta e simplesmente conexa. Sendo $P(x,y)=ye^{x}+\sin y$ e $Q(x,y)=e^{x}+x\,\cos y$, temos que $P$ e $Q$ possuem derivadas de primeira ordem contínuas. Também temos que

$$\frac{\partial P}{\partial y}=e^{x}+\cos y    \,\,\,  \text{   e   }   \,\,\,   \frac{\partial Q}{\partial x}=e^{x}+\cos y,$$

ou seja,

$$\frac{\partial P}{\partial y}=\frac{\partial Q}{\partial x}.$$

Assim, das condições acima verificadas, temos que ${\bf F}$ é um campo conservativo. Agora, vamos determinar $f$ tal que $\nabla f={\bf F}.$ Isto é, devemos encontrar $f$ tal que

$$f_{x}(x,y)=P(x,y)      \text{   e   }      f_{y}(x,y)=Q(x,y).$$

Como $f_{x}(x,y)=P(x,y)$ temos que

$$f_{x}(x,y)=ye^{x}+\sin y\Rightarrow f(x,y)=ye^{x}+x\,\sin y+g(y)$$

Assim obtemos que

$$f_{y}(x,y)=e^{x}+x\cos y+g'(y)$$

Mas, $f_{y}(x,y)=Q(x,y)$ logo obtemos que

$$e^{x}+x\cos y+g'(y)=e^{x}+x\,\cos y\Rightarrow g'(y)=0\Rightarrow g(y)=C.$$

Portanto,

$$f(x,y)=ye^{x}+x\sin y+C      \text{   e   }      \nabla f={\bf F}.$$


2704   

Considere a função

$$f(x,y)=\begin{cases}x+y, & \quad \text{se } xy=0,\\\kappa, & \quad \text{caso contrário},\\\end{cases}$$

em que $\kappa$ é um número real. Determine as derivadas parciais de primeira ordem de $f$ em $(0,0).$


$\displaystyle \frac{\partial f}{\partial x} (0,0) = \frac{\partial f}{\partial y} (0,0) = 1$.


2617   

Seja $C$ uma curva fechada, simples e lisa que está no plano $x+y+z=1$. Mostre que a integral de linha $\displaystyle\int_C zdx - 2xdy + 3ydz$ depende apenas da área da região englobada por $C$ e não da forma de $C$ ou de sua posição no plano.


$\displaystyle\int_C zdx - 2xdy + 3ydz = \dfrac{2}{\sqrt{3}} \times $ (área da região englobada por $C$).


1959   

Calcule a integral de linha $\displaystyle\int_{C}{\bf F}\cdot d{\bf r}$, onde ${\bf F}(x,y)=(y,3x)$ e $C$ é a elipse $\dfrac{x^{2}}{a^{2}}+\dfrac{y^{2}}{b^{2}}=1$, percorrida no sentido anti-horário.


$-2\pi ab.$


2236   

Seja $\mathbf{r} = x\mathbf{i} + y\mathbf{j} + z\mathbf{k}$ e $r=|\mathbf{r}|$. Verifique a identidade $\nabla\left(\dfrac{1}{r}\right) = -\dfrac{\mathbf{r}}{r^3}$.


$\nabla\left(\dfrac{1}{r}\right) =- \dfrac{\frac{1}{2\sqrt{x^2 + y^2 + z^2}} (2x)}{x^2 + y^2 + z^2} \mathbf{i} - \dfrac{-\frac{1}{2\sqrt{x^2 + y^2 + z^2}} (2y)}{x^2 + y^2 + z^2} \mathbf{j} - \dfrac{-\frac{1}{2\sqrt{x^2 + y^2 + z^2}} (2z)}{x^2 + y^2 + z^2} \mathbf{k}.$ (Note que: $r = \sqrt{x^{2} + y^{2} + z^{2}}.$)


2135   

Mostre que cada a equação a seguir define implicitamente pelo menos uma função diferenciável $z=z(x,y)$. 
Expresse $\partial z /\partial x$ e $\partial z/\partial y$ em termos de $x$, $y$ e $z.$
$e^{x+y+z}+xyz=1$


 $\displaystyle \frac{\partial z}{\partial x} = - \frac{e^{x + y + z} + yz}{e^{x + y + z} + xy}$ e $\displaystyle \frac{\partial z}{\partial y} = - \frac{e^{x + y + z} + xz}{e^{x + y + z} + xy}.$


1991   

Calcule $\dfrac{\mathrm{d}{\bf r}}{\mathrm{d}t}$ e $\dfrac{\mathrm{d}^{2}{\bf r}}{\mathrm{d}t^{2}}.$

  1. ${\bf r}(t)=(3t^{2},e^{-t},\ln(t^{2}+1))$
  2. ${\bf r}(t)=\sqrt[3]{t^{2}}{\bf i}+\cos(t^{2}){\bf j}+3t{\bf k}$
  3. ${\bf r}(t)=\sin(5t){\bf i}+\cos(4t){\bf j}-e^{-2t}{\bf k}$


1946   

Calcule a integral de linha, onde $C$ é a curva dada.

$\displaystyle\int_{C}2\,dx-dy$, $C$ tem por imagem $x^{2}+y^{2}=4$, $x\geq 0$ e $y\geq 0$; sentido de percurso é de $(2,0)$ para $(0,2).$


$\displaystyle -6.$


3106   

Encontre a área da superfície descrita como sendo a parte do cilindro \(y^2+z^2=9\) que está acima do retângulo \(\displaystyle R=\{(x,y)\in\mathbb{R}^2;\ 0\leq x\leq 2,\ -3\leq y\leq 3\}\).


\( 6\,\pi\)


2710   

Determine o plano que é paralelo ao plano $z = 2x + 3y$ e tangente ao gráfico de $f(x,y) = x^2 + xy$.


Considere

$$z-f(x_{0},y_{0})=\frac{\partial f}{\partial x}(x_{0},y_{0})(x-x_{0})+\frac{\partial f}{\partial y}(x_{0},y_{0})(y-y_{0})$$

o plano tangente ao gráfico de $f$. Assim,

$$z=\frac{\partial f}{\partial x}(x_{0},y_{0})\cdot x+\frac{\partial f}{\partial y}(x_{0},y_{0})\cdot y+\bigg[ f(x_{0},y_{0})-\frac{\partial f}{\partial x}(x_{0},y_{0})\cdot x_{0}-\frac{\partial f}{\partial y}(x_{0},y_{0})\cdot y_{0}\bigg].$$

Como tal plano é paralelo ao plano $z=2x+3y$, obtemos que

$$\frac{\partial f}{\partial x}(x_{0},y_{0})=2\;\;\;\;\;\;\; \mbox{e}\;\;\;\;\;\;\; \frac{\partial f}{\partial y}(x_{0},y_{0})=3.$$

Notemos que

$$\frac{\partial f}{\partial x}(x,y)=2x+y\;\;\;\;\;\;\; \mbox{e} \;\;\;\;\;\; \frac{\partial f}{\partial y}(x,y)=x.$$

Assim, temos o seguinte sistema de equações

$$\left \{\begin{array}{cc}2x_{0}+y_{0}=2 \\x_{0}=3\\\end{array}\right.$$

Logo, $x_{0}=3$ e $y_{0}=-4.$ A partir desses valores temos que $f(x_{0},y_{0})=-3$, $\dfrac{\partial f}{\partial x}(x_{0},y_{0})\cdot x_{0}=6$ e

$\dfrac{\partial f}{\partial y}(x_{0},y_{0})\cdot y_{0}=-12.$ Portanto, o plano desejado tem equação

$$z=2x+3y-3-6+12,$$

ou seja,

$$z=2x+3y+3.$$


2077   

Calcule a integral dupla.

  1. $\displaystyle\iint\limits_{R} (6x^{2}y^{3}-5y^{4})\,dA, \quad R=\{(x,y) \in \mathbb{R}^2:0\leq x\leq 3,\;0\leq y\leq 1\}.$

  2. $\displaystyle\iint\limits_{R} \dfrac{xy^{2}}{x^{2}+1}\,dA, \quad R=\{(x,y)\in \mathbb{R}^2:0\leq x\leq 1,\;-3\leq y\leq 3\}.$


  1. $\dfrac{21}{2}.$

  2. $9 \ln(2).$


2979   

Determine a imagem do conjunto $S$ sob a transformação dada. $S$ é o quadrado limitado pelas retas $u = 0$, $u = 1$, $v = 0$, $v = 1$;$x = v$, $y = u(1+v^2)$.



A região limitada por $y = 1 + x^2,$ pelo eixo $x$ e pelas retas $x = 0$ e $x = 1.$


2898   

Determine os pontos da elipse $\mathcal{D} = \left\{ (x,y) \in \mathbb{R}^2: \frac{x^2}{8} + \frac{y^2}{2} = 1 \right\}$ que fornecem o maior e o menor valor da função $f(x,y) = xy$.


Pontos de máximo: $(2,1)$ e $(-2,-1);$ pontos de mínimo: $(-2,1)$ e $(2,-1).$


2278   

Calcule $\nabla f(x,y)$.

$f(x,y) = \arctan{\dfrac{x}{y}}$


 $\displaystyle \nabla f(x,y) = \left(\frac{y }{x^{2} + y^{2}}, -\frac{x}{x^{2} + y^{2}} \right).$



2282   

Defina gradiente de uma função de três variáveis. Calcule $\nabla f(x,y,z)$.

$f(x,y,z) = z \arctan{\dfrac{x}{y}}$


$\displaystyle \nabla f(x,y,z) = \left(\frac{yz}{x^{2} + y^{2}},-\frac{xz}{x^{2} + y^{2}},\arctan\left(\frac{x}{y}\right) \right).$


2507   

Faça o mapa de contorno da função $f(x,y)=y-\ln{x}$ mostrando várias de suas curvas de nível.


$y = \ln(x) + C.$

ma211-list2-ex17_sol_b.png


2451   

Integre $g(x,y,z)=x+y+z$ sobre a superfície do cubo cortado do primeiro octante pelos planos $x=a$, $y=a$ e $z=a.$


$9a^3.$


3026   

Calcule $\int_{0}^{1}\!\int_{x}^{1}3y^{4}\cos(xy^{2})\,dy dx$. Esboce a região de integração.


$1 - \cos(1).$

ma211-list6-ex26_sol.png


2621   

Suponha que $S$ e $C$ satisfaçam as hipóteses do Teorema de Stokes e $f$ e $g$ tenham derivadas parciais de segunda ordem contínuas. Demonstre que $\displaystyle\int_C (f\nabla g + g\nabla f) \cdot d{\bf R} = 0$


Note que $\mbox{rot} (f\nabla g + g\nabla f) = {\bf 0}.$


2160   

Use o Teorema de Green para calcular a integral de linha ao longo da curva dada com orientação positiva. $\displaystyle\int_{C}e^y \, dx + 2xe^y \, dy$, $C$ é o quadrado de lados $x=0$, $x=1$, $y=0$ e $y=1$.


$e - 1.$


2643   

Determine a derivada parcial $f_{x}(3,4)$, onde $f(x,y)=\ln(x+\sqrt{x^{2}+y^{2}}).$


$f_{x}(3,4) = \frac{1}{5}$.


3068   

Calcule a integral de linha $\displaystyle\int_{C}\sqrt[3]{x}\,dx+\dfrac{dy}{1+y^{2}}$, onde $C$ é a curva na figura abaixo.

ma211-list10-ex16_a.png


$0.$


2577   

Determine se a função

$$f(x,y) = \begin{cases}e^{\left( \dfrac{1}{x^2 + y^2 - 1} \right)}, & \quad \text{se } x^2 + y^2 < 1, \\0, & \quad \text{se } x^2 + y^2 \geq 1.\end{cases}$$

é contínua em $\displaystyle{\left( \frac{\sqrt{2}}{2}, \frac{\sqrt{2}}{2} \right)}$. Justifique sua resposta.


$0.$


2427   

Calcule a integral tripla.

  1.  $\displaystyle\iiint\limits_{  E} x^2 e^y\,dV$, onde $E$ é delimitado pelo cilindro parabólico $z=1-y^{2}$ e pelos planos $z=0$, $x=1$ e $x=-1.$

  2.  $\displaystyle\iiint\limits_{  E}x\,dV$, onde $E$ é limitado pelo paraboloide $x=4y^{2}+4z^{2}$ e  pelo plano $x=4.$


  1.  $\dfrac{8}{3e}.$

  2.  $\dfrac{16\pi}{3}.$


3125   

Determine a equação do plano tangente à superfície descrita parametricamente por \(x=u\cosh v\), \(y=u\sinh v\), \(z=u^2\) no ponto \((-3,0,9)\).


2266   

Calcule $\displaystyle\iint\limits_{B}f(x,y)\,dx dy$ sendo dados:

  1.  $f(x,y)=1$ e $B$ a região compreendida entre os gráficos de $y=\sin{x}$ e $y=1-\cos{x}$, com $0\leq x\leq \dfrac{\pi}{2}.$

  2.  $f(x,y)=\sqrt{1+y^{3}}$ e $B=\{(x,y)\in \mathbb{R}^{2}|\;\sqrt{x}\leq y\leq 1 \}.$

  3.  $f(x,y)=x$ e $B$ é o conjunto de todos $(x,y)$ tais que $y\geq x^{2}$ e   $x\leq y\leq x+2.$

  4.  $f(x,y)=\dfrac{y}{x+y^{2}}$ e $B$ o conjunto de todos $(x,y)$ tais que $1\leq x\leq 4$ e $0\leq y\leq \sqrt{x}.$


  1.  $2 - \dfrac{\pi}{2}.$

  2.  $\dfrac{2(2\sqrt{2} - 1)}{9}.$

  3.  $\dfrac{13}{6}.$

  4.  $\dfrac{3 \ln(2)}{2}.$


1982   

  1.  Faça um esboço grande da curva descrita pela função vetorial ${\bf r}(t)=(t^{2},t)$, $0\leq t\leq 2$, e desenhe os vetores ${\bf r}(1)$, ${\bf r}(1,1)$ e ${\bf r}(1,1)-{\bf r}(1)$.

  2. Desenhe o vetor ${\bf r}(1)$ começando em $(1,1)$ e compare com o vetor $$\frac{{\bf r}(1,1)-{\bf r}(1)}{0,1}.$$


2867   

Estude com relação a máximos e mínimos a função dada com as restrições dadas.

$f(x,y) = x^2 - 2y^2$ e $x^2 + y^2 - 2x = 0.$


Ponto de máximo: $\displaystyle \left( 2,0 \right)$; pontos de mínimo: $\displaystyle \left( \frac{2}{3}, \frac{2\sqrt{2}}{3} \right)$ e $\displaystyle \left( \frac{2}{3}, \frac{-2\sqrt{2}}{3} \right)$.


2521   

Esboce o gráfico da função $f(x,y)=\sqrt{x^{2}+y^{2}}$. Em geral, se $g$ é uma função de uma variável, como saber o gráfico de $f(x,y)=g(\sqrt{x^{2}+y^{2}})$ a partir do gráfico de $g$?


O gráfico de $f(x,y) = g(\sqrt{x^{2} + y^{2}})$ pode ser obtido rotacionando o gráfico de $g$ no plano $xz$ ao redor do eixo $z.$

ma211-list2-ex22_sol_a.png


2354   

Calcule o volume do conjunto dado.

  1.  $x^{2}+y^{2}\leq a^{2}$ e $y^{2}+z^{2}\leq a^{2}$, $a >0.$

  2.  $x^{2}+y^{2}\leq z\leq 1-x^{2}.$


  1.  $\dfrac{16a^{3}}{3}.$

  2.  $\dfrac{\pi}{2\sqrt{2}}.$




2928   

Esboce o sólido descrito por $\rho \leq 2$, $0\leq \phi \leq \pi/2$ e $0\leq \theta \leq \pi/2.$



ma211-list9-ex9_sol.png


2516   

Encontre uma equação para a superfície de nível da função $f(x,y)=\ln (x^{2}+y^{2}+z^{2})$ que passa pelo ponto $(-1,2,1)$.


$x^{2} + y + z^{2} = 6.$


2856   

Determine as dimensões da caixa retangular de volume máximo, com faces paralelas aos planos coordenados, que possa ser inscrita no elipsóide $16x^{2}+4y^{2}+9z^{2}=144.$


$\displaystyle \frac{8}{\sqrt{3}} \times \frac{6}{\sqrt{3}} \times \frac{12}{\sqrt{3}}.$


2418   

Calcule a área limitada pelas curvas $x=y^{2}-1$ e $x=2y^{2}-2.$


$\dfrac{4}{3}.$


2438   

Calcule a integral de superfície $\displaystyle\iint\limits_{S}yz dS$, onde $S$ é a parte do plano $x+y+z=1$ que está no primeiro octante.


$\dfrac{\sqrt{3}}{24}.$


2869   

Estude com relação a máximos e mínimos a função dada com as restrições dadas.

$f(x,y) = x^2 - 2xy + 3y^2$ e $x^2 + 2y^2 = 1.$


Pontos de máximo: $\displaystyle \left( \frac{1}{\sqrt{3}}, -\frac{1}{\sqrt{3}} \right)$ e $\displaystyle \left( -\frac{1}{\sqrt{3}}, -\frac{1}{\sqrt{3}} \right)$; pontos de mínimo:  $\displaystyle \left( \frac{2}{\sqrt{6}}, \frac{1}{\sqrt{6}} \right)$ e $\displaystyle \left( -\frac{2}{\sqrt{6}}, -\frac{1}{\sqrt{6}} \right)$.


2127   

Seja $z=f(u+2v,u^{2}-v)$. Expresse $\partial z/\partial u$ e $\partial z/\partial v$ em termos das 

derivadas parciais de $f$. 


$\displaystyle \frac{\partial z}{\partial u}(u,v) = \frac{\partial f}{\partial x}(u + 2v,u^{2} - v) + 2u \frac{\partial f}{\partial y}(u + 2v,u^{2} - v)$ e\\ $\displaystyle \frac{\partial z}{\partial v}(u,v) = 2 \frac{\partial f}{\partial x}(u + 2v,u^{2} - v) - \frac{\partial f}{\partial y}(u + 2v,u^{2} - v).$


2184   

No item abaixo: 

  1. expresse $\partial w/\partial u$ e $\partial w/ \partial v$ como funções de $u$ e $v$, usando a Regra da Cadeia e também expressando $w$ diretamente em termos e $u$ e $v$ antes de diferenciar; 
  2. calcule $\partial w/\partial u$ e $\partial w/ \partial v$ no ponto dado $(u,v)$.

$w=xy+yz+xz$,  $x=u+v$, $y=u-v$,  $z=uv$;  $(u,v)=(1/2,1).$


  1. $\displaystyle w(u,v) = u^{2} - v^{2} + 2u^{2}v,$$\displaystyle \frac{\partial w}{\partial u}(u,v) = 2u + 4uv$ e $\displaystyle \frac{\partial w}{\partial v}(u,v) = -2v + 2u^{2}.$
  2. $\displaystyle \frac{\partial w}{\partial u}(-2,0) = 3$ e $\displaystyle \frac{\partial w}{\partial v}(-2,0) = -\frac{3}{2}.$


2489   

Esboce o gráfico da função $f(x,y)=\sqrt{x^{2}+y^{2}}$.


$z = \sqrt{x^{2} + y^{2}}$

ma211-list2-ex11_sol_f.png


3013   

A função densidade conjunta para um par de variáveis aleatórias $X$ e $Y$ é $$f(x,y) = \begin{cases} Cx(1 + y), & \quad \text{se } 0 \leq x \leq 1, \ 0 \leq y \leq 2,\\ 0, & \quad \text{caso contrário}.
\end{cases}$$

  1.  Determine a constante $C$.

  2.  Determine $P(X \leq 1, \ Y \leq 1)$.

  3.  Determine $P(X + Y \leq 1)$.


  1. $\dfrac{1}{2}.$

  2. $\dfrac{3}{8}.$

  3. $\dfrac{5}{48}$.



3087   

Suponha que a equação \(z=f(x,y)\) seja expressa na forma polar \(z=g(r,\theta)\) através da substituição \(x=r\cos\theta\) e \(y=r\sin\theta\).

  1.  Considere \(r\) e \(\theta\) como funções de \(x\) e \(y\) e use derivação implícita para mostrar que \[ \frac{\partial r}{\partial x} = \cos\theta \quad \text{e}\quad\frac{\partial\theta}{\partial x} =-\frac{\sin\theta}{r}.\]

  2.  Considere \(r\) e \(\theta\) como funções de \(x\) e \(y\) e use derivação implícita para mostrar que \[\dfrac{\partial r}{\partial y}=\sin\theta \quad \text{e}\quad \dfrac{\partial\theta}{\partial y}=\dfrac{\cos\theta}{r}.\]

  3.  Use os resultados anteriores para mostrar que \begin{align*} \dfrac{\partial z}{\partial x} & = \dfrac{\partial z}{\partial r}\cos\theta - \dfrac{1}{r}\dfrac{\partial z}{\partial\theta}\sin\theta \\ \dfrac{\partial z}{\partial y} & = \dfrac{\partial z}{\partial r}\sin\theta + \dfrac{1}{r}\dfrac{\partial z}{\partial\theta}\cos\theta\end{align*}

  4.  Use o resultado do item anterior para mostrar que \[ \left(\dfrac{\partial z}{\partial x}\right)^2 + \left(\dfrac{\partial z}{\partial y}\right)^2 = \left(\dfrac{\partial z}{\partial r}\right)^2 +\dfrac{1}{r^2}\left(\dfrac{\partial z}{\partial\theta}\right)^2. \]

  5.  Ainda usando o resultado do terceiro item, mostre que \(z=f(x,y)\) satisfaz a equação de Laplace \[ \dfrac{\partial^2z}{\partial x^2} + \dfrac{\partial^2z}{\partial y^2}= 0, \] se, e somente se, \(z=g(r,\theta)\) satisfaz a equação \[ \dfrac{\partial^2z}{\partial r^2} + \dfrac{1}{r^2}\dfrac{\partial^2z}{\partial\theta^2}+\dfrac{1}{r}\dfrac{\partial z}{\partial r} = 0. \] A última equação acima é chamada de forma polar da equação de Laplace.


3129   

Dado um campo vetorial \(\mathbf{F}\), uma curva \(C\) é chamada de linha de fluxo deste campo se \(\mathbf{F}\) for um vetor tangente a \(C\) em cada ponto ao longo de \(C\).

  1.  Sejam \(C\) uma linha de fluxo de \(\mathbf{F}(x,y)=-y\mathbf{i}+x\mathbf{j}\) e \((x,y)\) um ponto em \(C\) para o qual \(y\neq 0\). Mostre que as linhas de fluxo satisfazem a equação diferencial \[ \dfrac{dy}{dx} = -\dfrac{x}{y}. \]

  2. Resolva a equação diferencial do item anterior, por separação de variáveis, e mostre que as linhas de fluxo são círculos concêntricos centrados na origem, ou seja, da forma \(x^2+y^2=K\).


2873   

Utilize os multiplicadores de Lagrange para determinar os valores máximo e mínimo da função sujeita à(s) restrição(ões) dada(s).

$f(x,y,z) = 2x + 6y + 10z; \quad x^2 + y^2 + z^2 = 35.$  


Valor máximo: $70;$ valor mínimo: $-70.$


3046   

O campo vetorial $\mathbf{F}$ é mostrado no plano $xy$ e é o mesmo em todos os planos horizontais (em outras palavras, $\mathbf{F}$ é independente de $z$ e sua componente $z$ é 0).

  1. O $\text{div }{\mathbf{F}}$ será positivo, negativo ou nulo? Justifique.

  2. Determine se o $\text{rot }{\mathbf{F}} = 0$. Se não, em que direção rot $\mathbf{F}$ aponta?

ma211-list13-ex6_i.png


  1. Negativo.

  2. $\text{rot } \bf{F} = \bf{0}.$


2210   

Considere a integral

$$\int_{0}^{2}\int_{\frac{y}{2}}^{1}ye^{x^{3}}\,dx dy.$$

  1.  Faça um esboço da região de integração.

  2.  Calcule a integral sendo explícito se vai precisar mudar a ordem de integração.



  1. ...

  2.  $\dfrac{2(e - 1)}{3}.$


2787   

Utilize as curvas de nível da figura para predizer a localização dos pontos críticos de $f(x,y)=4+x^{3}+y^{3}-3xy$ e se $f$ tem um ponto de sela ou um máximo ou mínimo local em cada um desses pontos. Explique seu raciocínio. Em seguida, empregue o Teste da Segunda Derivada para confirmar suas predições.

ma211-list5-ex6_a.png


$f$ possui um ponto de sela em $(0,0)$ e um mínimo local em $(1,1).$



2837   

Calcule a integral dupla usando coordenadas polares: $\displaystyle\iint\limits_{R}\frac{x}{x^{2}+y^{2}}\,dA$, onde $R=\{(x,y)\in \mathbb{R}^{2}| x^{2}+y^{2}\leq 4, x\geq 1\}.$


$2\sqrt{3}.$


3119   

Use coordenadas esféricas para encontrar o volume do sólido: contido no interior do cone \(\phi=\pi/4\), entre as esferas \(\rho=1\) e \(\rho=2\).


2145   

Aplique o Teorema da Divergência para achar $\displaystyle\iint \limits_{S}{\bf F}\cdot {\bf n}\,dS,$  sendo ${\bf F}(x,y,z)=y\,\sin x\,{\bf i}+y^{2}z\,{\bf j}+(x+3z)\,{\bf k}$ e $S$ é a superfície da região delimitada pelos planos $x=\pm 1$, $y=\pm 1$ e $z=\pm 1.$


$24.$


2339   

Determine a área da superfície com equações paramétricas $x=u^{2}$, $y=uv$, $z=\dfrac{1}{2}v^{2}$, $0\leq u\leq 1$, $0\leq v\leq 2.$


$4.$


2443   

Calcule a integral de superfície $\displaystyle\iint\limits_{S}y^{2}dS$, onde $S$ é a parte da esfera $x^{2}+y^{2}+z^{2}=4$ que está dentro
do cilindro $x^{2}+y^{2}=1$ e acima do plano $xy.$


$\pi\left( \dfrac{32}{3} - 6\sqrt{3}\right).$


2071   

Seja $R$ o retângulo $1\leq x\leq 2$, $0\leq y\leq 1$. Calcule $\iint\limits_{ R} f(x,y)\,dxdy$, sendo $f(x,y)$ igual a

  1.  $x+2y$

  2.  $x-y$



  1.  $\dfrac{5}{2}.$

  2.  $1.$



2076   

Seja $R$ o retângulo $1\leq x\leq 2$, $0\leq y\leq 1$. Calcule $\iint\limits_{R} f(x,y)\,dxdy$, sendo $f(x,y)$ igual a

  1. $\dfrac{1}{(x+y)^{2}}$

  2. $\dfrac{1}{1+x^{2}+2xy+y^{2}}$


  1.  $\dfrac{3}{\pi}.$

  2.  $3\arctan(3) - 4\arctan(2) - \ln(2) + \dfrac{\ln(5)}{2} + \dfrac{\pi}{4}.$


3103   

Use uma integral dupla para calcular a área da região \(R\) entre a parábola \(y=\dfrac{1}{2}x^2\) e a reta \(y = 2x\).



Denotando por \(A(R)\) a área de \(R\), teremos que \begin{align*} A(R) & = \iint_R\,dA = \int_0^4\int_{x^2/2}^{2x}\,dydx = \int_0^4\left[y\right]_{y=x^2/2}^{2x}\,dx \\  & = \int_0^4\left(2x-\dfrac{1}{2}x^2\right)\,dx = \left[x^2-\dfrac{x^3}{6}\right]_0^4= \dfrac{16}{3}. \end{align*} De outra forma, fixando primeiro a variável \(y\), teríamos \begin{align*} A(R) & = \iint_R\,dA = \int_0^8\int_{y/2}^{\sqrt{2y}}\,dxdy = \int_0^8\left[x\right]_{x=y/2}^{\sqrt{2y}}\,dy \\  & = \int_0^8\left(2y-\dfrac{1}{2}y\right)\,dy = \left[\dfrac{2\sqrt{2}}{3}y^{3/2}-\dfrac{y^2}{4}\right]_0^8= \dfrac{16}{3}. \end{align*}


2573   

Calcule $\displaystyle \lim_{(x,y) \to (0,0)} \dfrac{x + y}{x - y}$, caso exista.


Não existe.


2607   

Use o Teorema de Stokes para calcular $\displaystyle\iint\limits_{ S}\mbox{rot}{\bf F}\cdot d{\bf S}.$

  • ${\bf F}(x,y,z) = x{\bf i} - z{\bf j} + y{\bf k}$, $S$ é a parte do plano $x+z=1$ dentro do cilindro $x^2+y^2 = 1$, com orientação para cima.


$2\pi.$


2574   

Calcule $\displaystyle \lim_{(x,y) \to (0,0)} \dfrac{xy}{y - x^3}$, caso exista.


Não existe.


2383   

Calcule a integral iterada.

  1. $\displaystyle\int_{1}^{4} \int_{1}^{2}\bigg(\dfrac{x}{y}+\dfrac{y}{x}\bigg)\,dy dx$

  2. $\displaystyle\int_{0}^{1} \int_{0}^{3}e^{x+3y}\,dx dy$



  1. $\dfrac{21}{2} \ln(2).$

  2. $\dfrac{(e^{3} - 1)^{2}}{3}.$


2524   

Esboce o gráfico da função $f(x,y)=\sin(\sqrt{x^{2}+y^{2}})$ .Em geral, se $g$ é uma função de uma variável, como saber o gráfico de $f(x,y)=g(\sqrt{x^{2}+y^{2}})$ a partir do gráfico de $g$?


O gráfico de $f(x,y) = g(\sqrt{x^{2} + y^{2}})$ pode ser obtido rotacionando o gráfico de $g$ no plano $xz$ ao redor do eixo $z.$

ma211-list2-ex22_sol_d.png


2447   

Calcule $\displaystyle\iint\limits_{S}g(x,y,z)dS,$ sendo $g(x,y,z)=x^{2}$ e $S$ o hemisfério superior de $x^{2}+y^{2}+z^{2}=a^{2}.$


$\dfrac{2\pi a^4}{3}.$


2712   

Determine uma equação do plano tangente à superfície no ponto especificado.

$z = 4x^2 - y^2 + 2y, \quad (-1,2,4)$.


$z = -8x - 2y$.


2840   

Calcule a integral dupla usando coordenadas polares: $\displaystyle\iint\limits_{R}\arctan\left(\dfrac{y}{x}\right)\,dA$, onde $R$ é a região do primeiro quadrante limitada pelo círculo $x^{2}+y^{2}=25.$


$\displaystyle \frac{25 \pi^2}{16}.$


2385   

Expresse a integral dupla, sobre a região $R$ indicada, como uma integral iterada e ache seu valor.

  1. $\displaystyle\iint\limits_{R}(y+2x)\,dA; \quad R$ região retangular de vértices $(-1,-1)$, $(2,-1)$, $(2,4)$ e $(-1,4).$

  2. $\displaystyle\iint\limits_{R}(x-y)\,dA; \quad R$ região triangular de vértices $(2,9)$, $(2,1)$ e $(-2,1).$

  3. $\displaystyle\iint\limits_{R}xy^{2}\,dA; \quad R$ região triangular de vértices $(0,0)$, $(3,1)$ e $(-2,1).$

  4. $\displaystyle\iint\limits_{R}e^{x/y}\,dA; \quad R$ região limitada pelos gráficos de $y=2x$, $y=-x$ e $y=4.$


  1.  $\displaystyle\int_{-1}^{4} \int_{-1}^{2} (y+2x)\,dx;dy = \dfrac{75}{2}.$

  2.  $\displaystyle\int_{-2}^{2} \int_{1}^{2x + 5} x - y\,dy;dx = -48.$

  3.  $\displaystyle\int_{0}^{1} \int_{-2y}^{3y} xy^{2}\,dx;dy = \dfrac{1}{2}.$

  4.  $\displaystyle\int_{0}^{4} \int_{-y}^{y/2} e^{x/y}\,dx;dy = 8(e^{1/2} - e^{-1}).$


2557   

Determine o limite, se existir, ou mostre que o limite não existe.

$\displaystyle \lim_{(x,y) \to (0,0)} \dfrac{xy}{\sqrt{x^2 + y^2}}$.


$0.$


2104   

Seja ${\bf F}(x,y)=\dfrac{-y\,{\bf i}+x\,{\bf j}}{x^{2}+y^{2}}.$

  1. Mostre que $\dfrac{\partial P}{\partial y}=\dfrac{\partial Q}{\partial x}.$

  2. Mostre que $\int_{C}{\bf F}\cdot d{\bf r}$ não é independente do caminho. [Sugestão: calcule $\int_{C_{1}}{\bf F}\cdot d{\bf r}$ e $\int_{C_{2}}{\bf F}\cdot d{\bf r}$, onde $C_{1}$ e $C_{2}$ são as metades superior e inferior do círculo $x^{2}+y^{2}=1$ de $(1,0)$ a $(-1,0)$.] Isso contraria o Teorema 6 (Seção 16.3 do Livro do James Stewart)?


  1. $\dfrac{\partial P}{\partial y}= \dfrac{y^{2} - x^{2}}{(x^{2} + y^{2})^{2}} = \dfrac{\partial Q}{\partial x}.$

  2. Tome $C_{1}$ a curva parametrizada por $\mathbf{r_{1}}(t) = (\cos(t), \sin(t)),$ $0 \leq t \leq \pi$ e $C_{2}$ a curva parametrizada por $\mathbf{r_{2}}(t) = (\cos(t), \sin(t)),$ de $t = 2\pi$ a $t = \pi.$ Segue que $\int_{C_{1}} \mathbf{F} \cdot d \mathbf{r}  = \pi \neq -\pi = \int_{C_{2}} \mathbf{F} \cdot d \mathbf{r}.$ Como o domínio de $\mathbf{F}$ é $\mathbb{R}^{2} \setminus \left\lbrace (0,0) \right\rbrace$ que não é simplesmente conexo, o resultado não contradiz o Teorema 6.


2434   

Calcule a integral tripla.

  1.  $\displaystyle\iiint\limits_{  E}e^{x^{2}}\;dx dy dz$, onde $E$ é o conjunto $0\leq x \leq 1$, $0\leq y \leq x$ e $0\leq z \leq 1.$

  2.  $\displaystyle\iiint\limits_{  E}x\;dx dy dz$, onde $E$ é o conjunto $x^{2}\leq y\leq x$, $0\leq z\leq x+y.$


  1.  $\dfrac{e - 1}{2}.$

  2.  $\dfrac{11}{120}.$


2483   

Use a integral tripla para determinar o volume do sólido dado.

  1.  $x^{2}+y^{2}\leq 1$ e $x^{2}+z^{2}\leq 1.$

  2.  $(x-a)^{2}+y^{2}\leq a^{2}$, $x^{2}+y^{2}+z^{2}\leq 4a^{2}$, $z\geq 0$ $(a>0).$

  3.  $x^{2}+y^{2}\leq a^{2}$ e $x^{2}+z^{2}\leq a^{2}$ $(a>0).$

  4.  $x^{2}+y^{2}+z^{2}\leq a^{2}$ e $z\geq \dfrac{a}{2}$ $(a>0).$


  1.  $\dfrac{16}{3}.$

  2.  $\dfrac{16a^3}{3} \left(\dfrac{\pi}{2} - \dfrac{2}{3}\right).$

  3.  $\dfrac{16a^3}{3}.$

  4.  $\dfrac{5\pi a^3}{24}.$


2116   

Se $z=f(x,y)$, onde $x=r^{2}+s^{2}$ e $y=2rs$, determine $\partial^{2}z/\partial r\partial s.$ 



$\displaystyle \frac{\partial^{2}z}{\partial r\partial s} = 4rs \frac{\partial^{2}z}{\partial x^{2}} + 4rs \frac{\partial^{2}z}{\partial y^{2}} + (4r^{2} + 4s^{2}) \frac{\partial^{2}z}{\partial x\partial y} +  2 \frac{\partial z}{\partial y}.$ 


3122   

Uma esfera astroidal tem equação \(x^{2/3}+y^{2/3}+z^{2/3}=a^{2/3}\). Encontre o volume do sólido compreendido por uma esfera astroidal usando uma integral tripla e a transformação \begin{align*}  x & = \rho (\sin\phi\cos\theta)^3, \\  y & = \rho (\sin\phi\sin\theta)^3, \\  z & = \rho (\cos\phi)^3, \end{align*} para a qual \(0\leq\rho\leq a\), \(0\leq\phi\leq\pi\), \(0\leq\theta\leq 2\pi\).


\(\dfrac{4}{35}\pi a^3\)


1998   

Determine o trabalho $W=\int_{C}{\bf F}\cdot d{\bf r}$ realizado pelo campo de força ${\bf F}(x,y)=x^{2}(x-y)\,{\bf i}+xy^{2}\,{\bf j}$ em uma partícula que se move da origem ao longo do eixo $x$ para $(1,0)$, em seguida ao longo de um segmento de arco de circunferência $x^{2}+y^{2}=1$ até $(0,1)$ e então volta à origem ao longo do eixo $y.$


$\dfrac{\pi}{8}.$


2854   

Determine a menor distância entre os planos paralelos $2x+3y-z=2$ e $2x+3y-z=4.$


$\displaystyle \frac{\sqrt{14}}{7}.$


2430   

Calcule a integral tripla.

  1.  $\displaystyle\iiint\limits_{  E}\sqrt{1-z^{2}}\;dx dy dz$, onde $E$ é o cubo $0\leq x\leq 1$, $0\leq y\leq 1$ e $0\leq z\leq 1.$

  2.  $\displaystyle\iiint\limits_{  E}\;dx dy dz$, onde $E$ é o conjunto $x^{2}+y^{2}\leq z \leq 2x.$


  1.  $\dfrac{\pi}{4}.$

  2.  $\dfrac{\pi}{2}.$


2638   

Determine as derivadas parciais de primeira ordem da função $f(x,y)=x^{5}+3x^{3}y^{2}+3xy^{4}$.


$\displaystyle \frac{\partial f}{\partial x} = 5x^{4} + 9x^{2}y^{2} + 3y^{4}\;\;\;\text{e}\;\;\; \frac{\partial f}{\partial y} = 2x^{3}y + 12xy^{3}$.


2760   

Verifique que a função $f(x,y) = \arctan{xy}$ é diferenciável.


As derivadas parciais $\frac{\partial f}{\partial x}$ e $\frac{\partial f}{\partial y}$ de cada função $f$ existem e são contínuas em todos os pontos do domínio.


1955   

Calcule a integral de linha $\int_{C}{\bf F}\cdot d{\bf r}$, onde $C$ é dada pela função vetorial ${\bf r}(t).$

${\bf F}(x,y)=x^{2}\,{\bf i}+(x-y)\,{\bf j}$, ${\bf r}(t)=(t,\sin t)$, $0\leq t\leq \pi.$


$\displaystyle \frac{\pi^{3}}{3} - 2.$


2403   

  1.  Duas superfícies são ditas \textbf{ortogonais} em um ponto de intersecção se suas normais são perpendiculares nesse ponto. Mostre que superfícies com equação $F(x,y,z) = 0$ e $G(x,y,z) = 0$ são ortogonais em um ponto $P$, em que $\nabla F \neq 0$ e $\nabla G \neq 0$, se, e somente se, em $P$, $$F_xG_x + F_yG_y + F_zG_z = 0.$$
  2. Use a parte 1. para mostrar que as superfícies $z^2 = x^2 + y^2$ e $x^2 + y^2 + z^2 = r^2$ são ortogonais em todo ponto de intersecção. Você pode ver isso sem fazer os cálculos?



  1.  Note que a direção da normal de $F$ é dada por $\nabla F,$ a de $G$ por $\nabla G$  e que duas normais em $P$ são perpendiculares se $\nabla F \cdot \nabla G = 0.$
  2.  Tome $F = x^2 + y^2 - z^2,$ $G = x^2 + y^2 + z^2 - r^2$ e verifique $(a).$ Para "ver" isso sem calcular, note que $F = 0$ é a equação de um cone circular com vértice na origem e $G = 0$ é a equação de uma esfera centrada na origem.


2378   

É dada uma curva $\gamma$ que passa pelo ponto $\gamma(t_0) = (1,3)$ e cuja imagem está contida na curva de nível $x^2 + y^2 = 10$. Suponha $\gamma'(t_0) \neq \bf{0}$.

  1.  Determine a equação da reta tangente a $\gamma$ no ponto $(1,3)$.
  2.  Determine uma curva $\gamma(t)$ satisfazendo as condições acima.


  1. $(x,y) = (1,3) + \lambda (-6,2),$ $\lambda \in \mathbb{R}.$
  2. $\gamma(t) = (\sqrt{10} \cos(t), \sqrt{10} \sin(t)).$


2718   

Explique por que a função é diferenciável no ponto dado. A seguir, encontre a linearização $L(x,y)$ da função naquele ponto. $f(x,y) = e^{-xy} \cos{y}, \quad (\pi,0)$.


As derivadas $f_{x}$ e $f_{y}$ de cada $f$ existem e são contínuas no ponto dado, logo $f$ é diferenciável.

$L(x,y) = 1 - \pi y$.


2161   

Use o Teorema de Green para calcular a integral de linha ao longo da curva dada com orientação positiva. $\displaystyle\int_{C}(y+e^{\sqrt{x}}) \, dx + (2x+\cos{y^2}) \, dy$, $C$ é a fronteira da região englobada pelas parábolas $y=x^2$ e $x=y^2$.


$\dfrac{1}{3}.$


2986   

Considere a transformação do plano $xy$ no plano $uv$ dada por $u=x-2y$ e $v=3x-y$.

  1. Inverta a transformação, isto é, obtenha as expressões da transformação do plano $uv$ no plano $xy$.

  2. Represente geometricamente a região $R$ no plano $xy$ obtida como imagem da transformação aplicada à região delimitada por $u=0$, $u=4$, $v=1$, $v=8$.

  3. Utilize a transformação dada para calcular a integral

    $$\iint\limits_{R}\dfrac{x-2y}{3x-y} \, dA.$$


  1. $x = \dfrac{2v - u}{5},$ $y = \dfrac{v - 3u}{5}.$

  2. Região delimitada pelas retas $x = 2y,$ $x = 2y + 4,$ $y = 3x - 1$ e $3x - 8.$

  3. $\dfrac{8 \ln(8)}{5}.$


2885   

Determine o ponto da parábola $y = x^2$ mais próximo de $(14,1)$.


$(2,4).$


1971   

Encontre ${\bf r}(t)$ se ${\bf r}'(t)=2t\;{\bf i}+3t^{2}\;{\bf j}+\sqrt{t}\;{\bf k}$ e ${\bf r}(1)={\bf i}+{\bf j}.$



Como ${\bf r}'(t)=2t\;{\bf i}+3t^{2}\;{\bf j}+\sqrt{t}\;{\bf k}$, temos que 
$${\bf r}(t)=t^{2}\,{\bf i}+t^{3}\,{\bf j}+\frac{2}{3}t^{3/2}\,{\bf k}+C.$$
Mas, ${\bf r}(1)={\bf i}+{\bf j}$, logo
$${\bf i}+{\bf j}={\bf i}+{\bf j}+\frac{2}{3}{\bf k}+C$$
implicando que $C=\dfrac{2}{3}\,{\bf k}.$
Portanto 
$${\bf r}(t)=t^{2}\,{\bf i}+t^{3}\,{\bf j}+\frac{2}{3}(t^{3/2}-1)\,{\bf k}.$$


2975   

Determine o jacobiano da transformação dada por: $x = \dfrac{u}{u+v}, \quad y = \dfrac{v}{u-v}$.


$0.$


2445   

Calcule a integral de superfície $\displaystyle\iint\limits_{S}xy dS$, onde $S$ é a superfície com equações paramétricas $x=u-v$, $y=u+v$, $z=2u+v+1$, $0 \leq u \leq 1$, $0 \leq v \leq u.$


2258   

Um sólido ocupa a região $E$ com superfície $S$ e está imerso em um líquido com densidade constante $\rho$. Escolhemos um sistema de
coordenadas de modo que o plano $xy$ coincida com a superfície do líquido e valores positivos de $z$ sejam medidos para baixo, adentrando o líquido. Então, a pressão na profundidade $z$ é $p=\rho g z$, onde $g$ é a aceleração da gravidade. A força de empuxo total sobre o sólido devida $\grave{a}$ distribuição de pressão é dada pela integral de superfície
${\bf F}=-\displaystyle\iint\limits_{S} p{\bf n}\,dS$ onde ${\bf n}$ é o vetor normal unitário apontando para fora. Use o resultado do exercício anterior para mostrar que ${\bf F}=-W{\bf k}$, onde $W$ é o peso do líquido deslocado pelo sólido. (Observe que ${\bf F}$ é orientado para cima porque $z$ está orientado para baixo.) O resultado é o Princípio de Arquimedes: a força de empuxo sobre um objeto é igual ao

peso do líquido deslocado.



Note que $\displaystyle {\bf F}=-\int_{S} p {\bf n} \,dS = -\iiint_{E} \nabla p\,dV = -\iiint_{E} \nabla p\,dV = - \iiint_{E} \nabla (\rho g z)\,dV.$

Conclua usando que $W = \rho g V(E),$ onde $V(E)$ é o volume de $E.$


2329   

Considere o vetor unitário $\bf{u} = (\sqrt{3}/2,1/2)$ e a função

$$f(x,y) = \begin{cases}
\dfrac{xy^2}{x^2 + y^4}, &  \text{se } (x,y) \neq (0,0),\\ 0, &  \text{se } (x,y) = (0,0).\end{cases}$$

  1. Determine a derivada direcional $D_{\bf{u}}f(0,0)$.
  2.  Explique por que o produto escalar $\nabla f(0,0) \cdot \bf{u}$ não fornece a derivada direcional de $f$ em $(0,0)$ na direção de $\bf{u}$.


  1. $\displaystyle \frac{\sqrt{3}}{6}.$
  2.  Pois $f$ não é diferenciável em $(0,0),$ já que não é contínua nesse ponto.


2791   

Determine os valores máximos e mínimos locais e pontos de sela da função $f(x,y)=e^{4y-x^{2}-y^{2}}$.


Ponto de máximo: $\displaystyle \left(0,2 \right).$


2180   

Se $\mathbf{F}(x,y) = (-y\mathbf{i} + x\mathbf{j})/(x^2+y^2)$, mostre que $\int_{C}\mathbf{F} \cdot d\mathbf{r} = 0$ para todo caminho fechado simples que não passe pela origem e nem a circunde.


Dica: como $C$ é um caminho fechado simples que não passa pela origem e não circunda a origem, então existe uma região aberta $A$ que ainda não contém a origem, mas contém $D,$ a região limitada por $C.$ Em $A,$ tanto $-y/(x^{2} + y^{2})$ quanto $x/(x^{2} + y^{2})$ possuem derivadas parciais contínuas e podemos aplicar o Teorema de Green.


2212   

Existe um campo vetorial $\bf{G}$ em $\mathbb{R}^3$ tal que $\text{rot }{\bf{G}} = (x\sin{y},\cos{y},z-xy)$? Justifique.



Suponha que existe um campo vetorial $\bf G$ tal que $\text{rot } G= (x\,\sin y, \cos y,z-xy)$. Vamos calcular $\text{div } \text{rot } {\bf G}.$

Temos que

$$\text{div } \text{rot } {\bf G}=\frac{\partial (x\,\sin y) }{\partial x}+\frac{\partial (\cos{y})}{\partial y}+\frac{\partial (z-xy)}{\partial z}$$

$$=\sin y- \sin y +1=1.$$


Sabemos que se ${\bf F}=P\,{\bf i}+Q\,{\bf j}+R\,{\bf k}$ é um campo vetorial sobre $\mathbb{R}^{3}$ e $P$, $Q$ e $R$ têm derivadas parciais de segunda ordem contínuas, então $\text{div } \text{rot } {\bf F}=0.$

Como $\text{div } \text{rot } {\bf G}\neq 0$, pela contrapositiva do resultado acima, temos que ${\bf G}$ não é um campo vetorial do $\mathbb{R}^{3}.$


2750   

Mostre que a função $f(x,y) = xy - 5y^2$ é diferenciável achando os valores $\varepsilon_1$ e $\varepsilon_2$ que satisfaçam a Definição $7$ da Seção $14.4$ do Stewart.


$\epsilon_{1} = \Delta y$ e $\epsilon_{2} = -5\Delta y$.


2722   

Determine a diferencial da função $R = \alpha\beta^2 \cos{\lambda}$.


$dR = \beta^{2} \cos(\gamma) d\alpha + 2\gamma \beta \cos (\gamma) d\beta - \alpha \beta^{2} \sin(\gamma) d\gamma$.


2079   

Sejam $f(x)$ e $g(x)$ duas funções contínuas, respectivamente, nos intervalos $[a,b]$ e $[c,d].$ Prove que $$\iint\limits_{R}f(x)g(y)\,dx dy=\bigg(\int_{a}^{b}f(x)\,dx\bigg)\bigg(\int_{c}^{d}g(y)\,dy\bigg),$$ onde $R$ é o retângulo $a\leq x\leq b$ e $c\leq y\leq d.$


Note que $$ \int_{c}^{d} \left[\int_{a}^{b}f(x)g(y)\,dx\right] \;dy = \int_{c}^{d} \left[\int_{a}^{b}f(x)\,dx\right]g(y)  \;dy = \left(\int_{a}^{b}f(x)\,dx\right) \int_{c}^{d} g(y)  \;dy.$$


2018   


  1.  Estime o volume do sólido que está abaixo da superfície $z = x + 2y^2$ e acima do retângulo $R = [0,2] \times [0,4]$. Use a soma de Riemann com $m = n = 2$ e escolha os pontos amostrais como os cantos inferiores direitos.

  2.  Use a Regra do Ponto Médio para dar uma estimativa da integral do item anterior.


  1.  $\approx 44.$

  2.  $\approx 88.$


3018   

Esboce a região de integração para a integral iterada $\displaystyle\int_{0}^{1}\!\!\int_{\sqrt{y}}^{3\sqrt{y}}f(x,y)\,dx dy$.


ma211-list6-ex24_sol_b.png

2065   

Dados ${\bf F}(x,y,z)=yz\,{\bf i}+xz\,{\bf j}+(xy+2z)\,{\bf k}$, $C$ é o segmento de reta de $(1,0,-2)$ a $(4,6,3).$

  1. Determine uma função $f$ tal que ${\bf F}=\nabla f$.

  2. Use o resultado anterior para calcular $\int_{C}{\bf F}\cdot d{\bf r}$ sobre a curva $C$ dada.


  1. $f(x,y,z) = xyz + z^{2};$

  2. $77.$


2162   

Use o Teorema de Green para calcular a integral de linha ao longo da curva dada com orientação positiva. $\displaystyle\int_{C} \sin{y} \, dx + x\cos{y} \, dy$, $C$ é a elipse $x^2 + xy + y^2 = 1$.


$0.$


1925   

Calcule a integral de linha, onde $C$ é a curva dada. $\displaystyle\int_{C}x^{2}y\sqrt{z}\,dz$, $C:\,x=t^{3},\, y=t,\, z=t^{2},\, 0\leq t\leq 1.$



As equações paramétricas de $C$ são

$$x=t^{3},\, y=t,\, z=t^{2},\, 0\leq t\leq 1.$$

Logo,

$$dx=3t^{2}\,dt,\, dy=dt,\, dz=2t\,dt.$$


Assim,

$$\int_{C}x^{2}y\sqrt{z}\,dz=\int_{0}^{1}((t^{3})^{2}\cdot t \cdot \sqrt{t^{2}})(2t\,dt)=2\int_{0}^{1}t^{9}\,dt$$

$$=2\cdot\frac{t^{10}}{10}\bigg|_{0}^{1}=2\cdot \frac{1}{10}=\frac{1}{5}.$$


2284   

Determine se os pontos $P(3,-1,5)$ e $Q(-1,3,4)$ estão na superfície ${\bf r}(u,v)=(u+v,u^{2}-v,u+v^{2})$.


$P$ está na superfície; $Q$ não está na superfície.


2804   

Determine os valores máximos e mínimos locais e pontos de sela da função $f(x,y)=-x^{2}+y^{2}+2xy+4x-2y$.


Ponto de sela: $\displaystyle \left(\frac{3}{2},-\frac{1}{2}\right).$


2087   

Calcule $\displaystyle\int_{C}\dfrac{-y}{x^{2}+y^{2}}\,dx+\dfrac{x}{x^{2}+y^{2}}\,dy$, onde $C:[0,1]\rightarrow \mathbb{R}^{2}$ é uma curva de classe $C^{1}$ por partes, com imagem contida no conjunto $\Omega=\{(x,y)\in \mathbb{R}^{2}| y>0\}\cup\{(x,y)\in \mathbb{R}^{2}|\,x<0\}$, tal que $C(0)=(1,1)$ e $C(1)=(-1,-1).$


$0.$


2458   

O índice I de temperatura-umidade (ou simplesmente humidex) é a temperatura aparente do ar quando a temperatura real é $T$ e a umidade relativa é $h$, de modo que podemos escrever $I=f(T,h)$. A tabela seguinte com valores de $I$ foi extraída de uma tabela do Environment Canada.

ma211-list2-ex5.png

  1. Qual é o valor de $f(35,60)$? Qual é o seu significado?

  2. Para que valor de $h$ temos $f(30,h)=36$?

  3. Para que valor de $T$ temos $f(T,40)=42$?

  4. Qual o significado de $I=f(20,h)$ e $I=f(40,h)$? Compare o comportamento dessas duas funções de $h.$


  1. 48, o que significa que quando a temperatura real é $35^\circ$C e a umidade relativa é $60\%,$ o humidex é $48^\circ$C.

  2. $50\%.$

  3. $35^\circ$C.

  4. $I = f(20,h)$ e $I = f(40,h)$ são funções de $h$ que fornecem os valores do humidex quando a temperatura real é $20^\circ$C e $40^\circ$C, respectivamente. Ambas as funções crescem com $h,$ porém $f(20,h)$ cresce aproximadamente a taxa constante, enquanto $f(40,h)$ cresce mais rapidamente a uma taxa crescente.


1970   

Se ${\bf r}(t)\neq {\bf 0}$, mostre que 

$$\frac{\mathrm{d}}{\mathrm{d}t}|{\bf r}(t)|=\frac{1}{|{\bf r}(t)|}{\bf r}(t)\cdot {\bf r}'(t).$$ 
(Sugestão: $|{\bf r}(t)|^{2}={\bf r}(t)\cdot {\bf r}(t)$).



Sabemos que $|{\bf r}(t)|^{2}={\bf r}(t)\cdot {\bf r}(t)$ ou $|{\bf r}(t)|= ({\bf r}(t)\cdot {\bf r}(t))^{1/2}.$ Então

\begin{eqnarray*}
\frac{d}{dt}|{\bf r}(t)|&=&\frac{d}{dt}[({\bf r}(t)\cdot {\bf r}(t))^{1/2}]\\
&=&\frac{1}{2}[{\bf r}(t)\cdot {\bf r}(t)]^{-1/2}[{\bf r}'(t)\cdot {\bf r}(t)+{\bf r}(t)\cdot {\bf r}'(t)]\\
&=& \frac{1}{2}[{\bf r}(t)\cdot {\bf r}(t)]^{-1/2}[2\,{\bf r}(t)\cdot {\bf r}'(t)]\\
&=&\frac{1}{2}\frac{1}{[{\bf r}(t)\cdot {\bf r}(t)]}[2\,{\bf r}(t)\cdot {\bf r}'(t)]\\
&=&\frac{1}{|{\bf r}(t)|}\,{\bf r}(t)\cdot {\bf r}'(t).
\end{eqnarray*}


2038   

 Utilize um diagrama em árvore para escrever a Regra da Cadeia para o caso dado. Suponha que todas as funções sejam diferenciáveis.

$w=f(r,s,t)$, onde $r=r(x,y)$, $s=s(x,y)$, $t=t(x,y)$.


$\displaystyle \frac{\partial w}{\partial x} = \frac{\partial w}{\partial r}\frac{\partial r}{\partial x} + \frac{\partial w}{\partial s}\frac{\partial s}{\partial x} + \frac{\partial w}{\partial t}\frac{\partial t}{\partial x}$ e $\displaystyle \frac{\partial w}{\partial y} = \frac{\partial w}{\partial r}\frac{\partial r}{\partial y} + \frac{\partial w}{\partial s}\frac{\partial s}{\partial y} + \frac{\partial w}{\partial t}\frac{\partial t}{\partial y}$


2950   

Calcule utilizando coordenadas esféricas. $\displaystyle\iiint\limits_{E}xyz\,dV$, onde $E$ é o sólido limitado pelos paraboloides $z=x^{2}+y^{2}$ e $z=8-x^{2}-y^{2}.$


$0.$


2255   

Demonstre a identidade abaixo, supondo que $S$ e $E$ satisfaçam as condições do Teorema do Divergente e que as funções escalares e as componentes dos campos vetoriais tenham derivadas parciais de segunda ordem contínuas.

  1. $\displaystyle\iint\limits_{S}(f\nabla g)\cdot {\bf n}\,dS=\displaystyle\iiint\limits_{E}(f\nabla^{2}g+\nabla f+\nabla g)\,dV.$



Note que $\displaystyle\iint\limits_{S}(f\nabla g)\cdot {\bf n}\,dS=\displaystyle\iiint\limits_{E} \mbox{div} (f\nabla g)\,dV.$


2494   

Dada a função $f(x,y)=x^{2}-y^{2}$.

  1. Encontre o domínio da função.

  2. Encontre a imagem da função.

  3. Descreva as curvas de nível da função.


  1. $D_{f} = \mathbb{R}^{2}.$

  2. $Im(f) = \mathbb{R}.$

  3. As curvas de nível são as hipérboles $x^{2} - y^{2} = C$ com foco no eixo $x$ se $C > 0;$ com foco no eixo $y$ se $C < 0$ e as retas $y = \pm x$ se $C = 0.$


2745   

Mostre que o plano tangente ao parabolóide $z = x^2 + y^2$ no ponto $(1,2,5)$ intercepta o plano $xy$ na reta

$$\begin{cases}2x + 4y - 5 = 0 \\z = 0\end{cases}.$$


Note que o plano tangente no ponto $(1,2,5)$ é $z = 2x + 4y - 5$.


2014   

Seja ${\bf E}(x,y)=\dfrac{1}{x^{2}+y^{2}}\dfrac{x\,{\bf i}+y\,{\bf j}}{\sqrt{x^{2}+y^{2}}}$ e seja $C$ a curva dada por $x=t$ e $y=1-t^{4}$, $-1\leq t\leq 1.$

  1. Que valor é razoável esperar para $\int_{C}{\bf E}\cdot d{\bf l}$? Por quê? (O ${\bf l}$ desempenha aqui o mesmo papel que ${\bf r}:{\bf l}(t)={\bf r}(t).$)

  2. Calcule $\int_{C}{\bf E}\cdot d{\bf l}.$


$0.$


2843   

Passe para coordenadas polares e calcule: $\displaystyle\int_{-1}^{1} \int_{0}^{\sqrt{1-x^{2}}}\,dy dx$


$\displaystyle \frac{\pi}{2}.$


2316   

Determine uma equação do plano tangente à superfície parametrizada dada no ponto especificado. $x=u+v$, $y=3u^{2}$, $z=u-v$; $(2,3,0).$


$3x - y + 3z = 3.$


2395   

Seja $g(x,y) = f(x^2 + y^2)$, em que $f$ é uma função diferenciável. Sabendo que $f'(2) = 1$, determine a equação da reta tangente à curva de nível de $g$ que passa pelo ponto $(1,1)$.


 $x + y = 2.$


2677   

Calcule as derivadas parciais de $s = f(x,y,z,w)$ dada por $s = xw \ln{(x^2 + y^2 + z^2 + w^2)}$.


$\begin{aligned}[t]\frac{\partial s}{\partial x} &= w \left( \frac{2x^{2}}{x^{2} + y^{2} + z^{2} + w^{2}} + \ln (x^{2} + y^{2} + z^{2} + w^{2})\right),\\\frac{\partial s}{\partial y} &= \frac{2xyw}{x^{2} + y^{2} + z^{2} + w^{2}},\;\;\;\; \frac{\partial s}{\partial z} = w \frac{2xzw}{x^{2} + y^{2} + z^{2} + w^{2}}\;\;\;\;\;\text{e}\\\frac{\partial s}{\partial w} &= x \left( \frac{2w^{2}}{x^{2} + y^{2} + z^{2} + w^{2}} + \ln (x^{2} + y^{2} + z^{2} + w^{2})\right).\end{aligned}$


1951   

Calcule a integral de linha $\int_{C}{\bf F}\cdot d{\bf r}$, onde $C$ é dada pela função vetorial ${\bf r}(t).$

${\bf F}(x,y,z)=\sin{x}\,{\bf i}+\cos{y}\,{\bf j}+xz\,{\bf k}$, ${\bf r}(t)=t^{3}\,{\bf i}-t^{2}\,{\bf j}+t\,{\bf k}$, $0\leq t\leq 1.$


$\dfrac{6}{5} - \cos(1) - \sin(1).$


3048   

O campo vetorial $\mathbf{F}$ é mostrado no plano $xy$ e é o mesmo em todos os planos horizontais (em outras palavras, $\mathbf{F}$ é independente de $z$ e sua componente $z$ é 0).

  1. O $\text{div }{\mathbf{F}}$ será positivo, negativo ou nulo? Justifique.

  2. Determine se o $\text{rot }{\mathbf{F}} = 0$. Se não, em que direção rot $\mathbf{F}$ aponta?

ma211-list13-ex6_iii.png


  1. Nulo.

  2. $\text{rot } \bf{F}$ aponta na direção negativa do eixo $z.$


2102   

Seja ${\bf F}=\nabla f$, onde $f(x,y)=\sin(x-2y)$. Determine curvas $C_{1}$ e $C_{2}$ que não sejam fechadas e satisfaçam a equação.

  1. $\displaystyle\int_{C_{1}}{\bf F}\cdot d{\bf r}=0$

  2. $\displaystyle\int_{C_{2}}{\bf F}\cdot d{\bf r}=1$


  1. $\mathbf{r}(t) = \pi t \mathbf{i} + \pi t \mathbf{j},$ $0 \leq t \leq 1.$

  2. $\mathbf{r}(t) = \dfrac{\pi}{2} t \mathbf{i},$ $0 \leq t \leq 1.$


2851   

Passe para coordenadas polares e calcule: $\displaystyle\iint\limits_{R}x\,dx dy$, onde $R$ é a região, no plano $xy$, limitada pela curva (dada em coordenadas polares) $\rho=\cos(3\theta)$, $-\dfrac{\pi}{6}\leq \theta \leq \dfrac{\pi}{6}.$


$\displaystyle \frac{81\sqrt{3}}{320}.$


2624   

Utilizando o Teorema de Stokes, transforme a integral $\iint_{ S}\mbox{rot}{\bf F}\cdot{\bf n}dS$ numa integral de linha e calcule.

  • ${\bf F}(x,y,z) = y{\bf i} + x^2{\bf j}+z{\bf k}$, $S$ a superfície parametrizada por ${\bf R}(u,v) = (u,v,2u+v+1)$, $u\geq 0$, $u+v\leq 2$, sendo ${\bf n}$ a normal apontando para baixo.


$-\dfrac{2}{3}.$


3058   

Esboce o campo vetorial ${\bf F}(x,y)=(x-y)\textbf{i} + x \textbf{j}$, desenhando um diagrama.


2780   

Calcule a integral dupla utilizando coordenadas polares: $\displaystyle\iint\limits_{R}(x^{2}+y^{2})\,dx dy$, onde $R=\{(x,y)\in \mathbb{R}^{2}| 1\leq x^{2}+y^{2}\leq 4\}.$


$\displaystyle \frac{15\pi}{2}.$


2448   

Calcule $\displaystyle\iint\limits_{S}g(x,y,z)dS,$ sendo $g(x,y,z)=x^{2}+y^{2}+z^{2}$ e $S$ a parte do plano $z=y+4$ interior ao cilindro $x^{2}+y^{2}=4.$


$76\pi \sqrt{2}.$


2788   

Utilize as curvas de nível da figura para predizer a localização dos pontos críticos de $f(x,y)=3x-x^{3}-2y^{2}+y^{4}$ e se $f$ tem um ponto de sela ou um máximo ou mínimo local em cada um desses pontos. Explique seu raciocínio. Em seguida, empregue o Teste da Segunda Derivada para confirmar suas predições.

ma211-list5-ex6_b.png


$f$ possui um ponto de máximo local em $(1,0),$ pontos de sela em $(1,1),$ $(1,-1)$ e $(-1,0)$ e pontos de mínimo local em $(-1,1)$ e $(-1,-1).$


2288   

Determine uma representação paramétrica para a superfície descrita a seguir. A parte do hiperboloide $x^{2}+y^{2}-z^{2}=1$ que está à direita do plano $xz.$


$x =u,$ $z = v,$ $y = \sqrt{1 - u^2 + v^2}.$


1980   

Mostre que a função vetorial
$$\textbf{r}(t) = (2\textbf{i} + 2\textbf{j} + \textbf{k}) + (\cos{t})\left( \dfrac{1}{\sqrt{2}}\textbf{i} - \dfrac{1}{\sqrt{2}}\textbf{j} \right) + (\sin{t})\left( \dfrac{1}{\sqrt{3}}\textbf{i} + \dfrac{1}{\sqrt{3}}\textbf{j} + \dfrac{1}{\sqrt{3}}\textbf{k} \right)$$
descreve o movimento de uma partícula no círculo de raio $1$ centrado no ponto $(2,2,1)$ e contido no plano $x + y - 2z = 2$.


2705   

Considere a função

$$f(x,y)= \begin{cases}\dfrac{xy^{2}}{x^{2}+y^{4}}, & \quad \text{se } (x,y)\neq (0,0),\\0, & \quad \text{se } (x,y)=(0,0).\\\end{cases}$$

  1. A função é contínua em $(0,0)$? Justifique sua resposta.

  2. Determine as derivadas parciais $\dfrac{\partial f}{\partial x}(0,0)$ e $\dfrac{\partial f}{\partial y}(0,0)$.


  1. Não, pois $\lim_{(x,y) \to (0,0)} f(x,y)$ não existe.

  2. $\displaystyle \frac{\partial f}{\partial x}(0,0) = \frac{\partial f}{\partial y}(0,0) = 0$.


3092   

  1.  Seja \(f(x,y)=x-2y\) e considere uma subdivisão uniforme do retângulo \(R=[0,2]\times[0,2]\) em \(16\) retângulos menores. Tome \((x_k^\ast,y_k^\ast)\) como sendo o centro do \(k\)-ésimo  retângulo e aproxime a integral dupla de \(f\) sobre \(R\) pela soma de Riemann resultante.

  2.  Compare o resultado obtido no item anterior com o valor exato da integral.


1981   

 Uma partícula se move no plano $xy$ de tal maneira que sua posição no instante $t$ é
$$\textbf{r}(t) = (t - \sin{t} )\textbf{i} + (1 - \cos{t})\textbf{j}.$$
Trace o gráfico de $\textbf{r}(t)$. A curva resultante é chamada de ciclóide.


2605   

Use o Teorema de Stokes para calcular $\displaystyle\iint\limits_{ S}\mbox{rot}{\bf F}\cdot d{\bf S}.$

  • ${\bf F}(x,y,z) = x^2z^2{\bf i} + y^2z^2{\bf j} + xyz {\bf k}$ e $S$ é a parte do parabolóide $z = x^2+y^2$ que está dentro do cilindro $x^2+y^2=4$, orientado para cima.


$0.$


2169   

Calcule as integrais iteradas.

  1. $\displaystyle\int_{0}^{1}\int_{0}^{x^{2}}(x+2y)\,dy dx$

  2. $\displaystyle\int_{0}^{1}\int_{x^{2}}^{x}(1+2y)\,dy dx$


  1.  $\dfrac{9}{20}.$

  2.  $\dfrac{3}{10}.$


1990   

 Se ${\bf u}(t)=(\sin{t}, \cos{t}, t)$ e ${\bf v}(t)=(t,t\cos{t},\sin{t})$, use a Fórmula $$\dfrac{d}{dt}\left[{\bf u}(t)\times{\bf v}(t)\right]={\bf u}'(t)\times{\bf v}(t)+{\bf u}(t)\times{\bf v}'(t)$$ para  encontrar $$\frac{\mathrm{d}}{\mathrm{d}t}[{\bf u}(t)\times {\bf v}(t)].$$


$\left\{t^2 \sin (t)-\sin ^2(t)+\cos ^2(t)-2 t \cos (t),2 t-2 \sin (t) \cos (t),-t \sin^2(t)+t \sin (t)+t \cos ^2(t)-\cos (t)+\sin (t) \cos (t)\right\}$


1986   

Determine a equação da reta tangente à trajetória da função dada, no ponto dado.

  1. ${\bf r}(t)=(\cos (t),\sin (t),t)$ e ${\bf r}(\pi/3).$
  2. ${\bf r}(t)=(t^{2},t)$ e ${\bf r}(1).$
  3. ${\bf r}(t)=\bigg(\dfrac{1}{t},\dfrac{1}{t},t^{2}\bigg)$ e ${\bf r}(2).$


2103   

Mostre que, se um campo vetorial ${\bf F}=P\,{\bf i}+Q\,{\bf j}+R\,{\bf k}$ é conservativo e $P$, $Q$, $R$ têm derivadas parciais de primeira ordem contínuas, então

$$\frac{\partial P}{\partial y}=\frac{\partial Q}{\partial x}, \,\,\,\,\,\frac{\partial P}{\partial z}=\frac{\partial R}{\partial x},\,\,\,\,\,\frac{\partial Q}{\partial z}=\frac{\partial R}{\partial y}$$


Se $f$ é uma função potencial de $\mathbf{F},$ então $f_{x} = P,$ $f_{y} = Q$ e $f_{z} = R.$ Como $P, Q$ e $R$ possuem derivadas parciais de primeira ordem contínuas, então pelo Teorema de Clairaut, temos $f_{xy} = f_{yx},$ $f_{yz} = f_{zy}$ e $f_{xz} = f_{zx}.$


2852   

Entre todos os pontos do gráfico de $z=10-x^{2}-y^{2}$ que estão acima do plano $x+2y+3z=0$, encontre o ponto mais afastado do plano.


$\displaystyle \left( \frac{1}{6}, \frac{1}{3}, \frac{355}{36} \right).$


2263   

Calcule $\displaystyle\iint\limits_{B}f(x,y)\,dx dy$ sendo dados:

  1.  $f(x,y)=x\cos{y}$ e $B=\{(x,y)\in \mathbb{R}^{2}|\;x\geq 0,\;x^{2}\leq y\leq \pi\}.$

  2.  $f(x,y)=xy$ e $B=\{(x,y)\in \mathbb{R}^{2}|\;x^{2}+y^{2}\leq 2,\;y\leq x\;e\;x\geq 0\}.$

  3.  $f(x,y)=x$ e $B$ o triângulo de vértices $(0,0)$, $(1,1)$ e $(2,0).$

  4.  $f(x,y)=xy\sqrt{x^{2}+y^{2}}$ e $B$ o retângulo $0\leq x\leq 1$, $0\leq y\leq 1.$

  5.  $f(x,y)=x+y$ e $B$ o paralelogramo de vértices $(0,0)$, $(1,1)$, $(3,1)$ e $(2,0).$


  1.  $-1.$

  2.  $-\dfrac{1}{4}$.

  3.  $1.$

  4.  $\dfrac{2(2\sqrt{2} - 1)}{15}.$

  5.  $4.$


2554   

Determine o limite, se existir, ou mostre que o limite não existe.

$\displaystyle \lim_{(x,y) \to (2,1)}\dfrac{4 - xy}{x^2 + 3y^2}$.


$\frac{2}{7}.$


1978   

Determine a função vetorial que representa a curva obtida pela intersecção do cone $z = \sqrt{x^2 + y^2}$ com o plano $z = 1 + y$.


2117   

Calcule $\mathrm{d} z/\mathrm{d} t$ por dois processos:

  1. substituindo as expressões para $x(t)$ e $y(t)$ em $z$ e depois derivando diretamente com relação a $t$
  2. aplicando a Regra da Cadeia: $\frac{dz}{dt}=\frac{\partial z}{\partial x}\frac{dx}{dt}+\frac{\partial z}{\partial y }\frac{dy}{dt}$.

$z=\sin(xy)$, $x=3t$ e $y=t^{2}.$


$\displaystyle \frac{dz}{dt} (t) =  9t^{2}\cos(3t^{3}).$


2530   

Represente graficamente o domínio da função $z=f(x,y)$ dada por $z=\sqrt{y-x^{2}}+\sqrt{2x-y}$.


$\left\lbrace (x,y); x^{2} \leq y \leq 2x \right\rbrace$

ma211-list2-ex25_sol_c.png


2343   

Seja $A=\{(0,y,z)\in \mathbb{R}^{3}| z^{2}+(y-2)^{2}=1\}$; ache a área da superfície gerada pela rotação em torno do eixo $Oz$ do conjunto $A.$


$8\pi^2.$


2762   

Determine o maior conjunto de pontos em que a função $f(x,y) = \begin{cases}\dfrac{x^3}{x^2 + y^2}, & \quad \text{se } (x,y) \neq (0,0),\\0, & \quad \text{se } (x,y) = 0\end{cases}$ é diferenciável. Justifique.


$\mathbb{R}^{2} \setminus \left\lbrace (0,0) \right\rbrace$.


3067   

Determine o trabalho $W=\int_{C}{\bf F}\cdot d{\bf r}$ realizado pelo campo de força ${\bf F}(x,y)=x\,{\bf i}+(x^{3}+3xy^{2})\,{\bf j}$ em uma partícula que inicialmente está no ponto $(-2,0)$, se move ao longo do eixo $x$ para $(2,0)$ e ao longo da semicircunferência $y=\sqrt{4-x^{2}}$ até o ponto inicial.


2219   

Seja $f$ um campo escalar e $\mathbf{F}$ um campo vetorial. Diga se cada expressão tem significado. Em caso negativo, explique por quê. Em caso afirmativo, diga se é um campo vetorial ou escalar. 

  1. $\text{rot }{f}$;

  2. $\text{div }{\mathbf{F}}$;

  3. $\text{grad }{\mathbf{F}}$.


  1. $\text{rot }{f}$ não tem significado, pois $f$ é um campo escalar.

  2. $\text{div }{\mathbf{F}}$ é um campo escalar.

  3. $\text{grad }{\mathbf{F}}$ não tem sifnificado, pois $\bf{F}$ não é um campo escalar.


2673   

Calcule as derivadas parciais de $f(x,y,z) = xe^{x - y - z}$.


$\displaystyle \frac{\partial f}{\partial x} = (1 + x)e^{x - y - z},\;\;\;\; \frac{\partial f}{\partial y} = -x e^{x - y - z}\;\;\;\;\;\text{e}\;\;\;\;\;\frac{\partial f}{\partial z} = -x e^{x - y - z}.$


2905   

Utilize coordenadas polares para determinar o volume do sólido dado: uma esfera de raio $a.$


$\displaystyle \frac{4\pi}{3}a^3.$


2026   

Use a Regra da Cadeia para determinar $\mathrm{d}z/\mathrm{d} t$ ou $\mathrm{d}w/ \mathrm{d}t.$

$z=x^{2}y+xy^{2}$, $x=2+t^{2}$, $y=1-t^{3}$.



$\displaystyle \frac{dz}{dt} = 4(2xy + y^{2} )^{3} - 3 (x^{2} + 2xy)t^{2}.$


2632   

Determine as derivadas parciais de primeira ordem da função $f(r,s)=r\ln(r^{2}+s^{2})$.



Sendo $f(r,s)=r\cdot \ln(r^{2}+s^{2})$, temos que as derivadas parciais em relação a $r$ e $s$, respectivamente, são:

$\bullet f_{r}(r,s)=1\cdot \ln(r^{2}+s^{2})+r\cdot \dfrac{1}{r^{2}+s^{2}}\cdot 2r=\ln(r^{2}+s^{2})+\dfrac{2r^{2}}{r^{2}+s^{2}}.$

$\bullet f_{s}(r,s)=0\cdot \ln(r^{2}+s^{2})+r\cdot \dfrac{1}{r^{2}+s^{2}}\cdot 2s=\dfrac{2rs}{r^{2}+s^{2}}.$


2882   

O plano $4x - 3y + 8z = 5$ intercepta o cone $z^2 = x^2 + y^2$ em uma elipse.

  1. Faça os gráficos do cone, do plano e da elipse.

  2. Use os multiplicadores de Lagrange para achar os pontos mais alto e mais baixo da elipse.


  1. Gráficos.

  2. Ponto mais alto: $\displaystyle \left( -\frac{4}{3}, 1,\frac{5}{3} \right)$ e ponto mais baixo: $\displaystyle \left( \frac{4}{13}, -\frac{3}{13},\frac{5}{13} \right).$



2015   

Calcule $\int_{C}{\bf E}\cdot d{\bf l}$, onde ${\bf E}(x,y)=\dfrac{1}{x^{2}+y^{2}}\dfrac{x\,{\bf i}+y\,{\bf j}}{\sqrt{x^{2}+y^{2}}}$ e $C$ é a curva dada por $x=2\,\cos t$, $y=\sin t$, com $0\leq t\leq \dfrac{\pi}{2}.$


$-\dfrac{1}{2}.$


2602   

Calcule, usando integração, o volume do sólido limitados pelas superfícies $z = 1$, $z = 2$ e $z = \sqrt{x^2 + y^2}.$


$\dfrac{7\pi}{6}.$


1967   

Seja $C: {\bf r}(t)=(R\,\cos t, R\,\sin t)$, $0\leq t \leq 2\pi$\,$(R>0).$ Mostre que

$$\int_{C}\frac{-y}{x^{2}+y^{2}}\,dx+\frac{x}{x^{2}+y^{2}}\,dy$$

não depende de $R.$


Note que o valor da integral é $2\pi,$ independente de $R.$


2690   

Passe para coordenadas polares e calcule.     

  1.  $\displaystyle\int_{0}^{a} \int_{0}^{x}\sqrt{x^{2}+y^{2}}\,dy dx$, em que $a>0$.

  2.  $\displaystyle\iint\limits_{ D}x\,dA$, onde $D$ é a região do primeiro quadrante compreendida entre os círculos x^2+y^2=4$ e  $x^2+y^2=2x.$ 



  1.  Temos que a região de integração é: $$ R=\{(x,y)\in \mathbb{R}^{2}|\,0\leq x \leq a,\, 0\leq y \leq x\}.$$ ma211-list7-ex1_sol_c.png

    Passando para coordenadas polares temos que $$\left\{ \begin{array}{cc} x=r\,\cos \theta\\ y=r\,\sin \theta\\ dy\,dx=r\,dr\,d\theta\\ \end{array} \right. $$ Como $0\leq x \leq a$, temos que $0\leq r\leq \dfrac{a}{\cos \theta}$ e também $0\leq \theta \leq \dfrac{\pi}{4}.$ Então, $$\int_{0}^{a}\int_{0}^{x}\sqrt{x^{2}+y^{2}}\,dy\,dx= \int_{0}^{\frac{\pi}{4}}\int_{0}^{\frac{a}{\cos \theta}}\sqrt{r^{2}\,\cos^{2}\theta +r^{2}\,\sin^{2}\theta}\,r\,dr\,d\theta$$ $$=\int_{0}^{\frac{\pi} {4}}\int_{0}^\frac{a}{\cos\theta}r^{2}\,dr\,d\theta=\int_{0}^\frac{\pi}{4}\frac{r^3}{3}\bigg|_{0}^{\frac{a}{\cos \theta}}d\theta $$ $$=\frac{a^{3}}{3}\int_{0}^{\frac{\pi}{4}}\frac{1}{\cos^{3}\theta}d\theta=\frac{a^{3}}{3}\int_{0}^{\frac{\pi}{4}}\sec^{3}\theta d\theta$$ $$=\frac{a^{3}}{3}\bigg(\frac{1}{2}\sec \theta \tan \theta+\frac{1}{2}\ln|\sec \theta+\tan \theta|\bigg)\bigg|_{0}^{\frac{\pi}{4}}$$ $$=\frac{a^{3}}{6}\bigg[\bigg(\sec\frac{\pi}{4}\cdot \tan\frac{\pi}{4}+\ln\bigg|\sec\frac{\pi}{4}+\tan\frac{\pi}{4}\bigg|\bigg)- \bigg(\sec 0\cdot \tan 0+\ln|\sec 0+\tan 0|\bigg)\bigg]$$ $$=\frac{a^{3}}{6}\bigg(\sqrt{2}+\ln(\sqrt{2}+1)\bigg)$$

  2.  A região  de integração $R$ é descrita na figura seguinte

    ma211-list7-ex1_sol_d.png

    Notemos que $x^{2}+y^{2}=2x\Leftrightarrow (x-1)^{2}+y^{2}=1.$ Assim, $$\iint\limits_{ R}x\,dA=\underbrace{\iint\limits_{   \substack{x^{2}+y^{2}\leq 4\\ x\geq 0\\ y\geq 0}}x\,dA}_{(1)} -\,\,\underbrace{\iint\limits_{\substack{(x-1)^{2}+y^{2}\leq 1 \\ y\geq 0}}x\,dA}_{(2)}$$ Para a integral $(1)$ temos em coordenadas polares que $$r^{2}\cos^{2}\theta+r^{2}\sin^{2}\theta=4\Rightarrow r^{2}=4\Rightarrow r=\pm 2.$$ Logo, $0\leq r\leq 2$ e $0\leq \theta \leq \dfrac{\pi}{2}.$ Para a integral $(2)$ temos em coordenadas polares que $$(r-\cos \theta-1)^{2}+r^{2}\sin^{2}\theta=1\Rightarrow r^{2}\cos^{2}\theta-2r\cos \theta+1+r^{2}\sin^{2}\theta=1$$ $$\Rightarrow r^{2}-2r\cos\theta=0\Rightarrow r(r-2\cos \theta)=0\Rightarrow r=0   \mbox{ou}    r=2\cos \theta.$$ Logo, $0\leq r\leq 2\cos \theta$ e $0\leq \theta \leq \dfrac{\pi}{2}.$ Assim,  $$\iint\limits_{       R}x\,dA=\int_{0}^{\frac{\pi}{2}}\int_{0}^{2}r\,\cos \theta \cdot r \,dr\,d\theta- \int_{0}^{\frac{\pi}{2}}\int_{0}^{2\cos \theta}r\cos \theta\cdot r\, dr\,d \theta$$ $$=\int_{0}^{\frac{\pi}{2}}\int_{0}^{2}r^{2}\,\cos \theta\,dr\,d\theta-\int_{0}^{\frac{\pi}{2}}\int_{0}^{2\cos \theta}r^{2}\,\cos \theta\,dr\,d\theta$$ $$=\int_{0}^{\frac{\pi}{2}}\cos\,d\theta \cdot \int_{0}^{2}r^{2}\,dr-\int_{0}^{\frac{\pi}{2}}\frac{r^{3}}{3}\cos \theta \bigg|_{0}^{2\cos \theta}\,d\theta$$ $$=\bigg(\sin\theta \bigg|_{0}^{\frac{\pi}{2}}\bigg)\cdot \bigg(\frac{r^{3}}{3}\bigg|_{0}^{2}\bigg)-\frac{8}{3}\int_{0}^{\frac{\pi}{2}}\cos^{4}\theta\,d\theta$$ $$=\bigg(\sin \frac{\pi}{2}-\sin 0\bigg)\cdot \bigg(\frac{8}{3}-0\bigg)-\frac{8}{3}\bigg(\frac{1}{4}\cos^{3}\theta\,\sin \theta+\frac{3}{8}\theta+\frac{3}{16}\sin 2 \theta\bigg)\bigg|_{0}^{\frac{\pi}{2}}$$ $$=\frac{8}{3}-\frac{8}{3}\bigg[\bigg(\frac{1}{4}\cos^{3}\frac{\pi}{2}\sin \frac{\pi}{2}+\frac{3}{8}\cdot\frac{\pi}{2}+\frac{3}{16}\sin2\cdot \frac{\pi}{2}\bigg) -\bigg(\frac{1}{4}\cos^{3}0\sin 0+\frac{3}{8}\cdot 0+\frac{3}{16}\sin 0\bigg)\bigg]$$ $$=\frac{8}{3}-\frac{8}{3}\cdot \bigg(\frac{3\pi}{16}\bigg)=\frac{8}{3}-\frac{\pi}{2}=\frac{16-3\pi}{6}.$$


2320   

Determine uma equação do plano tangente à superfície parametrizada dada no ponto especificado. ${\bf r}(u,v)=(3\sin 2u,6\sin^{2} u, v)$,$0\leq u\leq \pi$, no ponto ${\bf r}(\pi/3,0).$


$x^{2} + (y-3)^{2} = 9.$


2748   

Explique por que a função é diferenciável no ponto dado. $f(x,y) = \dfrac{x}{x+y}, \quad (2,1)$.


As derivadas $f_{x}$ e $f_{y}$ de cada $f$ existem e são contínuas no ponto dado, logo $f$ é diferenciável.


2271   

No item abaixo :

  1.  determine o gradiente de $f$; 
  2.  calcule o gradiente no ponto $P$; e 
  3.  determine a taxa de variação de $f$ em $P$ na direção do vetor $\bf{u}$.

$f(x,y) = 5xy^2 - 4x^3y,  P = (1,2),  \bf{u} = \left( \frac{5}{13},\frac{12}{13} \right)$.



  1. $\nabla f(x,y) = (5y^{2} - 12x^{2}y, 10xy - 4x^{3}).$
  2. $\nabla f(1,2) = (-4, 16).$
  3. $\displaystyle \frac{172}{13}.$


1933   

Calcule a integral de linha, onde $C$ é a curva dada.

$\displaystyle\int_{C}x\,dx-y\,dy$,   $C$ é o segmento de extremidades $(1,1)$ e $(2,3)$, percorrido no sentido de $(1,1)$ para $(2,3).$


$\displaystyle -\frac{5}{2}.$


2482   

Use a integral tripla para determinar o volume do sólido dado.

  1.  $x^{2}+y^{2}\leq 4$ e $x^{2}+y^{2}+z^{2}\leq 9.$

  2.  $x^{2}+4y^{2}+9z^{2}\leq 1.$

  3.  $\dfrac{x^{2}}{a^{2}}+\dfrac{y^{2}}{b^{2}}+\dfrac{z^{2}}{c^{2}}$,  $(a>0,\;b>0\;e\;c>0).$

  4.  $x^{2}+y^{2}\leq z \leq 4x+2y.$


  1.  $\left(36 - \dfrac{20\sqrt{5}}{3} \right)\pi.$

  2.  $\dfrac{2\pi}{9}.$

  3.  $\dfrac{4\pi abc}{3}.$

  4.  $\dfrac{25\pi}{4}$


2353   

Calcule o volume do conjunto dado.

  1.  $x^{2}+4y^{2}\leq 4$ e $x+y\leq z\leq x+y+1.$

  2.  $x\geq 0$, $x\leq y\leq 1$ e $0\leq z\leq e^{y^{2}}.$


  1.  $2\pi.$

  2.  $\dfrac{e - 1}{2}.$


2330   

Determine a taxa de variação máxima de $f$ no ponto dado e a direção em que isso ocorre.
$f(x,y) = \dfrac{y^2}{x},  (2,4).$


$4\sqrt{2}.$


2251   

Seja $S$ a parte do parabolóide $z=2-x^{2}-y^{2}$ que está acima do plano $z=1.$ Calcule o fluxo do campo vetorial ${\bf F}(x,y,z)=\frac{1}{(x^{2}+y^{2}+z^{2})^{3/2}}(x,y,z)$ através de $S.$


2830   

Determine três números positivos cuja soma é $100$ e cujo produto é máximo.


$\displaystyle x = y = z = \frac{100}{3}.$


2941   

Calcule utilizando coordenadas esféricas. $\displaystyle\iiint\limits_{B}(x^{2}+y^{2}+z^{2})^{2}\,dV$, onde $B$ é a bola com centro na origem e raio $5.$


$\dfrac{312500\pi}{7}.$


2749   

Explique por que a função é diferenciável no ponto dado. $f(x,y) = e^{-xy} \cos{y}, \quad (\pi,0)$.


As derivadas $f_{x}$ e $f_{y}$ de cada $f$ existem e são contínuas no ponto dado, logo $f$ é diferenciável.


3027   

Esboce o sólido cujo volume é dado pela integral iterada

$$\int_{0}^{1}\!\!\int_{0}^{1-x}(1-x-y)\,dy dx.$$


ma211-list6-ex31_sol.png

1956   

Calcule a integral de linha $\int_{C}{\bf F}\cdot d{\bf r}$, onde $C$ é dada pela função vetorial ${\bf r}(t).$

${\bf F}(x,y,z)=x^{2}\,{\bf i}+y^{2}\,{\bf j}+z^{2}\,{\bf k}$, ${\bf r}(t)=(2\cos t,3\sin t,t)$, $0\leq t\leq 2\pi.$


$\dfrac{8\pi^{3}}{3}.$


2678   

Seja $f(x,y,z) = \dfrac{x}{x^2 + y^2 + z^2}$.

Verifique que

$$x\dfrac{\partial f}{\partial x} + y\dfrac{\partial f}{\partial y} + z\dfrac{\partial f}{\partial z} = -f.$$


$\displaystyle \frac{\partial f}{\partial x} = \frac{-x^{2} + y^{2} + z^{2}}{(x^{2} + y^{2} + z^{2})^{2}},\;\;\;\; \frac{\partial f}{\partial y} = \frac{-2xy}{(x^{2} + y^{2} + z^{2})^{2}} \;\;\;\;\;\text{e}\;\;\;\;\;\frac{\partial f}{\partial z} = \frac{-2xz}{(x^{2} + y^{2} + z^{2})^{2}}.$


3082   

De acordo com a lei dos gases ideais, a pressão, a temperatura e o volume de um gás confinado estão relacionados por \( P=kT/V\), onde \(k\) é uma constante. Use diferenciais para aproximar a variação percentual na pressão se a temperatura de um gás tiver crescido em \(3\%\) e o volume tiver crescido em \(5\%\).


2239   

Use o Teorema de Green na forma $\oint_{C} \mathbf{F} \cdot \mathbf{n} \, ds = \iint\limits_{ D} \text{div }{\mathbf{F}(x,y)} \, dA$ para demonstrar a primeira identidade de Green:

$$\iint\limits_{ D} f\nabla^2g \, dA = \oint_{C}f(\nabla{g}) \cdot \mathbf{n} \, ds - \iint\limits_{ D}\nabla{f} \cdot \nabla{g} \, dA,$$

em que $D$ e $C$ satisfazem as hipóteses do Teorema de Green e as derivadas parciais apropriadas de $f$ e $g$ existem e são contínuas. (A quantidade $ \nabla{g} \cdot \mathbf{n} = D_{\mathbf{n}}g$ aparece na integral de linha. Essa é a derivada direcional na direção do vetor normal $\mathbf{n}$ e é chamada derivada normal de $g$.)


Note que $\oint_{C} f(\nabla{g}) \cdot \mathbf{n} \, ds = \iint\limits_{ D} \text{div }(f \nabla g) \, dA = \iint\limits_{ D} f\text{div }(\nabla g) + \nabla{f} \cdot \nabla{g} \, dA.$


3090   

Sejam \(\alpha\), \(\beta\) e \(\gamma\) os ângulos de um triângulo.

  1.  Use multiplicadores de Lagrange para determinar o valor máximo de \(f(\alpha,\beta,\gamma)=\cos\alpha\cos\beta\cos\gamma\) e determine os ângulos para os quais o máximo ocorre.

  2.  Expresse \(f(\alpha,\beta,\gamma)\) como uma função apenas de \(\alpha\) e \(\beta\) e use um software de apoio computacional para fazer o gráfico dessa função de duas variáveis. Confirme que o resultado obtido no item anterior é consistente com o gráfico.


  1. \(\alpha = \beta=\gamma=\pi/3\) e valor máximo \(=1/8\).


2834   

Três alelos (versões alternativas de um gene) $A$, $B$ e $O$ determinam os quatro tipos de sangue: $A$ ($AA$ ou $AO$), $B$ ($BB$ ou $BO$), $O$ ($OO$) e $AB$. A Lei de Hardy-Weinberg afirma que a proporção de indivíduos em uma população que carregam dois alelos diferentes é $P=2pq+2pr+2rq$, onde $p$, $q$ e $r$ são as proporções de $A$, $B$ e $O$ na população. Use o fato de que $p+q+r=1$ para mostrar que $P$ é no máximo $\dfrac{2}{3}$.


É preciso maximizar de $P = 2q - 2q^{2} + 2r - 2r^{2} -2rq$ no conjunto delimitado pelas retas $q = 0,$ $r = 0$ e $q + r = 1.$ O ponto de máximo ocorre em $\displaystyle \left( \frac{1}{3}, \frac{1}{3} \right),$ no qual o valor de $P$ é justamente $\dfrac{2}{3}.$


1943   

Calcule a integral de linha, onde $C$ é a curva dada.

$\displaystyle\int_{C}(x+yz)\,dx+2x\,dy+xyz\,dz$,   $C$ consiste nos segmentos de reta de $(1,0,1)$ a $(2,3,1)$ e de $(2,3,1)$ a $(2,5,2).$


$\dfrac{97}{3}.$


2377   

Mostre que a operação de calcular o gradiente de uma função tem a propriedade fornecida. Suponha que $u$ e $v$ sejam funções de $x$ e $y$, diferenciáveis, e $a$ e $b$ sejam constantes.

$\nabla (au + bv) = a \nabla u + b \nabla v$

$\nabla\left(\dfrac{u}{v}\right) = \dfrac{v \nabla u - u \nabla v}{v^2}$

$\nabla(uv) = u \nabla v + v \nabla u$

$\nabla u^n = nu^{n-1}\nabla u$


Pelas propriedades análogas para derivadas parciais e a linearidade de vetores, os quatro itens são válidos.


3134   

Considere o campo vetorial \[\mathbf{F}(x,y,z)=(x-z)\mathbf{i}+(y-x)\mathbf{j}+(z-xy)\mathbf{k}. \]

  1.  Use o Teorema de Stokes para encontrar a circulação em torno do triângulo de vértices \(A=(1,0,0)\), \(B=(0,2,0)\) e \(C=(0,0,1)\), orientado no sentido anti-horário quando visto da origem para o primeiro octante.

  2.  Encontre a densidade de circulação de \(\mathbf{F}\) na origem na direção de \(\mathbf{k}\), ou seja, \(\displaystyle\mathrm{rot\,}\mathbf{F}(\mathbf{0})\cdot\mathbf{k}\).

  3.  Encontre o vetor unitário \(\mathbf{n}\) tal que a densidade de circulação de \(\mathbf{F}\) na origem seja máxima na direção de \(\mathbf{n}\).


  1.  \(\dfrac{3}{2}\)

  2.  \(-1\)

  3.  \(\displaystyle \mathbf{n}= -\dfrac{1}{\sqrt{2}}\mathbf{j} -\dfrac{1}{\sqrt{2}}\mathbf{k} \)


2313   

Determine a derivada direcional da função no ponto dado e na direção do vetor $\bf{v}$.

$f(x,y,z) = xe^y + ye^z + ze^x,  (0,0,0),  \bf{v} = \left(5,1,-2\right).$


$\displaystyle \frac{4}{\sqrt{30}}.$


2122   

Expresse $\partial z/\partial t$ em termos das derivadas parciais de $f$, sendo $z=f(x,y)$ e $x=\sin{3t}$ e $y=\cos{2t}.$


 $\displaystyle \frac{dz}{dt} (t) = 3 \cos(3t) \frac{\partial f}{\partial x}(\sin(3t),\cos(2t)) - 2\sin(2t) \frac{\partial f}{\partial y}(\sin(3t),\cos(2t)).$


2486   

Esboce o gráfico da função $f(x,y)=10-4x-5y$.


$z = 10 - 4x - 5y.$

ma211-list2-ex11_sol_c.png


1927   

Determine o campo vetorial gradiente de  $f(x,y,z) = \sqrt{x^2+y^2+z^2}$.


$\nabla f(x,y,z) = \dfrac{x\textbf{i} + y\textbf{j} + z \textbf{k}}{\sqrt{x^{2} + y^{2} + z^{2}}}.$


2599   

Calcule as seguintes integrais triplas.

  1.  $\displaystyle\iiint\limits_{  E}  \sqrt{x^2 + y^2} \, dV$, em que $E$ é a região que está dentro do cilindro   $x^2 + y^2 = 16$ e entre os planos $z = -5$ e $z = 4$.

  2.  $\displaystyle\iiint\limits_{  E}  y \, dV$, em que $E$ é o sólido que está entre os cilindros $x^2 + y^2 = 1$ e $x^2 + y^2 = 4$, acima do plano $xy$ e abaixo do plano $z = x + 2$.

  3.  $\displaystyle\iiint\limits_{  E}  x \, dV$, em que $E$ está delimitidado pelos planos $z = 0$ e $z = x + y + 5$ e pelos cilindros $x^2 + y^2 = 4$ e $x^2 + y^2 = 9$.


  1.  $384\pi$.

  2. $0$.

  3. $\dfrac{65\pi}{4}$.


2525   

Esboce o gráfico da função $f(x,y)=g(\sqrt{x^{2}+y^{2}})$. Em geral, se $g$ é uma função de uma variável, como saber o gráfico de $f(x,y)=g(\sqrt{x^{2}+y^{2}})$ a partir do gráfico de $g$?


O gráfico de $f(x,y) = g(\sqrt{x^{2} + y^{2}})$ pode ser obtido rotacionando o gráfico de $g$ no plano $xz$ ao redor do eixo $z.$

ma211-list2-ex22_sol_e.png


2225   

Determine se o campo vetorial $\mathbf{F}(x,y,z) = y^2z^3\mathbf{i} + 2xyz^3\mathbf{j} + 3xy^2z^2\mathbf{k}$ é conservativo ou não. Se for conservativo, determine uma função $f$ tal que $\mathbf{F} = \nabla{f}$.


$\mathbf{F}$ é conservativo. $f(x,y,z) = xy^2 z^3.$


2746   

Considere a função

$$f(x,y)=\begin{cases}\dfrac{xy}{x^{2}+y^{2}}, & \quad \text{se } (x,y) \neq (0,0),\\0, & \quad \text{se } (x,y)=(0,0).\\\end{cases}$$

  1. A função $f$ é contínua em $(0,0)$? Justifique sua resposta.

  2. Calcule as derivadas parciais $\dfrac{\partial f}{\partial x}(0,0)$ e $\dfrac{\partial f}{\partial y}(0,0).$

  3. Determine $\dfrac{\partial f}{\partial x}(x,y)$ e $\dfrac{\partial f}{\partial y}(x,y)$ para $(x,y)\neq (0,0).$

  4. $f$ é diferenciável em $(0,0)$? Justifique sua resposta.


  1. Não, pois $\displaystyle \lim_{(x,y) \to (0,0)} f(x,y)$ não existe.

  2. $\displaystyle \frac{\partial f}{\partial x}(0,0) = \frac{\partial f}{\partial y}(0,0) = 0$.

  3. $\displaystyle \frac{\partial f}{\partial x} = \frac{y^{3} - x^{2}y}{(x^{2} + y^{2})^{2}}\;\;\;\text{e}\;\;\; \frac{\partial f}{\partial y} = \frac{x^{3} - xy^{2}}{(x^{2} + y^{2})^{2}}$.

  4. Não, pois $f$ não é contínua em $(0,0)$ (ou: pois suas derivadas parciais não são contínuas em $(0,0)$).


2247   

Prove que se $\mathbf{F} \cdot \mathbf{n}$ for constante sobre $Im\,\mathbf{r}$, então o fluxo de $\mathbf{F}$ sobre $\mathbf{r}$ é o produto de $\mathbf{F} \cdot \mathbf{n}$ pelo comprimento de $\mathbf{r}$, em que $\mathbf{n}$ é normal a $\mathbf{r}$.


Direto da definição do fluxo de $\mathbf{F}$ através de $\bf{r}$ na direção $\bf{n}.$


2335   

Mostre que uma função diferenciável $f$ decresce mais rapidamente em $\bf{x}$ na direção oposta à do vetor gradiente, ou seja, na direção de $-\nabla f(\bf{x})$.


Se $\bf{u}$ é um versor e $\theta$ é o ângulo entre $\nabla f$ e $\bf{u},$ então
$$
D_{\bf{u} f} = \nabla f \cdot \bf{u} = |\nabla f||\bf u| \cos(\theta) = |\nabla f|\cos(\theta).
$$
O valor mínimo de $\cos(\theta)$ é $-1$ e isto ocorre quando $\theta = \pi.$ Portanto o valor mínimo de $D_{\bf{u}} f$ é $-|\nabla f|$ e ocorre quando $\theta = \pi,$ ou seja, quando $\bf{u}$ tem a direção oposta à de $\nabla f.$


2974   

Determine o jacobiano da transformação dada por: $x = uv, \quad y = \dfrac{u}{v}$.


$-\dfrac{2u}{v}.$


2008   

A força em um ponto $(x,y,z)$ em três dimensões é dada por ${\bf F}(x,y,z)=y\,{\bf i}+z\,{\bf j}+x\,{\bf k}$. Ache o trabalho realizado por ${\bf F}(x,y,z)$ ao longo da cúbica reversa $x=t$, $y=t^{2}$, $z=t^{3}$ de $(0,0,0)$ a $(2,4,8).$


$\dfrac{412}{15}.$


2471   

Seja $f(x,y,z)=e^{\sqrt{z-x^{2}-y^{2}}}.$

  1. Calcule $f(2,-1,6).$

  2. Determine o domínio de $f$.

  3. Determine a imagem de $f$.


  1. $e.$

  2. $\left\lbrace (x,y,z): \;z \geq x^{2} + y^{2} \right\rbrace.$

  3. $[1,\infty).$


2499   

Dada a função $f(x,y)=\ln (x^{2}+y^{2})$.

  1. Encontre o domínio da função.

  2. Encontre a imagem da função.

  3. Descreva as curvas de nível da função.


  1. $D_{f} = \left\lbrace (x,y);\; (x,y) \neq (0,0) \right\rbrace$.

  2. $Im(f) = \mathbb{R}.$

  3. As curvas de nível são os círculos $x^{2} + y^{2} = C$ com $C > 0.$


3063   

Determine o campo vetorial gradiente $\nabla f$ de $f(x,y) = x^2-y$ e o esboce.


2625   

Utilizando o Teorema de Stokes, transforme a integral $\displaystyle\iint_{ S}\mbox{rot}{\bf F}\cdot{\bf n}dS$ numa integral de linha e calcule.

  • ${\bf F}(x,y,z) = y{\bf i} + x^2{\bf j} + z{\bf k}$, $S$ a superfície $x^2+y^2 = 1$, $0\leq z \leq 1$ e $y\geq 0$, sendo ${\bf n}$ a normal com componente $y\geq 0$.


$0$.


2090   

Considere o campo

$${\bf F}(x,y,z)=(e^{z},2yz, xe^{z}+y^{2}).$$


  1. Verifique se o campo ${\bf F}$ é conservativo.

  2. Se ${\bf F}$ for conservativo, calcule $f(x,y,z)$ tal que $\nabla f={\bf F}.$

  3. Calcule a integral de linha $\int_{C}{\bf F}\cdot d{\bf r}$ onde $C$ é dada por $r(t)=(\cos t, \sin t, t)$, $0\leq t\leq 2\pi.$


Sim.

$f(x,y) = x e^{z} + y^{2} z.$

$e^{2\pi} - 1.$


2311   

Determine a derivada direcional da função no ponto dado e na direção do vetor $\bf{v}$.

$f(x,y) = 1 + 2x\sqrt{y},  (3,4),  \bf{v} = \left(4, -3\right).$


$\displaystyle \frac{23}{10}.$


2022   

Considere a função
$$f(x,y) = \begin{cases}
x + y, & \quad \text{se } xy = 0,\\
1, & \quad \text{caso contrário}.
\end{cases}$$
Mostre que $f$ não possui derivada direcional em $(0,0)$ na direção de um vetor $\bf{v} = (a,b)$ com $a^2 + b^2 = 1$ e $ab \neq 0$.



Seja $\bf{v} = (a,b)$ um vetor unitário (isto é, $a^2+b^2 = 1$), em que $ab \neq 0$. A derivada direcional em $(0,0)$ na direção do vetor unitário $\bf{v}$ existe se o limite

$$\lim_{h \to 0} \frac{f(0+ah,0+bh)-f(0,0)}{h}$$

existir. Para $h \neq 0$, temos $(ah)(bh) \neq 0$. Logo $f(ah,bh) = 1$. Assim, o limite em questão se reduz a

$$\lim_{h \to 0} \frac{1}{h},$$

e esse limite não existe. Como o vetor $\bf{v}$ satisfazendo as hipóteses foi tomado arbitrariamente, concluímos que $f$ não possui derivada direcional em $(0,0)$ na direção de nenhum vetor $\bf{v} = (a,b)$ que satisfaça $a^2 + b^2 = 1$ e $ab \neq 0$.


2541   

Encontre o fluxo do campo ${\bf F}$ ao longo da porção da superfície dada no sentido especificado.

  • ${\bf F}(x,y,z)=-{\bf i}+2{\bf j}+3{\bf k}$; $S$ é a superfície retangular $z=0$, $0\leq x\leq 2$, $0\leq y \leq 3$, sentido ${\bf k}.$


$18.$



3079   

Mostre que os limites não existem, considerando que \((x,y)\rightarrow (0,0) \) ao longo dos eixos coordenados.

  1.  \[ \lim_{(x,y)\to(0,0)}\dfrac{3}{x^2+2y^2} \]

  2.  \[ \lim_{(x,y)\to(0,0)}\dfrac{x+y}{2x^2+y^2} \]


2618   

Uma partícula se move ao longo de segmentos de reta da origem aos pontos $(1,0,0)$, $(1,2,1)$, $(0,2,1)$ e de volta para a origem sob a influência do campo de forças ${\bf F}(x,y,z) = z^2{\bf i} + 2xy{\bf j} + 4y^2{\bf k}.$ Encontre o trabalho feito.


$3$.


3039   

Esboce a região cuja área é dada pela integral e calcule-a, sendo: $\displaystyle\int_{0}^{\pi/2}\int_{0}^{4\cos{\theta}}   r \,drd\theta.$


$2\pi;$ região de integração:

ma211-list7-ex6_sol_b.png



2043   

Aplique o Teorema da Divergência para achar $\displaystyle\iint\limits_{S}{\bf F}\cdot {\bf n}\,dS.$, sendo ${\bf F}(x,y,z)=y^{3}e^{z}\,{\bf i}-xy\,{\bf j}+x \cdot \arctan y\,{\bf k}$ e $S$ a superfície da região delimitada pelos planos coordenados e o plano $x+y+z=1.$



Pelo Teorema do Divergente, temos
$$\iint\limits_{S}{\bf F}\cdot {\bf n}\,dS = \displaystyle\iiint\limits_{E}\text{div }{\bf F}\,dV,$$
em que $E$ é o sólido

ma211-list15-ex1_sol.png

que pode ser escrito como
$E = \{(x,y,z) \in \mathbb{R}^3: 0 \leq x \leq 1, 0 \leq y \leq 1-x \mbox{ e } 0 \leq z \leq 1-x-y\}.$
Observe que
\begin{array}{rcl}\text{div }{\bf F} & = & \dfrac{\partial}{\partial x}(y^3e^z) + \dfrac{\partial}{\partial y}(-xy) + \dfrac{\partial}{\partial z}(x\arctan{y}) \\& = & 0 - x + 0 \\& = & -x.\end{array}
Assim,
\begin{array}{rcl}\iint\limits_{S}{\bf F}\cdot {\bf n}\,dS & = & \displaystyle\iiint\limits_{E}{\bf F}\,dV \\& = & \iiint\limits_{E}-x\,dV \\& = & \int_{0}^{1}\int_{0}^{1-x}\int_{0}^{1-x-y}-x\,dz dy dx \\& = & \int_{0}^{1}\int_{0}^{1-x}-x(1-x-y)\,dy dx \\& = & \int_{0}^{1}\left(-\frac{x}{2}+x^2-\frac{x^3}{3}\right)\,dx \\& = & -\frac{1}{12}.\end{array}


3142   

Considere o campo vetorial \(\mathbf{F}(x,y,z)=(x-y)\mathbf{i} + (y-z)\mathbf{j}+(z-x)\mathbf{k}\) e a superfície \(\sigma\)
descrita como sendo a porção do plano \(x+y+z=1\) no primeiro octante e orientada para cima. Verifique o Teorema de Stokes
calculando, separadamente, a integral de linha e a integral dupla e, em seguida, comparando os valores.


\(\dfrac{3}{2}\)


1984   

Determine a derivada da função vetorial.

  1. ${\bf r}(t)=(\tan (t), \sec (t), 1/t^{2})$
  2. ${\bf r}(t)=\sin^{-1}(t){\bf i}+\sqrt{1-t^{2}}{\bf j}+{\bf k}$


1985   

 Determine o vetor tangente unitário ${\bf T}(t)$ no ponto com valor de parâmetro $t$ dado, sendo 
${\bf r}(t)=\cos(t){\bf i}+3t{\bf j}+2\sin(2t){\bf k}$ e $t=0.$


2743   

Determine os planos tangentes ao gráfico de $f(x,y) = 2 + x^2 + y^2$ e que contenham o eixo $x$.


$z = 2\sqrt{2} y$ e $z = -2\sqrt{2} y.$


2639   

Determine as derivadas parciais de primeira ordem da função $f(x,y)=\dfrac{x-y}{x+y}$.


$\displaystyle \frac{\partial f}{\partial x} = \frac{2y}{(x + y)^{2}}\;\;\;\text{e}\;\;\; \frac{\partial f}{\partial y} = -\frac{2x}{(x + y)^{2}}$.


3019   

Esboce a região de integração para a integral iterada $\displaystyle\int_{\pi}^{2\pi}\!\!\int_{\sin{y}}^{\ln(y)}f(x,y)\,dx dy$.


ma211-list6-ex24_sol_c.png

2569   

Calcule $\displaystyle \lim_{(x,y) \to (0,0)} \dfrac{x}{\sqrt{x^2 + y^2}}$, caso exista.


Não existe.


2990   

Calcule a integral, efetuando uma mudança de variáveis apropriada. $\displaystyle\iint\limits_{R} x^2 \, dA$, em que $R$ é o conjunto de todos $(x,y)$ tais que $4x^2 + y^2 \leq 1$.


$\dfrac{\pi}{32}.$


2562   

Determine o maior conjunto no qual a função $F(x,y) = \dfrac{1}{x^2 - y}$ é contínua.


$\left\lbrace (x,y);\; y \neq x^{2} \right\rbrace.$


2136   

Mostre que cada a equação a seguir define implicitamente pelo menos uma função diferenciável $z=z(x,y)$. 
Expresse $\partial z /\partial x$ e $\partial z/\partial y$ em termos de $x$, $y$ e $z.$
$x^{3}+y^{3}+z^{3}=x+y+z$


 $\displaystyle \frac{\partial z}{\partial x} = -\frac{3x^{2} - 1}{3z^{2} - 1}$ e $\displaystyle \frac{\partial z}{\partial y} = -\frac{3y^{2} - 1}{3z^{2} - 1}.$


2826   

Suponha que $T(x,y)=4-x^{2}-y^{2}$ represente uma distribuição de temperatura em uma região que pode ser aproximada por um plano. Seja $D=\{(x,y)\in \mathbb{R}^2: x\geq 0,\;y\geq x\;\text{e}\;2y+x\leq 4\}$. Determine o ponto de $D$ de menor temperatura.


$(0,2).$


2357   

Calcule o volume do conjunto dado.

  1.  $4x+2y\geq z\geq 3x+y+1$, $x\geq 0$ e $y\geq 0.$

  2.  $0\leq z\leq \sin{y^{3}}$ e $\sqrt{x}\leq y\leq \sqrt[3]{\pi}.$


  1.  $\dfrac{1}{6}.$

  2.  $\dfrac{2}{3}.$


2020   

Utilize a Regra da Cadeia para determinar $\mathrm{\partial}z/\mathrm{\partial} s$ e $\mathrm{\partial}z/ \mathrm{\partial}t,$ onde
$$z=\sin{\theta}\cos{\phi}, \quad \theta=st^{2}, \quad \phi=s^{2}t.$$



Utilizando a Regra de Cadeia, obtemos
\begin{eqnarray*}
\frac{\partial z}{\partial s} & = & \frac{\partial z}{\partial \theta}\frac{\partial \theta}{\partial s}+\frac{\partial z}{\partial \phi}\frac{\partial \phi}{\partial s} \\
& = & (\cos{\theta}\cos{\phi})(t^2) + (\sin{\theta}(-\sin{\phi}))(2st) \\
& = & t^2\cos(st^2)\cos(s^2t) - 2st\sin(st^2)\sin(s^2t)
\end{eqnarray*}
e
\begin{eqnarray*}
\frac{\partial z}{\partial t} & = & \frac{\partial z}{\partial \theta}\frac{\partial \theta}{\partial t}+\frac{\partial z}{\partial \phi}\frac{\partial \phi}{\partial t} \\
& = & (\cos{\theta}\cos{\phi})(2st) + (\sin{\theta}(-\sin{\phi}))(s^2) \\
& = & 2st\cos(st^2)\cos(s^2t) - s^2\sin(st^2)\sin(s^2t).
\end{eqnarray*}


2654   

Determine as derivadas parciais de $f(x,y)=5x^{4}y^{2}+xy^{3}+4$.


$\displaystyle \frac{\partial f}{\partial x} = 20x^{3}y^{2} + y^{3}\;\;\;\;\;\;\text{e}\;\;\;\;\; \frac{\partial f}{\partial y} = 10x^{4}y + 3xy^{2}.$



2679   

Seja $s = f(x,y,z,w)$ dada por $s = e^{\frac{x}{y} - \frac{z}{w}}$. Verifique que

$$x\dfrac{\partial s}{\partial x} + y \dfrac{\partial s}{\partial y} + z \dfrac{\partial s}{\partial z} + w \dfrac{\partial s}{\partial w} = 0.$$


$\begin{aligned}[t]\frac{\partial s}{\partial x} &= \frac{1}{y} e^{\frac{x}{y} - \frac{z}{w}},\;\;\;\;\;\frac{\partial s}{\partial y} = -\frac{x}{y^{2}} e^{\frac{x}{y} - \frac{z}{w}},\\\frac{\partial s}{\partial z} &= -\frac{1}{w} e^{\frac{x}{y} - \frac{z}{w}}\;\;\;\;\;\text{e}\;\;\;\;\; \frac{\partial s}{\partial w} = \frac{z}{w^{2}} e^{\frac{x}{y} - \frac{z}{w}}.\end{aligned}$


2257   

Suponha que $S$ e $E$ satisfaçam as condições do Teorema do Divergente e que $f$ seja uma função escalar com derivadas parciais contínuas. Demonstre que $\displaystyle\iint\limits_{S}f{\bf n}\,dS=\iiint\limits_{E}\nabla f\,dV.$ Estas integrais de superfície e triplas de funções vetoriais são vetores definidos integrando cada função componente. [Sugestão: comece aplicando o Teorema do Divergente a ${\bf F}=f{\bf c}$, onde ${\bf c}$ é um vetor constante arbitrário.]



Note que se ${\bf n} = n_{1} {\bf i} + n_{2} {\bf j} + n_{3} {\bf k},$ então

\begin{align*} &\iint_{S} f \cdot {\bf n}\,dS \\ &= \left( \iint_{S} f n_{1}\,dS \right) {\bf i} + \left( \iint_{S} fn_{2}\,dS\right) {\bf j} + \left( \iint_{S} fn_{3}\,dS\right) {\bf k}\\ &= \left( \iiint_{E} \dfrac{\partial f}{\partial x}\,dV \right) {\bf i}+ \left( \iiint_{E} \dfrac{\partial f}{\partial y}\,dV\right) {\bf j} + \left( \iiint_{E} \dfrac{\partial f}{\partial z}\,dV \right) {\bf k}. \end{align*}


2629   

Utilizando o Teorema de Stokes, transforme a integral $\displaystyle\iint_{ S}\mbox{rot}{\bf F}\cdot{\bf n}dS$ numa integral de linha e calcule.

  • ${\bf F}(x,y,z) = -y{\bf i} + x{\bf j} + x^2{\bf k}$, $S$ a superfície $x^2+y^2+z^2 = 4$, $\sqrt{2} \leq z \leq \sqrt{3}$ e $y \geq 0$, sendo ${\bf n}$ a normal apontando para cima.


$\pi$.


2108   

Sejam $f(x)$ e $g(x)$ duas funções contínuas, respectivamente, nos intervalos $[a,b]$ e $[c,d].$ Use o seguinte resultado $$\iint\limits_{R}f(x)g(y)\,dx dy=\bigg(\int_{a}^{b}f(x)\,dx\bigg)\bigg(\int_{c}^{d}g(y)\,dy\bigg),$$ onde $R$ é o retângulo $a\leq x\leq b$ e $c\leq y\leq d$, para calcular as integrais

  1.  $\displaystyle\int\!\!\!\!\int\limits_{\!\!\!\!\!\! R} xy^{2}\,dx dy$, onde $R$ é o retângulo $1\leq x\leq 2,\;2\leq y\leq 3.$

  2.  $\displaystyle\int\!\!\!\!\int\limits_{\!\!\!\!\!\! R} x\cos(2y)\,dx dy$, onde $R$ é o retângulo $0\leq x\leq 1,\;-\dfrac{\pi}{4}\leq y\leq \dfrac{\pi}{4}.$


  1.  $\dfrac{19}{2}.$

  2.  $\dfrac{1}{2}.$


2449   

Calcule $\displaystyle\iint\limits_{S}g(x,y,z)dS,$ onde $g(x,y,z)=x+y$ e $S$ é parte do primeiro octante do plano $2x+3y+z=6.$


$5\sqrt{14}.$


2431   

Calcule a integral tripla.

  1.  $\displaystyle\iiint\limits_{  E}(x^{2}+z^{2})\;dx dy dz$, onde $E$ é o cilindro $x^{2}+y^{2}\leq 1$ e $0\leq z \leq 1.$

  2.  $\displaystyle\iiint\limits_{  E}\;dx dy dz$, onde $E$ é o conjunto $x^{2}+y^{2}\leq z \leq 2x+2y-1.$


  1.  $\dfrac{7\pi}{12}.$

  2.  $\dfrac{\pi}{2}.$


3137   

Calcule o trabalho realizado pelo campo vetorial \[ \mathbf{F}(x,y,z) = x^2\mathbf{i}+4xy^3\mathbf{j}+y^2x\mathbf{k}\] sobre uma partícula que percorre o caminho \(C\) definido como o bordo da superfície \(\sigma\) contida no plano \(z=y\) e cuja projeção no plano \(xy\) corresponde ao retângulo \(R=\{(x,y)\in\mathbb{R}^2; 0\leq x\leq 1\),\ \(0\leq y\leq 3\}\). O sentido de percurso é tal que a fronteira de \(R\) é percorrida no sentido horário.



Note que calcular o trabalho \(\displaystyle W= \oint_C\mathbf{F}\cdot\,d\mathbf{r}\) assim diretamente exigiria quatro integrações separadas, uma para cada lado do retângulo. Entretanto, usando o Teorema de Stokes podemos, em vez disso, calcular uma (única!) integral de superfície \[ W= \iint\limits_\sigma\mathrm{rot\,}\mathbf{F}\cdot\mathbf{n}\,dS \] na qual \(\sigma\) é tomada com a orientação para baixo, como requerido pelo Teorema de Stokes. Como a superfície \(\sigma\) está contida no plano \(z=y\) e \[\mathrm{rot\,}\mathbf{F} = \left|\begin{array}{ccc} \mathbf{i} & \mathbf{j} & \mathbf{k} \\ \dfrac{\partial}{\partial x} & \dfrac{\partial}{\partial y} & \dfrac{\partial}{\partial z} \\ x^2 & 4xy^3 & xy^2 \end{array}\right| = 2xy\mathbf{i}-y^2\mathbf{j}+4y^3\mathbf{k}, \] segue então que \begin{align*} W= \iint\limits_\sigma\mathrm{rot\,}\mathbf{F}\cdot\mathbf{n}\,dS & = \iint\limits_R\mathrm{rot\,}\mathbf{F}\cdot\left( \dfrac{\partial z}{\partial x}\mathbf{i} +\dfrac{\partial z}{\partial y}\mathbf{j} - \mathbf{k}\right)\,dA \\   & = \iint\limits_R\left(2xy\mathbf{i}-y^2\mathbf{j}+4y^3\mathbf{k}\right)\cdot\left(0\mathbf{i}+\mathbf{h}-\mathbf{k}\right)\,dA \\   & = \int_0^1\int_0^3(-y^2-4y^3)\,dydx \\   & = - \int_0^1\left[\dfrac{y^3}{3}+y^4\right]_{y=0}^3\,dx \\   & = -\int_0^1 90\,dx = -90. \end{align*}


2648   

São mostradas as curvas de nível de uma função $f.$ Determine se as seguintes derivadas parciais são positivas ou negativas no ponto $P.$

ma211-list3-ex14.png

  1. $f_{x}$

  2. $f_{xx}$

  3. $f_{yy}$$f_{y}$

  4. $f_{xy}$


  1. Negativa

  2. Positiva

  3. Positiva

  4. Negativa

  5. Positiva


2636   

O índice de sensação térmica $W$ é a temperatura sentida quando a temperatura real é $T$ e a velocidade do vento, $v$. Portanto, podemos escrever $W=f(T,v)$. Considerando a tabela abaixo:

ma211-list3-ex6.png

  1. Estime os valores de $f_{T}(-15,30)$ e $f_{v}(-15,30)$. Quais são as nterpretações práticas desses valores?

  2. Em geral, o que se pode dizer sobre o sinal de $\partial W/\partial T$ e $\partial W/\partial v$?

  3. Qual parece ser o valor do seguinte limite

    $$\lim_{v\rightarrow \infty}\frac{\partial W}{\partial v}?$$


  1. $f_{T}(-15,30) \approx 1.3$ Isto significa que quando a temperatura real é $-15º$C e a velocidade do vento é $30$km/h, a temperatura aparente aumenta cerca de $1.3º$C para cada $1º$C que a temperatura real aumenta;\\

  2. $f_{v}(-15,30) \approx -0.15$ Isto significa que quando a temperatura real é $-15º$C e a velocidade do vento é $30$km/h, a temperatura aparente diminui cerca de $0.15º$C para cada $1$km/h que a velocidade do vento aumenta.  

  3. $\frac{\partial W}{\partial T} > 0$ e  $\frac{\partial W}{\partial v} \leq 0.$

    $\lim_{v \to \infty} \frac{\partial W}{\partial v} = 0.$


2128   

Seja $z=f(u-v,v-u)$. Verifique que 
$$\frac{\partial z}{\partial u}+\frac{\partial z}{\partial v}=0.$$



Note que $\displaystyle \frac{\partial z}{\partial u}(u,v) = \frac{\partial f}{\partial x}(u-v,v-u) - \frac{\partial f}{\partial y}(u-v,v - u)$ e $\displaystyle \frac{\partial z}{\partial v}(u,v) = -\frac{\partial f}{\partial x}(u-v,v-u) + \frac{\partial f}{\partial y}(u-v,v - u).$


2012   

  1. Mostre que um campo de força constante realiza trabalho nulo sobre um partícula que dá uma única volta completa uniformemente na circunferência $x^{2}+y^{2}=1.$

  2. Isso também é verdadeiro para um campo de força ${\bf F}({\bf x})=k{\bf x}$, onde $k$ é uma constante e $\textbf{x}=x{\bf i}+y{\bf j}$?


  1. Dica: tome a parametrização do círculo $C$ dada por $x = cos(t)$ e $y = \sin(t),$ com $t \in [0,2\pi]$ e considere um campo constante arbitrário ${\bf F} = (a,b).$ Segue que $W = \int_{C} F\cdot d\textbf{r} = 0.$

  2. Sim. Realize o mesmo cálculo com ${\bf F}(x,y) = (k x, ky).$


2259   

Considere a integral $$\int_{0}^{1}\int_{3y}^{3}e^{x^{2}}\,dx dy.$$

  1.  Esboce a região de integração.

  2.  Calcule a integral usando a ordem de integração apropriada.


  1. (...)

  2.  $\dfrac{e^9 - 1}{6}.$


2756   

Verifique que a função $f(x,y) = x^4 + y^3$ é diferenciável.


As derivadas parciais $\frac{\partial f}{\partial x}$ e $\frac{\partial f}{\partial y}$ de cada função $f$ existem e são contínuas em todos os pontos do domínio.


2171   

Determine o trabalho $W = \int_{C}\mathbf{F}\cdot\, d\mathbf{r}$ realizado pelo campo de força

$$\mathbf{F}(x,y) = x\mathbf{i} + (x^3 + 3xy^2)\mathbf{j}$$

em uma partícula que inicialmente está no ponto $(-2,0)$, se move ao longo do eixo $x$ para $(2,0)$ e então se move ao longo da semicircunferência $y = \sqrt{4-x^2}$ até o ponto inicial.


$12\pi.$


2683   

Encontre $\partial f/\partial x$ e $\partial f/\partial y$ para $f(x,y)=e^{-x}\;\sin(x+y)$.


$\displaystyle \frac{\partial f}{\partial x} = -e^{-x} \sin(x + y) + e^{-x}\cos(x + y) \;\;\;\;\text{e}\;\;\;\; \frac{\partial f}{\partial y} = e^{-x}\cos(x + y)$.


2944   

Calcule utilizando coordenadas esféricas. $\displaystyle\iiint\limits_{E}xyz\,dV$, onde $E$ está entre as esferas $\rho=2$ e $\rho=4$ e acima do cone $\phi=\pi/3.$


$0.$


2616   

Verifique que o Teorema de Stokes é verdadeiro para o campo vetorial ${\bf F}$ dado e a superfície $S$.

  • ${\bf F}(x,y,z) = y{\bf i} + z{\bf j} + x{\bf k}$, $S$ é o hemisfério $x^2+y^2+z^2=1$, $y \geq 0$, orientado na direção positiva do eixo $y$.


$\displaystyle\int_{C} {\bf F} \cdot d{\bf R} = \displaystyle\iint_{S} \mbox{rot} {\bf F} \cdot d{\bf S} = -\pi$.


2388   

Determine as equações do plano tangente e da reta normal à superfície dada, no ponto dado.

$ze^{x - y} + z^3 = 2$ em $(2,2,1)$.


 Plano tangente: $x - y + 4z = 4$,
Reta normal: $(x,y,z) = (2,2,1) + \lambda (1,-1,4),$ $\lambda \in \mathbb{R}.$


2810   

Determine os valores máximos e mínimos locais e pontos de sela da função $f(x,y)=xy+2x-\ln(x^{2}y)$.


Ponto de mínimo: $\displaystyle \left(\frac{1}{2},2\right).$


2024   

Se $f$ é uma função constante, $f(x,y) = k$, e $R = [a,b] \times [c,d],$ mostre que $\iint \limits_{R} k \, dA = k(b-a)(d-c).$


Note que se $R$ for dividida em $mn$ subretângulos, vale $$ \sum^{m}_{i = 1} \sum^{n}_{j = 1} f(x_{ij}^{*}, y_{ij}^{*}) \Delta A = k \sum^{m}_{i = 1} \sum^{n}_{j = 1} \Delta A = k (b - a) (d - c), $$ independentemente dos pontos amostrais $(x_{ij}^{*}, y_{ij}^{*})$ escolhidos.


2933   

Calcule a integral em coordenadas esféricas. $\displaystyle\int_{0}^{3\pi/2}\int_{0}^{\pi}\int_{0}^{1}5\rho^{3}\sin^{3}{\phi}\,d\rho d\phi d\theta$.


$\dfrac{5\pi}{2}.$


3146   

Sejam \(\alpha\) e \(\beta\) dois ângulos que satisfazem \(\displaystyle 0<\beta-\alpha\leq 2\pi\) e suponha que \( r= f(\theta)\) seja uma curva polar lisa com \(f(\theta)>0\) no intervalo \([\alpha,\beta]\). Use a fórmula \[ A = \dfrac{1}{2}\int_C-y\,dx+x\,dy \] para encontrar a área da região \(R\) englobada pela curva \(r=f(\theta)\) e os raios \(\theta=\alpha\) e \(\theta=\beta\).


1961   

Calcule a integral de linha $\displaystyle\int_{C}(x-y)\,dx+e^{x+y}\, dy$, onde $C$ é a fronteira do triângulo de vértices $(0,0)$, $(0,1)$ e $(1,2)$, orientada no sentido anti-horário.


$\displaystyle \frac{e^{3}}{6} - \frac{e}{2} + \frac{5}{6}.$


2412   

Determine o volume do sólido que se encontra abaixo do plano $3x+2y+z=12$ e acima do retângulo $R=\{(x,y) \in \mathbb{R}^2|\;0\leq x\leq 1,\;-2\leq y\leq 3\}.$


$\dfrac{95}{2}.$


2795   

Determine os valores máximos e mínimos locais e pontos de sela da função $f(x,y)=xy-2x-y$.


Ponto de mínimo: $(2,1);$ ponto de sela: $(0,0).$


2771   

Uma carga elétrica é distribuída sobre um disco $x^2 + y^2 \leq 4$ de modo que a densidade de carga em $(x,y)$ é $\sigma(x,y) = x + y + x^2 + y^2$ (medida em coulombs por metro quadrado). Determine a carga total do disco.



Como a carga elétrica é distribuída sobre o disco $x^2 + y^2 \leq 4$, em coordenadas polares temos que $0\leq r \leq 2$ e $0\leq \theta \leq 2\pi.$ Temos que $$Q=\iint\limits_{D}\sigma(x,y)\,dA=\iint\limits_{D}(x+y+x^{2}+y^{2})\,dA$$ $$=\int_{0}^{2\pi}\int_{0}^{2}(r\,\cos \theta+r\,\sin \theta+r^{2})r\,dr\, d \theta=\int_{0}^{2\pi}\int_{0}^{2}(r^{2}\,\cos \theta+r^{2}\,\sin \theta+r^{3})\,dr\, d \theta$$ $$=\int_{0}^{2\pi}\bigg(\frac{r^{3}}{3}\cos \theta+\frac{r^{3}}{3}\sin \theta +\frac{r^{4}}{4}\bigg)\bigg|_{0}^{2}\,d\theta= \int_{0}^{2\pi}\bigg(\frac{8}{3}\cos \theta+\frac{8}{3}\sin \theta+4\bigg)\,d\theta$$ $$=\bigg(\frac{8}{3}\sin\theta-\frac{8}{3}\cos\theta+4\theta\bigg)\bigg|_{0}^{2\pi}=\bigg(-\frac{8}{3}+8\pi\bigg)-\bigg(-\frac{8}{3}\bigg)$$ $$=-\frac{8}{3}+8\pi+\frac{8}{3}=8\pi.$$


2544   

Encontre o fluxo exterior do campo ${\bf F}=2xy{\bf i}+2yz{\bf j}+2xz{\bf k}$ ao longo da superfície do cubo cortado do primeiro octante pelos planos $x=a$, $y=a$ e $z=a.$


$3\pi a^4.$


2359   

Seja $S$ a parte do cone $x^{2}=y^{2}+z^{2}$ que está dentro do cilindro $x^{2}+y^{2}=a^{2}$ e no primeiro octante. Determine a área da superfície $S.$


$\dfrac{\pi a^2}{4}$.


2338   

Determine a área da superfície $z=\frac{2}{3}(x^{3/2}+y^{3/2})$, $0\leq x \leq 1$ e $0\leq y\leq 1.$


$\dfrac{4}{15}(3^{5/2} - 2^{7/2} + 1).$


2199   

 Se $f(u,v,w)$ é diferenciável, $u=x-y$, $v=y-z$ e $w=z-x$, mostre que 
$$\frac{\partial f}{\partial x}+\frac{\partial f}{\partial y}+\frac{\partial f}{\partial z}=0.$$


Note que $\displaystyle \frac{\partial f}{\partial x} = \frac{\partial f}{\partial u} - \frac{\partial f}{\partial w}, $$\displaystyle \frac{\partial f}{\partial y} = -\frac{\partial f}{\partial u} + \frac{\partial f}{\partial v}$ e $\displaystyle \frac{\partial f}{\partial z} = -\frac{\partial f}{\partial v} + \frac{\partial f}{\partial w}.$


2293   

Determine uma representação paramétrica para a superfície descrita a seguir. O paraboloide $z=x^{2}+y^{2}$, \, $z\leq 4.$


$x = r \cos(\theta),$ $y = r \sin(\theta),$ $z = r^2,$ onde $0 \leq r \leq 2$ e $0\leq \theta \leq 2\pi.$


2857   

Seja

$$f(x,y)=k(x-y)^{2}+\frac{y^{4}}{2}-\frac{y^{2}}{2},\;\;k\neq 0.$$

  1. Encontre os pontos críticos da função $f$.

  2. Classifique os pontos críticos da função $f$ no caso em que $k>0$.


  1. $(0,0), (1,1)$ e $(-1,-1).$

  2. Pontos de mínimo: $(1,1)$ e $(-1,-1);$ ponto de sela: $(0,0).$


3111   

As equações paramétricas \[\begin{array}{lll} x=u, & y=u\cos v, & z=u\sin v \end{array}\] representam o cone que resulta quando a reta \(y=x\) do plano \(xy\) é girada em torno do eixo \(x\). Determine a área de superfície da parte do cone para a qual \(0\leq u\leq 2\) e \(0\leq v\leq 2\pi\).



Sendo \(\displaystyle\{\mathbf{i},\mathbf{j},\mathbf{k}\}\) a base canônica do espaço, a superfície pode ser representada vetorialmente como \[ \mathbf{r}=u\mathbf{i}+u\cos v\mathbf{j}+u\sin v\mathbf{k} \ \  \left(0\leq u\leq 2,\ 0\leq v\leq 2\pi\right). \] Assim, teremos  \begin{align*} \dfrac{\partial\mathbf{r}}{\partial u} & = \mathbf{i} + \cos v\mathbf{j} + \sin v\mathbf{k} \\ \dfrac{\partial\mathbf{r}}{\partial v} & = - u\sin v\mathbf{j} + u\cos v\mathbf{k} \\ \dfrac{\partial\mathbf{r}}{\partial u}\times\dfrac{\partial\mathbf{r}} {\partial v} & = \left|\begin{array}{ccc} \mathbf{i} & \mathbf{j} & \mathbf{k} \\ 1 & \cos v & \sin v \\ 0 & -u\sin v & u\cos v \end{array} \right| = u\mathbf{i} -u\cos v\mathbf{j} - u\sin v\mathbf{k} \\ \|\dfrac{\partial\mathbf{r}}{\partial u}\times \dfrac{\partial\mathbf{r}}{\partial v}\| & = \sqrt{u^2+(-u\cos v)^2+(-u\sin v)^2} = |u|\sqrt{2} = u\sqrt{2}. \end{align*} Segue, portanto, que \[ S = \iint\limits_R\|\dfrac{\partial\mathbf{r}}{\partial u}\times \dfrac{\partial\mathbf{r}}{\partial v}\|\,dA = \int_0^{2\pi}\int_0^2\sqrt{2}u\,dudv = 2\sqrt{2}\int_0^{2\pi}\,dv = 4\pi\sqrt{2}. \]


1942   

Calcule a integral de linha, onde $C$ é a curva dada.

$\displaystyle\int_{C}x^{2}y\sqrt{z}\,dz$,   $C:\,x=t^{3},\, y=t,\, z=t^{2},\, 0\leq t\leq 1.$


$\dfrac{1}{5}.$


2537   

Suponha que o sólido tenha densidade constante $k$. Encontre os momentos de inércia para um cubo com comprimento do lado $L$ se um vértice está localizado na origem e três arestas estão nos eixos coordenados.


$\displaystyle I_{x} = I_{y} = I_{z} = \dfrac{2kL^5}{3}.$


2095   

Seja ${\bf F}:\Omega\subset \mathbb{R}^{n}\rightarrow \mathbb{R}^{n}$ contínuo no aberto $\Omega$. Prove que uma condição necessária para que ${\bf F}$ seja

conservativo é que $\int_{C}{\bf F}\cdot d{\bf r}=0$ para toda curva $C$ fechada, de classe $C^{1}$ por partes, com imagem contida em $\Omega.$


Se $C$ é uma curva fechada em $\Omega$  parametrizada por $\mathbf{r}(t),$ com $a \leq t \leq b,$ $\mathbf{r}(a) = \mathbf{r}(b)$ e $\mathbf{F} = \nabla f,$ então $\int_{C}{\bf F}\cdot d{\bf r} = f(\mathbf{r}(a)) - f(\mathbf{r}(b)) = 0.$


2369   

A superfície de um lago é representada por uma região $D$ no plano $xy$, tal que a profundidade (em pés) sob o ponto correspondente a $(x,y)$ é dada por
$$f(x,y) = 300 - 2x^2 - 3y^2.$$
Se um nadador está no ponto $(4,9)$, em que direção deve nadar para que a profundidade sob ele decresça mais rapidamente?


 Na direção dada pelo vetor $(16,54).$


3051   

A figura mostra a região da integral

$$\int_{0}^{1}\int_{0}^{1-x^{2}}\int_{0}^{1-x}f(x,y,z)\;dy dz dx.$$

Reescreva essa integral como uma integral iterada equivalente nas cinco outras ordens.

ma211lista8q16.png


$\int_{0}^{1}\int_{\sqrt{x}}^{1}\int_{0}^{1-y}f(x,y,z)\;dz dy dx = \int_{0}^{1}\int_{0}^{y^2}\int_{0}^{1-y}f(x,y,z)\;dz dx dy $
$= \int_{0}^{1}\int_{0}^{1 - z}\int_{0}^{y^2}f(x,y,z)\;dx dy dz = \int_{0}^{1}\int_{0}^{1 - y}\int_{0}^{y^2}f(x,y,z)\;dx dz dy $
$= \int_{0}^{1}\int_{0}^{1 - \sqrt{x}}\int_{\sqrt{x}}^{1-z}f(x,y,z)\;dy dz dx = \int_{0}^{1}\int_{0}^{(1 - z)^2}\int_{\sqrt{x}}^{1-z}f(x,y,z)\;dy dx dz.$


2987   

Calcule a integral, efetuando uma mudança de variáveis apropriada. $\displaystyle\iint\limits_{R}\cos{\left(\dfrac{y - x}{y + x}\right)} \, dA$, em que $R$ é a região trapezoidal com vértices $(1,0)$, $(2,0)$, $(0,2)$ e $(0,1)$.


$\dfrac{3}{2} \sin(1).$


2692   

Encontre $f_{x}$, $f_{y}$ e $f_{z}$ para $f(x,y,z)=e^{-(x^{2}+y^{2}+z^{2})}$.


$\displaystyle f_{x} = -2xe^{-(x^{2} + y^{2} + z^{2})},\;\;\;\; f_{y} = -2ye^{-(x^{2} + y^{2} + z^{2})}\;\;\;\;\text{e}\;\;\;\; f_{z} = -2ze^{-(x^{2} + y^{2} + z^{2})}$.


2779   

Calcule a integral dupla usando coordenadas polares: $\displaystyle\iint\limits_{R}(x^{2}+2y)\,dx dy$, onde $R$ é o círculo $x^{2}+y^{2}\leq 4.$


$4\pi.$


2889   

Passe para coordenadas polares e calcule: $\displaystyle\int_{0}^{a}  \int_{0}^{\sqrt{a^{2}-x^{2}}}\sqrt{a^{2}-x^{2}-y^{2}}\,dy dx$, em que $a>0.$


$\displaystyle \frac{\pi a^3}{6}.$


2836   

Mostre que $(0,0)$ é um ponto crítico de $f(x,y)=x^{2}+kxy+y^{2}$, não importando o valor da constante $k$.


Note que $f_{x} (0,0) = f_{y} (0,0) = 0.$


2608   

Use o Teorema de Stokes para calcular $\displaystyle\iint\limits_{ S}\mbox{rot}{\bf F}\cdot d{\bf S}.$

  • ${\bf F}(x,y,z) = (e^{xy}\cos{z},(x^2+1)z,-y)$, $S$ é o hemisfério $x^2+y^2+z^2 = 1$, $x \geq 0$, orientado na direção positiva do eixo $x$.



$-2\pi$.


2911   

Ao calcular por integração dupla o volume $V$ do sólido situado abaixo do gráfico de $f(x,y)=e^{x^{2}+y^{2}}$ e limitado inferiormente por uma certa região $D$ no plano $xy$, chegou-se à seguinte expressão: $$V=\int_{0}^{2}  \int_{0}^{\sqrt{4-x^{2}}}e^{x^{2}+y^{2}}\,dy dx-\int_{0}^{1}  \int_{0}^{\sqrt{1-x^{2}}}e^{x^{2}+y^{2}}\,dy dx.$$

  1.  Esboce a região $D.$

  2.  Expresse $V$ numa única integral dupla em coordenadas polares.

  3.  Efetue a integração para calcular $V.$


  1.  $D = \left\lbrace (x,y); 1 \leq x^{2} + y^{2} \leq 2, x \geq 0, y \geq 0 \right\rbrace.$

  2.  $\displaystyle \int_{0}^{\frac{\pi}{2}} \int_{1}^{2} re^{r^2}  dr d\theta.$

  3.  $\dfrac{\pi}{4}(e^4 - 1).$


3065   

As linhas de escoamento (ou linhas de corrente) de um campo vetorial são as trajetórias seguidas por uma partícula cujo campo de velocidade é um campo vetorial dado. Assim, os vetores do campo vetorial são tangentes a suas linhas de escoamento.

Use um esboço do campo vetorial $\textbf{F}(x,y) = x\textbf{i} - y\textbf{j}$ para desenhar algumas linhas de escoamento. Desses seus esboços é possível descobrir qual é a equação das linhas de escoamento?
Se as equações paramétricas de uma linha de escoamento são $x=x(t)$ e $y=y(t)$, explique por que essas funções satisfazem as equações diferenciais $dx/dt = x$ e $dy/dt = -y$. Resolva então as equações de forma a obter uma equação da linha de escoamento que passe pelo ponto $(1,1)$.


2379   

Determine a equação da reta tangente à curva $\gamma$ no ponto $\gamma(t_0) = (2,5)$ sabendo-se que $\gamma'(t) \neq \bf{0}$ e que sua imagem está contida na curva de nível $xy = 10$. Qual a equação da reta normal a $\gamma$, neste ponto?


 Reta tangente: $(x,y) = (2,5) + \lambda (-2,5),$ $\lambda \in \mathbb{R},$
Reta normal: $(x,y) = (2,5) + \lambda (5,2),$ $\lambda \in \mathbb{R}.$


1941   

Calcule a integral de linha, onde $C$ é a curva dada.

$\displaystyle\int_{C}xyz\,ds$, onde $C$ é a hélice ${\bf r}(t)=(\cos t,\sin t,3t)$, $0\leq t\leq 4\pi.$


$-3\sqrt{10}\pi.$


2824   

Determine a menor distância entre o ponto $(2,1,-1)$ e o plano $x+y-z=1$.


$\sqrt{3}.$



A distância entre um ponto qualquer $(x,y,z)$ e o ponto $(2,1,-1)$ é \[ d=\sqrt{(x-2)^2+(y-1)^2+(z+1)^2} \] mas, se $(x,y,z)$ pertence ao plano $x+y-z=1$, então $z=x+y-1$, e assim temos \[ d=\sqrt{(x-2)^2+(y-1)^2+(x+y)^2}. \] Podemos minimizar $d$ minimizando a expressão mais simples \[ d^2=f(x,y)=(x-2)^2+(y-1)^2+(x+y)^2. \] Vamos encontrar os pontos críticos de $f$. Temos que $f_x(x,y)=2(x-2)+2(x+y)=4x+2y-4$ e $f_y(x,y)=2(y-1)+2(x+y)=4y+2x-2$. Fazendo $f_x=0$ obtemos \[ 4x+2y-4=0\Rightarrow 2x+y-2=0 \Rightarrow y=2-2x. \] Agora, fazendo $f_y=0$ e substituindo $y=2-2x$ obtemos: \[ 4y+2x-2=0\Rightarrow 4(2-2x)+2x-2=0\Rightarrow 8-8x+2x-2=0 \] \[ \Rightarrow -6x+6=0 \Rightarrow x=1 \] e, portanto, $y=2-2x=2-2\cdot 1=0$ é o único ponto crítico de $f$. Note que \[ f_{xx}(1,0)=4, f_{xy}(1,0)=2 \text{ e } f_{yy}(1,0)=4 \] assim $f_{xx}(1,0)f_{yy}(1,0)-(f_{xy}(1,0)) ^{2}=4\cdot 4-2^{2}=14-4=12>0$ e $f_{xx}(1,0)>0$. Portanto, pelo Teste da Derivada Segunda, $f$ tem um mínimo em $(1,0)$. Assim, a distância menor distância entre o ponto $(2,1,-1)$ e o plano $x+y-z=1$ é \[ d=\sqrt{(1-2)^2+(0-1)^2+(1+0)^2}=\sqrt{3}.

\]


2633   

Determine as derivadas parciais de primeira ordem da função $f(x,y)=\displaystyle\int_{y}^{x}\cos^2t \ \mathrm{d}t$.



Sendo $f(x,y)=\displaystyle\int_{y}^{x}\cos (t^{2})\,dt$, temos que as derivadas parciais em relação a $x$ e $y$, respectivamente, são:

$\bullet \dfrac{\partial}{\partial x}f(x,y)=\dfrac{\partial}{\partial x}\bigg(\displaystyle\int_{y}^{x}\cos(t^{2})\bigg)=\cos(x^{2}).$

$\bullet \dfrac{\partial}{\partial y}f(x,y)=\dfrac{\partial}{\partial y}\bigg(\displaystyle\int_{y}^{x}\cos(t^{2})\bigg)=\dfrac{\partial}{\partial y}\bigg(-\displaystyle\int_{x}^{y}\cos(t^{2})\bigg)=-\cos(y^{2}).$

Notemos que nas soluções das derivadas parciais acima utilizamos o Teorema Fundamental do Cálculo.


3057   

Faça uma correspondência entre as funções $f$ e os desenhos de seus campos vetoriais gradientes (rotulados de I-IV). Justifique.

  1. $f(x,y) = x^2+y^2$
  2. $f(x,y) = (x+y)^2$.
  3. $f(x,y) = x(x+y)$.
  4. $f(x,y) = \sin{\sqrt{x^2+y^2}}$.
I
ma211-list10-ex12_i.png

II

ma211-list10-ex12_ii.png

III

ma211-list10-ex12_iii.png

IV

ma211-list10-ex12_iv.png


  1. III.
  2. IV.
  3. II.
  4. I.

2067   

Dados ${\bf F}(x,y,z)=e^{y}\,{\bf i}+xe^{y}\,{\bf j}+(z+1)e^{z}\,{\bf k}$, $C: {\bf r}(t)=t\,{\bf i}+t^{2}\,{\bf j}+t^{3}\,{\bf k}$, $0\leq t\leq 1.$

  1. Determine uma função $f$ tal que ${\bf F}=\nabla f$.

  2. Use o resultado anterior para calcular $\int_{C}{\bf F}\cdot d{\bf r}$ sobre a curva $C$ dada.


  1. $f(x,y,z) = x e^{y} + ze^{z};$

  2. $2e.$


2118   

Calcule $\mathrm{d} z/\mathrm{d} t$ por dois processos:

  1. substituindo as expressões para $x(t)$ e $y(t)$ em $z$ e depois derivando diretamente com relação a $t$
  2. aplicando a Regra da Cadeia: $\frac{dz}{dt}=\frac{\partial z}{\partial x}\frac{dx}{dt}+\frac{\partial z}{\partial y }\frac{dy}{dt}$.

$z=x^{2}+3y^{2}$,$x=\sin{t}$ e  $y=\cos{t}.$


$\displaystyle \frac{dz}{dt} (t) = -4\sin(t)\cos(t).$


2002   

Uma partícula desloca-se em um campo de forças dado por ${\bf F}(x,y,z)=-y\,{\bf i}+x\,{\bf j}+z\,{\bf k}.$ Calcule o trabalho realizado por ${\bf F}$ no deslocamento da partícula de ${\bf r}(a)$ até ${\bf r}(b)$, sendo dados:

  1. ${\bf r}(t)=(\cos t, \sin t,t)$, $a=0$ e $b=2\pi.$

  2. ${\bf r}(t)=(2t+1,t-1,t)$, $a=1$ e $b=2.$

  3. ${\bf r}(t)=(\cos t,0, \sin t)$, $a=0$ e $b=2\pi.$


  1. $2\pi(1 + \pi).$

  2. $\dfrac{9}{2}.$

  3. $0.$


2827   

Determine o valor máximo de $f(x,y)=x+5y$, onde $x$ e $y$ estão sujeitos às restrições: $5x+6y\leq 30$, $3x+2y\leq 12$, $x\geq 0$ e $y\geq 0.$


$25.$


2862   

Estude com relação a máximos e mínimos a função dada com as restrições dadas.

$f(x,y) = x^2 + 2y^2$ e $3x + y = 1.$


Ponto de mínimo:  $\displaystyle \left( \frac{6}{19}, \frac{1}{19} \right)$.


2442   

Calcule a integral de superfície $\displaystyle\iint\limits_{S}y dS$, onde $S$ é a parte do parabolóide $y=x^{2}+z^{2}$ que está dentro do cilindro $x^{2}+z^{2}=4.$


$\dfrac{\pi(391\sqrt{17}+1)}{60}.$


2967   

Dentre as coordenadas cilíndricas ou esféricas, utilize a que lhe parecer mais apropriada para determinaretermine o volume da região limitada acima pelo paraboloide  $z=5-x^{2}-y^{2}$ e abaixo pelo paraboloide $z=4x^{2}+4y^{2}.$


$\dfrac{5\pi}{2}.$